Chapter 1: Geriatric Nephrology Has Come of Age: at Last

Download as pdf or txt
Download as pdf or txt
You are on page 1of 203

Chapter 1: Geriatric Nephrology Has Come of Age:

At Last
Dimitrios G. Oreopoulos* and Jocelyn Wiggins†

*Department of Medicine, University of Toronto, Toronto, Canada; and †University of Michigan, Ann Arbor,
Michigan

On May 23–24, 1985, the first International Sym- ease (MDRD) formula has revealed a large number
posium on Geriatric Nephrology was held in To- of patients who have impaired kidney function,
ronto. In his excellent review of the symposium,1 most of whom are elderly. Primary care physicians
Michael Kay commented on the challenges and are inundated with elderly patients with impaired
risks facing aged individuals and, in view of the in- kidney function and, in turn, are flooding the neph-
creased costs of their care, the ethical issues facing rologists with referrals.
the caregiver. His report stressed the fact that “the Nephrologists have had to take a serious look at
degree of humanity in our healthcare world will be the plight of elderly, and probably as a result, three
made evident in the way we treat (or do not treat) important developments have ushered in the new
our minorities, our underprivileged, our poor, our era of Geriatric Nephrology.
mentally infirm, those who have no voice to speak
1. For the first time, the American Society of Nephrology has in-
for themselves, and finally, the aged.” cluded in its annual Renal Week program a 2-d course on geri-
After that successful initial meeting, the Interna- atric nephrology that was sold out and that kept the interest of
tional Society of Geriatric Nephrology was formed, the participants to the end. All these presentations have been
with its own journal—the International Journal of taped and are available, with the accompanying slides, at http://
Geriatric Nephrology and Urology—and five addi- asn-online.org/education_and_meetings/media/geriatrics/. A
similar course has been planned for the 2009 ASN meeting.
tional international meetings of the International
Society were held at Salamanca, Lisbon, Atlanta, 2. Recognizing that geriatric nephrology is now essential to ne-
Thessaloniki, and Antalya. phrology training, the Accreditation Council for Graduate Med-
Despite all these efforts and activities, the interest ical Education (ACGME), in its program requirements for train-
among nephrologists concerning geriatric nephrology ing in nephrology, has mandated that “fellows must have formal
instruction, clinical experience and demonstrate competence in
did not increase and, if anything, was decreasing. the prevention, evaluation and management of geriatric aspects
Membership in the Society and participation to the of nephrology, including disorders of ageing kidney and urinary
meetings were small. Also subscriptions to and sub- tract.” In addition, the ACGME has mandated that “fellows must
mission of articles to the Society’s journal were not receive formal instruction in geriatric medicine, including phys-
sufficient to sustain it. As a result, the publisher de- iology and pathology of the ageing kidney, and drug dosing and
renal toxicity in the elderly patient.”
cided to publish the journal as a section in the journal
International Urology and Nephrology. In response to the above, the Chair of the ASN
All these regrettable circumstances seem to have Training Program Directors Committee (Dr. Donald
changed because of two important factors that have Kohan) invited a group of individuals to form a com-
contributed to a renewed interest in geriatric ne- mittee to design a curriculum by identifying the topics
phrology. First is the amazing increase in the inci- and authors to write the corresponding chapters and
dence of new patients with ESRD over the age of 65; we were honored to be asked to co-chair it. We were
this segment of our population is the fastest grow- impressed by the enthusiasm of all members of the
ing group of patients requiring dialysis, and it con- committee (Table 1). The committee identified 37
tinues to grow. Thus, nephrologists forced to prac-
tice as amateur geriatricians now recognize the need Correspondence: Dimitrios G. Oreopoulos, University Health
to master all aspects of geriatrics. Second, the intro- Network, 399 Bathurst Street, 8E-408, Toronto, Ontario M5T 2S8,
duction and automatic reporting of estimated GFR Canada. E-mail: [email protected]

(eGFR) using the Modification of Diet in Renal Dis- Copyright 䊚 2009 by the American Society of Nephrology

American Society of Nephrology Geriatric Nephrology Curriculum 1


Table 1. Geriatric Nephrology Curriculum Committee chapters (Table 2). These chapters are short (5 to 6 pages) and
Members (in alphabetical order) emphasize only knowledge related to geriatric nephrology. They
Abrass, Christine O’Hare, Ann provide key references for further reading with a powerpoint pre-
Abdel-Rahman, Emaad Oliver, Matthew sentation based on the content of each chapter and a number of
Campbell, Kellie Oreopoulos, Dimitrios multiple choice questions at its end.
Danziger, John Owens, Suzan
Dubeau, Catherine Patel, Sanjeevkumar 3. In 1998, ASN agreed to create a process to develop geriatric nephrologists. A
Dubose, Tom Quinn, Robert workshop was held to review the process and, in conjunction with the Associ-
Feinfeld, Don Reckelhoff, Jane ation of Specialty Professors (ASP), a program of career development awards
for geriatric nephrologists was initiated. One or two awards are made on a
Ferrucci, Luigi Rosner, Mitchell
competitive basis each year to a junior faculty nephrologist who commits to
Friedman, Eli Sands, Jeff
develop a career in some aspect of geriatric nephrology. Applications are re-
Gambert, Steven Schlanger, Lynn viewed by the ASN, and successful candidates are referred to ASP. ASP manages
Hartman, Erica Shim, Rosemare a fund provided by Atlantic Philanthropies and the Hartford Foundation and
Hollander, Jay Singh, Harmeet makes the T. Franklin Williams Scholarship Award each year. The scholarship
Jassal, Vanita Stankus, Nicole provides 2 yr of funding for career development in a combination of nephrology
Kuchel, George Striker, Gary and geriatric skills. The first scholarships were awarded in 2003, and a total of
Kujubu, Dean Swartz, Richard nine scholarships were awarded through 2008. It is anticipated that these schol-
Leinau, Lisa Swidler, Mark ars will form a group that will become leaders in this area and train others to
Mehta, Manisha Juthani Tamura, Manju provide age-sensitive nephrology care in the future. Several of these scholars
have contributed to the writing of the curriculum initiative discussed above.
Michelis, Michael Unruh, Mark
Miller, Myron Vijil, Julio In collaboration with various geriatric groups, our committee
Miyawaki, Bill Wiggins, Jocelyn will develop teaching tools in the diagnosis and management of
Mohamed, Maha Williams, Mark various geriatric entities.
Morley, John Winkelmayer, Wolfgang The curriculum will be available at the ASN⬘s website. We are
Munikrishnappa, Devraj Wright, Seth confident that it will strengthen the teaching of geriatric nephrol-
ogy not only in the United States but also throughout the world. A
number of colleagues from other countries have already expressed
great interest suggesting that, at last, geriatric nephrology has
come of age. We look forward to its rapid growth.

Table 2. Geriatric Nephrology Curriculum chapters


Table of Contents Author(s)
Geriatric Nephrology Has Come of Age: At Last Dimitrios Oreopoulos and Jocelyn Wiggins
Why Do We Need a Geriatric Nephrology Curriculum? Jocelyn Wiggins and Dimitrios Oreopoulos
The Coming Pandemic of Chronic Kidney Disease/ESKD and the Aging Jocelyn Wiggins and Sanjeet Kumar Patel
Population
Kidney Senescence Lynn Schlanger
Rate of Decline in eGFR and Clinical Evaluation of the Elderly With a Low eGFR Ann O’Hare and Rose Shim
Limitations of Various Formulae and Other Ways of Assessing GFR in the Elderly: Devraj Munikrishnappa
Is There a Role for Cystatin C?
Decline of Renal Function in Normal Aging, Role of Oxidants/Inflammation: When Helen Vlassara, Luigi Ferrucci, James Post and
Does It Begin, Is It Inevitable, Preventable, Treatable? Gary Striker
Diabetic CKD in the Elderly Mark Williams
Drug Dosing and Renal Toxicity in Elderly Patients William Bennett
Glomerular Disease in the Elderly Christine Abrass
Hypertension, Chronic Kidney Disease, and the Elderly Ann O’Hare
Cardiovascular Disease in the Elderly With Kidney Disease Wolfgang Winkelmayer
Vascular Disease in the Elderly Nobuyuki Miyawaki and Paula Lester
Bone Disease and Calcium Abnormalities in the Elderly With CKD Harmeet Singh
Anemia in the Geriatric Population With CKD Julio Vijil
Disorders of Serum Sodium Concentration in the Elderly Michael Michelis
Fluid Balance Disorders in the Elderly Myron Miller
Acute Kidney Injury in the Elderly Mitchell Rosner
Nocturia in the Elderly Persons and Nocturnal Polyuria Dean Kujubu
Hemodialysis in the Elderly Seth Wright
Vascular Access for Hemodialysis in the Elderly Seth Wright and John Danziger
Peritoneal Dialysis in the Elderly Seth Wright and John Danziger
Assisted Home Dialysis in the Elderly Matthew Oliver and Robert Quinn

2 American Society of Nephrology American Society of Nephrology


Table 2. Continued
Table of Contents Author(s)
Renal Transplantation in the Older Adult Erica Hartman
Interaction of Dialysis Teams With Geriatricians Nicole Stankus and Kellie Campbell
Comprehensive Geriatric Assessment: A Multidimensional Process Designed to Stephen Gambert
Assess an Elderly Person’s Functional Ability, Physical Health, Cognitive and
Mental Health, and Socio-Environmental Situation
Rehabilitation Services for Elderly Dialysis Patients Vanita Jassal
Integrated Care of the Elderly With ESRD Jocelyn Wiggins
Nutrition and the Kidney in the Elderly Patient John Morley
Urinary Incontinence in the Elderly George Kuchel and Catherine DuBeau
Lower Urinary Tract Conditions in the Elderly Population Damon Dyche and Jay Hollander
Urinary Tract Infections in Elderly Persons Manisha Juthani-Mehta
Falls in Elderly Patients With Kidney Disease John Morley
Association Between CKD and Frailty and Prevention of Functional Losses Emaad Abdel-Rahman
Methods to Assess Quality of Life and Functional Status and Their Applications in Tara Chang and Manjula Tamura
Clinical Care
Recognizing Delirium, Dementia, and Depression Manjula Tamura
Dialysis Decisions in the Elderly Patient With Advanced CKD and the Role of Mark Swidler
Nondialytic therapy
End of Life and Decision-Making in Elderly Patients With ESRD Richard Swartz and Erica Perry

American Society of Nephrology American Society of Nephrology 3


Chapter 2: Why Do We Need a Geriatric Nephrology
Curriculum?
Jocelyn Wiggins
University of Michigan, Ann Arbor, Michigan

In 2005, ACGME (Accreditation Council for Grad- tients with chronic kidney disease (CKD) usually
uate Medical Education) issued the following state- require complicated medication routines, complex
ment with respect to nephrology fellowship train- dietary restrictions, and frequent medical visits.
ing: Many patients in this age group have lost the ability
“Fellows must have formal instruction, clinical to administer their own pills, to buy and cook their
experience and demonstrate competence in the own groceries, or to drive themselves to office visits
prevention, evaluation and management of geriat- or dialysis units. Many patients, particularly in the
ric aspects of nephrology, including disorders of the diabetic population, have difficulties with basic
aging kidney and urinary tract.” In addition, the mobility. It is essential that the nephrologist be fa-
ACGME mandated that “fellows must receive for- miliar with and able to perform routine functional
mal instruction in geriatric medicine, including assessments of their older patients. This includes
physiology and pathology of the aging kidney; and evaluating cognitive, affective, functional, social,
drug dosing and renal toxicity in the elderly pa- economic, and environmental status. This enables
tient.” them to customize a regimen or direct the patient to
The following curriculum is an attempt to fulfill a living environment where such supportive care is
this mandate and prepare the next generation of available. In a busy practice, some aspects of this
nephrologists for the comprehensive care of the assessment can be allocated to other providers such
older population with kidney disease. as social workers, nurses, nurse practitioners, or
The over 65 population in the United States is physician assistants. The management of those el-
rapidly growing. During the next 20 yr, it is ex- derly who require chronic dialysis is even more
pected to double (Figure 1). This means that during complex. They frequently have more difficulty with
the professional lives of current fellows, they can vascular access; they have more cardiovascular dis-
expect to see an increasing number of older patients ease that leads to arrhythmias and hypotension
in their practice. Average life expectancy in 2004 while on dialysis. Furthermore, traveling to and
was 75.2 yr for men and 80.4 yr for women; by 2015, from the unit is a greater burden to them. Further
it is expected to be 76.2 and 82.2 yr, respectively, improvements in assisted dialysis at home will allow
and to continue growing. During the 1990s, the them to enjoy the benefits of treatment at home and
over 85-yr-old population was the fastest growing is an area that needs further exploration.
group at 38% growth. This older age group is the Patients in this older age group are likely to have
largest consumer of healthcare services. In 2005, multiple comorbidities. The average 75 yr old suf-
only 5% of the over 75-yr population had no health fers from 3.5 chronic diseases.1 Many symptoms in
visits, whereas fully 30% of those with 10 or more older patients are caused by multiple deficits and
visits were in this age group, although they consti- not by a single disease. These diseases and their
tute ⬍10% of the population.1 treatments are likely to interact and complicate one
Why should the aging of the population impact another. Murray2 has reported that up to 70% of
nephrologists? There are currently about 35 million dialysis patients 55 yr of age and older have chronic
people over 65 yr of age in the United States. Forty
percent of this population has some level of disabil- Correspondence: Jocelyn Wiggins, University of Michigan, 1150
ity: sensory, physical, mental, or self-care.1 Once a W. Medical Center Drive, 1560 MSRB II, Ann Arbor, MI 48109.
senior develops disability, it greatly impacts on their E-mail: [email protected]
ability to follow a complex medical regimen. Pa- Copyright 䊚 2009 by the American Society of Nephrology

American Society of Nephrology Geriatric Nephrology Curriculum 1


Number of Persons over 65 in Millions Table 1. Common geriatric conditions that impact on
nephrology care
80 69.4
70 Visual impairment
60 53.2 Hearing impairment
Millions

50 39.4
40 34.4 34.7 Malnutrition/weight loss
30 25.7 Urinary incontinence
16.7
20 9 Balance/gait impairment/falls
10 3.1 4.9
0 Polypharmacy
1900 1920 1940 1960 1980 1999 2000 2010 2020 2030 Cognitive impairment
Year Affective disorders
Functional limitations
Figure 1. Projected population over age 65 yr from the US Lack of social support
Census Bureau. Economic hardship
Home environment/safety
cognitive impairment of a level severe enough to impact on their
compliance and ability to make informed decisions.3,4 Prevalence
of depression is reported to be as high as 45% in the older dialysis were significantly more likely to be older (median, 63.2 versus
population.5–7 Metabolic bone disease is complicated by age-re- 59.3 yr; P ⬍ 0.05), because serum creatinine is a poor predictor
lated osteoporosis. The cardiovascular consequences of CKD are of renal function in the elderly. GFR declines with age in nor-
complicated by structural heart disease such as valvular insuffi- mal individuals; therefore, it can be difficult to distinguish age-
ciency and atrial fibrillation. Neurodegenerative disease impacts related decrease in GFR from CKD in the elderly. Older pa-
on the patient’s mobility and cognitive function. Osteoarthritis tients with mild decreased GFR and low risk for progressive
and neuropathy limit their physical activity. As age and disease decline in GFR need to be distinguished from those with pro-
advance, frailty becomes an issue. All of these things combine to gressive disease, because once identified, they probably do not
make their care much more complex than that of a younger pa- need to be followed by a nephrologist.
tient. Drug interactions and inappropriate dosing becomes an in- In conclusion, older patients will make up a growing pro-
creasing issue as the number of comorbidities and medications portion of the nephrologist’s practice. Thus, nephrologists
increases (Table 1). need to become comfortable with shouldering the full care of
In 1992, Nespor and Holley8 did a small study of in-center this segment of their patient population or work closely with a
hemodialysis patients in Pittsburgh. Eighty percent of these geriatrician and family physicians. As we become more willing
patients did not have a family physician and relied on their to offer life-prolonging technologies in the older age groups,
nephrologist for all of their medical care. Ninety-one percent we need to be willing to deal with the consequences of this
sought treatment from their nephrologist for minor acute ill- decision. Finally, with their elderly patients, nephrologists face
ness. Nephrologists were also providing ongoing treatment for challenging ethical problems, such as whether to withhold or
comorbid chronic illnesses such as diabetes and heart disease. withdraw dialysis. Unless addressed promptly and effectively,
In 1993, they went on to confirm similar statistics in their these ethical issues will greatly increase the stress on both the
chronic peritoneal dialysis patients.9 This would suggest that healthcare provider and family members.
the nephrologist needs to be prepared to take on the full com-
plexity of care for their older patients, particularly their
chronic dialysis population. In older patients, this would in- TAKE HOME POINTS
clude health maintenance screening and immunizations. Al- • A knowledge of geriatric medicine is required by ACGME for training in
though malignancies are more common in both the dialysis nephrology
population and in the posttransplantation population than in • The population over 65 yr of age will double in the next 20 yr
the general population, life expectancy, age, and cost effective- • This older population will bring their problems with them to the neph-
rologists
ness need to be considered by the nephrologists before order- • Dialysis patients rely on their nephrologists for most or all of their care
ing screening tests.
Patients with possible CKD are being referred to nephrolo-
gists in greater numbers since the introduction of formulae for
DISCLOSURES
estimating GFR. Most clinical laboratories supply an eGFR None.
when a serum creatinine is ordered. NHANES data estimates
that approximately 11% of the US population has CKD, and
this may be as high as 30% in the older population.10 A recent REFERENCES
Australian study showed that monthly referrals overall in-
creased by 40% after the introduction of eGFR reporting, and *Key References
this was most marked for the tertiary renal service (52% above 1. US Census Bureau: www.census.gov/
baseline).11 Patients referred after the introduction of eGFR 2. Murray AM: Cognitive impairment in the aging dialysis and chronic

2 American Society of Nephrology American Society of Nephrology


kidney disease populations: an occult burden. Adv Chronic Kidney chronic renal disease: where are we going? J Ren Nutr 18: 99 –103,
Disease 15: 123–132, 2008 2008
3. Murray AM, Tupper DE, Knopman DS, Gilbertson DT, Pederson SL, Li 8. Nespor SL, Holley JL: Patients on hemodialysis rely on nephrologists
S, Smith GE, Hochhalter AK, Collins AJ, Kane RL: Cognitive impair- and dialysis units for maintenance health care. ASAIO J 38: M279 –
ment in hemodialysis patients is common. Neurology 67: 216 –223, M281, 1992*
2006* 9. Holley JL, Nespor SL: Nephrologist-directed primary health care in
4. Madero M, Gul A, Sarnak MJ: Cognitive function in chronic kidney chronic dialysis patients. Am J Kidney Dis 21: 628 – 631, 1993*
disease. Semin Dial 21: 29 –37, 2008 10. Coresh J, Astor BC, Greene T, Eknoyan G, Levey AS: Prevalence of
5. Kimmel PL: Depression in patients with chronic renal disease: what chronic kidney disease and decreased kidney function in the adult US
we know and what we need to know. J Psychosom Res 53: 951–956, population: Third National health and Nutrition Examination Survey.
2002 Am J Kidney Dis 41: 1–12, 2003*
6. Watnick S, Kirwin P, Mahnensmith R, Concato J: The prevalence and 11. Noble E, Johnson DW, Gray N, Hollett P, Hawley CM, Campbell SB,
treatment of depression among patients starting dialysis. Am J Kidney Mudge DW, Isbel NM: The impact of automated eGFR reporting and
Dis 41: 105–110, 2003* education on nephrology service referrals. Nephrol Dial Transplant 23:
7. Kimmel PL, Cohen SD, Peterson RA: Depression in patients with 3845–3850, 2008

American Society of Nephrology American Society of Nephrology 3


REVIEW QUESTIONS: WHY DO WE NEED A 3. The prevalence of depression in the dialysis population is
GERIATRIC NEPHROLOGY CURRICULUM? a. 5%
b. 25%
1. The 85-yr-old population is growing at what rate per year?
c. 33%
a. 10%
d. 45%
b. 45%
e. 80%
c. 25%
d. 38%
4. What percent of dialysis patients receive all their medical care
2. The average 75-yr-old suffers from how many chronic dis- from their nephologist?
eases? a. 10%
a. 0 b. 20%
b. 5 c. 30%
c. 3.5 d. 50%
d. 10 e. 80%
e. 2

4 American Society of Nephrology American Society of Nephrology


Chapter 3: The Coming Pandemic of CKD/ESKD and
the Aging Population
Jocelyn Wiggins and Sanjeevkumar Patel
University of Michigan, Ann Arbor, Michigan

In chapter 1, data were presented about the growth hard to get an accurate measure of the extent of
of the older US population and their special needs. the increase.5,6 Coresh et al.7 have estimated that
In this chapter, we will discuss the epidemiology of the overall prevalence of CKD has increased from
the elderly patients with chronic kidney disease 10 to 13% of the US adult population since 1988.
(CKD) and end-stage kidney disease (ESKD). Because 50% of patients in the United States start
dialysis with no previous nephrology care, over-
referral is probably preferable to underreferral.8
CHRONIC KIDNEY DISEASE Another contentious area of debate is the decline
of renal function with age. It is generally accepted
There is much debate in the literature about that renal function declines about 1 ml/min per
whether the incidence of CKD is actually increas- year after the fourth decade of life, even in the ab-
ing or whether we are measuring changes in the sence of comorbidities such as diabetes and hyper-
way we detect and define CKD. In 1998, a report tension. Many nephrologists regard this as “normal
using NHANES data estimated the prevalence of aging” and do not feel that this constitutes a reason
CKD as about 11% in the US adult population.1 for referral. It is certainly true that only a very small
This estimate was based on routine creatinine fraction of these older patients will progress to end
measurements in a subset of the study popula- stage or die from renal failure. It is likely that they
tion. Since that data were published, several de- are at greater risk from vascular disease because
velopments have altered the way we define CKD. there is a very robust association between decline in
The Modification of Diet in Renal Disease GFR and vascular deaths.9
(MDRD) formula was developed and validated.2 So, which older patients should the busy neph-
The National Kidney Foundation created a panel rologists follow and which should be returned to
of experts, who redefined how CKD was classified the care of their primary care physician after an ini-
and staged.3 There has been a change in way cre- tial evaluation? Clearly signs of ongoing active renal
atinine is measured. Automated reporting of disease such as an active urine sediment or signifi-
eGFR based on MDRD formula was initiated in cant proteinuria are reason for a nephrologist’s
most clinical laboratories across the country. care. eGFR values between 45 and 59 ml/min per
These changes resulted in an apparent “pan- 1.73 m2 in those 70 yr of age and older should be
demic” of CKD. A study from an academic de- interpreted with caution. If other signs of kidney
partment in Australia tracked the level of ne- damage (e.g., proteinuria, hematuria) are not
phrology referrals after the implementation of present, a stable eGFR in this range may be consis-
automated eGFR reporting.4 General referrals in- tent with typical GFR for this age and an absence of
creased by 40%, whereas referrals to the tertiary CKD-related complications. Patients showing the
renal service were 52% above baseline. The pa- complications of decreased renal function such as
tients newly referred were significantly older: anemia, phosphorous retention, and hyperkalemia
63.2 versus 59.3 yr. However, the quality of the need nephrology management.
referrals declined with as many as 35% being in-
appropriate. It seems likely with the increases in Correspondence: Jocelyn Wiggins, University of Michigan, 1150
the incidence of diabetes, vascular disease, and W. Medical Center Drive, 1560 MSRB II, Ann Arbor, MI 48109.
the general aging of the population that the true E-mail: [email protected]
prevalence of CKD has increased, but it is very Copyright 䊚 2009 by the American Society of Nephrology

American Society of Nephrology Geriatric Nephrology Curriculum 1


Incident Rates of Treated ESKD
1600

1400
Rates per million pop

1200

1000

800

600

400

200

0 Figure 1. Incident rates of treated ESKD per million of


0-4 5-9 10-14 15-19 20-29 30-39 40-49 50-59 60-64 65-69 70-79 80+
population by decade. Data from the USRDS 2008
Age annual report.

END-STAGE KIDNEY DISEASE dialysis. The incident rates have been rising steadily over the
last 25 yr (Figure 2), with a narrowing gap between rates in the
Because Medicare mandates that all US dialysis units receiving 70- to 79-yr-old age group compared with the 80⫹-yr age
Medicare compensation report clinical data to the US Renal Data group. These data reflect numbers of patients who have sur-
System (USRDS), information on patients reaching ESKD is vived at least 90 d on dialysis and do not include those who get
much more robust than the data on CKD. The data for these acute dialysis in the hospital and do not progress to chronic
patients were all taken from the USRDS 2007 annual report.10 maintenance dialysis because of recovery or death.
Data collected by USRDS show that ESKD is a disease of the Rates of morbidity and mortality are higher in the ESKD
older population, with numbers starting to rise significantly population than in the general Medicare population. Hospital
after the age of 50. Mean age at the start of renal replacement admission rates are particularly high in the oldest patients,
therapy is 62.3 yr for men and 63.4 yr for women. Peak incident with cardiovascular disease being by far the most common
counts of treated ESKD occur in the 70- to 79-yr age group at cause for hospitalization. Patient admission rates increase lin-
⬎15,000 patients per year. Peak incident rates of treated ESKD early with age. A 20-yr-old patient with ESKD spends an aver-
occur in the 70- to 79-yr-old age group at 1543 per million age of 9 d per patient year in the hospital compared with 15.5 d
population (Figure 1). for patients over 70 yr of age. Older patients also carry signifi-
This probably reflects both a real increase in the rates of cant burden of disability. Overall, 10% carry a diagnosis of
patients reaching ESKD and an increase in the willingness to dementia, and this rises to 21% among those over 80 yr of age.
offer dialysis, regardless of age or comorbidity. The data show This is almost certainly an underestimate because dementia is
a drop off after 79 yr of age. This probably reflects the tendency often undiagnosed. As many as 20% of older patients with
of older patients, with significant burden of disease refusing ESKD have had a stroke that limits their mobility. These co-

Incident Rates of ESKD over Time

1800
Rate per million population

1600
1400
1200
1000 80+
800 70-79
600
400
Figure 2. Incident rates of treated ESKD per
200 million population from 1980 to 2005. Blue
0 bars represent patients over 80 yr of age. Red
1980 1985 1990 1995 2000 2005 bars represent the 70- to 79-yr-old age
group. Data from the USRDS annual report
Year for 2007.

2 American Society of Nephrology American Society of Nephrology


Probability of Survival ESRD patients

120

Probability of survival
100

80
1 yr
60
2 yr
40

20
Figure 3. Survival probabilities for patients
on dialysis: year 1 in blue and year 2 in red. 0
Data shown for patients across the life span.

+
4

9
9

9
9
9
4
9

9
10 9
14
All age groups show lower survival on dialysis
0-

80
-1
-2

-3
-4
-5
-6
-6

-7
o
5t
to
15
20

30
40
50
60
65

70
than age-matched controls with normal renal
function. Data from USRDS annual report
2007. Age in years

morbidities impact significantly on a patient’s ability to man- • Rates of treated ESRK are increasing in all older age groups
• ESKD carries a poor prognosis
age a complex medical regimen.
In addition to the significant burden of comorbidity, all-
cause mortality is six times higher in the ESKD population DISCLOSURES
than in the general Medicare population. When discussing di- None.
alysis, patients and families need to understand that, although
renal replacement therapy does prolong life, life expectancy is REFERENCES
very limited in the older population. Average 1-yr survival for *Key References
a 70 to 79 yr old is 70%, and for an 80 yr old is 60%. By 2 yr, 1. Jones CA, McQuillan GM, Kusek JW, Eberhardt MS, Herman WH,
survival drops to 52.7 and 39.7%, respectively (Figure 3). Coresh J, Salive M, Jones CP, Agodoa LY: Serum creatinine levels in
CKD and ESKD are huge financial burdens to our medical the US population: Third National Health and Nutrition Examination
system. In 2005, Medicare costs for CKD were $42 billion and for Survey. Am J Kidney Dis 32: 992–999, 1998*
2. Levey AS, Bosch JP, Lewis JB, Greene T, Rogers N, Roth D: A more
ESKD were $20 billion. The cost of ESKD was one half that of accurate method to estimate glomerular filtration rate from serum
CKD, although only a very small percentage of patients with CKD creatinine: a new prediction equation. Modification of Diet in Renal
progress to ESKD. According to NHANES data, about 11% of the Disease Study Group. Ann Intern Med 130: 461– 470, 1999
US population has CKD, whereas ⬍0.2% of the US population 3. Anonymous: K/DOQI clinical practice guidelines for chronic kidney
has ESKD. Despite this low prevalence, ESKD was responsible for disease: evaluation, classification, and stratification. Kidney Disease
Quality Initiative. Am J Kidney Disease 39: S1–S246, 2002
6.4% of the entire Medicare budget. The annual per person cost 4. Noble E, Johnson DW, Gray N, Hollett P, Hawley CM, Campbell SB, Mudge
for dialysis alone exceeded $65,000 in 2005. If all medical care is DW, Isbel NM: The impact of automated eGFR reporting and education on
included, this figure is even higher. For the 70- to 79-yr-old age nephrology service referrals. Nephrol Dial Transplant 23: 3845–3850, 2008
group, the per person annual cost of dialysis is more than $69,000 5. Glassock RJ, Winearls C: The Global Burden of Kidney Disease: How
and in the 80⫹-yr group is more than $74,000. valid are the estimates? Nephron 110: c39 – c47, 2008
6. Coresh J, Stevens LA, Levey AS: Chronic kidney disease is common:
In conclusion, CKD and ESKD are diseases of the elderly. The what do we do next? Nephrol Dial Transplant 23: 1122–1125, 2008
incidence and prevalence of these conditions are rising, especially in 7. Coresh J, Selvin E, Stevens LA, Manzi J, Kusek JW, Eggers P, Van
the older age groups. Progressing to ESKD carries a significant bur- Lente F, Levey AS: Prevalence of chronic kidney disease in the US
den of comorbidities and clearly shortens life expectancy. Treating during 1988 –1994 and 1999 –2004. JAMA 298: 2038 –2047, 2007*
patientsforESKDisevenmoreexpensiveintheolderagegroupsthan 8. Obrador GT, Ruthazer R, Arora P, Kausz AT, Pereira BJ: Prevalence of
and factors associated with suboptimal care before initiation of dialysis
for younger patients. Preventing progression of CKD should be an in the United States. J Am Soc Nephrol 10: 1793–1800, 1999
urgent priority for every nephrologist, even in the oldest patients. 9. Weiner DE, Tighiouart H, Elsayed EF, Griffith JL, Salem DN, Levey AS,
Sarnak MJ: Inflammation and cardiovascular events in individuals with
and without chronic kidney disease. Kidney Int. 73: 1406 –1412, 2008
TAKE HOME POINTS 10. US Renal Data System: USRDS 2007 Annual Data Report: Atlas of
• eGFR declines with age but does not necessarily indicate clinically Chronic Kidney Disease and End-Stage Renal Disease in the United
significant CKD States. Bethesda, MD, National Institutes of Health, National Institute
• Peak incidence of treated ESKD is in the 70- to 79-yr age group of Diabetes and Digestive and Kidney Diseases, 2007*

American Society of Nephrology American Society of Nephrology 3


REVIEW QUESTIONS: THE COMING PANDEMIC OF 3. A 70-yr-old dialysis patient will spend, on average, how many
CKD/ESKD AND THE AGING POPULATION days per year in the hospital?
1. The current prevalence of CKD in the US population is a. 2.5 d
a. 1 to 5% b. 6.1 d
b. 10 to 15% c. 8.3 d
c. 20 to 25%
d. 15.5 d
d. 30 to 35%
e. 21.7 d
e. None of the above
2. The peak incidence of treated ESKD falls in which of following 4. Compared with the general Medicare population, mortality in
age ranges?
patients with ESKD is how many fold higher?
a. 50 to 59 yr
b. 60 to 64 yr a. 2-fold
c. 65 to 69 yr b. 6-fold
d. 70 to 79 yr c. 10-fold
e. 80⫹ yr d. 19-fold

4 American Society of Nephrology American Society of Nephrology


Chapter 4: Kidney Senescence
Lynn Schlanger
Emory University and Veterans Affairs Medical Center at Atlanta, Atlanta, Georgia

In the United States, the elderly and the very elderly juxta-arcuate glomeruli until age 2. At this time, the
population has largely exceeded that of any other size of all of the glomeruli are the same, and the
age group.1 By 1994, the population of these demo- kidney is functioning at adult capacity.17The num-
graphic groups reached 36.5 million and has con- ber of glomeruli among individuals is quite vari-
tinued to increase over the last decade.1 This growth able, ranging from 247,652 to 1,825,380 per kidney,
parallels the increasing number of elderly persons and decreases with age14,18 at a rate of approxi-
classified with chronic kidney disease (CKD) stages mately 6752 glomeruli/yr after the age of 18.14
III through V. Moreover, an estimated 660,000 per- Renal mass increases from 50 g at birth to ⬎400
sons in the United States will have end-stage kidney g during the third and fourth decades of life before
disease (ESKD) by the year 2010, with the greatest decreasing to ⬍300 g by the ninth decade.5,13,14,18,19
growth rate occurring in the elderly and very elderly The latter decrease correlates with the loss of the
persons.1–3 Unfortunately, understanding of the renal cortex. Radiographically, the size of the kid-
normal biologic progression of renal disease in the ney has been shown to decrease in size by 10%after
elderly persons in the absence of comorbid fac- age 40 to 30% by age 80.20 –23 Using the Xenon
tors4 –9 or the progression of CKD is still not clearly washout technique, Hollenberg et al.22 noted that a
understood.10 decrease in the size of the kidney correlated with a
Cross-sectional and longitudinal studies have decrease in function and in the renal blood flow to
looked at the natural progression of the kidney with the cortex.22
aging.4,7,8,11 A linear relationship between aging and
a decline in the renal function was noted,7,8 but el-
derly persons who had no underlying disease had HISTOLOGY
adequate renal reserve.12–14 The Baltimore Longitu-
dinal study (BLS) from 1958 until 1981 studied a The histologic changes with aging observed in hu-
cohort of individuals for 8 or more yr who had a mans have been obtained from information from
least five 24-h urine collections for creatinine clear- autopsies or nephrectomies14,19,23–26 or studies in-
ance.7,8 There were three groups: group 1, CKD; volving laboratory animals.27,28 With aging, there
group 2, on anti-hypertension medications; group are certain universal findings in the cortex, medulla,
3, healthy patients. The overall rate of decline in and, in most cases, in the interstitium and vessels
creatinine clearance was 0.87 ml/min per year be- (Table 1; Figure 1). These histologic changes corre-
ginning at age 40 and was inversely related to age.7,8 late with functional changes observed with aging,
A rise in mean arterial pressure ⬎107 mmHg was including an inability to concentrate or dilute the
positively correlated with a decline in renal func- urine, an increased propensity toward salt reten-
tion.7 Interestingly, in the BLS, one third of the el- tion, dehydration, and acute kidney injury.
derly population had no decrease in renal function
as measured by creatinine clearance, and a small Glomerulus
segment actually had improvement in their renal With aging, hyaline expansion within the mesan-
function.8 gium results in the obliteration of the glomerular
In humans and some animals,14,15 the number of
glomeruli present in adulthood are predetermined
Correspondence: Lynn Schlanger, Assistant Professor, Emory
between weeks 32 and 36 of gestation,14,16 whereas University and Veterans Affairs Medical Center at Atlanta, Atlanta,
the number of glomeruli continue to increase in GA 30033. Phone: 404-727-2525; Fax: 404-727-3425; E-mail:
rats and mice after gestation.15 In humans, the su- [email protected]

perficial cortex glomeruli differ in size from the Copyright 䊚 2009 by the American Society of Nephrology

American Society of Nephrology Geriatric Nephrology Curriculum 1


Table 1. Histologic change in the aging kidney
Site Changes
Glomerulus Thickening basement membrane, increase mesangial matrix, focal global sclerosis, hypertrophy
Podocytes Fusion intermittent, detachment, vacuoles
Interstituim Tubular atrophy, tubular cast, monocytes infiltrates, interstitial fibrosis
Vessels Atrophy of afferent and efferent, hyalinosis of vessels, ⬙aglomeruls vessels⬙

loops28,29 and is associated with capillary tuft collapse, intra- ular weight proteins.37 These findings differ in the human
capsular fibrosis, and proteinuria.14 The sclerosis in the glo- GBM, where a decrease in hydroxylysine, 4- hydroxyproline,
meruli is primarily in the superficial cortex with sparse changes and glycosylation of collagen occurs with aging.26 The reason
in medulla.14,16,19 Cortical atrophy and loss of the renal paren- for these differences is not clear.
chyma result.18 One hundred forty-six cadaveric kidneys from
medical examiner offices and autopsies from hospital patients Tubulointerstitium
showed an increase in cortical glomerulosclerosis with age With aging, tubular dilation, intratubular cast formation,
from 5% at age 40 to 10% by the eighth decade.30The degree of thickening and splitting of the basement membrane, and fibro-
sclerosis was found to correlate with the degree of atheroscle- sis of the interstitium occurs.18,35,38 In 24-mo-old rats, scans of
rosis, suggesting a hemodynamic role in the aging process.31 the interstitium showed cellular infiltrates consisting predom-
The remaining glomeruli are enlarged to compensate for the inantly of macrophages and lymphocytes and an increase in
decrease in number of functioning cortex glomeruli.12,18,19,32,33 intracellular adhesion molecule (ICAM)-1, osteopontin, and
Electron scans showed podocyte injury with features that in- collagen IV. Areas were marked by an increase of apoptosis.
cluding hypertrophy, intracellular uptake of protein/absorp- None of these findings were detected in the 3-mo-old pups.38
tive droplets, foot process fusion, and detachment of the podo- After the administration of enalapril to 15-d-old CF1 mice, a
cytes from the glomerular basement membranes (GBMs).29,34 decrease in the peritubular and interstitial sclerosis occurred
by 18 mo of age compared with the control mice or mice
Glomerular Basement Membrane treated with nifedipine. A decrease in expression of SM-actin, a
The GBM increases in width with age.27,29,35 In Sprague-Daw- cytoskeleton protein commonly found in fibrosis and repair,
ley rats, the GBM increased in size from 1300 Å at birth to 4800 was also noted in the enalapril-treated group.38
Å at 24 mo.29 In humans, the basement membrane increases
until age 40, and after age 60, the surface area decreases with Vessels
wrinkling of the basement membrane with deposition of hya- An early angiographic study showed changes in the arteriole-
line.29 The composition of the basement membrane also glomerulus unit with aging.24 In the arterioles, hyaline deposi-
changes with aging.36,37 In older rats, the amino acid composi- tion within the vessels walls leads to obliteration of the lumen
tion shifts to a more collagen-like material marked by an in- and is associated with sclerotic glomeruli primarily in the cor-
crease in hydroxylysine, hydroxyproline, and glycine, and tex.22,24,31 Two structural types associated with the afferent and
more insoluble amino acids with higher content of low molec- efferent arterioles have been described.24 In the first case, oblit-

Lumen obliteration
Hyaline deposits in arterioles
RFB

Figure 1. Histologic changes in the aging


Glomerulosclerosis in cortex
kidney. There is a decrease in the renal blood
flow from hyaline deposition and obliteration
of the arterioles resulting in glomerular
RBF
changes such as wrinkling of the loops and
“Aglomerular” arterioles
Hypertrophy of glomeruli in noted hyaline deposition in the mesangium.
FF The loss of these glomeruli is primarily in the
cortex resulting in hypertrophy of the remain-
ing glomeruli. In the medulla, the arterioles
form “ aglomerular “ arterioles that results in
the shunting of blood to the medulla and an
RBF
increase in filtration fraction in the medulla
Peritubular capillary atrophy Tubulointerstitial fibrosis glomeruli. The tubulointerstituim in teh area
of glomerulosclerosis develop fibrosis, tubu-
lar atrophy with tubular casts, and inflamma-
tory cell infiltrates, and increase of peritubular
Decrease diluting and concentrating capacity
capillary atrophy.

2 Geriatric Nephrology Curriculum American Society of Nephrology


eration of efferent and afferent arterioles is associated with acute kidney injury in a low perfusion state because of attenu-
glomerular sclerosis, whereas in the second case, a continuous ated responses to vasodilators and an increase in response to
channel between the afferent and efferent arterioles results in a vasoconstrictors.5,22,39
sclerotic glomeruli, is called an “aglomerulus “ arteriole, and The “functional reserve “of the kidney is defined as the
shunts the blood to the medullary area.19,23,24 The small arter- acute rise in GFR that occurs after an infusion of amino ac-
ies show some elastic duplication, fibrous intimal thickening, ids.13,41 A lack of a rise in GFR with infusion of amino acid in
destructive changes in the media, narrowing of the lumen, and elderly persons with underlying renal disease may indicate that
lamination.24,29 The blood vessel changes play a major role in the kidney is working at maximal capacity and unable to re-
renal damage, compromising renal blood flow with subse- cruit additional nephrons in response to the increase in the
quent loss of renal mass.9,30 filtered load. However, the ability of the kidney to compensate
under stress may be limited in elderly persons. Fliser et al.13
Tubules found the functional reserve in healthy older volunteers to be
With aging, the length of the proximal convoluting tubule, the size around 15%, and this functional reserve was maintained until
of the proximal tubular epithelial cell, and the size of its respective age 80 in both men and women. This rise in the functional
nucleus decrease in parallel with the decrease in size of the glomer- reserve was not accompanied with a rise in effective renal blood
ulus.12,15,23,33 Electron micrographs of rat tubules showed non- flow (ERBF) or a significant decrease in renal vascular resis-
uniform thickening of the tubular basement membrane with tance. This suggests that the increase in the renal reserve is not
vacuoles in the proximal tubules, with intermittent loss of the related to vasodilatation in elderly persons as was commonly
microvilli,28 whereas the distal tubules are dilated with diver- found in younger adults.
ticular formation.28 Similar changes in the elderly may account
for an increased incidence of urinary tract infections.8 Tubular function
Elderly persons are not able to dilute or concentrate their urine
as well as younger healthy adults. This may stem from a com-
FUNCTIONAL CHANGES bination of interstitial damage, end organ resistance, or a de-
crease in production of various hormones.42– 44 As a conse-
With aging, renal blood flow decreases in both human and quence, elderly persons are more prone to water disorders and
animal populations13,19,22,27 (Table 2). Fliser et al.13 observed a volume depletion than the general population. The dysnatre-
marked decrease of about 10% per decade in the effective renal mias are the most common electrolyte disorders recorded
perfusion in healthy elderly volunteers compared with among elderly persons admitted to the hospital and are asso-
younger adults, with renal perfusion decreasing from 647 ml/ ciated with high morbidity and mortality.45 In older female
min per 1.73 m2 in younger volunteers to 339 ml/min per 1.73 WAG/Rij rats, aquaporin (AQP)-2 and -3 are downregulated
m2 in elderly volunteers. The lower renal plasma blood flow compared with in 3-mo-old rats. This correlated with labora-
and the decrease in GFR contribute to the increase in the fil- tory findings between the two age groups. There was a decrease
tration fraction found in the elderly persons. The decrease in in the urine output in older rats compared with younger rats
renal blood flow may result from an imbalance and alterations (3.9 ⫾ 0.3 versus 12.8 ⫾ 0.8 ml in 24 h). Older rats also had a
in the responsiveness to vasoactive substances, i.e., acetylcho- lower urine osmolality compared with younger rats (1042 ver-
line,22,39 or decrease in production of certain peptides with sus 2511 ⫾ 54 mosmol/kg).44 Moreover, there was no change
aging.13,22,40 Hollenger et al.22 performed Xenon washout stud- in the expression of AQP-1 in the proximal convoluting tu-
ies to evaluate potential transplant donors in ages ranging from bules and descending loop of Henle’s or in AQP-4 in the baso-
17 to 76 yr old and found a significant decrease in renal perfu- lateral membrane in the collecting tubules.44 These finding are
sion with aging that was associated with a reduction in cortical consistent with a decreased ability to concentrate urine in the
flow rate and kidney mass. He noted that the vasodilator re- elderly with normal levels of circulating vasopressin. This
sponse to acetylcholine was blunted in the elderly but found no could result in decreased insertion of apical AQP-2 into the
difference in the vasoconstrictor response to angiotensin.22 In apical membrane and an inability to concentrate urine.
older rats, the vasodilator response to nitric oxide and the en- The levels of serum renin, renin activity, and aldosterone42,43,46
dothelial-derived hyperpolarizing factor pathways are attenu- are low in elderly persons, and their response in a hypovolemic
ated.39 This suggests the elderly may be more susceptible to state is also blunted.12,44,47 Similarly, Sprague-Dawley adult rats
were found to have a downregulation of intrarenal mRNA renin
Table 2. Functional changes in the aging kidney and a blunted release of the serum renin in response to hypo-
Decrease renal blood flow by 10%/yr after age 40 tension compared with the younger rats.46
GFR decrease by 0.87 ml/min per year Despite a mild decline in renal function, elderly persons are
Increase in RVR capable of secreting an acid load when placed on a 70-g protein
Decrease diluting capacity diet and maintain normal serum bicarbonate levels and an
Decrease concentrating capacity appropriate urine pH.48 Although the serum aldosterone level
Normal renal reserve is decreased in the elderly, healthy elderly volunteers and those

American Society of Nephrology Geriatric Nephrology Curriculum 3


with CKD are able to excrete potassium to maintain normal function.56 Knockout mice for the SMP-30 gene showed an
serum potassium.49 increase in mortality and increase deposition of lipofuscin in
the renal tubular epithelial cells, marked degeneration in the
mitochondria, podocyte fusion, and an increase in apoptosis.56
MECHANISM FOR AGING Klotho is considered the anti-aging gene and has shown
expression in the kidney. The klotho mouse model (KI/KI) for
The biologic mechanisms for aging are still unknown. Various aging was genetically made by transgene disruption of the
possible mechanisms for aging have been touted and included klotho gene locus.57 The klotho (kl/kl) mouse exhibits many of
recruitment of senescence genes, changes in hormones related the phenotypic features of aging including the following: short
to gender,27,28,50 replicative senescence,51 damage caused by an lifespan, growth retardation, infertility, osteoporosis, athero-
unrestricted diet,52 and changes in oxidative stress53 (Figure 2). sclerosis, obstructive pulmonary disease, renal sclerosis, and
The cell cycle regulator gene regulator, p16INK4a, a cyclin-de- atrophy of the skin.57 Histologic changes in the kl/kl rat showed
pendent kinase inhibitor, and a possible senescence gene can- fibrosis in the renal arteries, the interstitium, and the glomer-
didate are found in the kidney of rats, mice, and humans.54 In uli, as well as calcification within the cortex in the older mice.57
the aging human kidney, there is an increase in p16 INK4a The imbalance between the accumulation and degradation
mRNA expression in the cortex.54 In vitro experiments in the of extracellular matrix (ECM) may play a role in fibrosis. A
aging human kidney suggested other gene candidates. Increase homozygote TIMP-1 transgenic mice was constructed to study
expression of p16INK4a and p53 was found in the sclerotic glo- the effect of TIMP-1 on ICAM-1 and fibrosis in the aging kid-
meruli areas, whereas p16INK4a, p53, cyclooxygenase-1 (COX- ney.58 TIMP-1 is a tissue inhibitor of metalloproteinase
1), transforming growth factor (TGF)-␤, and heat shock pro- (MMP), which is known to increase the degradation of ECM
tein A5 (HSPA5) were found in the interstitium.54 and ICAM-1. In the aging rat, there was an upregulation of
Knockout mice or transgenic animals have been created to TIMP-1 correlating with upregulation of ICAM-1 and TGF ␤.
evaluate the correlation between certain proteins and aging. In Fibrosis seems to be promoted by the regulation of profibrotic
transgenic male rats, the antisense growth hormone (GH) proteins and inhibition of the breakdown of ECM.58
showed a suppression of the expression of GH/insulin-like Besides genetic programming, there seems to be a sex di-
growth factor-1 (IGF-1) activity.55 The suppression of this ac- morphism in the development of glomerulosclerosis in ani-
tivity prevented histologic changes normally seen in the aging mals, with the female gender being protected until meno-
rat kidney. Associated with the decrease activity of GH/IGF-1 pause.27,28,59 This relationship may not hold for humans. 17␤-
was a decrease in macrophage infiltrates, the extracellular ma- estradiol seems to have many protective functions including
trix, and collagen production. The decrease activity of GH/ inhibition of apoptosis in mesangial cells, an increase in the
IGF-1 seems to be renoprotective,55 and upregulation may expression of metalloproteinase, and a decrease in collagen
contribute to sclerosis found in certain disease states such as production, which would point to a beneficial effect on ag-
diabetes mellitus. Another candidate gene for senescence is the ing.28,59 Aging female Dahl salt-sensitive rats were placed on
SMP-30 gene, which seems to be important in anti-apoptotic 17␤-estradiol replacement therapy after undergoing ovariec-

Fas
HSP47
GH/IGF-1
TIMP-1
HSP47 +apoptosis
Androgens Klotho gene
pINK4a MMP
p53 Aging -sclerosis Estradiol
ROS Kidney Ace inhibitor
PAI-1 +sclerosis ET- inhibitor
COX-1
TGFβ -apoptosis

Restricted calories
SMP-30 gene
Klotho gene

Figure 2. Aging kidney. A schematic outline of the various modulators that may be responsible for damage or reno-protection of the
aging kidney. The restricted caloric intake has a reno-protective effect through modulating various proteins by suppressing GH/IGF-1
activity, Fas, and HSP47. The upregulation of MMP, downregulation of TIMP-1 and ICAM-1, and decrease in oxidative stress results in
a decrease in matrix dysregulation and inflammation. The SMP-30 and klotho genes are anti-apoptotic. The klotho gene seems to be
reno-protective by decreasing sclerosis. The inhibition of ET-1 and angiotensin II are known to decrease sclerosis. Areas of fibrosis are
found to have an increase in PA1–1, COX-1, TGF-␤, PINK4a, and p53. The hormones estradiol and androgen have opposite effects on
aging.35,40,50 –59

4 Geriatric Nephrology Curriculum American Society of Nephrology


tomy at 3 mo and showed less renal scarring at 12 mo than the • The common histologic changes include cortical glomerular sclerosis,
loss of afferent and efferent arterioles in the cortex, and shunting of the
unsupplemented ovariectomy female rats.59 This suggests that renal blood flow to the medulla
estradiol has a protective role in sclerosis.59 Other studies sug- • Functional reserve and electrolyte balance are maintained under nor-
gest a role of androgens mediating sclerosis.27 mal condition in the elderly population
The role of endothelin (ET) in aging was explored by treat- • Biology of aging is not well known, but there may be an interaction
ing male Wistar rats at 2 and 23 mo after treatment for 28 d between senescence genes, hormones, and diet

with a nonpeptide endothelin A receptor antagonist.34 There


was a 50% decrease in glomerulosclerosis in the treated group
at 23 mo compared with the control group, as well as a signif- DISCLOSURES
icant decrease in proteinuria in the treated group compared None.
with the control group (51 versus 307 mg/kg; P ⬍ 0.0102).34
The telomeres are DNA repeat sequences of TTAAGGG ⫻
repeats that are attached to the somatic chromosome. With REFERENCES
replication, certain base pairs are lost. There are a finite num-
ber of replicate cycles that somatic cultured cells can undergo *Key References
and then replication ceases. This is termed replicative senes- 1. Spencer G: Projections of the Population of the United States by Age,
Sex and Race: 1988 to 2080. Current Population Reports. Washington,
cence (Hayflick limit).51 In human kidneys, the cortical telo-
DC, Government Printing Office, 1989, 1–17
mere length decreases with aging at a loss of 0.029 kbp/yr, 2. Levey AS, Bosch JP, Lewis JB, Greene T, Rogers N, Roth D: A more
which is greater than seen in the medulla.51 The significance of accurate method to estimate glomerular filtration rate from serum
this needs to be further evaluated. creatinine. Ann Intern Med 130: 461– 470, 1999
The biology of aging is an important area with many unan- 3. US Renal Data System: Excerpts from 2000 U.S. Renal Data system
annual data report: atlas to end stage renal disease in the United
swered questions. The understanding of the variability of the
States. Am J Kidney Dis 36: S1–S240, 2000
aging process among humans and animals may improve our 4. Baggio B, Budakovic A, Perissinotto E, Maggi S, Cantaro S, Enzi G,
fundamental knowledge of disease mechanisms and the possi- Grigoletto F, ILSA Working Group: Atherosclerotic risk factors and
bility of preventing the progression of renal disease. Presently, renal function in the elderly: the role of hyperfibrinogenaemia and
it seems that BP control, decrease in caloric intake, and an smoking. Results from the Italian Longitudinal Study on Ageing (ILSA).
Nephrol Dial Transplant 20: 114 –123, 2005
angiotensin-converting enzyme inhibitor may delay the pro-
5. Epstein M: Aging and the kidney. J Am Soc Nephrol 7: 1106 –1122,
gression of aging in the kidney. 1996
6. Hemmelgarn BR, Zhang J, Manns BJ, Tonelli M, Larsen E, Ghali WA,
Southern DA, McLaughlin K, Mortis G, Culleton BF: Progression of
kidney dysfunction in the community-dwelling elderly. Kidney Int 69:
CONCLUSION 2155–2161, 2006
7. Lindeman RD, Tobin JD, Shock NW: Association between blood pres-
The elderly and the very elderly population is the fastest grow- sure and the rate of decline in the renal function with age. Kidney Int
ing in the United States and accounts for a large percentage of 26: 861– 868, 1984
8. Lindeman RD, Tobin J, Shock NW: Longitudinal studies on the rate of
those with CKD. Cross-sectional and longitudinal studies have
decline in renal function with age. J Am Geriatr Soc 33: 278 –285, 1985
shown a decrease in renal function with aging beginning at age 9. Lindeman RD: Is the decline in renal function with normal aging
40, with the exception of a small population showing no de- inevitable? Geriatr Nephrol Urol 8: 7–9, 1998
cline with age. The comorbid problems accompanying the el- 10. Razzaque MS: Does renal aging affect survival? Ageing Res Rev 6:
derly population make it difficult to decipher the true course of 211–222, 2007*
11. Danziger RS, Tobin JD, Becker LC, Lakatta EE, Fleg JL: The age
aging within the kidney. Even so, there are some common his-
associated decline in glomerular filtration in healthy normotensive
tologic findings and functional changes in the kidney with ag- volunteers lack of relationship no cardiovascular performance. J Am
ing. The biologic mechanism for the changing with age are not Geriatr Soc 38: 1127–1132, 1990
well known, but recent identification of senescence genes, the 12. Fehrman- Ekholm I, Skeppholm L: Renal function in the elderly(⬎70
role of hormones, and diet may improve our understanding year old) measured by means of iohexol clearance, serum creatinine,
serum urea and estimated clearance. Scand J Urol Nephrol 38: 73–77,
and slow the decline in kidney function.
2004
13. Fliser D, Zeler M, Nowack R, Ritz E: Renal functional reserve in healthy
elderly subjects. Am J Soc Nephrol 3: 1371–1377, 1993
14. Hoy WE, Douglas-Denton RN, Hughson MD, Cass A, Johnson K,
TAKE HOME POINTS
Bertram JF: A stereological study of the glomerular number and
• The elderly and the elderly population is the fasting growing age group volume: preliminary findings in a multiracial study of kidneys at au-
in the United States, and they encompass the largest group with CKD topsy. Kidney Int 63: S31–S37, 2003
• The decrease in GFR begins at age 40 and is around 0.87 min/ml per 15. Spitzer A, Brandis M: Functional and morphological maturation of the
year superficial nephrons. J Clin Invest 53: 279 –287, 1974
• Gross changes with age include 30% loss in size of the kidney by the 16. Mañalich R, Reyes L, Herrera M, Melendi C, Fundora I: Relationship
eighth decade and decrease in the renal mass to ⬍300 g by the ninth between weight and the number and size of renal glomeruli in hu-
decade mans: a histomorphometric study. Kidney Int 58: 770 –773, 2000

American Society of Nephrology Geriatric Nephrology Curriculum 5


17. Souster LP, Emery JL: The sizes of renal glomeruli in fetuses and Messeri G, Masotti G: Excessive vasoconstriction after stress by the
infants. J Anatom 130: 595– 602, 1980 aging kidney: inadequate prostaglandin modulation of increased en-
18. Mudler WJ, Hillen HFP: Renal function and renal disease in the elderly: dothelin activity. J Lab Clin Med 132: 186 –194, 1998
part I. Eur J Intern Med 12: 86 –97, 2001* 41. Bosch JP, Saccaggi A, Lauer A, Ronco C, Belledonne M, Glabman S:
19. Anderson S, Brenner BM: Effect of aging on the renal glomerulus. Renal functional reserve in humans— effect of protein intake on glo-
Am J Med 80: 436 – 442, 1986 merular filtration rate. Am J Med 75: 943–950, 1983
20. Emamian SA, Nielsen MB, Pedersen JF, Ytte L: Kidney dimensions at 42. Corman B, Barrault MB, Klinger C, Houot AM, Michel JB, Della Bruna
sonography: correlation with age, sex, and habitus in 655 adult vol- R, Pinet F, Soubrier F: Renin gene expression n the aging kidney:
unteers. AJR 160: 83– 86, 1993 effect of sodium restriction. Mechan Aging Dev 84: 1–13, 1995
21. Gourtsoyiannis N, Prassopoulos P, Cavouras D, Pantelidis N: The 43. Weidmann P, De Myttenaere-Bursztein S, Maxwell MH, de Lima J:
thickness of the renal parenchyma decreases with age: a CT study of Effect of aging on plasma renin and aldosterone in normal man.
360 patients. AJR 155: 541–544, 1990 Kidney Int 8: 325–333, 1975
22. Hollenberg NK, Adams DF, Solomon HS, Rashid A, Abrams HL, Merrill 44. Preisser L, Teillet L, Aliotti S, Gobin R, Berthonaud V, Chevalier J,
JP: Senescence in the renal vasculature in normal man. Circ Res 34: Corman B, Verbavatz JM: Downregulation of aquaporin-2 and -3 in
309 –316, 1974 aging kidney is independent of V(2) vasopressin receptor. Am J
23. Lamb EJ, O’Riordan SE, Delaney MP: Kidney function in older people: Physiol Renal Physiol 279: F144 –F152, 2000
pathology, assessment, and management. Clin Chim Acta 334: 24 – 45. Palevsky PM, Bhagrath R, Greenberg A: Hypernatremia in hospitalized
40, 2003* patients. Ann Intern Med 124: 197–203, 1996
24. Takazakura E, Sawabu N, Handa A, Takada A, Shinoda A, Takeuchi J: 46. Jung FF, Kennefick TM, Ingelfinger JR, Vora JP, Anderson S: Down-
Intrarenal vascular changes with age and disease. Kidney Int 2: 224 – regulation of the intrarenal renin-angiotensin system in the aging rat.
230, 1972 J Am Soc Nephrol 5: 1573–1580, 1995
25. Terry S, Hoy WE, Douglas-Denton R, et al.: Determinants of glomer- 47. Luckey AE, Parsa CJ: Fluid and electrolytes in the aged. Arch Surg
ular volume in different cortical zones of the human kidney. J Am Soc 138: 1055–1060, 2003
Nephrol 16: 3102–3109, 2005 48. Wagner EA, Falciglia GA, Amlal H, Levin L, Soleimani M: Short-term
26. Thomas SE, Anderson S, Gordon KL, Oyama TT, Shankland SJ, John- exposure to a high-protein diet differentially affects glomerular filtra-
son RJ: Tubulointerstitial disease in aging: evidence for underlying tion rate but not acid-base balance in older compared to younger
peritubular capillary damage, a potential role for renal ischemia. J Am adults. J Am Dietetic Assoc 107: 1404 –1408, 2007
Soc Nephrol 9: 231–242, 1998 49. Musso CG, Miguel R, Algranati L, Farias Edos R: Renal potassium
27. Baylis C, Corman B: The aging kidney: insights from experimental excretion: comparison between chronic renal disease patients and old
studies. J Am Soc Nephrol 9: 699 –709, 1998* people. Inter Urol Nephrol 37: 167–170, 2005
28. Zheng F, Plati AR, Potier M, Schulman Y, Berho M, Banerjee A, 50. Baylis C: Change in renal hemodynamics and structure in the aging
Leclercq B, Zisman A, Striker LJ, Striker GE: Resistance to glomerulo- kidney; sexual dimorphism and the nitric oxide system. Exper Geron-
sclerosis in B6 mice disappears after menopause. Am J Pathol 162: tol 40: 271–278, 2005
1339 –1348, 2003 51. Melk A, Ramassar V, Helms LM, Moore R, Rayner D, Solez K, Halloran
29. Bolton WK, Sturgill BC: Spontaneous glomerular sclerosis in aging- PF: Telomere shortening in kidneys with age. J Am Soc Nephrol 11:
Sprague-Dawley rats. Am J Pathol 98: 339 –350, 1980 444 – 453, 2000
30. Kaplan C, Pasternack B, Shah H, Gallo G: Age-related incidence of 52. Razzaque MS, Shimokawa I, Koji T, Higami Y, Taguchi T: Life-long
sclerotic in human kidneys. Am J Pathol 80: 227–234, 1975 caloric restriction suppresses age-associated Fas expression in the
31. Kasiske BL: Relationship between vascular disease and age-associated Fischer 344 rat kidney. Mol Cell Biol Res Commun 1: 82– 85, 1999
changes in the human kidney. Kidney Int 31: 1153–1159, 1987 53. Melk A: Senescence of renal cells: molecular basis and clinical impli-
32. Newbold KM, Sandison A, Howie AJ: Comparison of size of juxtamed- cations. Nephrol Dialysis Transplant 18: 2474 –2478, 2003
ullary and outer cortical glomeruli in normal adult kidney. Virchows 54. Melk A, Schmidt BM, Takeuchi O, Sawitzki B, Rayner DC, Halloran PF:
Arch Pathol Anat 420: 127–129, 1992 Expression of pk16INK4a and other cell cycle regulator and senes-
33. Sato T, Tauchi H: Age changes of mitochondria of rat kidney. Mech cence associated genes in aging human kidney. Kidney Int 65: 510 –
Aging Dev 20: 111–126, 1982 520, 2004
34. Ortmann J, Amann K, Brandes RP, Kretzler M, Münter K, Parekh N, 55. Zha Zha Y, Le VT, Higami Y, Shimokawa I, Taguchi T, Razzaque MS:
Traupe T, Lange M, Lattmann T, Barton M: Role of podocytes for Life-long suppression of growth hormone-insulin-like growth factor I
reversal of glomerulosclerosis and proteinuria in the aging kidney after activity in genetically altered rats could prevent age-related renal
endothelin inhibition. Hypertension 44: 974 –981, 2004 damage. Endocrinology 147: 5690 –5698, 2006
35. Abrass CK, Adcox MJ, Raugi GJ: Aging associated changes in renal 56. Maruyama N, Ishigami A, Kuramoto M, Handa S, Kubo S, Imasawa T,
extracellular matrix. Am J Pathol 146: 742–752, 1995 Seyama K, Shimosawa T, Kasahara Y: Senescence marker protein-30
36. Langeveld JP, Veerkamp JH, Trijbels JM, Duyf CM, Monnens LH: knockout mouse as an aging model. Ann NY Acad Sci 1019: 383–387,
Chemical composition and solubility of human glomerular and tubular 2004
basement membranes of adult and senescent men. Int J Biochem 16: 57. Takeshita K, Yamamoto K, Ito M, Kondo T, Matsushita T, Hirai M,
1255–1264, 1984 Kojima T, Nishimura M, Nabeshima Y, Loskutoff DJ, Saito H, Murohara
37. Taylor SA, Price RT: Age-related changes in rat glomerular basement T: Increased expression of plasminogen activator inhibitior-1 with
membrane. J Biochem 14: 201–206, 1982 fibrin deposition in a urine model of aging, “Klotho” mouse. Sem
38. Inserra F, Romano LA, de Cavanagh EM, Ercole L, Ferder LF, Gomez Thromb Haemost 2: 545–553, 2002
RA: Renal interstitial sclerosis in aging: effects of enalapril and nifed- 58. Zhang X, Chen X, Hong Q, Lin H, Zhu H, Liu Q, Wang J, Xie Y, Shang
ipine. J Am Soc Nephrol 7: 676 – 680, 1996 X, Shi S, Lu Y, Yin Z: TIMP-1 promotes age-related renal fibrosis
39. Long DA, Newaz MA, Prabhakar SS, Price KL, Truong LD, Feng L, Mu through upregulating ICAM-1 in human TIMP-1 transgenic mice. J
W, Oyekan AO, Johnson RJ: Loss of nitric oxide and endothelial- Gerontol 61: 1130 –1143, 2006
derived hyperpolarized factor-mediated responses in age. Kidney Int 59. Maric C, Sandberg K, Hinojosa-Laborde C: Glomerulosclerosis and
68: 2154 –2163, 2005 tubulointerstitial fibrosis are attenuated with 17beta-estradiol in
40. Castellani S, Ungar A, Cantini C, La Cava G, Di Serio C, Altobelli A, the aging Dahl salt sensitive rat. J Am Soc Nephrol 15: 1546 –1556,
Vallotti B, Pellegri M, Brocchi A, Camaiti A, Coppo M, Meldolesi U, 2004

6 Geriatric Nephrology Curriculum American Society of Nephrology


REVIEW QUESTIONS: KIDNEY SENESCENCE a. Glomerulosclerosis in the medulla more than the cortex
b. The arterioles in the cortex become sclerosed leading to
1. A 68-yr-old Caucasian male has been quite concerned after “aglomeruli”
reading in his local newspaper about the increased incidence of c. There is shunting of blood to the medullary region second-
chronic kidney diseases in the elderly population. He was ary arteriolar structural changes
never told by his primary care physician he had any kidney d. There is thickening of the glomerular membrane and
problems. He made an appointment to see a nephrologist to change in composition with aging
discuss his possible kidney disease. He presents at the clinic,
3. Community-dwelling elderly persons are able to maintain
and his BP is 125/70 mmHG, and there are no pertinent find-
electrolyte balance secondary to adequate residual renal re-
ings on his physical exam. He does not take any prescribed
serve
medications and only vitamins. Laboratory values show a se-
a. True
rum creatinine of 1.2 mg/dl. Which the following is true?
b. False
a. The rate of decline in his renal function is normal for his
age group 4. The biologic causes of aging are unknown; however, there are
b. He will be on renal replacement therapy by the time he certain medical interventions that may slow down renal loss in
reaches 90 yr old elderly person with normal renal function. The correct answer
c. Elevated BP has no effect on his progression is which of the following?
d. The aging process affects all organs and all elderly people a. Controlled hypertension
have progression of their renal function b. High caloric intake
c. Low protein diet
2. The histologic changes in the kidney with age are the following d. Testesterone
except

American Society of Nephrology Geriatric Nephrology Curriculum 7


Chapter 5: Rate of Decline in eGFR and Clinical
Evaluation of the Elderly With a Low eGFR
Rosemarie L. Shim* and Ann M. O’Hare†
*Department of Medicine, Ohio State University, Columbus, Ohio; and †Department of Medicine, University of
Washington, and VA Puget Sound Healthcare System, Seattle, Washington

AGE AND RATE OF DECLINE OF RENAL change in eGFR seems to be somewhat complex and
FUNCTION perhaps dependent on baseline level of eGFR.
Among a national cohort of veterans with an eGFR
In cross-sectional studies, levels of renal function
⬍60 ml/min per 1.73 m2, eGFR declined more rapidly
are on average lower in older compared with
for older than for younger patients at higher levels of
younger participants.1–3 However, the extent to
eGFR (i.e., ⱖ45 ml/min per 1.73 m2). However, the
which this phenomenon results from an age-asso-
opposite was true at lower levels of eGFR (i.e., ⬍45
ciated decline in renal function versus a higher prev-
ml/min per 1.73 m2), where eGFR declined more
alence of comorbidities linked to chronic kidney
slowly in older than in younger patients.9 Collectively,
disease (CKD) in the elderly is uncertain. Relatively
these data seem to suggest that, although older pa-
few studies have explicitly examined rates of decline
tients are more likely to develop CKD, those who sur-
in renal function across age groups. Most of what
vive long enough to reach more advanced stages of
we know about longitudinal changes in renal func-
CKD are actually less likely than their younger coun-
tion comes from the Baltimore Longitudinal Study
terparts to experience progressive loss of eGFR.
of Aging (BLSA).4 – 6 A subset of participants in this
study underwent serial creatinine clearance mea-
surements over time. Observations on these pa-
AGE AND RISK OF PROGRESSION TO END-
tients have provided some important insights into
STAGE RENAL DISEASE
the effect of age on change in level of renal function.
First, even in individuals without known comorbid
Studies of rate of change in measured or estimated
conditions and without intrinsic renal disease or
renal function can be difficult to interpret for a va-
proteinuria, level of creatinine clearance declined
riety of reasons: (1) progression may not occur in a
on average by 0.75 ml/min per year.4 Second, renal
predictable and linear fashion; (2) the clinical sig-
function was stable and even improved in some
nificance of changes renal function, particularly
subjects.4 Hemmelgarn et al.7 reported a similar
within the normal range, is uncertain; and (3) it can
phenomenon among community-dwelling elderly
be difficult to account for differences in survival
in Canada followed over a 2-yr period (Figure 1).
and follow-up among participants. Thus, results of
Thus, these reports suggest that, on average, renal
studies reporting change in level of renal function as
function declines with increasing age even in the
an outcome are probably quite sensitive to the an-
absence of comorbidity. At the same time, decline
alytic approach selected. Progression to end-stage
in renal function does not seem to be an inevitable
kidney disease (ESKD) often represents a more
consequence of aging.
meaningful clinical outcome than change in level of
Among participants in the BLSA without CKD,
renal function. This outcome is easily defined and
the rate at which creatinine clearance declined over
identified, and the clinical significance of ESKD (de-
time was greater among older participants.6 Consis-
tent with these results and with prior cross-sec-
tional studies showing lower levels of renal function Correspondence: Ann M. O’Hare, MA, MD, Division of Nephrol-
ogy, VA/Puget Sound Medical Center, Nephrology and Renal
among older people, older age seems to be a risk Dialysis Unit, Building 100, Room 5B113, 1660 S. Columbian Way,
factor for the development of CKD, defined as an Seattle, WA 98108. Phone: 206-277-3192; Fax: 206-764-2022;
estimated GFR (eGFR) ⬍60 ml/min per 1.73 m2.8 E-mail: [email protected]

However, the relationship between age and rate of Copyright 䊚 2009 by the American Society of Nephrology

American Society of Nephrology Geriatric Nephrology Curriculum 1


ysis are nevertheless elderly.9 In fact, patients 75 yr and older
currently represent one of the fastest growing contingents of
the ESKD population, most likely reflecting both population
aging and the high overall prevalence of CKD in the elderly. 12
Thus, a critical challenge for health systems and providers car-
ing for older patients with CKD lies in identifying the relatively
small proportion, but large absolute number, of older patients
with CKD who are at greatest risk for progressive loss of renal
function and ultimate need for dialysis.
In general, relatively little is known about what predicts
more rapid loss of renal function in elderly patients with CKD
and whether risk factors are similar in older and younger pa-
Figure 1. Change in eGFR over a 2-yr period among members of tients. Male gender is a strong risk factor for progression of
an elderly community cohort. (Source: Hemmelgarn BR, et al.: kidney disease in the general population, and this may also be
Kidney Int. 69: 2155–2161, 2006.) This material is copyrighted by the case in the elderly. For example, among an elderly Cana-
the ISN. dian cohort, rate of progression was greater in men than in
women.7 African-American race is a strong risk factor for pro-
fined as treatment with chronic dialysis or transplant) is beyond gression of renal disease in the general population. However, a
dispute. Rates of progression to ESKD among patients with dif- study of patients with incident ESKD suggested that the risk of
ferent levels of eGFR vary substantially with age.9,10 Although ESKD related to African-American race may be greatest in
eGFR is an excellent predictor of who will progress to ESKD middle age.13 To date, the effect of age on the relationship
among patients of all ages, there are large differences in the abso- between race and progression has not been studied prospec-
lute risk of ESKD among patients of different ages with similar tively. Similarly, although hypertension is a prototypical risk
levels of eGFR, with older patients being less likely to progress to factor for progression of renal disease in the overall popula-
ESKD at any given level of eGFR.9 –11 This phenomenon likely tion, the association of hypertension with progression of renal
reflects a number of different factors including a greater compet- disease may be attenuated at older ages.14 Diabetes seems to be
ing risk of mortality in older patients, slower rates of progression a risk factor for progression of CKD in older patients as it is in
among older patients with advanced CKD, and age differences in younger patients, although it is unclear whether the strength of
acceptance or receipt of chronic dialysis when indications arise. this association is the same in patients of different ages.7
Mortality rates among elderly people with CKD are much Level of proteinuria shows some promise as a marker for
higher than among their younger counterparts.2,3 Consequently, clinically significant outcomes in elderly patients with CKD.
many older patients with CKD will not survive long enough to The presence of microalbuminuria and macroalbuminuria has
progress to the point where they need dialysis. Furthermore, as been shown to be associated with mortality in a variety of dif-
described above, CKD is often either slowly progressive or non- ferent populations, including patients with and without diabe-
progressive in the elderly. Thus, at any given level of renal func-
tion, there are large differences between younger and older pa-
tients in the most common clinical outcomes. For example,
among younger members of a national cohort of VA patients,
progression to ESKD was a more likely outcome than death
among those with relatively high levels of eGFR (e.g., ⬍45 ml/min
per 1.73 m2 for those 18 to 44 yr of age). However, among older
patients, death was a more likely outcome than ESKD even among
those with very low levels of eGFR (e.g., ⬍15 ml/min per 1.73 m2
for those 65 to 84 yr of age) (Figure 2). Patients 85 yr and older
were more likely to die than to reach ESKD at all levels of eGFR.
Thus, among the vast majority of older persons with CKD, even
when this is quite advanced (i.e., eGFR 15 to 29 ml/min per 1.73 m2),
death is a more common outcome than progression to ESKD.

RISK FACTORS FOR PROGRESSION IN THE ELDERLY


Figure 2. Relationship between age, eGFR, and risk of death in
Although older patients are less likely to progress to ESKD than relation to risk of ESKD. (Source: O’Hare AM, et al.: J Am Soc
their younger counterparts with similar levels of renal func- Nephrol 18: 2758 –2765, 2007.) Copyright 䊚2007 American So-
tion, most patients who progress to the point of needing dial- ciety of Nephrology.

2 American Society of Nephrology American Society of Nephrology


tes.15–25 Several recent studies have shown an additive risk of be very important in shaping future management decisions.
death among those with microalbuminuria and a low However, making this distinction represents one of the major
eGFR.26 –29 One of these studies specifically confirmed the challenges to clinicians caring for older patients with CKD. As
presence of this association in the elderly.26 Hallan et al.26 de- discussed earlier, although traditional risk factors such as dia-
scribed the relationship between eGFR and urinary albumin- betes, hypertension, and black race are potentially important
to-creatinine ratio among 9709 participants in the Hunt II predictors of progression in the elderly as they are in younger
study, a large-scale Norwegian general health survey. Among patients with CKD, few studies have examined the importance
patients with a similar level of eGFR, mortality risk increased of these risk factors in the elderly. While not specifically stud-
with increasing level of urinary albumin excretion. However, ied, if prior records exist, it may simply be more helpful in
mortality risk increased with falling eGFR only in those with many instances to review each patient’s prior trajectory of re-
microalbuminuria and not among those with lower levels of nal function to determine whether their CKD is rapidly pro-
urinary albumin excretion. Of note, these associations were gressive, slowly progressive, or nonprogressive.
present both among those younger and older than 70 yr. These Although primary renal disease processes can manifest at all
findings suggest that urinary albumin measurement may be ages (e.g., glomerulonephritis, polycystic kidney disease, lupus
helpful in identifying the subset of older persons with very nephritis), in many elderly people, CKD may function more as
moderate reductions in eGFR (e.g., 45 to 60 ml/min per 1.73 a marker for a variety of other comorbid conditions (e.g., ath-
m2) who are at greatest risk for mortality. To date, no informa- erosclerosis, diabetes, and hypertension). Furthermore, the
tion is available on the prognostic significance of urinary pro- presence of CKD in an older person may reflect the cumulative
tein excretion for progression to ESKD in the elderly, but nu- end result of a variety of different known and unknown pro-
merous studies in younger cohorts have identified proteinuria cesses that have occurred during the course of that patient’s
as a risk factor for progression of renal disease. lifetime. Thus, in many elderly patients, the future course of
Perhaps more valuable than knowing whether an older per- CKD may have more to do with that person’s overall health,
son with CKD has traditional risk factors for progressive renal level of comorbidity, and level of renal reserve than with pro-
disease may be an understanding of what the course of that gression of a single underlying renal disease process (as is more
person’s renal functional decline has been to date (Figure 1). likely to be the case in younger people with CKD). Related to
Regardless of the underlying etiology and presence or absence this, it may be more difficult to predict the future trajectory of
of known risk factors for progression, information on a pa- renal functional decline in older compared with younger pa-
tient’s past trajectory of renal function may help to predict how tients if this will primarily reflect events and disease processes
their renal function will change in the future, whether this is that impact the kidney only indirectly.
likely to occur in a predictable fashion, and under what cir- Although, in general, identifying the underlying etiology of
cumstances progression is most likely to occur (e.g., hospital- CKD is often viewed as a critical first step in patient assessment,
ization). Even if the course of an older patient’s prior trajectory in many older patients, it may not be possible to come up with
of renal function is found to be variable, making it difficult to a single etiology for their CKD. However, much depends on
accurately predict future trends, it may still be helpful for the the clinical presentation. In older as in younger patients, an
patient and provider to be aware of this uncertainty. aggressive diagnostic work up to uncover the underlying etiol-
ogy of CKD has the potential to provide important informa-
tion on prognosis and help guide management when a single
CLINICAL ASSESSMENT OF THE OLDER PATIENT dominant renal process is suspected. For example, if an older
WITH CKD patient experiences rapid loss of renal function in the setting of
heavy proteinuria and active urinary sediment, the results of a
Because heterogeneity increases with age, perhaps the only renal biopsy and serologic work up might provide critical in-
statement that can be made with any certainty about the assess- formation that could change management and alter prognosis.
ment and management of older patients with CKD is that a However, for many older patients with CKD, particularly in the
single approach is unlikely to be appropriate for all older pa- absence of proteinuria, active urinary sediment, and rapid pro-
tients. However, there may be some general differences in the gression, a single unifying diagnosis may not be possible. In these
ideal emphasis of the assessment of CKD in older and younger situations, rather than trying to come up with a definitive diagno-
patients related to general differences in the expected course of sis, it may be more helpful to view the diagnostic evaluation of
CKD at different ages. CKD as an opportunity to rule out potentially reversible processes
The cornerstone of the evaluation of all patients with CKD that are common at older ages (e.g., obstructive uropathy, renal
is assessment of risk for future adverse events (e.g., progression artery stenosis, medication-induced interstitial nephritis).
of renal disease, mortality, cardiovascular events, complica- Because the burden of comorbidity is so high in many older
tions of CKD). Because so many older patients with CKD do patients with CKD, it is also critically important in managing
not progress to the point of needing dialysis, assessment of older patients with a low eGFR to weigh the clinical signifi-
each patient’s risk for progressive disease and likelihood for cance of this finding in relation to other comorbid conditions
requiring dialysis in relation to the competing risk of death can that might be present in a given patient.7,9 If these other co-

American Society of Nephrology American Society of Nephrology 3


morbid conditions are more likely to impact overall health and 4. Lindeman RD, Tobin J, Shock NW: Longitudinal studies on the rate of
decline in renal function with age. J Am Geriatr Soc 33: 278 –285, 1985
quality of life, interventions to slow progression of CKD 5. Lindeman RD, Tobin JD, Shock NW: Association between blood pres-
should perhaps not be prioritized to the same extent as these sure and the rate of decline in renal function with age. Kidney Int 26:
other comorbid conditions. On the other hand, if the patient 861– 868, 1984
has few comorbid conditions and CKD is their primary clinical 6. Rowe JW, Andres R, Tobin JD, Norris AH, Shock NW: The effect of
problem, a stronger focus on efforts to diagnose and manage age on creatinine clearance in men: a cross-sectional and longitudinal
study. J Gerontol 31: 155–163, 1976
progressive CKD may be very appropriate, particularly if their 7. Hemmelgarn BR, Zhang J, Manns BJ, Tonelli M, Larsen E, Ghali WA,
CKD is clearly progressive or they have risk factors for progres- Southern DA, McLaughlin K, Mortis G, Culleton BF: Progression of
sion such as proteinuria. kidney dysfunction in the community-dwelling elderly. Kidney Int 69:
2155–2161, 2006*
8. Fox CS, Larson MG, Leip EP, Culleton B, Wilson PW, Levy D: Predic-
tors of new-onset kidney disease in a community-based population.
CONCLUSION JAMA 291: 844 – 850, 2004
9. O’Hare AM, Choi AI, Bertenthal D, Bacchetti P, Garg AX, Kaufman JS,
Among patients with similar levels of eGFR, clinical outcomes Walter LC, Mehta KM, Steinman MA, Allon M, McClellan WM, Land-
efeld CS: Age affects outcomes in chronic kidney disease. J Am Soc
vary substantially by age. In general, older patients are more Nephrol 18: 2758 –2765, 2007*
likely than their younger counterparts to have a low eGFR but 10. Eriksen BO, Ingebretsen OC: The progression of chronic kidney dis-
are less likely to experience progression to ESKD. At the same ease: a 10-year population-based study of the effects of gender and
time, older patients represent the largest and fastest growing age. Kidney Int 69: 375–382, 2006
contingent of the ESKD population. Therefore, the main chal- 11. Evans M, Fryzek JP, Elinder CG, Cohen SS, McLaughlin JK, Nyrén O,
Fored CM: The natural history of chronic renal failure: results from an
lenge in managing older patients with CKD is to identify the unselected, population-based, inception cohort in Sweden. Am J
small proportion but large number who are most likely to Kidney Dis 46: 863– 870, 2005
progress to ESKD and who may benefit the most from aggres- 12. Collins AJ, Foley R, Herzog C, Chavers B, Gilbertson D, Ishani A,
sive efforts to diagnose and treat their underlying renal disease. Kasiske B, Liu J, Mau LW, McBean M, Murray A, St Peter W, Xue J, Fan
Q, Guo H, Li Q, Li S, Li S, Peng Y, Qiu Y, Roberts T, Skeans M, Snyder
J, Solid C, Wang C, Weinhandl E, Zaun D, Zhang R, Arko C, Chen SC,
Dalleska F, Daniels F, Dunning S, Ebben J, Frazier E, Hanzlik C,
TAKE HOME POINTS Johnson R, Sheets D, Wang X, Forrest B, Constantini E, Everson S,
Eggers P, Agodoa L: Excerpts from the United States Renal Data
• A growing number and proportion of all patients initiating chronic System 2007 annual data report. Am J Kidney Dis 51: S1–S320, 2008
dialysis are 75 yr and older 13. Lopes AA, Hornbuckle K, James SA, Port FK: The joint effects of race
• Most older patients who meet criteria for CKD are much more likely to and age on the risk of end-stage renal disease attributed to hyper-
die before they reach ESKD; this is true even for older patients with tension. Am J Kidney Dis 24: 554 –560, 1994
severe reductions in eGFR 14. Klag MJ, Whelton PK, Randall BL, Neaton JD, Brancati FL, Ford CE,
• It is often difficult to know which subset of older patients with CKD will Shulman NB, Stamler J: Blood pressure and end-stage renal disease in
progress to ESKD men. New Engl J Med 334: 13–18, 1996
• Most older patients who meet the criteria for CKD have other health 15. Yuyun MF, Khaw KT, Luben R, Welch A, Bingham S, Day NE, Wareham
conditions NJ; European Prospective Investigation into Cancer in Norfolk (EPIC-
• The importance of interventions to slow progression of CKD should be Norfolk) population study: Microalbuminuria independently predicts
weighed against other, perhaps competing, health priorities all-cause and cardiovascular mortality in a British population: The
European Prospective Investigation into Cancer in Norfolk (EPIC-Nor-
folk) population study. Int J Epidemiol 33: 189 –198, 2004
16. Romundstad S, Holmen J, Hallan H, Kvenild K, Ellekjaer H: Microalbu-
DISCLOSURES minuria and all-cause mortality in treated hypertensive individuals:
None. does sex matter? The Nord-Trondelag Health Study (HUNT). Norway
Circulation 108: 2783–2789, 2003
17. Klausen K, Borch-Johnsen K, Feldt-Rasmussen B, Jensen G, Clausen
P, Scharling H, Appleyard M, Jensen JS: Very low levels of microalbu-
REFERENCES minuria are associated with increased risk of coronary heart disease
and death independently of renal function, hypertension, and diabe-
*Key References tes. Circulation 110: 32–35, 2004
1. Coresh J, Selvin E, Stevens LA, Manzi J, Kusek JW, Eggers P, Van 18. Jager A, Kostense PJ, Ruhé HG, Heine RJ, Nijpels G, Dekker JM,
Lente F, Levey AS: Prevalence of chronic kidney disease in the United Bouter LM, Stehouwer CD: Microalbuminuria and peripheral arterial
States. JAMA 298: 2038 –2047, 2007* disease are independent predictors of cardiovascular and all-cause
2. O’Hare AM, Bertenthal D, Covinsky KE, Landefeld CS, Sen S, Mehta K, mortality, especially among hypertensive subjects: five-year follow-up
Steinman MA, Borzecki A, Walter LC: Mortality risk stratification in of the Hoorn Study. Arterioscler Thromb Vasc Biol 19: 617– 624, 1999
chronic kidney disease: one size for all ages? J Am Soc Nephrol 17: 19. Hillege HL, Janssen WM, Bak AA, Diercks GF, Grobbee DE, Crijns HJ,
846 – 853, 2006* Van Gilst WH, De Zeeuw D, De Jong PE; Prevend Study Group:
3. Raymond NT, Zehnder D, Smith SC, Stinson JA, Lehnert H, Higgins Microalbuminuria is common, also in a nondiabetic, nonhypertensive
RM: Elevated relative mortality risk with mild-to-moderate chronic population, and an independent indicator of cardiovascular risk fac-
kidney disease decreases with age. Nephrol Dial Transplant 22: 3214 – tors and cardiovascular morbidity. J Intern Med 249: 519 –526, 2001
3220, 2007* 20. Gerstein HC, Mann JF, Yi Q, Zinman B, Dinneen SF, Hoogwerf B, Hallé

4 American Society of Nephrology American Society of Nephrology


JP, Young J, Rashkow A, Joyce C, Nawaz S, Yusuf S; HOPE Study microalbuminuria in predicting cardiovascular mortality in treated hy-
Investigators: Albuminuria and risk of cardiovascular events, death, pertensive men with and without diabetes mellitus. Risk Factor Inter-
and heart failure in diabetic and nondiabetic individuals. JAMA 286: vention Study Group. Am J Cardiol 80: 164 –169, 1997
421– 426, 2001 26. Hallan S, Astor B, Romundstad S, Aasarod K, Kvenild K, Coresh J:
21. Arnlöv J, Evans JC, Meigs JB, Wang TJ, Fox CS, Levy D, Benjamin EJ, Association of kidney function and albuminuria with cardiovascular
D’Agostino RB, Vasan RS: Low-grade albuminuria and incidence of mortality in older vs younger individuals: the HUNT II study. Arch
cardiovascular disease events in nonhypertensive and nondiabetic Intern Med 167: 2490 –2496, 2007
individuals: the Framingham Heart Study. Circulation 112: 969 –975, 27. Foster MC, Hwang SJ, Larson MG, Parikh NI, Meigs JB, Vasan RS,
2005 Wang TJ, Levy D, Fox CS: Cross-classification of microalbuminuria and
22. Ljungman S, Wikstrand J, Hartford M, Berglund G: Urinary albumin reduced glomerular filtration rate: associations between cardiovascu-
excretion—a predictor of risk of cardiovascular disease. A prospective lar disease risk factors and clinical outcomes. Arch Intern Med 167:
10-year follow-up of middle-aged nondiabetic normal and hyperten- 1386 –1392, 2007
sive men. Am J Hypertens 9: 770 –778, 1996 28. Astor BC, Hallan SI, Miller ER III Yeung E, Coresh J: Glomerular
23. Dinneen SF, Gerstein HC: The association of microalbuminuria and filtration rate, albuminuria, and risk of cardiovascular and all-cause
mortality in non-insulin-dependent diabetes mellitus. A systematic mortality in the US population. Am J Epidemiol 167: 1226 –1234,
overview of the literature. Arch Intern Med 157: 1413–1418, 1997 2008
24. Damsgaard EM, Froland A, Jorgensen OD, Mogensen CE: Microalbu- 29. Brantsma AH, Bakker SJ, Hillege HL, de Zeeuw D, de Jong PE,
minuria as predictor of increased mortality in elderly people. BMJ 300: Gansevoort RT: Cardiovascular and renal outcome in subjects with
297–300, 1990 K/DOQI stage 1–3 chronic kidney disease: the importance of urinary
25. Agewall S, Wikstrand J, Ljungman S, Fagerberg B: Usefulness of albumin excretion. Nephrol Dial Transplant 23: 3851–3858, 2008

American Society of Nephrology American Society of Nephrology 5


REVIEW QUESTIONS: RATE OF DECLINE IN EGFR c. See her back monthly for surveillance for the complications
AND CLINICAL EVALUATION OF THE ELDERLY of chronic kidney disease
WITH A LOW EGFR d. Avoid potassium-containing foods
1. An 86-yr-old woman in good health with no comorbidities is
3. An 85-yr-old man has an eGFR of 25 ml/min per 1.73 m2, an
found to have an eGFR of 54 ml/min per 1.73 m2 on routine
albumin to creatinine ratio of 9000 mg/g, hematuria with dys-
testing. Three months later, her eGFR is 55 ml/min per 1.73
morphic red blood cells, normal complements, negative
m2. Urine testing shows an albumin to creatinine ratio of 12
ANCA, anti-GBM, hepatitis serologies, UPEP and SPEP, and
mg/g, and her urinalysis is bland. Her primary care physician
an ANA of 1:80. In the last year, his eGFR has gone from 65 to
asks you whether a renal consultation is needed. Which of the
25 ml/min per 1.73 m2. In the past and as recently as 1 yr ago,
following do you think is the most appropriate response to his
his urinalyses showed no proteinuria on dipstick. He is feeling
or her question?
very fatigued and listless compared with usual and has not
a. At this point, probably little to gain from renal consulta-
been able to do his usual daily 3-mile walk. Which do you
tion; however, would make sure all her medications are
think is the most appropriate next step?
appropriately dosed and reasonable to follow serum creat-
a. Refer the patient for a renal biopsy to identify the underly-
inine to make sure her eGFR does not fall markedly
ing cause of his renal disease
b. Would recommend seeing the patient for thorough review
b. Start the patient on an ACE inhibitor
of possible causes for her chronic kidney disease and appro-
c. Discuss dialysis modality and send the patient for vascular
priate management
access placement if they choose hemodialysis
c. Would recommend checking for complications of chronic
d. Discuss end of life issues and hospice placement given the
kidney disease and seeing the patient at least annually in
patient’s poor prognosis
nephrology for management of any complications of
chronic kidney disease she might develop
4. A new patient comes to see you. She is a 78-yr-old nondiabetic
d. Would recommend referral to nephrology with consider-
woman with an eGFR of 30 ml/min per 1.73 m2 that has de-
ation for renal biopsy to identify the underlying cause of
creased from 40 ml/min per 1.73 m2 in the last year. She has an
this patient’s chronic kidney disease
albumin to creatinine ratio of 1000 mg/g. She is on amlodop-
2. A 72-yr-old woman has an eGFR of 25 ml/min per 1.73 m2 that ine for hypertension. Her BP today is 170/80 mmHg. She
has been stable for 5 yr. Her albumin to creatinine ratio is 21 comes into your office quite upset because she got lost on her
mg/g and her urinalysis is bland. Her hematocrit is 35, her way to clinic and actually forgot where she was. This is the first
calcium is 9 mEq/L, phosphorus is 4 mEq/L, and parathyroid time this has ever happened to her. Her vital signs are other-
hormone is 120 mEq/L. Her serum potassium is 5.0 mEq/L. wise stable and her blood sugar is 110 mg/dl. What should be
Every day she takes Lisinopril 40 mg, amlodopine 10 mg, lasix the first priority for her care?
40 mg, and a multivitamin. In the past, she used nonsteroidal a. Review her urinalysis to determine whether she needs a
agents heavily but discontinued these about 5 yr ago when she biopsy
found out about her kidney disease. Which of the following do b. Obtain a serologic work up to identify the cause of her
you regard the most appropriate next step? proteinuria
a. Make sure that all her medications are appropriately dosed c. Talk with her primary care provider and review her medical
and advise her to consult her pharmacist any time a new record to determine whether she has any underlying cog-
medication is started to make sure it is not nephrotoxic and nitive issues to determine whether she needs an acute neu-
is appropriately dosed rologic evaluation
b. Discuss dialysis treatment modality with a view for sending d. Start her on an ACE inhibitor
her for vascular access placement if she starts hemodialysis

6 American Society of Nephrology American Society of Nephrology


Chapter 6: Limitations of Various Formulae and Other
Ways of Assessing GFR in the Elderly: Is There a Role
for Cystatin C?
Devraj Munikrishnappa
Department of Internal Medicine, The Nephrology Division, St. Louis University School of Medicine, St. Louis,
Missouri

GFR is the best index available to assess kidney and diet are stable, serum creatinine could be used
function in disease and in health in an individual. It for monitoring GFR more closely. In general, a
is 120 to 130 ml/min per 1.73 m2 in young, healthy change in serum creatinine ⬎15% is likely to indi-
adults, and it decreases by about 0.8 ml/min per cate a significant fall in GFR in an individual patient
1.73 m2 per year after 40 yr of age. However, it is rather than being caused by simple biologic and an-
important to note that, in the Baltimore Longitudi- alytical variations. Table 1 shows other limitations
nal Study on Aging, about one third of the patients of Scr.
that were followed did not have a decrease in GFR
with aging. Creatinine Clearance
The GFR cannot be measured directly in an in- Creatinine clearance as measured from a 24-h urine
dividual. Therefore, it is assessed using either exog- collection can be used to measure GFR, but it is vital
enous markers or endogenous markers in their to remember the high likelihood of inaccurate col-
steady states as shown in Table 1. lection, especially in some elderly people with cog-
nitive impairment or the bed bound. It is impor-
tant, therefore, to check for adequacy of urinary
MEASURING GFR USING EXOGENOUS collection before interpretation of clearance. The
MARKERS collection is said to be adequate if the creatinine
excretion is 20 to 25 mg/kg per day in a young
The direct methods in the general population and healthy man and if it is 15 to 20 mg/kg per day in a
in elderly persons are riddled with a number of young healthy woman. In elderly people, adequacy
problems (Table 1). Therefore, except in rare situ- is similarly checked because it is assumed that the
ations such as in a prospective kidney donor with muscle mass (and hence creatinine generation) and
borderline GFR for eligibility, these methods are renal function decline simultaneously with age.
not used in clinical practice. Caution must, therefore, be used if this assumption
cannot be made in individual instances. If this
method is used in the setting of acute renal failure
METHODS OF GFR ESTIMATION USING or rapidly changing serum creatinine, it is necessary
ENDOGENOUS MARKERS to measure an average from simultaneous serial se-
rum creatinine values during urine collection. Cre-
Serum Creatinine (Scr) atinine clearance systematically overestimates GFR
GFR estimation based on serum creatinine alone is because of tubular secretion of creatinine. The 24-h
not an ideal method, especially in elderly persons urine collection for the estimation of GFR has been
because it is influenced by a number of variables shown by many studies to not be any more reliable,
such as age, gender, muscle mass, diet, and medica-
tions that block creatinine’s tubular secretion. For
Correspondence: Devraj Munikrishnappa, Department of Inter-
example, despite reductions in GFR to ⬍60 ml/min nal Medicine, The Nephrology Division, St. Louis University
per 1.73 m2, there may not be a significant increase School of Medicine, 1402 South Grand, St. Louis, MO 63104.
in creatinine in the elderly persons with decreased E-mail: [email protected]

muscle mass. On the other hand, if the muscle mass Copyright 䊚 2009 by the American Society of Nephrology

American Society of Nephrology Geriatric Nephrology Curriculum 1


Table 1. Methods of determining GFR and their limitations
Method of Determining GFR Limitations
Using exogenous markers: GFR can be Ideal filtration marker: inulin clearance Gold standard: for GFR assessment but is
measured as the urinary or plasma difficult to use in routine practice
clearance of an ideal filtration
marker or of alternative exogenous
markers
Alternate exogenous markers to inulin: Expensive, less widely available and complex
tests
Iohexol
51
Cr EDTA
125
I-iothalamate, 99mTc-DTPA
Using endogenous markers: Serum creatinine (Scr) Factors influencing Scr:
endogenous markers, such as serum
creatinine or serum cystatin C can
be used to estimate GFR from their
serum levels if they are in a steady
state.1 The estimation equations
using these endogenous markers
adjust to other variables in an
attempt to improve accuracy of
estimation of GFR from these
markers.
(1) Creatinine production: e.g., muscle mass,
ingested cooked meat, protein restriction
(2) creatinine filtration: with age, excretion
decreases resulting in underestimation of
GFR
(3) creatinine secretion:
(i) because of tubular secretion, tends to
overestimate GFR by about 10%,
increasing significantly as GFR declines
(ii) cimetidine, trimethoprim inhibit secretion
(4) Creatinine assay:
(i) Calibration bias: large variations between
laboratories in calibration of the creatinine
assays may lead to differences in
interpretation of values
(ii) Factors influencing assay methods-Jaffè
reaction based assays-glucose, ketones,
bilirubin, cephalosporins and enzymatic
method- flucytosine
(5) Extrarenal elimination may be increased
with decreasing GFR (degradation of
creatinine by intestinal bacteria)
Serum cystatin C See below
Measured urinary clearance using Prone to errors, unpleasant, inconvenient
creatinine and adequacy of urinary collection needs
to ascertained prior to interpretation. The
limitations of Scr also applies to this
method.
Serum creatinine-based estimation See below
equations:
1. CG formula
2. MDRD formula
Estimation of GFR from combined serum Recently published—experience limited
creatinine and cystatin C–based
equation

2 Geriatric Nephrology Curriculum American Society of Nephrology


and frequently less reliable, than serum creatinine-based equa- munity-related reasons, and not for reasons related to the kid-
tions. However, in individuals with variation in dietary intake ney.
(e.g., vegetarian diet, creatine supplements) or muscle mass Tables 2 and 3 show factors likely to influence cystatin C
(e.g., amputation, malnutrition, muscle wasting), as is seen in levels and limitations, respectively.5
many elderly persons, this may be a preferred method because
many of these factors are not specifically taken into account in
prediction equations. Table 1 shows other limitations.
GFR ESTIMATION FROM SERUM CREATININE-
Creatinine clearance (CCr) can be calculated if values for
BASED EQUATIONS
urine creatinine concentration (UCr), urine flow rate (V), and
plasma creatinine concentration (PCr) are known:
The two most commonly used equations to estimate GFR are
C cr⫽(UCr ⫻ V)/PCr serum creatinine based: Cockcroft-Gault (CG) and the Modi-
fication of Diet in Renal Disease (MDRD) equations. Essen-
For the creatinine clearances to be comparable among individ- tially, compared with serum creatinine, these equations in-
uals of different sizes, it is often corrected to that of an average- crease the accuracy of estimated GFR (eGFR) to the measured
sized person, which is 1.73 m2 and expressed as ml/min per GFR by accounting for variables such as age and weight in the
1.73 m2. If the sizes are extreme, Ccr should be corrected for former equation and age, gender, and race in the latter one.
their actual body surface area as follows:
CG Equation
Corrected Ccr ⫽ 共Ccr ⫻ 1.73)/Actual body surface area This is one of the most widely used equations, even among
elderly people, although it was originally derived from mostly
younger subjects, with only a 4% female representation.6 The
GFR Estimation by Serum Cystatin C
main intention of the equation was to predict creatinine clear-
Cystatin C is an endogenous substance like creatinine but is
ance instead of GFR, and hence, it was validated against mea-
constitutively produced by all nucleated cells, freely filtered,
sured creatinine clearance. Creatinine clearance, as we know,
reabsorbed, and catabolized by the kidney. Most studies have
overestimates GFR; therefore, the CG equation that estimates
shown that serum cystatin C levels correlate better with GFR
creatinine clearance should also overestimate GFR. However,
than does serum creatinine alone, especially at higher levels of
studies indicate that it actually underestimates GFR in the el-
GFR. Its physiologic role is that it is a cysteine proteinase in-
derly, especially at higher GFRs. According to one study, for
hibitor with important roles in extracellular proteolysis, im-
instance, both the MDRD and CG equations underestimated
mune modulation, and antibacterial and antiviral activities.
GFR in hospitalized older individuals, but CG did so more
Some in vitro studies have indicated that cystatin C may be
than MDRD.7 Nevertheless, most of the estimated values using
affected by some stimuli such as steroids and transforming
this equation (a median of 75%) were within 30% of measured
growth factor ␤. Additionally, cystatin C was thought to be
GFR, which was acceptable for good clinical decision making
either less influenced or not influenced at all by certain demo-
and superior to serum creatinine alone. Table 4 shows addi-
graphic factors such as age, race, gender, or muscle mass com-
tional limitations.
pared with serum creatinine in reflecting GFR until recently.
The CG equation is as follows:
There are now emerging data showing that it is, in fact, influ-
enced by some of these factors. For instance, a recent study, For men: CrCl 共ml/min兲 ⫽ 兵关共140 – Age in共yr兲兴
although not necessarily focusing entirely on elderly people,
with subjects with a mean age of 52 yr, concluded that cystatin ⫻ Weight共kg兲其/SCr共mg/dl兲 ⫻ 72
C was 9% lower in women and 6% higher in blacks for a given
GFR2,3. Similarly, another recent study that reported popula- where CrCl is creatinine clearance and SCr is serum creatinine.
tion distributions of cystatin C in the United States using sera For women, the above equation should be multiplied by
samples from the Third National Health and Nutritional Ex- 0.85.
amination Survey noted that abnormal cystatin C was more In cases of persons of extreme weights, some have used lean
prevalent with increasing age from 1% in the 20- to 40-yr-old body mass, whereas others have used correction of the eCrCl to
group to ⬎50% in persons over age 80 yr of age within each average body surface area.
demographic subgroup.3,4 However, the definition of abnor-
mal cystatin C levels was chosen as the 99th percentile distri-
Table 2. Factors considered to influence cystatin c levels
bution among 20 to 40 yr olds, without hypertension or dia-
betics. Although it is plausible that cystatin C rises with age for Serum Cystatin C
Factors
the reason that there is, in general, a decline in kidney function Level
as noted after the age of 40 yr, it is equally plausible that it could Elevated Hypothyroidism, steroid use,
be elevated in the elderly, at least partly, for reasons that are Rheumatoid arthritis
related to the primary function of cystatin C, for example, im- Reduced Hyperthyroidism

American Society of Nephrology Geriatric Nephrology Curriculum 3


Table 3. Advantages and limitations of cystatin C in GFR estimation
Advantages Limitations
Possible role as an alternative to serum creatinine or its estimation Normal range not defined, method of measurements are
equations in elderly people prone for muscle mass variation and not standardized for clinical practice
hence, creatinine generation
Possible role as an alternative to serum creatinine or its estimation The test is neither widely used nor easily available at this
equations in identifying elderly persons with early decline in time and has higher cost compared to serum
GFR creatinine
Serum cystatin C–based GFR estimates were closely comparable Possible increase in its extra-renal excretion in renal
to MDRD (abbreviated) estimates and an equation combining it insufficiency
with serum creatinine, age, sex and race yielded the best
possible estimates of GFR
Conflicting studies regarding factors influencing its
ability to estimate GFR

MDRD Equation surement, because it only requires serum creatinine, age, gen-
The original equation was derived from a study of 1628 mid- der, and race, but not weight or any urine collections.
dle-aged, nondiabetic, chronic renal insufficiency patients that Differences in calibration of creatinine assays between labora-
used a directly measured GFR by urinary clearance of 125I- tories can lead to differences in GFR estimation and thus is an
Iothalamate.8 It has several advantages over the CG equation important limitation of estimation equations in general. The
including providing an estimate of GFR rather than creatinine four-variable MDRD was therefore re-expressed in 2005 (as
clearance, and in addition, a greater percent of these estimates shown below) for use with creatinine methods calibrated to the
are within the clinically useful range for decision making: 90% reference assay method. It is important to note that laborato-
of the MDRD based estimates were within 30% of the mea- ries without calibrations of their serum creatinine assays cali-
sured GFR compared with about 75% of CG-based estimates. brated to the reference method, the isotope– dilution mass
However, the MDRD equation also has several limitations in- spectrometry (IDMS) method, should report eGFR using the
cluding that it is less accurate at levels above 60 ml/min per 1.73 original four-variable MDRD study equation, recognizing it is
m2. Consequently, it may lead to misdiagnosis and misclassi- less accurate, especially at higher levels of GFR.13
fication of CKD in individuals with mild renal insufficiency.1,9 In recent years, many laboratories in the United States
Table 4 shows additional limitations. started reporting, along with the serum creatinine, MDRD-
There have been some validation studies of the MDRD based eGFR values in routine chemistry laboratories and, in
equation in elderly people concluding that is it is better than some instances, with separate values for African Americans
the CG equation.10 –12 The most widely used form of MDRD in and non-African Americans. However, if the value of eGFR is
elderly people is the four variable version or the version that ⬎60 ml/min per 1.73 m2, no specific values are mentioned but
was abbreviated from the original six variable version (shown reported simply as ⬎60 ml/min per 1.73 m2. As mentioned
below). This is especially advantageous for elderly people com- above, this is because of a lack of precision of estimation at
pared with the CG formula or the creatinine clearance mea- higher levels of GFR. If the specific value ⬎60 ml/min per 1.73

Table 4. Limitations of CG and MDRD equations


Equations Limitations Examples
Both CG and MDRD equations Renal or extrarenal conditions affecting steady Initially in ARF, when the serum creatinine is
state of creatinine in plasma fluctuating or a recovering frail cachectic
elderly person
Interference with creatinine’s assay Ketones, glucose, medications such as
cephalosporins interfering with assay
Variations in muscle mass or diet Extremes of body mass, pure vegetarian diet
or protein-rich diet or amputees
Calibration bias
CG equation Values not adjusted for body surface area Loss of precision in obese individuals
MDRD equation Reliability and accuracy decreased in extremes Healthy adults, dialysis patients
of GFR
Lack of validation studies in some ethnic groups Some Asian ethnic groups
Dosing of drugs for renal impairment patients
was widely published based on creatinine
clearance even before the origin of MDRD
and is not adapted for it yet

4 Geriatric Nephrology Curriculum American Society of Nephrology


m2 is needed for some reason, one can use the web-based CONCLUSION
MDRD calculators to find it.
A pertinent point here is, in case of elderly people, if such an GFR is the best index available to assess kidney function. Ex-
estimate of GFR is in the range of 60 to 89 ml/min/1.73 m2 and cept in situations such as drug dosage adjustment and in some
these individuals have no known kidney damage markers, it is cases of offering transplant options, practically, the change in
unclear whether all of the reduced GFR is attributable to age- GFR is more important than the absolute cut-off value. Al-
related decline or part of it is caused by existing kidney disease though novel methods such as cystatin C– based ones are ex-
that the current kidney damage markers are unable to identify. plored, GFR estimation is still largely creatinine based. As such,
According to NHANES III data, about 75% of persons older we are faced with a number of limitations, especially in elderly
than 70 yr of age may have a GFR ⬍90 ml/min per 1.73 m2 and persons, because the variables affecting creatinine tend to be
approximately 25% may have a GFR of ⬍60 ml/min per 1.73 more pronounced because of comorbid conditions. Currently,
m2. There are no good studies investigating such subsets of MDRD is the most widely used method to estimate GFR in
elderly persons with decreased GFR but without identifiable elderly persons. However, other methods may be preferred in
evidence of kidney disease with respect to their long-term out- certain situations, for example, in extremes of weight, using the
comes. Nevertheless, the NKF-KDOQI recommendation for 24-h collection for creatinine clearance. Caution needs to be
elderly persons with eGFR in the range of 60 to 89 ml/min per exercised to ascertain that the serum creatinine is stable while
1.73 m2 without any kidney damage markers is to assess for using any of the estimation methods. Future studies in this
CKD risk factors, screen for kidney damage markers, and carry field is very important and should focus on issues such as the
out interventions to decrease risk of kidney injury. elderly subset of patients with decreased GFR but no identifi-
The six variable MDRD equation is as follows: able markers of kidney damage and novel methods of assessing
kidney function by attempting to directly assess nephron mass
GFR ⫽ 170 ⫻ 共SCr兲⫺0.999 ⫻ (Age)⫺0.176 and function instead of expanding the pool of estimation equa-
tions.
⫻ 0.762 共if patient is female)
⫻ 1.18 共if patient is black兲 ⫻ 共BUN兲⫺0.170 ⫻ (Alb)0.318
TAKE HOME POINTS
where BUN is blood urea nitrogen and Alb is albumin.
• All of the currently available methods of estimation of GFR have several
The abbreviated version or four variable version of the limitations in the elderly persons as in the general population
MDRD equation (ml/min per 1.73 m2) is as follows: • Serum creatinine and GFR estimation equations can be applied only
when creatinine is stable
GFR ⫽ 186 ⫻ 共SCr兲⫺1.154 ⫻ 共Age兲⫺0.203 • Age-related decline in GFR may not always occur as noted in Baltimore
Longitudinal Study on Aging and in some elderly persons it may be
⫻ 0.742 共if patient is female兲 ⫻ 1.212 共if patient is black兲 difficult to differentiate age-related decline from chronic kidney disease
• The MDRD method is currently the most widely used method of esti-
The re-expressed MDRD equation (abbreviated version; ml/ mation in the elderly persons

min per 1.73 m2) after IDMS-traceable calibration is as follows:

GFR ⫽ 175 ⫻ 共SCr兲⫺1.154 ⫻ 共Age兲⫺0.203 DISCLOSURES


None.
⫻ 0.742 共if patient is female兲 ⫻ 1.212 共if patient is black兲

REFERENCES
GFR ESTIMATION FROM SERUM CREATININE AND
SERUM CYSTATIN C–BASED EQUATIONS *Key References
1. Stevens LA, Coresh J, Greene T, Levey AS: Assessing kidney func-
tion—measured and estimated glomerular filtration rate. N Engl
As mentioned above, a recent study involving a pooled analysis J Med 354: 2473–2483, 2006*
of individuals with chronic kidney disease proposed an estima- 2. Stevens LA, Coresh J, Schmid CH, Feldman HI, Froissart M, Kusek J,
tion equation that included serum cystatin C in addition to Rossert J, Van Lente F, Bruce RD 3rd, Zhang YL, Greene T, Levey AS:
serum creatinine, age, sex, and race. The study concluded this Estimating GFR using serum cystatin C alone and in combination with
serum creatinine: a pooled analysis of 3,418 individuals with CKD.
equation provided the most accurate estimates. However, fur-
Am J Kidney Dis 51: 395– 406, 2008
ther studies are needed to confirm this, especially in elderly 3. Shlipak MG: Cystatin C: research priorities targeted to clinical decision
persons.2 making. Am J Kidney Dis 51: 358 –361, 2008
4. Köttgen A, Selvin E, Stevens LA, Levey AS, Van Lente F, Coresh J:
eGFR ⫽ 177.6 ⫻ Scr⫺0.65 ⫻ CysC⫺0.57 ⫻ 共Age兲⫺0.20 Serum cystatin C in the United States: the Third National Health and
Nutrition Examination Survey (NHANES III). Am J Kidney Dis 51:
⫻ 0.82 共if female兲 ⫻ 1.11 共if black兲 385–394, 2008

American Society of Nephrology Geriatric Nephrology Curriculum 5


5. Munikrishnappa D: Chronic kidney disease (CKD) in the elderly—a years old) measured by means of iohexol clearance, serum creatinine,
geriatrician’s perspective. Aging Male 10: 113–137, 2007* serum urea and estimated clearance. Scand J Urol Nephrol 38: 73–77,
6. Cockcroft DW, Gault MH: Prediction of creatinine clearance from 2004
serum creatinine. Nephron 16: 31– 41, 1976 11. Lamb EJ, Webb MC, O’Riordan SE: Using the modification of diet in
7. Pedone C, Corsonello A, Incalzi RA: Estimating renal function in older renal disease (MDRD) and Cockcroft and Gault equations to estimate
people: a comparison of three formulas. Age Ageing 35: 121–126, 2006* glomerular filtration rate (GFR) in older people. Age Ageing 36: 689 –
8. Levey AS, Bosch JP, Lewis JB, Greene T, Rogers N, Roth D: A more 692, 2007*
accurate method to estimate glomerular filtration rate from serum 12. Verhave JC, Fesler P, Ribstein J, du CG, Mimran A: Estimation of renal
creatinine: a new prediction equation. Modification of Diet in Renal function in subjects with normal serum creatinine levels: influence of
Disease Study Group. Ann Intern Med 130: 461– 470, 1999 age and body mass index. Am J Kidney Dis 46: 233–241, 2005
9. Froissart M, Rossert J, Jacquot C, Paillard M, Houillier P: Predictive 13. Levey AS, Coresh J, Greene T, Marsh J, Stevens LA, Kusek JW, Van
performance of the modification of diet in renal disease and Cock- Lente F; Chronic Kidney Disease Epidemiology Collaboration: Ex-
croft-Gault equations for estimating renal function. J Am Soc Nephrol pressing the Modification of Diet in Renal Disease Study equation for
16: 763–773, 2005 estimating glomerular filtration rate with standardized serum creati-
10. Fehrman-Ekholm I, Skeppholm L: Renal function in the elderly (⬎70 nine values. Clin Chem 53: 766 –772, 2007

6 Geriatric Nephrology Curriculum American Society of Nephrology


REVIEW QUESTIONS: LIMITATIONS OF VARIOUS family needs to start planning for him to be initiated on dialysis
FORMULAE AND OTHER WAYS OF ASSESSING GFR anytime soon. On reviewing the records, you conclude that his
IN THE ELDERLY: IS THERE A ROLE FOR CYSTATIN C? eGFR is approximately 82 ml/min per 1.73 m2 based on
MDRD without any known risk factors for kidney disease.
1. An active, 87-yr-old African-American female resident from a
Which of the following statements is true?
nursing home develops pneumonia requiring prolonged hos-
a. eGFR by MDRD is less accurate at GFR above 60 ml/min
pitalization and complicated by diarrhea. She returns to the
per m2
nursing home on a common oral antibiotic for one more week
b. Periodic follow-up for risk factor assessment recom-
duration in addition to metronidazole. She is noticeably weak,
mended
frail, and has lost significant weight. A chemistry panel ob-
c. MDRD is currently the most widely used equation for
tained on her return to the nursing home shows a creatinine of
eGFR in elderly persons
1 mg/dl, which is an increase of 0.1 mg/dl from the previous
d. All of the above
one obtained at the nursing home before her hospitalization.
e. None of the above
The facility pharmacy consultant reviews the antibiotic dose
and recommends a significant decrease in dosage. Presuming
3. The following statements are true of cystatin C in the elderly
the consultant made the right recommendation, what is the
except:
most likely explanation warranting a decrease in the dose?
a. serum cystatin C likely correlates better with GFR than
a. Mild decline in kidney function
does serum creatinine alone in a frail elderly with mild
b. Mild increased creatinine
kidney dysfunction
c. Significant loss of muscle mass and kidney function
b. Increased tubular secretion compared with creatinine
d. Pharmacist is likely incorrect
c. Steroids may influence its level
2. A robust, healthy elderly person 72 yr of age with no medical d. Age and muscle mass may also influence its level according
condition comes to your nephrology clinic. He inquires if the to some studies

American Society of Nephrology Geriatric Nephrology Curriculum 7


Chapter 7: Decline of Renal Function in Normal Aging,
Role of Oxidants/Inflammation: When Does It Begin:
Is It Inevitable, Preventable, or Treatable?
Helen Vlassara,* Luigi Ferrucci,† James Post,‡ and Gary Striker§
*Division of Experimental Diabetes and Aging, Department of Geriatrics, Mount Sinai School of Medicine, New
York, New York; †Longitudinal Studies Section, Baltimore Longitudinal Study of Aging, Clinical Research Branch,
National Institute on Aging, National Institutes of Health, Bethesda, Maryland; ‡Department of Nephrology, Bronx
Medical Center, Division of Nephrology, Mount Sinai School of Medicine, New York, New York; and §Division of
Nephrology, Department of Medicine, and Division of Experimental Diabetes and Aging, Department of Geriatrics,
Mount Sinai School of Medicine, New York, New York

This chapter will include two large longitudinal, ob- unique for two reasons: (1) both cross-sectional
servational studies of renal function changes in nor- and longitudinal observations were used in the pre-
mal aging, a smaller cross-sectional and interventional diction and (2) only individuals considered healthy,
study focusing on oxidants and inflammation, and a based on strict, standard criteria, were selected for
consideration of the causes of decreased function in these analyses. Reports on the longitudinal studies
aging in both animal models and aging normal hu- only used data from BLSA participants followed for
mans. We chose to focus on oxidant stress (OS) and at least 10 yr and who had five or more serial mea-
inflammation because they increase in aging and are sures. Overall, the longitudinal data analysis con-
thought to both underlie aging-related diseases, in- firmed the cross-sectional observations, although
cluding decreased kidney function. Importantly, it is the accelerated decline of kidney function with age
now possible to reduce OS and inflammation in was more accentuated and statistically significant.
both normal adults and patients with chronic kid- The estimated average annual change in CrCl was
ney disease (CKD). Therefore, if OS and inflamma- ⫺0.26 ml/min per 1.73 m2 in the age group 20 to 39
tion are critical in the pathogenesis of reduced renal yr and became 1.51 ml/min per 1.73 m2 after the age
function in aging, and progression in CKD, it is of 80 yr.3 A study of water restriction confirmed
incumbent on the renal community to recognize/ that older BLSA participants have impaired re-
reduce their levels as a part of normal care or in the sponse of renal tubules to change in plasma osmo-
construction of clinical trials aimed at reducing CKD lality associated with impaired sodium homeostasis,
or cardiovascular disease (CVD). Finally, increased previously reported in smaller studies.3 Although
OS and inflammation may reduce the ability of the these data are a widely cited reference for kidney aging,
aging person to sustain metabolic or physical stress. they have some intrinsic limitations. First, they
were estimated from men only. Second, the number
of very long-lived individuals (⬎85 yr of age) was
LONGITUDINAL, OBSERVATIONAL quite small. Third, while BLSA participants were
STUDIES OF RENAL FUNCTION IN ADULTS selected to be “healthy,” the diagnostic technology
available at that time may have not detected sub-
The Baltimore Longitudinal Study of Aging clinical cardiovascular and kidney disease. This is
The Baltimore Longitudinal Study of Aging important because the distinction between aging-
(BLSA), the first continuing scientific examination related renal changes and progressive renal insuffi-
of human aging, was started in 1958 and has been an ciency are associated with a different prognosis.
important source of information on the aging kid-
ney.1 In 1976, the BLSA confirmed the previously
Correspondence: Helen Vlassara, MD, Division of Experimental
postulated progressive decline of renal function Diabetes and Aging, Department of Geriatrics, Mount Sinai
with aging, using age-adjusted standards for creat- School of Medicine, 1 Gustave Levy Place, New York, NY 10029.
inine clearance (CrCl).2 The BLSA data were Copyright 䊚 2009 by the American Society of Nephrology

American Society of Nephrology Geriatric Nephrology Curriculum 1


Nevertheless, the findings of the BLSA remain the best descrip- significant predictor of mortality4 (Figure 2). The 24-h creati-
tion of the course of renal function with aging. Ongoing re- nine clearance (CrCl)-, Cockcroft-Gault (C-G)-, and Modifi-
search in the BLSA has added studies of renal function in cation of Diet in Renal Disease (MDRD)-derived equations
women and will consider genetic and environmental factors (full and simplified) were calculated at enrollment
that affect renal function with aging. Perhaps one of the most (1998 –2000), and all-cause mortality and cardiovascular mor-
important findings of the BLSA for future research was that tality were prospectively ascertained by Cox regression over a
kidney function varied between persons at all ages, and when 6-yr follow-up. Participants with CrCl 60 to 90 ml/min per
declines with aging were noted, they occurred at substantially 1.73 m2 and CrCl ⬍60 ml/min per 1.73 m2 were, respectively,
different rates. In some individuals with an accelerated decline 1.70 (95% CI: 1.02 to 2.83) and 1.91 (95% CI: 1.11 to 3.29)
in CrCl, the presence of undetected, subclinical diseases could times more likely to die over the follow-up compared with
not be excluded. Indeed, Rowe et al.3 showed that in restricting those with CrCl ⬎90 ml/min per 1.73 m2. Using the C-G
the analysis to individuals without diabetes or any degree of equation, the group with values ⬍60 ml/min per 1.73 m2
hypertension, the decline was much less accentuated. Perhaps had a significantly higher all-cause mortality compared with
more important for nephrologists is the fact that some BLSA those with values ⬎90 ml/min per 1.73 m2 (HR: 2.59, 95%
participants showed periods of 5 yr or even 10 yr without a signif- CI: 1.13 to 5.91). Interestingly, classification based on the
icant decline of renal function. Although the number of these MDRD formulae did not provide any significant prognostic
individuals was small and few were older than 70 yr of age, they information for mortality. A possible interpretation of these
challenge the notion that the decline of kidney function with differences is that C-G includes information on chronological
age is unavoidable and calls for further investigation (Figure 1). age, whereas the MDRD formulae do not directly include age.
Despite the reduction of CrCl in many adults with aging, Although the data from the BLSA and InCHIANTI studies
serum creatinine remains relatively stable and, in observa- indirectly suggested that the age-associated decline of CrCl
tional studies of older individuals, is only slightly correlated may have little effect on health and mortality in the absence
with age. A change in body composition, especially the crude of other disease, the presence of even small amounts of pro-
and relative decline of muscle mass, may partly explain this teinuria (microalbuminuria) is associated with an increased
finding since skeletal muscle is the major source of creatinine risk of CVD.5
production. Therefore, some suggest that information on lean
body mass, such as those that can be gathered by dual-energy
x-ray absorptometry (DEXA), could be used to modify the
CrCl estimating equation. This was applied to a representative RENAL FUNCTION, INFLAMMATION, AND OS IN
aging population in Italy, the InCHIANTI study, and used an NORMAL AGING ADULTS: ROLE OF DIETARY
estimating equation that included information on grip ADVANCED GLYCATION ENDPRODUCTS
strength, a surrogate of muscle mass; however, the improve-
ment in prediction was small and of questionable clinical Advanced Glycation Endproducts
value, and no other study that includes information on muscle The pro-oxidants considered in this chapter are representa-
mass/strength has been validated in large samples. tives of pro-oxidants in general and belong to a common class
of compounds referred to as advanced glycation endproducts
The InCHIANTI Study (AGEs). AGEs form spontaneously between sugars and the
Longitudinal data from this study addressed the question of amino groups of proteins lipids and nucleic acids.6 They are
whether measured and estimated CrCl in older individuals is a highly reactive, generating reactive oxygen species (ROS) and
inflammation responses both intracellularly and extracellu-
200
larly. While it has generally been thought that AGEs are prin-
150

100
cipally derived from cellular metabolism, it is now widely ap-
Creatinine Clearance (mL/min)

50 preciated that a large part of the total oxidant burden derives


0
200 from the diet in normal adults and in chronic diseases, such as
150
CKD and diabetes.7–17
100

50

0
200
Uptake, Detoxification, and Elimination of AGEs
150 AGEs react with cell surface receptors, which mediate opposite
100
responses. One receptor (AGER1) lowers OS and inflamma-
50

0
tory reactions,18 –20 whereas another (RAGE) mediates in-
30 40 50 60 70 30 40 50 60 70 80 40 50 60 70 30 40 50 60 70 30 40 50 60 70
creases in these parameters.9,21–23 Both receptors are driven by
Age (yrs)
the ambient levels of ligand (AGEs) in normal, healthy sub-
Figure 1. Representative subjects from the Baltimore Longitudi- jects. Serum AGEs, often bound to small peptides, are filtered
nal Study of Aging, who did not show a significant decline in by the kidneys and AGEs. They can also be metabolized to
measured creatinine clearance values over a period of several years. inactive molecules and excreted in the renal tubules.

2 Geriatric Nephrology Curriculum American Society of Nephrology


24h-ClCr Cockcroft-Gault
1.00 >90 ml/min 1.00 >90 ml/min

Survival Distribution Function


Survival Distribution Function
60-90 ml/min
60-90 ml/min
0.75 < 60 ml/min 0.75 < 60 ml/min

0.50 0.50

0.25 0.25

0.00 0.00
0 1 2 3 4 5 6 0 1 2 3 4 5 6

Years Since Enrollment Years Since Enrollment

MDRD-4 MDRD-6
1.00 1.00
Survival Distribution Function

Survival Distribution Function


>90 ml/min >90 ml/min

60-90 ml/min 60-90 ml/min


0.75 0.75
< 60 ml/min
< 60 ml/min

0.50 0.50

0.25 0.25

0.00
0 1 2 3 4 5 6 0 1 2 3 4 5 6
Years Since Enrollment Years Since Enrollment

Figure 2. Comparisons of different methods of calculating GFR in the InCHIANTI study individuals with respect to predictions of
survival. Note that there was considerable variation in the predictive value of the various equations, suggesting that additional factors
may have to be considered in the aging population.

Oxidant Stress and Inflammation in Normal Adults a decrease in eGFR with aging (Figure 3C, inset). Nonetheless,
From Early to Late Adulthood urinary excretion of AGEs was significantly lower in older
Many studies have shown that oxidant stress and inflamma- adults, consistent with a lower intake.
tion generally increase with aging; however, few have consid-
ered adults older than 75 yr old.2,24 When we studied a normal
cohort residing in New York that included older adults, we
found that there was considerable heterogeneity between indi- SOURCES OF PRO-OXIDANTS IN ADULTS
viduals (Figure 3).14 Although, on average, there is an increase
in oxidant stress and inflammation in aging, this was not a Generally speaking, the amount of pro-oxidants in food in-
universal finding in the normal population. For instance, al- creases when food is cooked at high heat and without wa-
though there was an overall increase in carboxymethyl lysine ter.25,26 We find that the way the food is cooked, rather than the
(CML; Figure 3A) and methylglyoxal (MG; Figure 3B), there composition of the diet, is the critical factor in the amount of
was a substantial number of older individuals who had normal oxidants in the food (Table 1). For instance, meat cooked with
levels (upper limit of normal for CML ⫽ 12 to 15 and MG ⫽ water (steamed or boiled) has a much lower AGE content than
1.0). There was an inverse correlation between CML levels and broiled meat. In addition, in food that is cooked in the presence
estimated GFR (eGFR; MDRD; Figure 3C), but, as with the of lipids at high temperature (as in fries), the amount of AGEs
BLSA, there was a number of normal adults that did not show is markedly increased. Additional ways to reduce oxidant for-

A. Serum CML (U/ml) B. Serum MG (nmol/ml) C. Serum CML vs. eGFRCr


60 3.0
r = 0.647 r = 0.553, r = - 0.647 250 r = - 0.447
p = 0.0001 p = 0.0001 60
p = 0.0001 200 p = 0.0001
50 150
2.0
40 100
40
50
30 0
20 40 60 80 100
1.0 age (years)
20
20

10
0 0.0 0
20 40 60 80 100 20 40 60 80 100 0 50 100 150 200 250
age (years) age (years) eGFRCr

Figure 3. Data from a cross-section of subjects without obvious concurrent disease. (A) Serum carboxymethyllysine, (B) serum
methylglyoxal, and (C) serum CML versus eGFRCr (inset, eGFRCr versus age). Note that the slope of the curves in A and B is largely
driven by the large range in CML and MG levels in the aged. The eGFRCr values in the aged also show a large spread.

American Society of Nephrology Geriatric Nephrology Curriculum 3


Table 1. Thermally modulated
odulated AGE
GE content
ontent in common
ommon foods
oods
mediators and oxidant stress (Figure 5A). Those who main-
tained their normal diet had a stable or a modest increase in OS
Regular Diet
et (U/mg) Low AGE Diet
iet
and inflammation. However, the energy intake remained con-
Beef: broiled 5367 STEWED 2000
stant in the two groups. Importantly, this intervention was
done in normal subjects at their own home, they continued to
Chicken: broiled 5245 STEWED 1011
prepare their usual types of foods, and they did not require
Salmon: broiled 1348 RAW 502 extensive supervision to maintain adherence to the recom-
Potato: fried 1522 STEAMED 17 mended cooking methods. Since this intervention turned out
to be practical and economical, it may be applicable to the
The levels of AGEs in most foods, not only red meats, depend
largely on the method of cooking (see potato). Data are general population. The length of time required to uncover
shown as CML-immunoreactivity, based on ELISA.
A
Healthy Participants on a Low-AGE Diet
mation in foods include marinating with acidic substances (to-
matoes, red wine, vinegar, lemon juice, etc.).
40

∆%
MODIFYING OXIDANT INTAKE 20

Studies in Normal Adults 0


Generally, the adult, healthy population maintains a constant
level of oxidant intake at any given period of life, and this -20

∆%
intake directly correlates with serum AGEs14(Figure 4A). Al-
though this has not been extensively analyzed in the CKD pop- -40 * * * *
ulation, many of the same principles are likely to apply. * *
Namely, adults have constant habits for cooking and order -60 * *
*

TN -1
foods cooked in similar ways in restaurants. Surprisingly, it

6 hc
1

GE
AM
GE

F␣
so

al
ML

ER

p6 s
appears that this behavior can be readily modified. We enrolled G

dC
dA
8-i

RA

VC
sM
sC

AG
adults in the highest tertile of consumption of AGEs among
normal subjects of all ages (Figure 4B), randomly divided them
into two groups, and followed them for 4 mo.27 One group B CKD Patients on a Low-AGE Diet

maintained their normal cooking methods and food types. The


60 *
AGER1
second was provided with instructions on how to cook the
∆%

40
food that they normally eat so that the formation of AGEs
would be lowered. Two results were noted: first, there was a
20
substantial reduction in the amount of AGEs consumed by the
subjects who modified their food preparation methods. Sec-
0
ond, this intervention was associated with an approximate de-
crease of 30 to 60% in the amount of circulating inflammatory -20
∆%

A. 8-isoprostane (pg/ml) B. hsCRP TNF- *


* -40 * *
**
600 r = 0.723, p=0.000 *
8
y = 22.24x + 10.8 16 *
6 -60 * *
pg/mg

400 12
-1
6 hc
sM L

F␣

GE
AM
mg/l

so
G

4
al
GE
M

p6 s

TN

dC
sC

dA
8-i

VC

8
RA

200
2
4
0 0 Figure 5. The influence of a reduction of the intake of AGEs on
0 5 10 15 20 <15.4 >23 <15.4 >23
sAGE (U/ml) the blood levels of AGEs, measures of OS and inflammation, and
Dietary AGEs (Eq/day)
AGE receptors. (A) After a 4-mo period on a low-AGE diet (⬃50%
Figure 4. Correlations of serum levels of AGEs and levels of reduction), the levels of serum markers of OS, inflammation, and
oxidants markers of inflammation. (A) Serum levels of AGEs di- AGE receptors were all substantially reduced, whereas calorie
rectly correlate with markers of lipid oxidation (8-isoprostanes). (B) intake remained essentially unchanged. (B) After a 4-wk period on
Those subjects who consumed lower levels of AGEs had lower a low-AGE diet, CKD 2– 4 patients had a similar reduction of
serum levels of hsCRP and TNF␣. hsCRP, high-sensitivity C-reac- markers of OS and inflammation. Note that whereas RAGE levels
tive protein; TNF, tumor necrosis factor; sAGE, serum advanced were reduced by the dietary intervention, AGER1 levels returned
glycation factor. to normal levels.

4 Geriatric Nephrology Curriculum American Society of Nephrology


significant changes in OS and inflammation using this simple INFLAMMATION/OXIDANT STRESS IN PATIENTS
dietary maneuver was found to be much shorter (4 mo) than WITH CKD AND NEUROLOGIC SYMPTOMS
we had anticipated. In addition, both glucose and insulin lev-
els, two indicators of the metabolic syndrome, were coordi- It is well known that adults with elevated oxidant stress and/or
nately lowered in the normal subjects consuming a diet with inflammation have an increased risk of aging-related diseases:
lower AGEs content. It is important to note that the decrease in CVD, Alzheimer’s disease (AD), atherosclerosis, stroke, CKD,
the AGEs content of the diet was, on average, only approxi- and diabetes.29 –33 The time at which the increased risk begins is
mately 30 to 40%. This suggests that the normal diet often not known, but the above data suggest that an increase in the
contains a sufficiently large amount of oxidants that the anti- risk factors can begin in young adulthood in the normal pop-
oxidant systems are saturated. Importantly, a modest reduc- ulation. There has been increasing evidence for a kidney—
tion in oxidant intake can reduce OS and inflammation in cognition connection or a renocerebral syndrome. Moderate
normal adults. Thus, the diet may underlie the high OS found CKD defined as an eGFR of 30 to 59 ml/min per 1.73 m2 has
in normal aging. been associated with poor baseline cognitive function and an
increased risk for cognitive decline in both elderly and young
Modifying the Levels of Oxidants and Inflammation in populations versus those with an eGFR ⱖ60 ml/min per 1.73
Patients With CKD m2.34 –36 The impaired cognition in patients with CKD seems
Somewhat surprisingly, the high baseline levels of OS and in- to be related to the severity of renal disease and is more severe
flammation in patients with CKD also responded to a similar in those receiving hemodialysis.36 In patients receiving outpa-
reduction in dietary AGE intake.27 A 4-wk low-AGE dietary tient hemodialysis, cognitive impairment, defined as mini
diet was given to patients with stage 2 to 4 CKD [using food mental state examination (MMSE) ⱕ 24, has a prevalence re-
prepared by the general clinical research center (GCRC)]. Both ported to range from 30 to 60%.37,38 Milder forms of cognitive
OS and inflammation were reduced, paralleling the percent impairment have also been reported in 34% of outpatient he-
change reached in healthy adults (Figure 5B). This result sug- modialysis patients compared with age-, gender-, and comor-
gests that the reduced excretion of oxidants in patients with bidity-matched controls.39
CKD may be amenable to treatment without drugs and could Patients with CKD have a high prevalence of subclinical
be both practical and economical. It remains to be seen in subcortical white matter damage on neuroimaging and a high
larger longitudinal studies whether this reduction results in incidence rate of stroke.40,41 These findings are likely related to
improved cardiac, renal, or central nervous system (CNS) out- a high burden of traditional and nontraditional vascular risk
comes in aging and chronic diseases. factors in these patients including hypertension, diabetes, hy-
perlipidemia, elevated oxidative stress, and elevated inflamma-
tory state. Since positive outcomes in patients with advanced
CKS and receiving HD require adherence to complex treat-
FACTORS INFLUENCING THE REMOVAL OF PRO- ment regiments, identification of neurocognitive deficits that
OXIDANTS AND REDUCTION OF INFLAMMATION may be a prodrome to later vascular dementia may provide an
IN NORMAL AGING AND IN PATIENTS WITH CKD opportunity to slow the progression of cognitive decline and
provide a method to identify modifiable factors associated with
As noted above, AGEs react with cell surface receptors that early cognitive decline. The effect of antihypertensives, statins,
mediate opposite responses. AGER1 reduces both OS and in- ARB/ACE inhibitors, antioxidants, and modifiers of inflam-
flammation, whereas RAGE promotes OS and inflammatory mation on the high prevalence of cognitive impairment and
reactions. Both receptors are driven by the ambient levels of stroke in this population are potential areas of future research.
ligand (AGEs) in healthy subjects. Thus, the levels of both re-
ceptor types were decreased when the levels of oxidants in the
diet were lowered (Figure 5A). While AGER1 levels have been UPTAKE OF DIETARY AGES IN HEALTHY ADULTS
shown to be decreased in patients with CKD,10,18,20,28 a reduc- AND PATIENTS WITH EARLY DIABETIC
tion in the dietary load of oxidants resulted in restoration of NEPHROPATHY
AGER1 levels to near normal values in these patients (Figure
5B). This surprising result must be verified in larger studies, Healthy adults exposed to diets with a low or high content of
but it further supports the view that OS and inflammation may AGEs show rapid absorption of AGEs, resulting in a spike in
be amenable to reduction in CKD patients without drugs. The serum levels, followed shortly thereafter by an increase in the
take-home message from these studies is that oxidants in the content of AGEs in the urine (Figure 6A).14 Those fed a diet
diet directly lead to increased serum levels of inflammatory rich in AGEs had higher peak serum and urine AGE levels than
mediators and OS, but this process can be interrupted by sim- those fed a low-AGE diet. However, the levels rapidly returned
ply changing the methods of processing and/or cooking the to baseline.
usual foods consumed by adults. Importantly, the same out- Two groups of patients with diabetes, with either micro- or
comes may also be seen in CKD patients. macroalbuminuria and either a normal or only slightly re-

American Society of Nephrology Geriatric Nephrology Curriculum 5


Serum AGE (U/ml) Urine AGE (Ux103/ml) both cases. Aminoguanidine was the first to be developed and
A. NORMAL SUBJECTS
tested in animals and then in patients.49 This drug interferes
sAGE (u/ml)
30 60
25 High-AGE with the generation of AGEs intracellularly. It did not meet the
Low-AGE 45
20
15
required outcome of decreasing the rise in serum creatinine by
30
10 50% in a single trial.50 The second drug, a vitamin B6 analog,
5 15
0
pyridoxamine, has long been used in the treatment of hyper-
0
0 20 40
hours
60 0 20 40
hours
60 oxaluria and has few side effects. It serves to block the early
B. DIABETIC 60
SUBJECTS
glycation derivatives after the formation of Schiff bases and is
sAGE (u/ml)

30 60 Ccr=100, µ/alb also a chelator. In a phase 2 clinical trial, it reduced proteinuria


25
45 Ccr=84, in diabetic patients with progressive renal disease.51
20 proteinuria
15 30
10 15
5
0 0 STUDIES IN HEALTHY AGING MICE
0 20 40 0 20 40 60
hours hours
Rodent food contains a relatively large amount of oxidants
Figure 6. The levels of AGEs in the serum and urine of normal
because of the increased temperature used in its preparation,
subjects and patients with CKD after eating a meal with either a
low or high AGE content. (A) The levels of serum AGEs and urine
zzespecially that amount needed to pelletize the granular prod-
rapidly rose in normal subjects, varying directly with the amount uct. The significance of increased oxidants in the diet becomes
of AGEs in the diet. (B) Patients with diabetes and either mi- clear when comparing kidney and heart lesions in mice given
croalbuminuria or proteinuria, with relatively preserved renal isocaloric diets prepared with standard heating (high-AGE
function, showed higher baseline levels of AGEs, higher and diet) or with less heating during manufacture (low-AGE diet).
more prolonged peak levels, and a smaller increase in urine AGE Namely, both kidney and cardiovascular lesions were mark-
levels. edly reduced, and lifespan was significantly extended in mice
given the low-AGE diet.7,8 To determine if the inflammation
duced eGFR, were given either a standard or AGE-rich meal and OS was caused by the AGEs in the diet, we added a specific
(Figure 6B). Their responses to a regular meal were quite dif- AGE to a low-AGE diet. Adding methylglyoxal (MG) led to an
ferent.10 Namely, the baseline levels of AGEs were higher than increase in both OS and inflammation in the serum, paralleling
normal, and the high peak serum levels were maintained for a standard mouse chow.8 The only other major intervention that
prolonged period. Note that these changes were present in di- reduces OS and inflammation, prevents CKD and CVD in ag-
abetics that had either a normal or only slightly reduced eGFR ing, and prolongs life is caloric restriction.52 It is not clear
and that urinary excretion of AGEs was also diminished. These whether the beneficial effects of calorie restriction are caused
data suggest that the kidney responds to increased oxidant by a 40 to 60% reduced energy intake, since reduction of food
stress and inflammation by reducing the excretion of AGEs. intake concomitantly restricts the intake of oxidant by the
The data also suggest that AGEs may be actively excreted by the same amount. Therefore, we designed experiments in which
tubules and filtered at the glomerulus.28,42 mice were pair-fed the regular diet throughout their lifetime
(which has a high AGE content) and compared them with mice
fed a calorie-restricted diet (40% decrease in food) and mice
DRUGS AND THE REDUCTION OF PRO-OXIDANT fed the same calorie-restricted diet that had a high content of
MOLECULES (AGES) AGEs, prepared with an additional heating step. At sacrifice in
late life, the kidney lesions were largely prevented in the calo-
Vitamins and Other Dietary Supplements rie-restricted mice, and the heart was essentially normal. How-
In general, the results of trials of a wide range of oral anti- ever, mice given either of the high AGE diets had sclerotic
oxidants have largely been disappointing.43– 46 However, none cardiac and renal lesions. The two high AGE diets (normal
of these studies included an attempt to control baseline levels mouse chow and calorie restricted, high AGE) were also asso-
of OS and inflammation. If our data apply to the general pop- ciated with a decreased life span. These findings suggest that
ulation (Figure 3),14the great variation in the baseline levels of the beneficial effects of calorie restriction may be partly ex-
oxidants may partly explain these results. This conclusion is plained by the concomitant reduction in oxidant intake.
supported by the fact that many studies have shown that the
Mediterranean Diet, which is low in AGEs, is associated with
low levels of OS and inflammation and seems to have beneficial CONCLUSIONS
effects on general health.47,48
CrCl declines, on average, with aging even in individuals with-
Anti-AGE/Oxidant Products out clinical kidney disease. Although this decline occurs in
Two drugs have been tested for their ability to reduce the levels most, a small number of individuals maintain intact renal
of AGEs in clinical trials. The focus has been on diabetics in function up to a very old age, raising the issue that a decline

6 Geriatric Nephrology Curriculum American Society of Nephrology


may not be obligatory. Furthermore, the handling of oxidants 9. Kilhovd BK, Juutilainen A, Lehto S, Rönnemaa T, Torjesen PA, Birke-
land KI, Berg TJ, Hanssen KF, Laakso M: High serum levels of ad-
by the kidney may be suppressed before a decline in eGFR. One vanced glycation end products predict increased coronary heart dis-
problem in the assessment of renal function in aging is the lack ease mortality in nondiabetic women but not in nondiabetic men: a
agreement among nephrologists and geriatricians about which population-based 18-year follow-up study. Arterioscler Thromb Vasc
of the currently available formulae to calculate GFR is most Biol 25: 815– 820, 2005
appropriate in normal aging. This topic awaits further study. 10. Koschinsky T, He CJ, Mitsuhashi T, Bucala R, Liu C, Buenting C,
Heitmann K, Vlassara H: Orally absorbed reactive glycation products
In addition, there is currently no biomarker allowing determi- (glycotoxins): an environmental risk factor in diabetic nephropathy.
nation of whether impaired renal function in an older individ- Proc Natl Acad Sci USA 94: 6474 – 6479, 2007*
ual is cuased by “normal” aging or the effect of some factor(s) 11. Makita Z, Bucala R, Rayfield EJ, Friedman EA, Kaufman AM, Korbet
as yet unidentified. A study of oxidant stress and inflammation SM, Barth RH, Winston JA, Fuh H, Manogue KR, et al: Reactive
and the role of current high-AGE diets in altering kidney func- glycosylation endproducts in diabetic uraemia and treatment of renal
failure. Lancet 343: 1519 –1522, 1994
tion(s) may also be a fruitful area for future investigation, since 12. Makita Z, Radoff S, Rayfield EJ, Yang Z, Skolnik E, Delaney V, Fried-
there are economical and efficient interventions that reduce man EA, Cerami A, Vlassara H: Advanced glycosylation end products
oxidant stress and inflammation in both normal adults of all in patients with diabetic nephropathy. N Engl J Med 325: 836 – 842,
ages and patients with CKD. 1991
13. Stirban A, Negrean M, Stratmann B, Gawlowski T, Horstmann T,
Götting C, Kleesiek K, Mueller-Roesel M, Koschinsky T, Uribarri J,
Vlassara H, Tschoepe D: Benfotiamine prevents macro- and microvas-
TAKE HOME POINTS cular endothelial dysfunction and oxidative stress following a meal rich
• Oxidants in the diet directly lead to increased serum levels of inflam- in advanced glycation end products in individuals with type 2 diabe-
matory mediators and OS tes. Diabetes Care 29: 2064 –2071, 2006
• This process can be interrupted by simply changing the methods of 14. Uribarri J, Cai W, Peppa M, Goodman S, Ferrucci L, Striker G, Vlassara
processing and/or cooking the usual foods consumed by adults H: Circulating glycotoxins and dietary advanced glycation endprod-
• The same outcomes may also be seen in patients with CKD ucts: two links to inflammatory response, oxidative stress, and aging.
J Gerontol A Biol Sci Med Sci 62: 427– 433, 2007*
15. Uribarri J, Peppa M, Cai W, Goldberg T, Lu M, He C, Vlassara H:
Restriction of dietary glycotoxins reduces excessive advanced glyca-
DISCLOSURES tion end products in renal failure patients. J Am Soc Nephrol 14:
None. 728 –731, 2003
16. Uribarri J, Stirban A, Sander D, Cai W, Negrean M, Buenting CE,
Koschinsky T, Vlassara H: Single oral challenge by advanced glycation
end products acutely impairs endothelial function in diabetic and
REFERENCES nondiabetic subjects. Diabetes Care 30: 2579 –2582, 2007*
17. Vlassara H, Cai W, Crandall J, Goldberg T, Oberstein R, Dardaine V,
*Key References Peppa M, Rayfield EJ: Inflammatory mediators are induced by dietary
1. Ruggiero C, Metter EJ, Melenovsky V, Cherubini A, Najjar SS, Ble A, glycotoxins, a major risk factor for diabetic angiopathy. Proc Natl Acad
Senin U, Longo DL, Ferrucci L: High basal metabolic rate is a risk factor Sci USA 99: 15596 –15601, 2004*
for mortality: the Baltimore Longitudinal Study of Aging. J Gerontol A 18. He CJ, Koschinsky T, Buenting C, Vlassara H: Presence of diabetic
Biol Sci Med Sci 63: 698 –706, 2008 complications in type 1 diabetic patients correlates with low expres-
2. Lindeman RD, Tobin J, Shock NW: Longitudinal studies on the rate of sion of mononuclear cell AGE-receptor-1 and elevated serum AGE.
decline in renal function with age. J Am Geriatr Soc 33: 278 –285, Mol Med 7: 159 –168, 2001
1985* 19. Cai W, He JC, Zhu L, Chen X, Striker GE, Vlassara H: AGE-receptor-1
3. Rowe JW, Andres R, Tobin JD, Norris AH, Shock NW: The effect of counteracts cellular oxidant stress induced by AGEs via negative
age on creatinine clearance in men: a cross-sectional and longitudinal regulation of p66shc-dependent FKHRL1 phosphorylation. Am J
study. J Gerontol 31: 155–163, 1976 Physiol Cell Physiol 294: C145–C152, 2008
4. Pizzarelli F, Lauretani F, Bandinelli S, Windham GB, Corsi AM, Gian- 20. Lu C, He JC, Cai W, Liu H, Zhu L, Vlassara H: Advanced glycation
nelli SV, Ferrucci L, Guralnik JM: Predictivity of survival according to endproduct (AGE) receptor 1 is a negative regulator of the inflamma-
different equations for estimating renal function in community-dwell- tory response to AGE in mesangial cells. Proc Natl Acad Sci USA 101:
ing elderly subjects. Nephrol Dial Transplant 24: 1197–1205, 2009 11767–11772, 2004
5. Färbom P, Wahlstrand B, Almgren P, Skrtic S, Lanke J, Weiss L, 21. Chen Y, Akirav EM, Chen W, Henegariu O, Moser B, Desai D, Shen
Kjeldsen S, Hedner T, Melander O: Interaction between renal function JM, Webster JC, Andrews RC, Mjalli AM, Rothlein R, Schmidt AM,
and microalbuminuria for cardiovascular risk in hypertension: the nor- Clynes R, Herold KC: RAGE ligation affects T cell activation and
dic diltiazem study. Hypertension 52: 115–122, 2008* controls T cell differentiation. J Immunol 181: 4272– 4278, 2004
6. Baynes JW, Thorpe SR: Role of oxidative stress in diabetic complica- 22. Hudson BI, Kalea AZ, Del Mar Arriero M, Harja E, Boulanger E, D’Agati
tions: a new perspective on an old paradigm. Diabetes 48: 1–9, 1999 V, Schmidt AM: Interaction of the RAGE cytoplasmic domain with
7. Cai W, He JC, Zhu L, Chen X, Wallenstein S, Striker GE, Vlassara H: diaphanous-1 is required for ligand-stimulated cellular migration
Reduced oxidant stress and extended lifespan in mice exposed to a through activation of Rac1 and Cdc42. J Biol Chem 283: 34457–
low glycotoxin diet: association with increased AGER1 expression. 34468, 2008
Am J Pathol 170: 1893–1902, 2007 23. Wendt T, Tanji N, Guo J, Hudson BI, Bierhaus A, Ramasamy R, Arnold
8. Cai W, He JC, Zhu L, Chen X, Zheng I, Striker GE, Vlassara H: Oral B, Nawroth PP, Yan SF, D’Agati V, Schmidt AM: Glucose, glycation,
glycotoxins determine the effects of calorie restriction on oxidant and RAGE: implications for amplification of cellular dysfunction in
stress, age-related diseases, and lifespan. Am J Pathol 173: 327–336, diabetic nephropathy. J Am Soc Nephrol 14: 1383–1395, 2008*
2008* 24. Ferrucci L: The Baltimore Longitudinal Study of Aging (BLSA): a 50-

American Society of Nephrology Geriatric Nephrology Curriculum 7


year-long journey and plans for the future. J Gerontol A Biol Sci Med nition, and implications of mental impairment among hemodialysis
Sci 63: 1416 –1419, 2008 patients. Am J Kidney Dis 30: 41– 49, 1997
25. Goldberg T, Cai W, Peppa M, Dardaine V, Baliga BS, Uribarri J, 39. Murray AM, Tupper DE, Knopman DS, Gilbertson DT, Pederson SL, Li S,
Vlassara H: Advanced glycoxidation end products in commonly con- Smith GE, Hochhalter AK, Collins AJ, Kane RL: Cognitive impairment in
sumed foods. J Am Diet Assoc 104: 1287–1291, 2004* hemodialysis patients is common. Neurology 67: 216 –223, 2006
26. Woodruff S, Goodman S, Pyzik R, Yong A, Zhu L, Neade N, Beeri M, 40. Martinez-Vea A, Salvadó E, Bardajı́ A, Gutierrez C, Ramos A, Garcı́a C,
Silverman JM, Ferrucci L, Tansman L, Strker GE, Uribarri J: Control of Compte T, Peralta C, Broch M, Pastor R, Angelet P, Marcas L, Saurı́ A,
toxic advanced glycation end products in foods: methods and impli- Oliver JA: Silent cerebral white matter lesions and their relationship
cations for clinical practice (in press) with vascular risk factors in middle-aged predialysis patients with CKD.
27. Vlassara H, Cai W, Goodman S, Pyzik R, Yong A, Zhu L, Neade N, Beeri Am J Kidney Dis 47: 241–250, 2006
M, Silverman JM, Ferrucci L, Tansman L, Striker GE, Uribarri J: Pro- 41. Seliger SL, Gillen DL, Longstreth WT Jr, Kestenbaum B, Stehman-
tection against increased oxidant stress and the loss of innate de- Breen CO: Elevated risk of stroke among patients with end-stage renal
fenses with aging by a low dietary AGE intake: role of AGER1, an disease. Kidney Int 64: 603– 609, 2003
anti-inflammatory AGE receptor (in press) 42. Miyata T, Ueda Y, Horie K, Nangaku M, Tanaka S, van Ypersele de
28. Gugliucci A, Bendayan M: Renal fate of circulating advanced glycated Strihou C, Kurokawa K: Renal catabolism of advanced glycation end
end products (AGE): evidence for reabsorption and catabolism of products: the fate of pentosidine. Kidney Int 53: 416 – 422, 1998
AGE-peptides by renal proximal tubular cells. Diabetologia 39: 149 – 43. Bleys J, Miller ER 3rd, Pastor-Barriuso R, Appel LJ, Guallar E: Vitamin-
160, 1996 mineral supplementation and the progression of atherosclerosis: a
29. Dukkipati R, Adler S, Mehrotra R: Cardiovascular implications of meta-analysis of randomized controlled trials. Am J Clin Nutr 84:
chronic kidney disease in older adults. Drugs Aging 25: 241–253, 2008 880 – 887, 2006
30. Perkovic V, Verdon C, Ninomiya T, Barzi F, Cass A, Patel A, Jardine M, 44. de Bree A, Verschuren WM, Blom HJ, Nadeau M, Trijbels FJ, Krom-
Gallagher M, Turnbull F, Chalmers J, Craig J, Huxley R: The relation- hout D: Coronary heart disease mortality, plasma homocysteine, and
ship between proteinuria and coronary risk: a systematic review and B-vitamins: a prospective study. Atherosclerosis 166: 369 –377, 2003
meta-analysis. PLoS Med 5: e207, 2008 45. Traber MG: Heart disease and single-vitamin supplementation. Am J
31. Ballantyne CM, Hoogeveen RC, McNeill AM, Heiss G, Schmidt MI, Duncan Clin Nutr 85: 293S–299S, 2007
BB, Pankow JS: Metabolic syndrome risk for cardiovascular disease and 46. Ward NC, Wu JH, Clarke MW, Puddey IB, Burke V, Croft KD, Hodgson
diabetes in the ARIC study. Int J Obes (Lond) 32(Suppl 2): S21–S24, 2008 JM: The effect of vitamin E on blood pressure in individuals with type
32. Eckel RH, Wassef M, Chait A, Sobel B, Barrett E, King G, Lopes-Virella M, 2 diabetes: a randomized, double-blind, placebo-controlled trial.
Reusch J, Ruderman N, Steiner G, Vlassara H: Prevention Conference VI: J Hypertens 25: 227–234, 2007
diabetes and cardiovascular disease: writing group II: pathogenesis of 47. Sofi F, Cesari F, Abbate R, Gensini GF, Casini A: Adherence to Mediter-
atherosclerosis in diabetes. Circulation 105: e138 – e143, 2002 ranean diet and health status: meta-analysis. BMJ 337: a1344, 2008
33. Navarro-Dı́az M, Serra A, López D, Granada M, Bayés B, Romero R: 48. Roman B, Carta L, Martı́nez-González MA, Serra-Majem L: Effective-
Obesity, inflammation, and kidney disease. Kidney Int 74(Suppl 111): ness of the Mediterranean diet in the elderly. Clin Interv Aging 3:
S15–S18, 2008 97–109, 2008
34. Kurella M, Chertow GM, Fried LF, Cummings SR, Harris T, Simonsick 49. Brownlee M, Vlassara H, Kooney A, Ulrich P, Cerami A: Aminoguani-
E, Satterfield S, Ayonayon H, Yaffe K: Chronic kidney disease and dine prevents diabetes-induced arterial wall protein cross-linking. Sci-
cognitive impairment in the elderly: the health, aging, and body ence 232: 1629 –1632, 1986
composition study. J Am Soc Nephrol 16: 2127–2133, 2005* 50. Bolton WK, Cattran DC, Williams ME, Adler SG, Appel GB, Cartwright
35. Hailpern SM, Melamed ML, Cohen HW, Hostetter TH: Moderate K, Foiles PG, Freedman BI, Raskin P, Ratner RE, Spinowitz BS, Whittier
chronic kidney disease and cognitive function in adults 20 to 59 years FC, Wuerth JP; ACTION I Investigator Group: Randomized trial of an
of age: Third National Health and Nutrition Examination Survey inhibitor of formation of advanced glycation end products in diabetic
(NHANES III). J Am Soc Nephrol 18: 2205–2213, 2007 nephropathy. Am J Nephrol 24: 32– 40, 2004*
36. Kurella M, Chertow GM, Luan J, Yaffe K: Cognitive impairment in 51. Williams ME, Bolton WK, Khalifah RG, Degenhardt TP, Schotzinger RJ,
chronic kidney disease. J Am Geriatr Soc 52: 1863–1869, 2004 McGill JB: Effects of pyridoxamine in combined phase 2 studies of
37. Fazekas G, Fazekas F, Schmidt R, Kapeller P, Offenbacher H, Krejs GJ: patients with type 1 and type 2 diabetes and overt nephropathy. Am J
Brain MRI findings and cognitive impairment in patients undergoing Nephrol 27: 605– 614, 2007
chronic hemodialysis treatment. J Neurol Sci 134: 83– 88, 1995 52. Weindruch R: Aging in rodents fed restricted diets. J Am Geriatr Soc
38. Sehgal AR, Grey SF, DeOreo PB, Whitehouse PJ: Prevalence, recog- 33: 125–132, 1985

8 Geriatric Nephrology Curriculum American Society of Nephrology


REVIEW QUESTIONS: DECLINE OF RENAL b. Serum AGE levels are unaffected by dietary intake in dia-
FUNCTION IN NORMAL AGING AND ROLE OF betics with nephropathy
OXIDANTS/INFLAMMATION: WHEN DOES IT BEGIN c. AGE excretion in patients with diabetic nephropathy is
AND IS IT INEVITABLE, PREVENTABLE, OR increased diabetics with normal renal function compared
TREATABLE? with normal adults
d. AGE excretion in patients with diabetic nephropathy is
1. Which of the following are true? decreased diabetics with normal renal function, compared
a. CrCl remains normal with aging in most persons to normal adults
b. Most of the estimates of CrCl give the same results in aging e. AGE excretion in patients with diabetic nephropathy in
c. CrCl, on average, decreases with age diabetics is impaired only when eGFR is ⬍80 ml/min
d. CrCl may remain in the normal range in some individuals
5. Choose the correct statement from the following statements.
2. Which of the following statements about oxidants are true? a. Reducing AGEs in the diet has been shown to reduce
a. The levels of serum oxidants increases with age, on average markers of vascular injury in humans
b. The levels of oxidants vary among individuals depending b. The reduction of oxidants in humans has not proven to be
on dietary intake possible, at present
c. The levels of serum oxidants correlates with CrCl c. Reducing the levels of oxidants in the diet of patients with
d. The sources of serum oxidants includes the diet diabetic nephropathy rapidly results in normalization of
the levels of oxidants in the blood
3. Choose the two best answers to describe the effect (s) of de- d. Reducing the levels of oxidants in the diet of patients with
creasing oxidant intake in CKD patients. diabetic nephropathy results in normalization of the ex-
a. Decreasing oxidant intake in healthy adults has no influ- cretion of AGEs
ence on inflammation
b. Decreasing oxidant intake in healthy adults reduces in- 6. Which of the following statements are false.
flammation a. Anti-oxidant vitamins have been shown to reduce oxi-
c. Decreasing oxidant intake reduces the levels of RAGE dants in controlled clinical trials
(pro-oxidant), and increases the levels of AGER1 (anti- b. Drugs that control the formation of AGEs have been con-
oxidant) clusively shown to reduce diabetic nephropathy in con-
d. Decreasing oxidant intake in patients with diabetic ne- trolled clinical trials
phropathy reduces the levels of RAGE (pro-oxidant), and c. The “Mediterranean Diet” has been shown to be associ-
increases the levels of AGER1 (anti-oxidant). ated with reduced serum levels of oxidants and inflamma-
tion
4. Choose the two best answers from the three statements below. d. The content of oxidants in the diet, rather than the energy
a. Serum AGE levels are paralleled by urinary excretion of intake, has been shown to be critical to reducing organ
AGEs in normal adults damage in experimental studies

American Society of Nephrology Geriatric Nephrology Curriculum 9


Chapter 8: Kidney Disease in Elderly Diabetic Patients
Mark E. Williams and Robert C. Stanton
Renal Section, Joslin Diabetes Center, Boston, Massachusetts

Diabetes is a major health issue affecting the aging has better accuracy than the Cockroft formula in
US population: epidemiologic studies show an in- moderate and severe kidney function.9 Pathologi-
creased prevalence of diabetes with aging. The re- cally, the aging kidney may be associated with
ported incidence of diabetes in the elderly US pop- changes of basement membrane thickening and
ulation is at least 10 to 17%, caused by factors such mesangial expansion that are also key histologic
as obesity, decreased activity, insulin resistance, and features of diabetic glomerulopathy.1 Global glo-
increased oxidative tissue damage. The prevalence merulosclerosis affecting the kidneys of elderly per-
of metabolic syndrome also increases with age and sons may relate to hyperperfusion, also observed in
is frequently complicated by hypertension and diabetes. However, studies of the diagnosis and
chronic kidney disease (CKD). With further ad- prevalence of diabetic kidney disease in the elderly
vances in the treatment of diabetes, longer lifespan are lacking. Thus, there could be a higher preva-
is leading to more diabetes-related complications. lence of unusual presentations of diabetic kidney
Kidney disease secondary to diabetes has an in- disease (e.g., decreased GFR without albumin-
creased prevalence in the geriatric population,1 uria).10 Nondiabetic glomerular syndromes present
which comprises the fastest-growing subgroup of more commonly in geriatric patients because of
CKD and end-stage kidney disease (ESKD) in the conditions such as vasculitis, amyloidosis, parapro-
United States. About one third of older diabetic in- teinemia, membranous glomerulopathy, and anti-
dividuals have microalbuminuria,2 and an equal glomerular basement membrane (GBM) disease.
fraction have depressed kidney function. However, Another factor that needs to be considered in el-
CKD care of the elderly diabetic patient remains derly persons is the existence of renal artery stenosis
underemphasized, and nephrology consultation re- caused by atherosclerotic disease.
mains underused. Clinical guidelines for type 2 di-
abetes in the elderly do not address CKD, and
guidelines for diabetic CKD have not distinguished TREATMENT
age groups.3
The standard therapy of diabetic kidney disease is
the triad of blood glucose control, BP control, and
KIDNEY FUNCTION IN THE ELDERLY administration of angiotensin converting enzyme
DIABETIC PATIENT inhibitors (ACEIs) or angiotensin receptor blockers
(ARBs). The goals that have been established
Renal blood flow and GFR diminish over time in through many clinical studies are a hemoglobin A1c
elderly persons, minimized by a rise in the filtration of ⬍7%, a BP of ⬍130/80 (with weak data support-
fraction.4 In older diabetic patients, the decrease in ing a lower systolic goal if proteinuria persists), and
kidney mass, particularly from the renal cortex, and reduction of total urine protein to ⬍500 mg/g of
the histologic changes of diabetic nephropathy are creatinine or of urine albumin to ⬍300 mg/g of
compounded by advanced vascular changes.5 The creatinine. Although these goals have been vali-
term “concealed renal failure” has been applied to
elderly patients with normal serum creatinine but
decreasing GFR.6,7 The Modification of Diet in Re-
nal Disease (MDRD) is increasingly used in the Correspondence: Mark Williams, Renal Section, Joslin Diabetes
United States and has been found to be accurate in Center, One Joslin Place, Boston, MA 02215. Phone: 617-732-
diabetic kidney disease.8 A recent study of 160 dia- 2477; E-mail: [email protected]

betic patients reported that the MDRD equation Copyright 䊚 2009 by the American Society of Nephrology

American Society of Nephrology Geriatric Nephrology Curriculum 1


dated in a young to middle-aged population, they have not A principal outcome goal of these interventions is reduction
been tested in the elderly diabetic population. of proteinuria.18 –20 A recent study explored the association of
Management of blood glucose level has been shown to be microalbuminuria in patients with and without hypertension
of major importance in patients with type 1 and type 2 dia- and diabetes in a group that was 65 yr of age and older.21 The
betes mellitus in the prevention of complications. Recently results showed that there was a close correlation of microalbu-
the ADVANCE study11 showed that tight control of blood minuria and cardiovascular disease, inflammatory markers
glucose (glycated hemoglobin of 6.5 versus 7.3%) led to a (such as C-reactive protein), and systolic BP with increasing
21% reduction in the incidence of diabetic nephropathy age. A variety of other factors including anemia affects the care
over a 5-yr period. The average age of the participants of this of the elderly patient with diabetic nephropathy.22 There is a
study was 66 yr old. In addition, an update to the UKPDS danger of polypharmacy or confusion in the proper intake of
study on type 2 diabetic patients was recently published12 prescribed medications. There may be significant cost limita-
and showed that, although there was a loss of difference in tions for elderly patients because of fixed income or rules of
the glycated hemoglobin between the conventional and in- healthcare insurance coverage. As with ACEI/ARBs, there is an
tensive treatment group over time, there were still signifi- increased concern for side effects of medications. Thus, all of
cant benefits in cardiovascular outcomes and death rates for these factors should be taken into account when deciding on a
the original intensive treated group. Taken together, these particular treatment regimen. Unfortunately, epidemiologic
results showed that tight glucose control in diabetic patients studies suggest that elderly patients with CKD are not being
is important in preserving kidney function in an age range referred for specialty care. In a report of mostly male patients
(54 to 73 yr) that will be relevant to the geriatric population. with an average age of 66 yr, CKD, and diabetes over a 3-yr
BP control may be the most important factor in slowing period from 2000 to 2002,23 the authors estimated GFR using
progression of renal disease. For example, the Systolic Hyper- the MDRD formula. In the nearly 10,000 patients evaluated,
tension in the Elderly Program evaluated the effects of systolic almost one half of the patients had CKD. Of these, only 7.2%
hypertension, the most common pattern in elderly patients. A were referred to a nephrologist for care.
cohort of 2181 patients who were in the placebo arm of the
study was examined to see the relationship between BP and
decline in kidney function.13 In general, the systolic BP ranged END-STAGE KIDNEY DISEASE
from 160 to 200 and the diastolic BP from 70 to 90 mmHg. The
results showed that systolic hypertension (and not diastolic Although the number of elderly adults with diabetes has
hypertension) strongly correlated with declining renal function. increased dramatically over the past two decades, it is far
Because diabetic elderly patients are clearly at higher risk for sig- surpassed by their increase in ESKD, because of several fac-
nificant decreases in BP, physicians caring for patients with dia- tors including the willingness of providers to proceed with
betic kidney disease need to be aware of these to prescribe drugs renal replacement therapy. A recent observational study of
appropriately and to determine how low to push the BP. Of note, the US Renal Data System, for example, reported on octo-
a recently published follow-up of the UKPDS study on effects of genarians (78,419) and nonagenarians (5577) initiating di-
BP on any diabetes-related outcome was not maintained unless alysis between 1996 and 2003.24 There was an average an-
the decrease in BP was maintained chronically.14 nual increase of about 10% in dialysis initiation in these very
Third is the use of ACEIs and ARBs in diabetic elderly pa- elderly patients. The most recent USRDS report indicates
tients. These drugs slow progression of diabetic kidney disease that incident rates of patients reported with diabetic ESKD
in both type 1 and type 215,16 diabetic patients. The acknowl- and ⬎75 yr old almost doubled between 1996 and 2006, the
edged current standard of care is to start an ACE-I or ARB in most recent year with data available, and prevalence rates
any patient with microalbuminuria or overt proteinuria. How- have more than doubled.25
ever, a recent study showed that many elderly patients are not Little attention has been paid to this population in existing
being prescribed ACEI or ARBs,17 presumably because of con- guidelines. In elderly persons, mortality rates worsen with kid-
cerns over lack of benefit or untoward effects of ACE inhibitors ney disease more than in other groups. In the general US pop-
and ARBs such as hyperkalemia and decreased GFR. No study ulation, persons 75 to 79 yr of age have an expected remaining
has focused exclusively on the role of ACEI/ARBs in elderly life duration of 10.4 yr; for the elderly patient with ESKD, it is
persons, but considering their utility in other studies, some of 2.6 yr,26 and in the presence of diabetes, at least 25% less.
which included elderly patients, it is reasonable to prescribe Unique challenges include the high number of comorbid con-
them at this time pending further studies. Both ACE inhibitors ditions, including ischemic coronary disease, congestive heart
and ARBs will decrease GFR modestly, particularly initially. In failure, and peripheral vascular disease. Cardiovascular disease
general, if the drop is ⬍30% and the GFR subsequently re- develops in ⬎90% of elderly diabetic patients before starting
mains stable, therapeutic benefit is achieved. To monitor sig- dialysis. Cognitive and psychiatric disorders, malnutrition,
nificant changes in potassium or GFR, it is routine to check poor compliance, postdialysis hypotension, and dialysis access
potassium and serum creatinine 1 wk after starting or changing failure are common problems. ADA and other clinical practice
these medicines. guideline recommendations acknowledge differences in the

2 Geriatric Nephrology Curriculum American Society of Nephrology


risks and benefits of glycemic control among individuals. Be- survival in elderly patients transplanted compared with those
cause uncontrolled hyperglycemia produces fluid overload or on the waiting list were especially true for elderly diabetic pa-
other problems such as decreased cognition, nocturia, incon- tients (53% reduction).
tinence, and impaired cognition, treatment is indicated. How-
ever, geriatric patients with ESKD may be less likely to benefit
from long-term glycemic control in general and more likely to
TAKE HOME POINTS
suffer from hypoglycemia. The elderly patient is at higher risk
for drug-associated hypoglycemia. Nonetheless, monitoring • Diabetes is the most common etiology of CKD/ESKD in the elderly
glycemic control in diabetic ESKD remains far below recom- • Therapeutic trials of diabetic CKD/ESKD in elderly patients and guide-
line recommendations are insufficient
mended levels for hemoglobin A1c testing and prescription of • Optimal management of diabetic CKD/ESKD in elderly patients may
diabetic test strips.27 involve heightened risks and must take into account their increased
Annual ESKD mortality risk in the elderly population is mortality rates
almost 50% and is higher for very elderly persons. Survival • Increased number of elderly patients are being managed with dialysis,
data for elderly diabetic patients are seldom reported sepa- and some may benefit from a kidney transplant compared with remain-
ing on dialysis
rately.24 Regarding choice of dialysis modality, one clinical
study of USRDS patients from 1987 to 1989 comparing the
mortality with treatment assignment (hemodialysis or peri-
toneal dialysis) suggested that the higher mortality risk with DISCLOSURES
peritoneal dialysis (PD) patients was accentuated in diabetic None.
and older patients.28 Diabetic ESKD is the highest risk group
for cardiac death. Diabetes and ESKD are both precursors to
accelerated vascular calcification, involving coronary arter- REFERENCES
ies, and are associated with poorer outcomes from percuta-
neous coronary interventions and coronary bypass surgery. *Key References
Elderly diabetic patients with ESKD will also be at high risk 1. Zhou XJ, Rakheja D, Yu X, Saxena R, Vaziri ND, Silva FG: The aging
for operative management of left main or multivessel dis- kidney. Kidney Int 74: 710 –720, 2008*
2. Coresh J, Astor BC, Greene T, Eknoyan G, Levey AS: Prevalence of
ease, in which case medical therapy might be preferred.
chronic kidney disease and decreased kidney function in the adult US
ESKD costs for diabetic patients exceed those for nondia- population: Third National Health and Nutrition Examination Survey.
betic patients by 15 to 30%.25 Am J Kidney Dis 41: 1–12, 2003
3. Abaterusso C, Lupo A, Ortalda V, De Biase V, Pani A, Muggeo M,
Gambaro G: Treating elderly people with diabetes and stages 3 and
4 chronic kidney disease. Clin J Am Soc Nephrol 3: 1185–1194, 2008*
TRANSPLANTATION
4. Baid-Agrawal S, Reinke P, Schindler R, Tullius S, Frei U: WCN 2003
satellite symposium on kidney transplantation in the elderly, Weimar,
At a time when the maximum age of kidney patients consid- Germany, June 12–14, 2003. Nephrol Dial Transplant 19: 43– 46, 2004
ered for transplantation continues to increase, the growing 5. Adler S: Diabetic nephropathy: linking histology, cell biology, and
numbers of elderly diabetic patients with CKD/ESKD raises genetics. Kidney Int 66: 2095–2106, 2004
6. Corsonello A, Pedone C, Corica F, Mazzei B, Di Iorio A, Carbonin P,
significant questions regarding the appropriateness of kidney
Incalzi RA; Gruppo Italiano di Farmacovigilanza nell’Anziano (GIFA):
transplantation. The number of elderly patients on waiting lists Concealed renal failure and adverse drug reactions in older patients
for kidney transplantation began to increase in the 1990s, and with type 2 diabetes mellitus. J Gerontol A Biol Sci Med Sci 60:
transplant outcomes in elderly patients improved over a 15-yr 1147–1151, 2005
period.29 Concerns about limited life expectancy, high comor- 7. Kampmann J, Siersbaek-Nielsen K, Kristensen M, Hansen JM: Rapid
evaluation of creatinine clearance. Acta Med Scand 196: 517–520,
bidity rates, and relative organ shortages persist. The propor-
1974
tion of transplanted patients with diabetes is much lower 8. Lewis J, Agodoa L, Cheek D, Greene T, Middleton J, O’Connor D, Ojo
(23%) than the percent of patients on dialysis who have diabe- A, Phillips R, Sika M, Wright J Jr; African-American Study of Hyper-
tes. Both elderly patients and those with diabetic ESKD are tension and Kidney Disease: Comparison of cross-sectional renal func-
more likely to be considered for expanded kidney donor lists. tion measurements in African Americans with hypertensive nephro-
sclerosis and of primary formulas to estimate glomerular filtration rate.
Older recipients are more likely to die with a functioning graft,
Am J Kidney Dis 38: 744 –753, 2001
decreased initial function, and delayed graft function.30 A ret- 9. Rigalleau V, Lasseur C, Perlemoine C, Barthe N, Raffaitin C, Liu C,
rospective analysis of 5667 kidney transplant candidates ⱖ70 Chauveau P, Baillet-Blanco L, Beauvieux MC, Combe C, Gin H: Esti-
yr old using the Scientific Registry of Transplant Recipients mation of glomerular filtration rate in diabetic subjects: Cockcroft
database was recently reported.31 Between 1990 and 2004, 2078 formula or modification of diet in renal disease study equation? Dia-
betes Care 28: 838 – 843, 2005
received a deceased donor kidney transplant and 360 received a
10. Rosolowsky ET, Niewczas MA, Ficociello LH, Perkins BA, Warram JH,
living donor transplant. Mortality risk for those transplanted Krolewski AS: Between hyperfiltration and impairment: demystifying
were compared with candidates remaining on the transplant early renal functional changes in diabetic nephropathy. Diabetes Res
list. Findings of 41% lower long-term mortality risk and better Clin Pract 82(Suppl 1): 546 –553, 2008

American Society of Nephrology Geriatric Nephrology Curriculum 3


11. The ADVANCE Collaborative Group: Intensive control of blood glu- risk factors to microalbuminuria in older adults with or without diabe-
cose and vascular outcomes in patients with type 2 diabetes. N Engl tes mellitus or hypertension: the cardiovascular health study. Am J
J Med 358: 2560 –2572, 2008 Kidney Dis 44: 25–34, 2004
12. Holman RR, Paul SK, Bethel MA, Matthews DR, Neil HA: 10-year 22. Mohanram A, Zhang Z, Shahinfar S, Keane WF, Brenner BM, Toto RD:
follow-up of intensive glucose control in type 2 diabetes. N Engl Anemia and end-stage renal disease in patients with type 2 diabetes
J Med 359: 1577–1589, 2008 and nephropathy. Kidney Int 66: 1131–1138, 2004
13. Young JH, Klag MJ, Muntner P, Whyte JL, Pahor M, Coresh J: Blood 23. Patel UD, Young EW, Ojo AO, Hayward RA: CKD progression and
pressure and decline in kidney function: findings from the Systolic mortality among older patients with diabetes. Am J Kidney Dis 46:
Hypertension in the Elderly Program (SHEP). J Am Soc Nephrol 13: 406 – 414, 2005*
2776 –2782, 2002 24. Kurella M, Covinsky KE, Collins AJ, Chertow GM: Octogenarians and
14. Holman RR, Paul SK, Bethel MA, Neil HA, Matthews DR: Long-term nonagenarians starting dialysis in the United States. Ann Intern Med
follow-up after tight control of blood pressure in type 2 diabetes. 146: 177–183, 2007*
N Engl J Med 359: 1565–1576, 2008 25. US Renal Data Systems: USRDS 2008 Annual Data Report: Atlas of
15. Brenner BM, Cooper ME, de Zeeuw D, Keane WF, Mitch WE, Parving Chronic Kidney Disease and End-Stage Renal Disease in the United
HH, Remuzzi G, Snapinn SM, Zhang Z, Shahinfar S; RENAAL Study States. Bethesda, MD, National Institutes of Health, National Institute
Investigators: Effects of losartan on renal and cardiovascular outcomes of Diabetes and Digestive and Kidney Diseases, 2008
in patients with type 2 diabetes and nephropathy. N Engl J Med 345: 26. US Renal Data Systems: USRDS 2005 Annual Data Report. Bethesda,
861– 869, 2001 MD, National Institutes of Health, National Institute of Diabetes and
16. Lewis EJ, Hunsicker LG, Clarke WR, Berl T, Pohl MA, Lewis JB, Ritz E, Digestive and Kidney Diseases, 2005
Atkins RC, Rohde R, Raz I; Collaborative Study Group: Renoprotective 27. Williams M, Stanton R: Kidney dysfunction in older adults with diabe-
effect of the angiotensin-receptor antagonist irbesartan in patients tes. In: Geriatric Diabetes, edited by Munshi M, Lipsitz L, New York,
with nephropathy due to type 2 diabetes. N Engl J Med 345: 851– Informa Healthcare USA, 2007, pp 193–205*
860, 2001 28. Bloembergen WE, Port FK, Mauger EA, Wolfe RA: A comparison of
17. Winkelmayer WC, Glynn RJ, Levin R, Owen WF Jr, Avorn J: Determi- mortality between patients treated with hemodialysis and peritoneal
nants of delayed nephrologist referral in patients with chronic kidney dialysis. J Am Soc Nephrol 6: 177–183, 1995
disease. Am J Kidney Dis 38: 1178 –1184, 2001 29. Jager KJ, van Dijk PC, Dekker FW, Stengel B, Simpson K, Briggs JD;
18. Basi S, Lewis J: Microalbuminuria as a target to improve cardiovascular ERA-EDTA Registry Committee: The epidemic of aging in renal re-
and renal outcomes. Am J Kidney Dis 47: 927–946, 2006 placement therapy: an update on elderly patients and their outcomes.
19. Borch-Johnsen K, Feldt-Rasmussen B, Strandgaard S, Schroll M, Jensen Clin Nephrol 60: 352–360, 2003
JS: Urinary albumin excretion: an independent predictor of ischemic 30. Pirsch JD, Stratta RJ, Armbrust MJ, D’Alessandro AM, Sollinger HW,
heart disease. Arterioscler Thromb Vasc Biol 19: 1992–1997, 1996 Kalayoglu M, Belzer FO: Cadaveric renal transplantation with cyclo-
20. Miettinen H, Haffner SM, Lehto S, Rönnemaa T, Pyörälä K, Laakso M: sporine in patients more than 60 years of age. Transplantation 47:
Proteinuria predicts stroke and other atherosclerotic vascular disease 259 –261, 1989
events in nondiabetic and non–insulin-dependent diabetic subjects. 31. Rao Rao PS, Merion RM, Ashby VB, Port FK, Wolfe RA, Kayler LK:
Stroke 27: 2033–2039, 1996 Renal transplantation in elderly patients older than 70 years of age:
21. Barzilay JI, Peterson D, Cushman M, Heckbert SR, Cao JJ, Blaum C, results from the Scientific Registry of Transplant Recipients. Transplan-
Tracy RP, Klein R, Herrington DM: The relationship of cardiovascular tation 83: 1069, 2007

4 Geriatric Nephrology Curriculum American Society of Nephrology


REVIEW QUESTIONS: KIDNEY DISEASE IN ELDERLY c. Increase in nondiabetic glomerular syndromes
DIABETIC PATIENTS d. Increased incidence of renovascular disease
e. Reliance on validated measure of eGFR in the MDRD
1. The number of elderly diabetic patients with CKD/ESKD is equation
increasing because of all of the below EXCEPT
a. Increasing rates of metabolic syndrome patients 3. Data regarding therapy for diabetic CKD indicate that
b. Higher referral rates for specialty care a. Tight glycemic control is valuable and carries minimal risk
c. The overall risk of CKD increased with age b. Mild initial loss of GFR does not require cessation of ACEI
d. Age-related loss of kidney function therapy
c. Diastolic BP control is more important than systolic con-
2. Evaluation of the elderly diabetic patient with kidney disease trol
must take into consideration all EXCEPT d. ACEI/ARBs are overused in this population
a. Histologic changes of diabetic glomerulopathy and aging e. There should be minimal concern about medication side
b. Progression may occur independent of albuminuria effects

American Society of Nephrology Geriatric Nephrology Curriculum 5


Chapter 9: Drug Dosing and Renal Toxicity in the
Elderly Patient
Ali J. Olyaei* and William M. Bennett†
*Division of Nephrology and Hypertension, Oregon Health and Science University, Portland, Oregon; and

Northwest Renal Clinic, Legacy Good Samaritan Hospital Transplant Services, Portland, Oregon

Aging is a natural process of human development day (Figure 3). These numbers are significantly
and is characterized by a progressive loss of physio- higher for patients with CKD, including those un-
logic and reproductive functions.1,2 Despite signif- dergoing kidney dialysis. In the elderly, new-onset
icant advances in the fields of human physiology, adverse drug reactions are commonly mistaken by
pharmacology, and pathology, as well as medical healthcare providers as a new-onset disease or mor-
and clinical interventions, aging continues to be a bidity related to aging.13 Approximately 15 to 45%
significant risk factor and strong predictor of mor- of older adults develop moderate to severe forms of
bidity and mortality. memory impairment from medication use.14 This
Geriatric patients, while accounting for 15% of cognitive impairment may position older adults at a
the population, use 30% of all prescription drugs higher risk of overdose or nonadherence compared
prescribed in the United States. By 2020, ⬎50 mil- with the rest of the population.15 In addition, older
lion people will be over 65 yr of age and account for adults with kidney disease are cared for by a variety
25% of the US population (Figure 1). It is estimated of healthcare providers such as nephrologists, car-
that by 2030, the population of individuals over 65 diologists, general practitioners, and pharmacists
yr of age will increase by four-fold, and this age and may lack good continuity of care. Medication
group is the fastest growing segment of the US pop- reconciliation is still a major problem in this era of
ulation. This increase will add to healthcare costs the electronic medical record. Failure to disclose a
and strengthen the economic tsunami that our complete list of current medications taken to each
country is facing today.3,4 provider because of cognitive impairment may
Chronic kidney disease (CKD) is a common and have a profound impact on potential risk of drug–
progressive condition that continues to rise in the drug interactions or disease– drug interactions.16
United States.5–7 Limited data exist regarding drug The overall incidence of adverse drug reactions
use in elderly patients with CKD. Proven therapies is three- to ten-fold higher in older adults with kid-
are often underused in the geriatric population. For ney disease compared with those without CKD.17
example, recent data indicated that aggressive man- The incidence of adverse drug reactions correlates
agement of hypertension may be beneficial in older exponentially with renal function.18 Most drugs
patients.8,9 However, most physicians hesitate to and/or their metabolites are excreted renally
treat older patients more aggressively. This is, in through glomerular filtration. The overall size,
part, because of a high risk of drug– drug interac- mass, and effective area of filtration decreases with
tions, adverse drug reactions, and lack of clinical increasing age.19,20 These morphologic changes in-
data in this population. Monitoring the medica- crease the risk for drug and/or active metabolite ac-
tions used in older adults and identifying drug in- cumulation in older patients with kidney disease.
teractions and adverse events are crucial. Drug ther- After the age of 50, the number of nephrons pro-
apy management in older adults is challenging, and gressively declines from approximately ⬎1,000,000
many factors related to normal aging, disease states,
and lifestyle should be considered before initiation Correspondence: Ali J. Olyaei, PharmD, BCPS, Associate Profes-
of pharmacotherapy (Figure 2).10,11 sor of Medicine, Division of Nephrology and Hypertension, Ore-
In a recent study, Qato et al.12 documented that gon Health Sciences University, 3314 SW US Veterans Hospital
Road, Portland, OR 97201. Phone: 503-494-8007; Fax: 503-494-
91% of older adults regularly use one prescription 2994; E-mail: [email protected]
drug and ⬎50% use five or more prescriptions per
Copyright 䊚 2009 by the American Society of Nephrology

American Society of Nephrology Geriatric Nephrology Curriculum 1


Altered Pharmacokinetics

Age-related

Drug-related Disease-related

Figure 1. Estimated population of individuals (%) over 65 yr of


age by 2100 in the United States.
Figure 3. Altered pharmacokinetics.

to ⬍500,000. In addition, up to 35% of nephrons show clini-


cally important evidence of sclerosis.21 cardiovascular disease as one ages. Increased renal nerve activ-
Drug dosing in CKD has been reviewed extensively in other ity, increased production and release of angiotensin II, de-
review papers.17,18 This chapter will examine how aging and creased local prostaglandins, and increased release of endothe-
physiologic changes secondary to aging effect drug elimination lin may contribute to vasoconstriction and decreased renal
and provide general dosing guidelines. However, it is very im- blood flow during later stages of life.23,24 A higher incidence of
portant to mention that very few drugs and clinical interven- vascular disease, hypertension, diabetes, and smoking and a
tions have been well studied in older adults, particularly those high protein diet are common among older adults in industri-
with CKD. Older adults are in general underrepresented in alized nations and may further distress renal function. In the
most clinical studies. Most studies exclude patients over 65 yr Unites States, approximately 65% of older adults have a diag-
of age, and few studies allow the inclusion of nursing home nosis of hypertension.25 Hypertensive nephropathy accounts
patients. for 25% of all ESRD that requires dialysis. These pathologic
In an important study, Lindeman et al.22 showed that a de- processes independent of aging alter drug absorption, protein
crease in renal GFR occurs during the aging process but much binding, volume of distribution, and elimination.26
less than previously perceived. These alterations in renal func- The GFR is closely correlated with renal drug elimination
tion are largely clinically insignificant and have a limited effect and is useful in determining dosage adjustments.27 The age-
on the life of adult patients without comorbid conditions. dependent alterations to renal anatomy and physiology in
Many investigators have discovered small artery fibrosis and older adults make the kidneys more susceptible to environ-
decreased renal blood flow after four decades of life. Decreased mental and pathologic nephrotoxins. To avoid toxicity in older
renal blood flow can happen independent of cardiac output or patients with kidney disease, the drug dosage should be ad-

Prevalence of Polypharmacy for Prescription Medications


69%
57-64 19%
2%
82%
Men 65-74 28%
6%
88%
75-85 36%
4%

81%
57-64 27%
5%
87%
Women 65-74 33%
5%
92%
75-85 37%
4%

0 .2 .4 .6 .8 1
Figure 2. Percent of polypharmacy at different age
1+ 5+ 10+
groups: ⬎1 drug, ⬎5 drugs, and ⬎10 drugs.

2 American Society of Nephrology American Society of Nephrology


Table 1. Agents to be avoided or used with caution in patients with CKD
Class Examples
Antibiotics Aminoglycosides, vancomycin, sulfamethoxazole
Antifungals Amphotericin B
Antivirals Foscarnet, indinavir, cidofovir
Anticoagulants Low molecular weight heparins, warfarin
Cardiac drugs Digoxin, sotalol, ACE-I, ARB, DRIs
Opioids Morphine, meperidine, prophoxyphene
Psychotropics/anticonvulsants Amisulpride, gabapentin, lithium, levetiracetam, topiramate, vigabatrin
Hypoglycemic drugs Metformin, glyburide, insulin
Drugs for gout Allopurinol, colchicine
Others Methotrexate, penicillamine, nonsteroidal anti-inflammatory drugs
ACE-I, angiotensin-converting enzyme; ARB, angiotensin II receptor blockers; DRI, direct renin inhibitor.

justed according to estimated creatinine clearance. There are a both prescription and nonprescription, should be obtained to
number of methods to estimate creatinine clearance; however, identify potential nephrotoxins or interacting medications. A
the most commonly used method is the Cockcroft-Gault thorough medication history should be obtained to identify
method. There are a number of limitations when using the drug allergies or intolerances and previous adverse drug reac-
Cockcroft-Gault method in older patients. The production tions. Body mass index (BMI) and ideal body weight (IBW)
and elimination of creatinine decreases with age.26 This may should be calculated using the following formulas: BMI ⫽
overestimate renal function and mask the early stage of renal weight in kilograms divided by height in meters squared; IB-
dysfunction. In older adults, the use of Modification of Diet in W(men) ⫽ 50.0 kg ⫹ 2.3 kg for every 2.5 cm over 152 cm; and
Renal Disease (MDRD) may provide a better estimate of renal IBW(women) ⫽ 45.5 kg ⫹ 2.3 kg for every 2.5 cm over 152 cm.
function. Some drugs may increase the metabolic load by in- Volume status, both intracellular and extracellular, should
creasing creatinine production and/or urea production (e.g., be assessed frequently. Shifts in extracellular fluid volume may
glucocorticoids and androgens). Some agent may also interfere change the volume of distribution of many drugs. Patients with
with creatinine tubular secretion (e.g., cimetidine and tri- dehydration have a higher predisposition to drug toxicity. In
methoprim). Ketosis, hyperbilirubinemia, and some cephalo- older adults, total body volume decreases by 10 to 15%. Muscle
sporins may influence the measurement of plasma creatinine atrophy, reduced tissue perfusion, and increase in fat content
and cause renal function assessment inaccurate when serum change the volume of distribution of most hydrophilic agents.
creatinine is used. The plasma concentration of drugs with a narrow therapeutic
Considering these limitations, estimating renal function is window and small volume of distribution (aminoglycosides,
difficult in the elderly population. However, the use of the lithium) may alter considerably with any change in extracellu-
Cockcroft-Gault method is the safest and most effective ap- lar fluids. Coexisting hepatic dysfunction may alter protein
proach for dosage adjustments in patients with renal impair- binding, volume of distribution, and intravascular volume and
ment. necessitate further dosage modification.28

Step 2: Renal Function Assessment


APPROACH TO DOSAGE ADJUSTMENT IN OLDER The Cockcroft-Gault formula is a simple and widely used
ADULTS method to estimate GFR. A 24-h creatinine clearance (Clcr) is
an approximation of GFR, and it is practical and useful for
Most guidelines recommend drug dosing adjustments in older estimating renal function in drug dosage modification in pa-
adults with or without renal disease. The following recommen- tients with stable renal dysfunction. In patients with acute kid-
dations provide a simple approach for healthcare providers ney injury, rising serum creatinine, and low urine output, cre-
and attempt to reduce the risk of drug toxicity and improve atinine clearance should be assumed to be generally ⬍10 ml/
pharmacotherapeutic efficacy. However, patient-specific fac- min. The Cockcroft-Gault formula includes the variables of
tors, comorbid condition, drug interactions, and healthcare age (yr), IBW (kg), and serum creatinine (Scr) (mg/dl) and
insurance coverage should be considered and monitored dur- calculates the Clcr (ml/min): Clcr ⫽ (140 ⫺ age) ⫻ IBW/72 ⫻
ing therapy.19 Scr (the result should be multiplied by 0.85 for women).
Use of Iohexol is a new method to estimate GFR but is
Step 1: Medical History and Physical Examination currently being used only in the clinical research setting. Io-
A complete medical history should be obtained, and a physical hexol provides a more accurate measurement of renal function
exam should be performed. The etiology and duration of renal without exposing the patient to radiolabeled material. Finally,
dysfunction should be determined and defined as acute or the MDRD study recently reported a new formula to estimate
chronic whenever possible. A review of current medications, renal function. This method provides a more accurate estima-

American Society of Nephrology American Society of Nephrology 3


Table 2. Therapeutic drug monitoring
Drug Name Therapeutic Range When to Draw Sample How Often to Draw Levels
Aminoglycosides Gentamicin and Trough: immediately before Check peak and trough with third dose
(conventional dosing) tobramycin: dose
Trough: 0.5–2 mg/L Peak: 30 min after a 30- to For therapy less than 72 h, levels not necessary.
45-min infusion Repeat drug levels weekly or if renal function
changes
Gentamicin, tobramycin, Peak: 5–8 mg/L
amikacin Amikacin:
Peak: 20–30 mg/L
Trough: ⬍10 mg/L
Aminoglycosides (24-h 0.5–3 mg/L Obtain random drug level After initial dose. Repeat drug level in 1 wk or if
dosing) gentamicin, 12 h after dose renal function changes
tobramycin, amikacin
Carbamazepine 4–12 ␮g/ml Trough: immediately before Check 2–4 days after first dose or change in
dosing dose
Cyclosporin 150–400 ng/ml Trough: immediately before Daily for first week and then weekly.
dosing
Digoxin 0.8–2.0 ng/ml 12 h after maintenance 5–7 d after first dose for patients with normal
dose renal and hepatic function; 15–20 d in
anephric patients
Lidocaine 1–5 ␮g/ml 8 h after intravenous
infusion started or
changed
Lithium Acute: 0.8–1.2 Trough: before a.m. dose
mmol/L at least 12 h since last
dose
Chronic: 0.6–0.8
mmol/L
Phenobarbital 15–40 ␮g/ml Trough: immediately before Check 2 wk after first dose or change in dose.
dosing Follow-up level in 1–2 mo
Phenytoin 10–20 ␮g/ml Trough: immediately before 5–7 d after first dose or after change in dose
dosing
Free phenytoin 1–2 ␮g/ml
Procainamide NAPA (n- 4–10 ␮g/ml Trough: immediately before
acetyl procainamide) a next dose or 12–18 h
procainamide metabolite after starting or changing
an infusion
Trough: 4 ␮g/ml Draw with procainamide
sample
Peak: 8 ␮g/ml
10–30 ␮g/ml
Quinidine 1–5 ␮g/ml Trough: immediately before
next dose
Sirolimus 10–20 ng/dl Trough: immediately before
next dose
Tacrolimus (FK-506) 10–15 ng/ml Trough: immediately before Daily for first week and then weekly.
next dose
Theophylline p.o. or 15–20 ␮g/ml Trough: immediately before
Aminophylline i.v. next dose
Valproic acid (divalproex 40–100 ␮g/ml Trough: immediately before Check 2–4 d after first dose or change in dose
sodium) next dose
Vancomycin Trough: 5–15 mg/L Trough: immediately before With third dose (when initially starting therapy,
dose or after each dosage adjustment). For therapy
⬍72 h, levels not necessary. Repeat drug
levels if renal function changes
Peak: 25–40 mg/L Peak: 60 min after a 60-min
infusion

4 American Society of Nephrology American Society of Nephrology


tion of renal function in patients with low muscle mass, high portant to know the exact dose given, the route of administra-
protein intake, or geriatric populations and is subject to less tion, time of administration, and time since the last dose. Peak
artifact interference. In dialysis patients, with a residual renal levels are meaningful for only few drugs. Peak drug levels rep-
function, the residual renal function may to a large extent con- resent the highest drug concentration achieved after initial
tribute to the elimination of drugs and their active metabolites. rapid distribution. For most drugs, trough levels are obtained
The effect of residual renal function on drug elimination in immediately before the next dose, represent the lowest serum
dialysis patients with urine output ⬍500 ml/d is very difficult concentration, and predict drug toxicity. For most drugs,
to estimate. Residual renal function decreases over time and is TDM is not available and could be very costly. TDM should be
usually ⬍5 ml/min after 1 yr on hemodialysis. used only for drugs in which plasma concentrations correlate
with toxicity or efficacy. It is important to mention that, even
Step 3: Loading Dose Determination with TDM, drug toxicities can not always be avoided. For ex-
For most drugs, the loading dose should not be adjusted. In ample, aminoglycoside antibiotics can accumulate in tissues
patients with normal renal function, steady-state drug concen- such as the inner ear and renal tubules. Aminoglycoside toxic-
tration is reached after approximately five half-lives. The half- ity can occur after a single dose or in some cases without asso-
life of drugs that are excreted renally may be significantly pro- ciated high plasma concentrations. Clinical practitioners must
longed in patients with CKD. To rapidly achieve a therapeutic incorporate ongoing clinical assessment and TDM simulta-
plasma concentration, a loading dose should be administered. neously. Assessment of adverse drug reactions is important
However, when the volume of distribution is altered secondary even when drug levels are within the established therapeutic
to CKD, a smaller loading dose may be required for some range. Finally, in older adults with kidney disease, protein
agents. For example, in dialysis patients, the loading dose of binding is altered significantly. For highly protein-bound
digoxin should be reduced by 25 to 50% to avoid toxicity. drugs, the free fraction can be elevated significantly, whereas
Although routine plasma concentration monitoring of digoxin the total plasma concentration is within therapeutic range.
is not necessary in patients with normal renal function, in di- Most assays do not distinguish between free and protein-
alysis patients and patients with a history of hypokalemia, the bound drug in the plasma. An increase in unbound drug is
plasma concentration should be monitored very closely.29 The common in patients with renal failure. For example, a dialysis
loading dose can be calculated as follows to achieve a desirable patient with albumin of ⬍2 g/dl and phenytoin level of 6 mg/L
plasma concentration: LD ⫽ Vd (L/kg) ⫻ IBW (kg) ⫻ [Cp], could experience phenytoin toxicities while phenytoin levels
where Cp is the desired plasma concentration (mg/L). are within the therapeutic range (therapeutic level: 10 to 20
mg/L). Free phenytoin levels provide better therapeutic drug
Step 4: Maintenance Dose Determination monitoring in older patients with renal impairment.31,32 Table
The maintenance of drugs that are primarily eliminated un- 2 summarizes the therapeutic drug monitoring parameters in
changed through the kidneys should be modified in patients renal insufficiency for drugs in which monitoring of levels is
with CKD. Dosage modification in older adults with kidney routinely recommended.
disease can be accomplished by dose reduction, dosing interval
prolongation, or both methods. For drugs whose clinical effi-
cacy correlates with adequate peak concentrations (aminogly- TAKE HOME POINTS
cosides, cephalosporins), the dosing interval should be ad-
• Review patients past medical history and medication profiles for any
justed. For agents whose efficacy correlates with area under of possible drug– drug interactions
the curve or a rapid rise in plasma concentration correlate with • For GFR ⬍50 ml/min, renally excreted drugs should be adjusted ac-
toxicity, the dosage interval should be prolonged. In general, cording to the renal function
for most drugs, a combined approach using both the dose re- • Dosage modification can be accomplished by dose reduction, dosing
interval prolongation, or both methods
duction and interval prolongation methods is often used. This
• If needed, consider therapeutic drug monitoring (TDM) in older pa-
method provides a constant plasma drug concentration with- tients with renal impairment
out increasing the risk of toxicity from high peak or trough
levels.30 Finally some medications should be avoided or be
used with caution in patients with CKD (Table 1).
DISCLOSURES
None.
Step 5: Drug Level Monitoring
Despite dosage adjustments and prolongation dosage interval,
drug toxicity is a common problem in older adults with CKD.
REFERENCES
Because of inter- and intraindividual pharmacokinetic vari-
abilities, comorbid conditions, and drug interactions, thera-
*Key References
peutic drug monitoring (TDM) is important in older patients 1. Vellai T: Autophagy genes and ageing. Cell Death Differ 16: 94 –102,
with renal impairment. The diagnostic value of TDM relies on 2009
correct interpretation of plasma drug concentrations. It is im- 2. Garinis GA, van der Horst GT, Vijg J, Hoeijmakers JH: DNA damage

American Society of Nephrology American Society of Nephrology 5


and ageing: new-age ideas for an age-old problem. Nat Cell Biol 10: 18. Cantu TG, Ellerbeck EF, Yun SW, Castine SD, Kornhauser DM: Drug
1241–1247, 2008 prescribing for patients with changing renal function. Am J Hosp
3. Culberson JW, Ziska M: Prescription drug misuse/abuse in the elderly. Pharm 49: 2944 –2948, 1992*
Geriatrics 63: 22–31, 2008 19. Turnheim K: Pharmacokinetic dosage guidelines for elderly subjects.
4. Le Couteur DG, Kendig H: Pharmaco-epistemology for the prescrib- Expert Opin Drug Metab Toxicol 1: 33– 48, 2005*
ing geriatrician. Australas J Ageing 27: 3–7, 2008 20. Turnheim K: When drug therapy gets old: pharmacokinetics and phar-
5. Saran AM, DuBose TD Jr: Cardiovascular disease in chronic kidney macodynamics in the elderly. Exp Gerontol 38: 843– 853, 2003
disease. Ther Adv Cardiovasc Dis 2: 425– 434, 2008 21. McLean AJ, Le Couteur DG: Aging biology and geriatric clinical phar-
6. Hossain MP, Goyder EC, Rigby JE, El Nahas M: CKD and poverty: a macology. Pharmacol Rev 56: 163–184, 2004*
growing global challenge. Am J Kidney Dis 53: 166 –174, 2009 22. Lindeman RD, Tobin J, Shock NW: Longitudinal studies on the rate
7. Abdelhafiz AH, Tan E, El Nahas M: The epidemic challenge of chronic of decline in renal function with age. J Am Geriatr Soc 33: 278 –285,
kidney disease in older patients. Postgrad Med 120: 87–94, 2008* 1985
8. McDonald M, Hertz RP, Unger AN, Lustik MB: Prevalence, awareness, 23. Zhou XJ, Rakheja D, Yu X, Saxena R, Vaziri ND, Silva FG: The aging
and management of hypertension, dyslipidemia, and diabetes among kidney. Kidney Int 74: 710 –720, 2008
United States adults aged 65 and older. J Gerontol A Biol Sci Med Sci 24. Tedla FM, Friedman EA. The trend toward geriatric nephrology. Prim
64: 256 –263, 2009 Care 35: 515–530, 2008
9. Aronow WS: Treatment of hypertension in the elderly. Compr Ther 34: 25. Burt VL, Cutler JA, Higgins M, Horan MJ, Labarthe D, Whelton P,
171–176, 2008 Brown C, Roccella EJ: Trends in the prevalence, awareness, treatment,
10. Ferrario CG: Geropharmacology: A primer for advanced practice. and control of hypertension in the adult US population. Data from the
Acute care and critical care nurses, part II. AACN Adv Crit Care 19: health examination surveys, 1960 to 1991. Hypertension 26: 60 – 69,
134 –149, 2008 1995
11. Ferrario CG: Geropharmacology: A primer for advanced practice 26. Odden MC, Shlipak MG, Tager IB: Serum creatinine and functional
acute care and critical care nurses, part I. AACN Adv Crit Care 19: limitation in elderly persons. J Gerontol A Biol Sci Med Sci 64: 370 –
23–35, 2008 376, 2009
12. Qato DM, Alexander GC, Conti RM, Johnson M, Schumm P, Lindau 27. Munar MY, Singh H: Drug dosing adjustments in patients with chronic
ST: Use of prescription and over-the-counter medications and dietary kidney disease. Am Fam Physician 75: 1487–1496, 2007
supplements among older adults in the United States. JAMA 300: 28. El Desoky ES: Pharmacokinetic-pharmacodynamic crisis in the elderly.
2867–2878, 2008 Am J Ther 14: 488 – 498, 2007
13. Snowden A: Medication management in older adults: a critique of 29. Vivo RP, Krim SR, Perez J, Inklab M, Tenner T Jr, Hodgson J: Digoxin:
concordance. Br J Nurs 17: 114 –119, 2008* current use and approach to toxicity. Am J Med Sci 336: 423– 428,
14. Gray SL, Lai KV, Larson EB: Drug-induced cognition disorders in the 2008
elderly: incidence, prevention and management. Drug Saf 21: 101–122, 30. Pollock B, Forsyth C, Bies R: The critical role of clinical pharmacology
1999 in geriatric psychopharmacology. Clin Pharmacol Ther 85: 89 –93,
15. Bushardt RL, Massey EB, Simpson TW, Ariail JC, Simpson KN: Poly- 2009
pharmacy: misleading, but manageable. Clin Interv Aging 3: 383–389, 31. Hope OA, Zeber JE, Kressin NR, Bokhour BG, Vancott AC, Cramer JA,
2008 Amuan ME, Knoefel JE, Pugh MJ: New-onset geriatric epilepsy care:
16. Perhats C, Valdez AM, St Mars T: Promoting safer medication use race, setting of diagnosis, and choice of antiepileptic drug. Epilepsia
among older adults. J Emerg Nurs 34: 156 –158, 2008* 57: 237–242, 2009
17. Muhlberg W, Platt D: Age-dependent changes of the kidneys: phar- 32. Leppik IE: Choosing an antiepileptic. Selecting drugs for older pa-
macological implications. Gerontology 45: 243–253, 1999* tients with epilepsy. Geriatrics 60: 42– 47, 2005

6 American Society of Nephrology American Society of Nephrology


REVIEW QUESTIONS: DRUG DOSING AND RENAL a. Pharmacokinetic and pharmacodynamic properties of the drugs
TOXICITY IN THE ELDERLY PATIENT b. Intended therapeutic indication
c. Both a and b
1. Of the following, which would be of important consideration d. Neither a nor b
in assessing renal function in older patients?
a. Serum creatinine 3. The elderly are subject to a greater risk of ADR and DDIs
b. Weight and muscle mass because:
c. Urine output a. Comorbidity
d. All of the above b. Alter pharmacokinetics
c. Seen by a number of health care providers
2. Drug–drug interaction occurs as a result of: d. All of the above

American Society of Nephrology American Society of Nephrology 7


Chapter 10: Glomerular Disease in the Elderly
Christine K. Abrass
Department of Medicine, Division of Gerontology and Geriatric Medicine, University of Washington School of
Medicine, Seattle, Washington

APPROACH TO AND UNIQUE FEATURES of the benefit of diagnosis and appropriate inter-
OF GLOMERULONEPHRITIS IN ELDERLY vention.
PERSONS

Recognition, diagnosis, and management of glo- GENDER-, ETHNIC-, AND AGE-RELATED


merular disease in elderly persons have several DISPARITIES IN HEALTHCARE
unique challenges. Reductions in GFR are common
in individuals over the age of 75. Because this so Studies continue to show that underrepresented
often occurs in the setting of aging nephropathy, minorities, women, and elderly persons are pro-
hypertension, or vascular disease, other intrinsic vided standard care and included in clinical trials at
kidney diseases are not considered in the differen- rates below those for white men6,7; however, these
tial diagnosis. Aging nephropathy is characterized same groups have higher rates of certain diseases
by loss of GFR of approximately 1.7 ml/min per and worse outcomes. This relatively poorer prog-
year1; thus, renal function seems to be stable over a nosis argues for a more aggressive approach. Given
several year window of time. However, this can rep- that women make up a steadily higher proportion
resent a significant loss of renal function over the of the elderly population with each year of increas-
final 30 yr of life. Within a short time frame, any ing age, disparities in approach to their care is even
change in renal function should indicate the possi- more relevant in this age group. With any given
bility of an alternative diagnosis. When prior mea- disease, women lose renal function at a slower rate
sures of renal function are not available, estimation than men until after menopause when this benefit is
of GFR using the Cockcroft-Gault formula or an- lost. Recognition of changes in risk factors and in-
other formula that takes into account age and an cidence of specific forms of GN throughout the
assumed serum creatinine of 0.8 mg/dl will allow lifespan is critical to resolving health disparities.
calculation of a rough estimate of age-adjusted
GFR. Should the measured eGFR be substantially
below that predicted on the basis of age alone, other URINARY PROTEIN EXCRETION IN
diagnoses should be considered. Using analysis of ELDERLY PERSONS
large databases, O’Hare et al.2,3 have shown that ag-
ing nephropathy per se is not usually associated with Standard teaching often taught that urinary protein
proteinuria. Because many of these individuals excretion increases with age. Because these con-
have not been subjected to renal biopsy, the actual cepts were developed based on studies of subjects
pathology is unknown; however, some have argued living in nursing homes, it was thought that these
that the absence of proteinuria excludes significant data reflected disease, often urinary tract infection,
pathology. Based on these arguments, the presence and thus were thought not to be relevant to the
of proteinuria or active urinary sediment should be elderly population at large. Subsequent studies8
considered as evidence of renal disease other than
age-associated reduction in GFR,4,5 and the possi-
bility of glomerulonephritis (GN) should be con- Correspondence: Christine K. Abrass, MD, Professor of Medi-
cine, Division of Gerontology and Geriatric Medicine, University
sidered. Despite these caveats, urinary abnormali- of Washington School of Medicine, UW Medicine Lake Union, 815
ties and reductions in GFR are frequently Mercer Street, Seattle, WA 98109. Phone: 206-897-1966; Fax:
erroneously attributed to aging per se and not pur- 206-897-1300; Email: [email protected]
sued further, thus depriving the elderly individual Copyright 䊚 2009 by the American Society of Nephrology

American Society of Nephrology Geriatric Nephrology Curriculum 1


showed that healthy elderly subjects generally do not have pro- elderly subjects show a low rate of complications and iden-
teinuria; however, in those who do, there is significant loss of tification of treatable forms of kidney disease; however,
renal function over 5 yr of follow-up. Recent studies have rates of biopsy in elderly individuals continue to be low.
shown that rates of albuminuria in community-dwelling el- This raises the possibility that more biopsies should be done
derly subjects increase with age and may reach 37% in individ- in elderly persons. Indications for renal biopsy are shown in
uals over the age of 80.9 As in other age groups, the presence of Table 1.
albuminuria shows increased risk for dementia,9 hypertension,
cardiovascular disease, and progressive renal disease. Based on
these findings, proteinuria in elderly individuals should be
viewed as an indicator of renal disease and a predictor of other POSTINFECTIOUS GLOMERULONEPHRITIS
disorders that have implications for diagnosis, therapy, and
outcomes. These findings are somewhat difficult to reconcile The most common organism associated with acute prolifera-
with epidemiologic database studies such as those reported by tive glomerulonephritis is Group A streptococcus. This entity
O’Hare et al., which indicate that elderly individuals with GFR is uncommon in adults, and in general, its incidence is declin-
⬍60 ml/min usually do not have proteinuria. Additional stud- ing in the developed world. A recent review13 of 86 cases
ies are needed to clarify the nature and natural history of aging showed a mean age of 56 yr, with 34% being over the age of 64.
nephropathy per se; but all studies indicate that, when protein- Unlike what occurs in children, as many as 38% had an under-
uria is present, specific diagnoses are usually identified on bi- lying disorder associated with immunocompromise. Com-
opsy and outcomes are poorer. plete remission occurred in slightly more than one half of all
patients and was less likely in those with pre-existing renal
disease or illnesses associated with immunocompromise.
RENAL BIOPSY IN ELDERLY PERSONS Among individuals with pre-existing diabetic glomeruloscle-
rosis, remission was rare and rapid progression to end-stage
Despite the frequency of urinary abnormalities and reduc- kidney disease (ESKD) often followed. Thus, outcomes seem
tions in GFR among elderly individuals, only 15% of renal to be worse in elderly persons.
biopsies are from individuals over the age of 65 and even No studies have addressed the incidence or outcomes in
fewer are in individuals over the age of 75. As a result, very older individuals with postinfectious GN caused by organisms
few data are available that inform our understanding of GN other than Group A streptococcus. Given the frequency of
in elderly individuals.5 The limited number of biopsies in pneumococcal pneumonia and other infections in elderly in-
the face of high rates of proteinuria suggests that many older dividuals, infections that are associated with postinfectious GN
individuals with kidney disease are not provided a specific less often in younger individuals may contribute a higher pro-
diagnosis or offered disease-specific treatments to slow the portion in elderly persons.
rate of progression. It is well documented that inflamma-
tory symptomatology and thus clinical manifestations are
blunted in elderly persons, which leads to atypical presen- LUPUS NEPHRITIS
tations even for common diseases. This also can contribute
to missed or delayed diagnosis in this age group. Nephrotic Lupus is generally a disease of women of child-bearing age;
syndrome is often misdiagnosed as congestive heart failure. however, 10 to 20% occurs in older individuals including those
As elderly individuals with nephrotic syndrome had focal over the age of 65. Arthritis, fever, serositis, sicca symptoms,
sclerosis (FSGS; 23%), minimal change disease (MCD; Raynaud’s syndrome, lung disease, and neuropsychiatric
19%), and membranous nephropathy (MN; 15%), thera- symptoms are more common in elderly patients, whereas ma-
peutic intervention would be expected to modify their out- lar rash, discoid lupus, and GN are less common. Serologic
comes. Among those individuals with an acute nephritic manifestations include ANA, anti-Ro, and anti-La, whereas
syndrome, most had pauci-immune, MPO-ANCA positive, anti-DNA is less common. When lupus with or without ne-
crescentic GN, whereas the remainder had acute interstitial phritis occurs in elderly individuals, diagnosis is often delayed.
nephritis.10 One study focused on individuals over the age of When present, lupus nephritis should be treated the same as in
80 (3% of the biopsies in their series).11 Similar to the pre- younger subjects.14
vious study, with advanced age, the majority of individuals
had crescentic GN, whereas only 15% had MN. IgA ne-
phropathy and lupus nephritis were uncommon in all se- Table 1. Indications for consideration of renal biopsy in
ries.12 Even in the very old, 40% had a treatable entity iden- the elderly
tified, and in the remainder, prognostic information was GFR reduced to a degree greater than predicted for age alone
attained and potentially harmful empiric therapy was Acute change in GFR
avoided. Elderly persons who are treated respond similarly Proteinuria
to younger individuals. All published studies of biopsy in Active urinary sediment

2 Geriatric Nephrology Curriculum American Society of Nephrology


MINIMAL CHANGE DISEASE to improved outcomes. Delays in diagnosis and initiation of
therapy are uniformly associated with poor outcomes; thus,
As many as 15 to 20% of older individuals with nephrotic syn- recognition of this cause of acute change in renal function in
drome will have MCD.10,11 Their response to treatment is com- contrast to other forms of acute kidney injury is essential. In all
parable to younger age groups; thus, they should be treated. studies reported, this form of crescentic GN is the only one that
Risk for development of diabetes and infection is somewhat is relatively unique to elderly persons. Many studies have doc-
greater in elderly than in younger individuals, but untreated umented changes in immune function with age, particularly in
individuals of all ages have a high rate of infection-associated innate immunity17; however, these alone have not provided
death. Among adults with nephrotic syndrome, MCD disease insights into the unique risk that elderly individuals have for
occurs more often in the elderly than younger adults, and in developing this form of GN.
this age group, it is often misdiagnosed as congestive heart
failure.
MANAGEMENT OF NEPHROTIC SYNDROME IN
ELDERLY PERSONS
FOCAL AND SEGMENTAL GLOMERULOSCLEROSIS
The most important first step in the management of nephrotic
Current evidence shows that this pathologic pattern is a heter- syndrome in elderly persons is to recognize that edema, short-
ogeneous group of diseases, and focal areas of scarring can ness of breath, and cardiomegaly are caused by nephrotic syn-
occur in a variety of diseases including aging nephropathy. The drome rather than congestive heart failure. Once the etiology
idiopathic form of FSGS typically presents with proteinuria of nephrotic syndrome has been defined, this should be the
and reduced GFR in young adults, and it is likely caused by a primary guide for therapy.18 Older individuals may be sensitive
variety of gene mutations. This form of GN is relatively un- to diuretics and develop prerenal azotemia; thus, they require
common in elderly persons and is not thought to result from cautious dosing and careful follow-up.
pre-existing gene mutations; however, this has not been con-
clusively examined.
USE OF IMMUNOSUPPRESSIVE AGENTS IN ELDERLY
PERSONS
MEMBRANOUS NEPHROPATHY
With aging, fat as a proportion of lean body mass increases,
In adults of all ages, membranous nephropathy is the most and hepatic clearance and renal function diminish. Each of
common cause of nephrotic syndrome; however, the incidence these factors influences drug pharmacokinetics; thus, drug
is greatest in individuals 40 to 60 yr of age. After age 60, mul- doses may need to be adjusted in elderly persons. Although
tiple myeloma and amyloidosis become much more common there are no data specifically looking at outcomes or modified
than in younger age groups, and thus, make up 15 to 20% of therapies in elderly individuals per se, most treatment studies
nephrotic syndrome in older individuals. Because MCD is sim- indicate that elderly persons fair favorably compared with
ilarly more common in elderly than younger adults, membra- younger age groups in frequency of response rates and compli-
nous nephropathy accounts for only 15% of elderly individuals cations. Based on limited data, recommendations are to select
with nephrotic syndrome. Treatment approaches and out- and tailor therapy in the elderly using the same criteria as for
comes are similar in older and younger individuals.15 Although younger individuals. Preserving renal function even when the
the incidence of cancer increases with age, there is no evidence risk of progression to ESKD is not a concern will facilitate
that MN and cancer are linked in elderly persons other than by treatment of and reduce side effects and complications of all
coincidence. other health related issues in the elderly.

CRESCENTIC GLOMERULONEPHRITIS CONCLUSIONS

Pauci-immune, MPO-ANCA positive, crescentic glomerulo- Although GN is relatively uncommon in elderly individuals, it
nephritis is the most common form of GN in elderly persons, does occur. Because treatment response rates and complica-
and its incidence steadily increases with increasing age. Un- tions are comparable for individuals in all age groups, elderly
treated, this form of GN progresses rapidly to ESKD. Differ- persons will benefit from diagnosis and treatment using the
ences in the forms of vasculitis and implications for therapy same criteria for biopsy and intervention as in younger indi-
were recently reviewed by Jennette and Falk.16 Individuals with viduals. This recommendation is particularly justified given
this disease can have a significant and sustained remission with that ANCA-positive, pauci-immune, crescentic GN is the most
appropriate therapy; thus, recognition of this disorder in el- common form of GN in elderly individuals, and left untreated,
derly individuals, with prompt biopsy and treatment, is critical this disease rapidly leads to ESKD. Furthermore, following

American Society of Nephrology Geriatric Nephrology Curriculum 3


progression to ESKD, outcomes in elderly persons are quite efeld CS: Age affects outcomes in chronic kidney disease. J Am Soc
Nephrol 18: 2758 –2765, 2007
poor; thus, interventions that slow the rate of progression are 4. Abrass CK: Glomerulonephritis in the elderly. Am J Nephrol 5: 409 –
of particular benefit. To improve outcomes in elderly persons, 418, 1985
more studies that include elderly subjects are needed. To 5. Abrass CK. Renal biopsy in the elderly. Am J Kidney Dis 35: 544 –546,
achieve this goal, biases that exclude elderly subjects from stan- 2000*
dard care and research protocols need to be modified. 6. Wesson DE: Is the ethnic disparity in CKD a symptom of dysfunctional
primary care in the US? J Am Soc Nephrol 19: 1249 –1251, 2008
7. Norris K, Nissenson AR: Race, gender, and socioeconomic dispar-
ities in CKD in the United States. J Am Soc Nephrol 19: 1261–1270,
TAKE HOME POINTS 2008
8. Landahl S, Aurell M, Jagenburg R: Glomerular filtration rate at the age
• Aging nephropathy contributes to increased risk for cardiovascular
of 70 and 75. J Clin Exp Gerontol 3: 29 – 45, 1981
disease and frailty
9. Barzilay JI, Fitzpatrick AL, Luchsinger J, Yasar S, Bernick C, Jenny NS,
• Proteinuria and sudden changes in GFR indicate the presence of kidney
Kuller LH: Albuminuria and dementia in the elderly: a community
disease
study. Am J Kidney Dis 52: 216 –226, 2008
• Elderly persons should be evaluated and treated using the same criteria
10. Uezono S, Hara S, Sato Y, Komatsu H, Ikeda N, Shimao Y, Hayashi T,
as in younger individuals
Asada Y, Fujimoto S, Eto T: Renal biopsy in elderly patients: a clini-
• Pauci-immune, crescentic GN is a disease of elderly persons
copathological analysis. Ren Fail 28: 549 –555, 2006
11. Nair R, Bell JM, Walker PD: Renal biopsy in patients aged 80 years and
older. Am J Kidney Dis 44: 618 – 626,2004*
DISCLOSURES 12. Shin JH, Pyo HJ, Kwon YJ, Chang MK, Kim HK, Won NH, Lee HS, Oh
None. KH, Ahn C, Kim S, Lee JS: Renal biopsy in elderly patients: clinico-
pathological correlation in 117 Korean patients. Clin Nephrol 56:
19 –26, 2001
13. Nasr SH, Markowitz GS, Stokes MB, Said SM, Valeri AM, D’Agati VD:
REFERENCES Acute postinfectious glomerulonephritis in the modern era: experi-
ence with 86 adults and review of the literature. Medicine 87: 21–32,
*Key References 2008*
1. Hemmelgarn BR, Zhang J, Manns BJ, Tonelli M, Larsen E, Ghali WA, 14. Lazaro D: Elderly-onset systemic lupus erythematosus: prevelance,
Southern DA, McLaughlin K, Mortis G, Culleton BF: Progression of clinical course and treatment. Drugs Aging 24: 701–715, 2007
kidney dysfunction in the community-dwelling elderly. Kidney Int 69: 15. Abrass CK: Treatment of membranous nephropathy in the elderly.
2155–2161, 2006* Semin Nephrol 23: 373–378, 2003*
2. O’Hare AM, Bertenthal D, Covinsky KE, Landefeld CS, Sen S, Mehta K, 16. Jennette JC, Falk RJ: Nosology of primary vasculitis. Cur Opin Rheu-
Steinman MA, Borzecki A, Walter LC: Mortality risk stratification in matol 19: 10 –16, 2007*
chronic kidney disease: one size for all ages? J Am Soc Nephrol 17: 17. Gomez CR, Nomellini V, Faunce DE, Kovacs EJ: Innate immunity and
846 – 853, 2006 aging. Exp Gerontol 43: 718 –728, 2008
3. O’Hare AM, Choi AI, Bertenthal D, Bacchetti P, Garg AX, Kaufman JS, 18. Abrass CK: Clinical spectrum and complications of the nephrotic
Walter LC, Mehta KM, Steinman MA, Allon M, McClellan WM, Land- syndrome. J Investig Med 45: 143–153, 1997

4 Geriatric Nephrology Curriculum American Society of Nephrology


REVIEW QUESTIONS: GLOMERULAR DISEASE IN a. Minimal change disease
THE ELDERLY b. Membranous nephropathy
c. Lupus nephritis
1A.An 85-yr-old man is seen with complaints of joint pains caused
d. Amyloidosis
by osteoarthritis. Before starting an NSAID, you measure his
e. Focal and segmental glomerulosclerosis
creatinine. It is 1.2 mg/dl. His estimated GFR is:
a. 100 ml/min 4A.A female presents complaining of increasing fatigue, slight an-
b. 75 ml/min kle edema, headaches, dark colored urine, and morning nau-
c. 45 ml/min sea of 3-wk duration. Urinalysis reveals 2⫹ protein, 3⫹ blood,
d. 25 ml/min granular, and red cell casts. SCr is 2.6 mg/dl. If she is 8 yr old,
1B. Is his eGFR equal to or different from what you expect would which of the following is the most likely diagnosis?
based on his age alone? a. Pauci-immune crescentic glomerulonephritis
a. Equal to b. Postinfectious glomerulonephritis
b. Better than c. Lupus nephritis
c. Worse than d. Acute cystitis
e. Membranous nephropathy
2. Proteinuria is a normal consequence of the aging process?
a. True 4B. If she is 24 yr old, which of the following is the most likely
b. False diagnosis?
a. Pauci-immune crescentic glomerulonephritis
3A.A male person presents with a 4-wk history of progressive an-
b. Postinfectious glomerulonephritis
kle and periorbital edema. Urinalysis reveals 4⫹ protein and
c. Lupus nephritis
no blood. SCr is 1.2 mg/dl. Which of the following is the most
d. Acute cystitis
likely diagnosis if the individual is 10 yr old?
e. Membranous nephropathy
a. Minimal change disease
b. Membranous nephropathy 4C.If she is 83 yr old, which of the following is the most likely
c. Lupus nephritis diagnosis?
d. Amyloidosis a. Pauci-immune crescentic glomerulonephritis
e. Focal and segmental glomerulosclerosis b. Postinfectious glomerulonephritis
3B. Which of the following is the most likely diagnosis if the indi- c. Lupus nephritis
vidual is 45 yr old? d. Acute cystitis
a. Minimal change disease e. Membranous nephropathy
b. Membranous nephropathy
5. Treatment responses in individuals with glomerulonephritis
c. Lupus nephritis
are dependent on diagnosis and therapy. These principles of
d. Amyloidosis
treatment and the response rates are comparable in elderly and
e. Focal and segmental glomerulosclerosis
younger individuals.
3C.Which of the following is the most likely diagnosis if the indi- a. True
vidual is 80 yr old? b. False

American Society of Nephrology Geriatric Nephrology Curriculum 5


Chapter 11: Hypertension, Chronic Kidney Disease,
and the Elderly
Ann M. O’Hare
Department of Medicine, University of Washington and Staff Physician, VA Puget Sound Healthcare System, Seattle,
Washington

The major goals of lowering BP in patients with to lower than usual BP targets is not associated with
chronic kidney disease (CKD) include reduction of an increased risk of adverse outcomes.
mortality, cardiovascular events, and slowing pro-
gression. Key considerations in the management of
hypertension include selection of a target BP and BP TARGETS IN THE ELDERLY
selection of agents used to attain the chosen target.
This chapter outlines key considerations in apply- In applying this recommendation to the elderly, it is
ing current guidelines for the management of BP to worth noting that none of the trials used to support
older patients with CKD. the safety of lower than usual BP targets in patients
with CKD enrolled any participants older than 75
(Table 1). Thus, the safety of treating to a lower than
BP TARGETS usual BP level in older patients with CKD is not
known. Indeed, in the very elderly (e.g., 85 yr or
older), observational data showed that there is a J-
Many clinical practice guidelines recommend a
shaped relationship between BP and survival and
lower than usual BP target for patients with CKD.
that optimal BP may be higher than in younger peo-
For example, the Kidney Disease Outcomes Quality
ple.2,3 Although a number of trials have specifically
Initiative (KDOQI) recommends a target BP of
examined the effect of BP lowering on nonrenal
⬍130/80 mmHg for all patients with CKD, which is
outcomes in the elderly, and in many instances have
defined as an eGFR ⬍60 ml/min per 1.73 m2 or
shown a benefit, these trials have tended to target a
“kidney damage” (specified as microalbuminuria
higher than usual (rather than lower than usual or
or macroalbuminuria for patients with diabetes).
even usual) BP.2,4 –7
Citing KDOQI, the seventh report from the Joint
In pursuing a lower than usual BP target in an
National Committee on Prevention, Detection,
older person, it is important to assess the impor-
Evaluation, and Treatment of High Blood Pressure
tance to that patient of the implicit goals of this
(JNC 7) also recommends a target BP of ⬍130/80
intervention (e.g., slowing progression of CKD and
mmHg for all patients with CKD defined as an
reducing cardiovascular risk) as well as the poten-
eGFR ⬍60 ml/min per 1.73 m2 or protein-to-creat-
tial harms. For a variety of reasons, it is likely that
inine ratio ⱖ200 mg/g.1 A target BP ⬍130/80
for many patients who meet criteria for CKD, the
mmHg is also recommended by the American Dia-
risk-to-benefit ratio of BP lowering may differ from
betes Association (ADA) and by JNC 7 for all pa-
that in younger patients. First, CKD in the elderly is
tients with diabetes.
often slowly progressive or nonprogressive, and the
Despite the consistency of guideline recommen-
risk of progression to end-stage kidney disease
dations for lower than usual BP targets in patients
(ESKD) is lower for older than for younger patients
with CKD, these recommendations are based on
opinion rather than the results of randomized con-
trolled trials. Few trials have shown that treatment Correspondence: Ann M. O’Hare, MA, MD, Division of Nephrol-
ogy, VA/Puget Sound Medical Center, Nephrology and Renal
to lower than usual BP targets slows progression of Dialysis Unit, Building 100, Room 5B113, 1660 S. Columbian Way,
CKD or reduces other clinically significant out- Seattle, WA 98108. Phone: 206-277-3192; Fax: 206-764-2022;
comes in patients with CKD (Table 1). On the other E-mail: [email protected]

hand, available evidence also suggests that treating Copyright 䊚 2009 by the American Society of Nephrology

American Society of Nephrology Geriatric Nephrology Curriculum 1


Table 1. Trials examining the effect of blood pressure lowering on progression of CKD by diabetic status
Blood Pressure Subgroup or Secondary Age Inclusion Mean
Outcome
Targets Analyses Criteria (yr) Age (yr)
Diabetes
Estacio et al.23 and DBP ⬍ 75 versus No difference in 40–74 59.5
Schrier et al.24 80–89 progression
Lewis et al.25 MAP ⬍ 92 versus No difference in 18–40 37
100–107 progression
UKPDS26 ⬍150/85 versus No difference in Reduced risk of 25–65 56
⬍180/105 progression microalbuminuria at 6 yr
No diabetes
Ruggenenti et al.27 DBP ⬍ 90 versus No difference in 18–70 54
130/80 progression
Wright et al.28 MAP 102–107 versus No difference in Trend toward greater benefit in 18–70 54.6
ⱕ92 (125/75) progression patients with proteinuria
Klahr et al.29* MAP ⱕ107 versus ⬍92 No difference in Secondary analysis showed 18–70 52
progression greater benefit in patients
with greater degrees of
proteinuria
*The MDRD study had different blood pressure targets based on age and required a MAP ⱕ 107 (equivalent to 140/90) for patients 18 to 60 yr of age and
MAP ⱕ 113 for patients ⱖ61 yr of age (equivalent to 160/90) or low blood pressure ⬍92 (125/75) for patients 18 to 60 yr of age and MAP ⱕ 98 mmHg
(equivalent to 145/75) for patients ⱖ61 yr of age.
DBP, diastolic blood pressure; MAP, mean arterial pressure.

with similar levels of eGFR.8,9 Thus, there may be less to be nician must consider each individual patient’s likelihood of
gained from slowing progression if this is slow to begin with. It experiencing progressive loss of renal function and mortality
is also not clear that mortality risk for most elderly patients in relation to their age peers and in the context of their risk for
with a low eGFR is any higher than for their age peers with a other (perhaps competing) health outcomes and their risk for
“normal” eGFR.10,11 The majority of older patients who meet adverse events as a result of BP lowering.
criteria for CKD have very moderate reductions in eGFR (e.g.,
45 to 59 ml/min per 1.73 m2). For many of these patients, the
relative and absolute risk of death may be no greater than for CHOICE OF AGENTS
patients of the same age whose eGFR falls in the normal
range.10,11 Thus, it is not even clear that reducing mortality risk In addition to lowering BP, progression of CKD can probably
provides a compelling rationale for a lower than usual BP in also be slowed by reducing proteinuria.14 For this reason, an-
many older patients with an eGFR ⬍60 ml/min per 1.73 m2. giotensin-converting enzyme inhibitor (ACEI) and angioten-
Second, it is not clear that slowing progression of CKD is al- sin II receptor antagonists (ARBs) are considered first-line
ways the most meaningful goal of anti-hypertensive therapy in agents for patients with CKD in a number of different clinical
older patients with a low eGFR.9,12Their risk for other out- practice guidelines. KDOQI recommends that these agents be
comes such as cardiovascular events, disability, and cognitive prescribed for patients with diabetic CKD (defined as an eGFR
insufficiency is often much higher than that for ESKD. CKD in ⬍60 ml/min per 1.73 m2 or micro- or macroalbuminuria) and
the elderly rarely occurs in the absence of other comorbid con- for those with nondiabetic proteinuric CKD, even in the ab-
ditions.9,13 The presence of multiple comorbid conditions in sence of hypertension.1,15,16 Based on recommendations from
older patients with CKD may complicate the management of KDOQI and a subset of trials referenced in KDOQI, JNC 7
CKD by creating potentially conflicting or competing treat- identifies CKD, defined as an eGFR ⬍60 ml/min per 1.73 m2 or
ment goals. Third, the potential harms of BP lowering may be a protein-to-creatinine ratio ⱖ200 mg/g, as a compelling indi-
greater in the elderly. Most elderly patients with CKD have cation for the use of ACEI or ARB.
isolated systolic hypertension. Thus, theoretically, treatment
of their systolic hypertension may have the unintended effect
of lowering diastolic pressure to suboptimal levels, leading to CHOICE OF AGENTS IN THE ELDERLY
impaired perfusion during diastole. Orthostatic hypotension is
also more common in the elderly and may be aggravated by In applying these guidelines to the management of older pa-
treatment to lower than usual BP targets. Finally, elderly pa- tients with CKD, it is important to note that many of the key
tients, particularly those who are frail, may be more likely to studies supporting these recommendations did not include
experience injury as a result of an episode of hypotension. participants older than 70.17–19 Nevertheless, a subgroup anal-
In summary, in deciding whether to target a lower than ysis among participants older than 65 enrolled in the RENAAL
usual BP in older patients who meet criteria for CKD, the cli- trial, a trial among type II diabetics with macroalbuminuria,20

2 Geriatric Nephrology Curriculum American Society of Nephrology


Losartan was similarly renoprotective in these older partici- with CKD, because of the complexity of medical decision mak-
pants as it was in the overall study population, suggesting that ing in the elderly, age differences in the clinical implications of
this agent is equally efficacious in elderly patients with albu- CKD, and the high burden of other comorbidities in older
minuria. Like RENAAL, most trials of ACEI and ARB in pa- patients with CKD, strict adherence to guidelines for the man-
tients with CKD that are cited in contemporary guidelines ex- agement of hypertension may not always represent the most
plicitly or implicitly selected for proteinuria. However, the trial patient-centric approach.
that enrolled by far the largest number of older participants
with CKD (the Antihypertensive and Lipid Lowering to Pre-
vent Heart Attack or ALLHAT) did not select for proteinuria TAKE HOME POINTS
(because level of urinary protein was not ascertained at base-
• Be aware that there is little evidence to support current guidelines for
line or follow-up).21 Participants in ALLHAT with an eGFR
the management of BP in chronic kidney disease in the elderly because
⬍60 ml/min per 1.73 m2 had a mean age of 70.8 yr and had a most trials did not include participants older than 70
similar risk of ESKD regardless of whether they received an • Realize that older patients with a low eGFR often have a variety of other
ACEI, thiazide diuretic, or calcium channel blocker. This was comorbidities and competing treatment goals and that optimization of
true both overall and for the subset with diabetes. This study BP, particularly if the primary goal is slowing progression of CKD, may
not always be the most important treatment priority for an individual
certainly raises the question of whether ACEI confer greater
patient
protection against renal outcomes in elderly patients with
CKD unselected for proteinuria.
Furthermore, similar principles apply to choice of agent as ap-
ply to BP targets in the management of hypertension in elderly DISCLOSURES
Dr. O’Hare receives research support from the NIA and the CDC. She re-
patients with a low eGFR. Use of ACEI and ARB is recommended
ceives royalties from UpToDate.
for the management of hypertension in CKD based on the ability
of these agents to slow progression, which for reasons outlined
above, may not always represent the most patient-centric goal of
therapy. In older patients who meet criteria for CKD, the goals of REFERENCES
anti-hypertensive therapy in many instances should perhaps be
*Key References
directed at other outcomes such as cardiovascular events, cogni-
1. Chobanian AV, Bakris GL, Black HR, Cushman WC, Green LA, Izzo JL
tive insufficiency, disability, and mortality if these pose a greater Jr, Jones DW, Materson BJ, Oparil S, Wright JT Jr, Roccella EJ;
risk to the patient than progression of kidney disease. Changing National Heart, Lung, and Blood Institute Joint National Committee
the goals of anti-hypertensive therapy may in some instances per- on Prevention, Detection, Evaluation, and Treatment of High Blood
mit greater flexibility in choice of anti-hypertensive regimen.22 Pressure; National High Blood Pressure Education Program Coordi-
nating Committee: The Seventh Report of the Joint National Commit-
The potential burden of using an ACEI or ARB must also be fac-
tee on Prevention. Detection, evaluation, and treatment of high blood
tored into the decision as to whether to prescribe these in prefer- pressure: the JNC 7 report. JAMA 289: 2560 –2572, 2003
ence to other agents. Prescription of ACEI and ARB in patients 2. Boshuizen HC, Izaks GJ, van Buuren S, Ligthart GJ: Blood pressure
with CKD mandates careful monitoring for acute renal failure and mortality in elderly people aged 85 and older: community based
and hyperkalemia often requiring extra laboratory testing and study. BMJ 316: 1780 –1784, 1998
3. Mattila K, Haavisto M, Rajala S, Heikinheimo R: Blood pressure and
clinic visits after initiation of these agents and after any change in
five year survival in the very old. Br Med J 296: 887– 889, 1988
dose. In addition, administration of these agents in patients with 4. Prevention of stroke by antihypertensive drug treatment in older per-
CKD often requires dietary modification and chronic administra- sons with isolated systolic hypertension. Final results of the Systolic
tion of ion-exchange resins, and can also limit the use other med- Hypertension in the Elderly Program (SHEP). SHEP Cooperative Re-
ications that also raise serum potassium (e.g., spironolactone).15 search Group. JAMA 265: 3255–3264, 1991*
5. Beckett NS, Peters R, Fletcher AE, Staessen JA, Liu L, Dumitrascu D,
Thus, in deciding whether to treat an older patient with CKD with
Stoyanovsky V, Antikainen RL, Nikitin Y, Anderson C, Belhani A, For-
an ACEI or ARB to slow progression of CKD, the clinician should ette F, Rajkumar C, Thijs L, Banya W, Bulpitt CJ; HYVET Study Group:
consider whether the patient has proteinuria, whether their CKD Treatment of hypertension in patients 80 years of age or older. New
is clearly progressive, whether they have other health concerns or Engl J Med 358: 1887–1898, 2008*
priorities that might make another anti-hypertensive agent pref- 6. Staessen JA, Fagard R, Thijs L, Celis H, Arabidze GG, Birkenhäger WH,
Bulpitt CJ, de Leeuw PW, Dollery CT, Fletcher AE, Forette F, Leonetti
erable, and whether the additional burden that these agents may
G, Nachev C, O’Brien ET, Rosenfeld J, Rodicio JL, Tuomilehto J,
impose are justified and acceptable to the patient. Zanchetti A: Randomised double-blind comparison of placebo and
active treatment for older patients with isolated systolic hypertension.
The Systolic Hypertension in Europe (Syst-Eur) Trial Investigators.
CONCLUSION Lancet 350: 757–764, 1997
7 Dahlof B, Lindholm LH, Hansson L, Schersten B, Ekbom T, Wester PO:
Morbidity and mortality in the Swedish Trial in Old Patients with
Current guidelines for BP targets and choice of anti-hyperten- Hypertension (STOP-Hypertension). Lancet 338: 1281–1285, 1991*
sive agents in patients with CKD are age neutral. Although 8. Hemmelgarn BR, Zhang J, Manns BJ, Tonelli M, Larsen E, Ghali WA,
these guidelines may be appropriate for many older patients Southern DA, McLaughlin K, Mortis G, Culleton BF: Progression of

American Society of Nephrology Geriatric Nephrology Curriculum 3


kidney dysfunction in the community-dwelling elderly. Kidney Int 69: 20. Winkelmayer WC, Zhang Z, Shahinfar S, Cooper ME, Avorn J, Brenner
2155–2161, 2006* BM: Efficacy and safety of angiotensin II receptor blockade in elderly
9. O’Hare AM, Choi AI, Bertenthal D, Bacchetti P, Garg AX, Kaufman JS, patients with diabetes. Diabetes Care 29: 2210 –2217, 2006*
Walter LC, Mehta KM, Steinman MA, Allon M, McClellan WM, Land- 21. Rahman M, Pressel S, Davis BR, Nwachuku C, Wright JT Jr, Whelton
efeld CS: Age affects outcomes in chronic kidney disease. J Am Soc PK, Barzilay J, Batuman V, Eckfeldt JH, Farber M, Henriquez M, Kopyt
Nephrol 18: 2758 –2765, 2007 N, Louis GT, Saklayen M, Stanford C, Walworth C, Ward H, Wiegmann
10. O’Hare O’Hare AM, Bertenthal D, Covinsky KE, Landefeld CS, Sen S, T: Renal outcomes in high-risk hypertensive patients treated with an
Mehta K, Steinman MA, Borzecki A, Walter LC: Mortality risk stratifi- angiotensin-converting enzyme inhibitor or a calcium channel blocker
cation in chronic kidney disease: one size for all ages? J Am Soc vs a diuretic: a report from the Antihypertensive and Lipid-Lowering
Nephrol 17: 846 – 853, 2006 Treatment to Prevent Heart Attack Trial (ALLHAT). Arch Intern Med
11. Raymond NT, Zehnder D, Smith SC, Stinson JA, Lehnert H, Higgins 165: 936 –946, 2005*
RM: Elevated relative mortality risk with mild-to-moderate chronic 22. Blood Pressure Lowering Treatment Trialists’ Collaboration, Turnbull
kidney disease decreases with age. Nephrol Dial Transplant 22: 3214 – F, Neal B, Ninomiya T, Algert C, Arima H, Barzi F, Bulpitt C, Chalmers
3220, 2007 J, Fagard R, Gleason A, Heritier S, Li N, Perkovic V, Woodward M,
12. Eriksen BO, Ingebretsen OC: In chronic kidney disease staging the use MacMahon S: Effects of different regimens to lower blood pressure on
of the chronicity criterion affects prognosis and the rate of progres- major cardiovascular events in older and younger adults: meta-analy-
sion. Kidney Int 72: 1242–1248, 2007 sis of randomised trials. BMJ 336: 1121–1123, 2008
13. Gullion CM, Keith DS, Nichols GA, Smith DH: Impact of comorbidities 23. Estacio RO, Jeffers BW, Hiatt WR, Biggerstaff SL, Gifford N, Schrier
on mortality in managed care patients with CKD. Am J Kidney Dis 48: RW: The effect of nisoldipine as compared with enalapril on cardio-
212–220, 2006 vascular outcomes in patients with non-insulin-dependent diabetes
14. Bakris GL: Slowing nephropathy progression: focus on proteinuria and hypertension. New Engl J Med 338: 645– 652, 1998
reduction. Clin J Am Soc Nephrol 3(Suppl 1): S3–S10, 2008 24. Schrier RW, Estacio RO, Esler A, Mehler P: Effects of aggressive blood
15. K/DOQI: Clinical practice guidelines on hypertension and antihyper- pressure control in normotensive type 2 diabetic patients on albumin-
tensive agents in chronic kidney disease. Am J Kidney Dis 43: S1– uria, retinopathy and strokes. Kidney Int 61: 1086 –1097, 2002
S290, 2004 25. Lewis JB, Berl T, Bain RP, Rohde RD, Lewis EJ: Effect of intensive
16. KDOQI: Clinical practice guidelines and clinical practice recommen- blood pressure control on the course of type 1 diabetic nephropathy.
dations for diabetes and chronic kidney disease. Am J Kidney Dis 49: Collaborative Study Group. Am J Kidney Dis 34: 809 – 817, 1999
S12–154, 2007 26. UKPDS: Tight blood pressure control and risk of macrovascular and
17. Agodoa LY, Appel L, Bakris GL, Beck G, Bourgoignie J, Briggs JP, microvascular complications in type 2 diabetes: UKPDS 38. UK Pro-
Charleston J, Cheek D, Cleveland W, Douglas JG, Douglas M, Dowie spective Diabetes Study Group. BMJ 317: 703–713, 1998
D, Faulkner M, Gabriel A, Gassman J, Greene T, Hall Y, Hebert L, 27. Ruggenenti P, Perna A, Loriga G, Ganeva M, Ene-Iordache B, Turturro
Hiremath L, Jamerson K, Johnson CJ, Kopple J, Kusek J, Lash J, Lea M, Lesti M, Perticucci E, Chakarski IN, Leonardis D, Garini G, Sessa A,
J, Lewis JB, Lipkowitz M, Massry S, Middleton J, Miller ER 3rd, Norris Basile C, Alpa M, Scanziani R, Sorba G, Zoccali C, Remuzzi G; REIN-2
K, O’Connor D, Ojo A, Phillips RA, Pogue V, Rahman M, Randall OS, Study Group: Blood-pressure control for renoprotection in patients
Rostand S, Schulman G, Smith W, Thornley-Brown D, Tisher CC, Toto with non-diabetic chronic renal disease (REIN-2): multicentre, random-
RD, Wright JT Jr, Xu S; African American Study of Kidney Disease and ised controlled trial. Lancet 365: 939 –946, 2005
Hypertension (AASK) Study Group: Effect of ramipril vs amlodipine on 28. Wright JT Jr, Bakris G, Greene T, Agodoa LY, Appel LJ, Charleston J,
renal outcomes in hypertensive nephrosclerosis: a randomized con- Cheek D, Douglas-Baltimore JG, Gassman J, Glassock R, Hebert L,
trolled trial. JAMA 285: 2719 –2728, 2001 Jamerson K, Lewis J, Phillips RA, Toto RD, Middleton JP, Rostand SG;
18. Lewis EJ, Hunsicker LG, Bain RP, Rohde RD: The effect of angiotensin- African American Study of Kidney Disease and Hypertension Study
converting-enzyme inhibition on diabetic nephropathy. The Collabo- Group: Effect of blood pressure lowering and antihypertensive drug
rative Study Group. New Engl J Med 329: 1456 –1462, 1993 class on progression of hypertensive kidney disease: results from the
19. Lewis EJ, Hunsicker LG, Clarke WR, Berl T, Pohl MA, Lewis JB, Ritz E, AASK trial. JAMA 288: 2421–2431, 2002
Atkins RC, Rohde R, Raz I; Collaborative Study Group: Renoprotective 29. Klahr S, Levey AS, Beck GJ, Caggiula AW, Hunsicker L, Kusek JW, Striker
effect of the angiotensin-receptor antagonist irbesartan in patients G: The effects of dietary protein restriction and blood-pressure control on
with nephropathy due to type 2 diabetes. New Engl J Med 345: the progression of chronic renal disease. Modification of Diet in Renal
851– 860, 2001 Disease Study Group. New Engl J Med 330: 877– 884, 1994

4 Geriatric Nephrology Curriculum American Society of Nephrology


REVIEW QUESTIONS: HYPERTENSION, CHRONIC 60. His serum potassium is 4.5 mEq/L and his urine sediment
KIDNEY DISEASE, AND THE ELDERLY is bland. He has no other comorbidities and lives indepen-
dently. He is taking amlodopine 10 mg, lasix 40 mg, and meto-
1. An 83-yr-old woman with osteoporosis and cataracts but no
prolol 25 mg twice daily. His primary care provider sends him
other health problems has a stable eGFR of 55 ml/min per 1.73
to you for advice on how to manage his BP. The patient is
m2, an albumin to creatinine ratio of 22 mg/g, and a BP of
already following a low sodium diet and walks daily. Which of
135/85 mmHg. She is not taking any BP medications. Which of
the following strategies do you think is most appropriate.
the following BP management strategies do you think is most
a. Start an ACE inhibitor and see him back in 1 to 2 wk for
appropriate.
follow-up on his serum creatinine and potassium
a. Start an ACE inhibitor with a goal BP ⬍130/80 mmHg
b. Start minoxidil or hydralazine
b. Start a thiazide with a goal BP ⬍130/80 mmHg
c. Leave well alone because this is a normal BP for his age and
c. Encourage her to follow a low sodium diet
adding another BP medication may cause more harm than
d. Do nothing
good
2. A 95-yr-old man with diabetes, an eGFR of 24 ml/min per 1.73 d. Start an ACE inhibitor and an ARB for control of both BP
m2, and 5 g/d of proteinuria that is presumed to be caused by and proteinuria and see him back in 1 to 2 wk for fol-
diabetic nephropathy has a BP of 170/85 mmHg and a pulse of low-up of his serum creatinine and potassium

American Society of Nephrology Geriatric Nephrology Curriculum 5


Chapter 12: Cardiovascular Disease in the Elderly With
Kidney Disease
Wolfgang C. Winkelmayer
Renal Division and Division of Pharmacoepidemiology and Pharmacoeconomics, Brigham and Women’s Hospital
and Harvard Medical School, Boston, Massachusetts

Heart disease constitutes the leading cause of death CKD and 32% for those with both CKD and diabe-
in the United States. Age is an important, albeit tes.8
nonmodifiable, risk factor for cardiovascular dis- Ample information is available on the epidemi-
ease in the general population, as well as in patients ology of cardiovascular disease in older individuals
with chronic kidney disease (CKD). The prevalence and its relationship with kidney function, including
of chronic ischemic heart disease in men and key prospective studies in elderly individuals such
women ⱖ65 yr of age in the United States in 1995 as the Cardiovascular Health Study. In a prospec-
was 83 per 1000 men and 90 per 1000 women. tive study of traditional and novel cardiovascular
Among those ⱖ75 years of age, the prevalences risk factors, diabetes, hypertension, smoking, low
were 217 per 1000 for men and 129 per 1000 for physical activity, left ventricular hypertension, and
women. Increasing evidence has accumulated that nonuse of alcohol were all predictors of subsequent
elderly individuals with cardiovascular disease can cardiovascular mortality, whereas high-density li-
benefit greatly from several aspects of secondary poprotein (HDL)-cholesterol, low-density lipopro-
prevention.1 tein (LDL)-cholesterol, triglycerides, and obesity
Kidney disease has been shown to be an impor- were not associated with such risk.9 None of the
tant determinant of cardiovascular disease,2,3 and novel cardiovascular risk factors that were tested
patients with CKD should be regarded a “highest were independently associated with cardiovascular
risk” group for cardiovascular disease, irrespective mortality, including C-reactive protein and anemia
of levels of traditional cardiovascular disease among others. Of note, homocysteine and phos-
(CVD) risk factors (http://www.kidney.org/profes- phorus were not evaluated in that study, and the
sionals/kdoqi/guidelines_ckd/p7_risk_g15.htm). other negative associations need to be interpreted
Furthermore, several cardiovascular risk factors are in light of the relatively low power of this study.
increasingly prevalent with declining kidney func- Table 1 provides a list of established and novel car-
tion.4 Interestingly, the Framingham Risk Score is diovascular risk factors in patients with CKD.
only poorly predictive for CVD in patients with In contrast, little evidence has been generated on
CKD, and standard factors only account for a small the efficacy and safety of standard curative or pre-
proportion of the observed risk in these patients.5 ventive cardiovascular interventions in patients
Finally, older age is an important determinant of with CKD. Most landmark trials have explicitly ex-
kidney function (as indicated by its representation cluded patients with CKD,10,11 and similarly, older
in the Modification of Diet in Renal Disease subjects were also barred from participation in
(MDRD) estimation equation for GFR6). It has most of these trials. These two independent phe-
been estimated that more than a third of US indi- nomena jointly explain the particular evidence vac-
viduals over age 70 have CKD Stages 3 to 5 and the uum for the population of older adults with CKD.
prevalence is increasing over time.7 One can postu-
late that the older individual with CKD is at the
highest risk of CVD, and even more so if additional Correspondence: Wolfgang C. Winkelmayer, Renal Division and
comorbid conditions including diabetes (DM), hy- Division of Pharmacoepidemiology and Pharmacoeconomics,
Brigham and Women’s Hospital and Harvard Medical School,
pertension, obesity, and other vascular disease are 1620 Tremont Street, Suite 3-030, Boston, MA 02120. Phone:
present. Indeed, among adults over age 67, 2-yr car- 617-278-0036; Fax: 617-232-8602; E-mail: wwinkelmayer@
diovascular mortality was 10% for those without partners.org

diagnosed CKD or diabetes but 30% for those with Copyright 䊚 2009 by the American Society of Nephrology

American Society of Nephrology Geriatric Nephrology Curriculum 1


Table 1. Cardiovascular risk factors in CKD (established or proposed, selection)
Traditional Risk Factors Novel Risk Factors, Some CKD-related
Older age Decreased kidney function
Male gender Proteinuria
White race Renin-angiotensin-system activity
Higher blood pressure Extracellular fluid overload
Higher LDL-cholesterol Higher phosphorus concentration
Lower HDL-cholesterol Hyperparathyroidism
Diabetes mellitus/impaired glucose tolerance Vitamin D deficiency
Tobacco use Dyslipidemia
Physical inactivity Anemia
Menopause Malnutrition
Psychosocial stress Inflammation
Family history of cardiovascular disease Oxidative stress
Infection
Higher homocysteine
Thrombogenic factors
Modified and updated from Sarnak and Levey.35

The fundamental question is whether evidence can be extrap- emia is clearly accepted as an important cardiovascular risk
olated to older patients with CKD from trials that effectively factor, and medical treatment, predominantly with statins, is
excluded those patients or contained only few such patients? well established for both primary and secondary cardiovascu-
Or should we require that specific trials be conducted in this lar prevention. Cardiovascular prevention with statins has also
relatively small segment of the population? Alternatively, been studied in a trial dedicated to the older population. In the
should we require that prespecified and sufficiently powered PROspective Study of Pravastatin in the Elderly at Risk (PROS-
tests for interaction of drug efficacy with age and kidney func- PER) trial, patients aged between 70 and 82 yr were enrolled
tion be planned and conducted? Although it would be desir- and compared with placebo. Treatment with 40 mg of prava-
able to inform more evidence-based practice in geriatric ne- statin per day conferred a 15% reduction in the risk of the
phrology, it is unlikely that such information will become primary combined cardiovascular endpoint (fatal or nonfatal
available on a larger scale anytime soon. Only recently, studies myocardial infarction or stroke) and a 19% reduction in the
were conducted, at the very least, that specifically focused on secondary endpoint of fatal or nonfatal myocardial infarction.
the older population.12 Additional top-level evidence has been These risk reductions were found to be in line with those found
made available from post hoc analyses of individual or pooled in trials of younger patients. In a post hoc analysis of data from
data from randomized trials. The vast majority of the evidence three pravastatin trials, it was found that statins were also effi-
on cardiovascular risk interventions in older patients with cacious in reducing cardiovascular outcomes in patients with
CKD, however, has come from retrospective pharmaco-epide- CKD Stage 3.13 The mean age in this study was 65.7 yr. Al-
miologic studies, often with serious methodological limita- though an interaction test with age was not conducted in this
tions. The following aims to provide evidence on a selected analysis, it is probably safe to assume that lipid-lowering treat-
number of cardiovascular risk factors and interventions in el- ment using statins is also efficacious in older patients with
derly patients with CKD: lipid disorders and lipid-lowering CKD. The optimal dose of specific agents or any preferred lipid
therapy, C-reactive protein and inflammation, homocysteine, targets, however, is not clearly established. Statins were also
as well as hyperphosphatemia and use of phosphate binders. efficacious in reducing cardiovascular events in kidney trans-
Other risk factors are covered in other chapters of this curric- plant patients.14 Whether statins are also efficacious in patients
ulum, notably diabetes and proteinuria, hypertension, and on hemodialysis is unclear. At the very least, chronic dialysis
anemia. patients with diabetes did not benefit from statin treatment in
a large randomized trial.15 Further evidence can be expected in
the near future when the results from the large Study of Heart
CARDIOVASCULAR RISK FACTORS IN CKD and Renal Protection (SHARP) trial will be released.

Traditional Risk Factors Smoking and Physical Activity.


Lipids. Smoking is a strong cardiovascular risk factor in the elderly,
Although HDL-cholesterol, LDL-cholesterol, and triglycerides and smoking cessation reduces overall morbidity and mortal-
were not associated with cardiovascular mortality in the Car- ity rates in patients with myocardial infarction (MI) and coro-
diovascular Heath Study, the wide confidence limits of these nary artery bypass graft surgery, including those older than 70
estimates are compatible with substantial risk increases associ- yr.16,17 Furthermore, smoking is a risk factor for progression of
ated with these factors. In the general population, hyperlipid- kidney disease. Although specific studies of smoking cessation

2 Geriatric Nephrology Curriculum American Society of Nephrology


in elderly patients with CKD or ESRD are lacking, smoking peutic lowering of homocysteine using vitamin therapy cannot
cessation counseling seems to be a prudent approach in this be recommended.
population. Similarly, physical activity has been shown to be
associated with CVD in older adults with CKD,9 but trials sup- Phosphorus.
porting specific interventions are not available in that popula- Serum phosphate has evolved as a recent risk factor for all-
tion. cause and cardiovascular mortality and morbidity. The preva-
lence of hyperphosphatemia increases with decreasing kidney
Novel Risk Factors function. It is an important therapeutic goal to maintain a
C-Reactive Protein. normal phosphorus concentration and a normal calcium-
C-reactive protein (CRP) is an acute-phase protein and has phosphate product, but dietary measures are often insufficient
evolved as a major cardiovascular risk factor in the general to achieve these goals. Phosphate binders have become impor-
population and in other subpopulations. CRP is higher, on tant tools to maintain phosphate control in patients with ESRD
average, in older individuals, and some studies have also shown but are also used in patients with CKD who are not on dialysis.
that CRP concentrations tend to be higher in patients with An ongoing debate has focused on the preferred choice of
CKD. In a study of patients with CKD Stages 3 and 4 enrolled in phosphate binder and calcium-containing binders have been
the MDRD study, CRP was positively associated with mortality associated with an increased risk of vascular and valvular cal-
from any cause. Elevated concentrations of this marker were cification in dialysis patients. The largest randomized trial of
also independently associated with a doubling in cardiovascu- calcium-containing phosphate binders versus sevelamer, a cal-
lar mortality.18 Similarly, CRP was associated with mortality in cium-free binder, showed no differences in the prespecified
kidney transplant recipients.19 Most recently, therapeutic ap- endpoint of all-cause or cardiovascular mortality.24 An inter-
proaches to lowering CRP and to reduce cardiovascular mor- action with age was found, however, in that, among patients
tality through it have become available. The recently published older than 65 yr, sevelamer conferred a significant reduction in
Justification for the Use of Statins in Prevention: an Inter- mortality. Because these results arose from subgroup analyses
vention Trial Evaluating Rosuvastatin (JUPITER) enrolled pa- with the possibility of a false-positive result from multiple test-
tients with normal lipid concentrations and no history of ing, these findings need to be interpreted with caution.
cardiovascular disease, but elevated CRP concentrations.20
Rosuvastatin 40 mg lowered CRP concentrations by 37% and
reduced the risk of a composite cardiovascular endpoint by DIAGNOSIS OF ACUTE CORONARY SYNDROMES
44%. Although one half of the enrolled patients were older
than 66 yr, ⬍25% of patients had an estimated GFR ⬍60 ml/ One important issue with coronary artery disease in older pa-
min per 1.73 m2. Thus, the applicability of these results to tients with CKD is the fact that it is harder to diagnose than in
older patients with CKD is uncertain, but subsequent younger patients free from CKD. Noninvasive tests seem to
subgroup analysis by kidney function can certainly be ex- have different sensitivities and specificities in patients with
pected. CKD than in the general population. For example, ST-eleva-
tion myocardial infarction is considerably less likely in patients
Homocysteine. with CKD compared with patients without it.25 Patients with
Homocysteine is a sulfur-containing amino acid that occupies CKD are less likely to present with arm, shoulder, or chest pain
an important role in 1-carbon metabolism and nucleotide syn- or pressure, or with diaphoresis, but more likely complain of
thesis. Both age and reduced kidney function are important cough or dyspnea.26 Standard laboratory markers such as cre-
determinants of total homocysteine plasma concentrations. atine kinase and its myocardial subfraction and troponins are
Elevated homocysteine, in turn, has been shown to be a pow- frequently and intermittently elevated in patients with kidney
erful cardiovascular risk factor in the general population, as disease.27–29 Thus, diagnosing acute coronary syndromes in
well as in the elderly and in patients with diabetes. In kidney older patients with CKD can be challenging at times and fur-
disease, this association is less clear: an association with mor- ther research on improved diagnostic tests or algorithms in this
tality has been shown in kidney transplant recipients21 but was subgroup of patients is warranted.
absent in patients enrolled in the MDRD study, although the
confidence intervals in this study were rather wide.22 It is well
established that high-dose vitamin therapy with folic acid, pyr- USE OF ACUTE AND CHRONIC CARDIOVASCULAR
idoxine (vitamin B6), and/or cyanocobalamin (vitamin B12) INTERVENTIONS
can reduce plasma homocysteine concentrations quite signifi-
cantly, especially in patients with kidney disease. Several large Numerous studies have shown that older individuals are less
randomized trials were conducted, and their results were ex- likely to receive recommended medications, both in cardiovas-
pected with great enthusiasm and hope. The findings, how- cular disease and in other diseases and disorders. Such treat-
ever, were sobering. The majority of these trials showed no ment bias has sometimes been termed “Ageism.” Similarly,
beneficial effect on cardiovascular outcomes.23 Thus, thera- Chertow has coined the term “Renalism” based on the obser-

American Society of Nephrology Geriatric Nephrology Curriculum 3


vation that patients with kidney disease also were less likely to 3. Anavekar NS, McMurray JJ, Velazquez EJ, Solomon SD, Kober L,
Rouleau JL, White HD, Nordlander R, Maggioni A, Dickstein K, Zelen-
receive standard therapies compared with others without kid- kofske S, Leimberger JD, Califf RM, Pfeffer MA: Relation between
ney disease but of similar age. Again, at the intersection of renal dysfunction and cardiovascular outcomes after myocardial infarc-
Ageism and Renalism, older individuals with kidney disease tion. N Engl J Med 351: 1285–1295, 2004
are least likely to receive recommended interventions and 4. Uhlig K, Levey AS, Sarnak MJ: Traditional cardiac risk factors in indi-
treatments. For cardiovascular disease, this has been observed viduals with chronic kidney disease. Semin Dial 16: 118 –127, 2003
5. Weiner DE, Tighiouart H, Elsayed EF, Griffith JL, Salem DN, Levey AS,
for acute coronary interventions as well as for chronic treat- Sarnak MJ: The Framingham predictive instrument in chronic kidney
ments for secondary prevention including statins, ␤-blockers, disease. J Am Coll Cardiol 50: 217–224, 2007
aspirin, and inhibitors of the rennin-angiotensin system25,30 –33 6. Levey AS, Coresh J, Greene T, Marsh J, Stevens LA, Kusek JW, Van
(see Wetmore and Shireman for a comprehensive review of Lente F; Chronic Kidney Disease Epidemiology Collaboration: Ex-
this literature.34). It is unclear on what basis such discrimina- pressing the Modification of Diet in Renal Disease Study equation for
estimating glomerular filtration rate with standardized serum creati-
tion occurs: the lack of available evidence in this specific pop- nine values. Clin Chem 53: 766 –772, 2007
ulation group? Perceived futility in a population of assumed 7. Coresh J, Selvin E, Stevens LA, Manzi J, Kusek JW, Eggers P, Van
low benefit in light of the naturally shortened lifespan? Cost- Lente F, Levey AS: Prevalence of chronic kidney disease in the United
effectiveness considerations in that older patients with CKD States. JAMA 298: 2038 –2047, 2007
may not live long enough to reap the benefits from chronic 8. Collins AJ, Li S, Gilbertson DT, Liu J, Chen SC, Herzog CA: Chronic
kidney disease and cardiovascular disease in the Medicare population.
preventive treatment? Presence of multiple comorbid condi- Kidney Int Suppl S24 –S31, 2003
tions that may diffuse focus on cardiovascular care? Probably 9. Shlipak MG, Fried LF, Cushman M, Manolio TA, Peterson D, Stehman-
all of the above contribute. Further research is necessary to Breen C, Bleyer A, Newman A, Siscovick D, Psaty B: Cardiovascular
provide the evidence needed and focused integrative curricula mortality risk in chronic kidney disease: comparison of traditional and
such as this geriatric nephrology effort are necessary to im- novel risk factors. JAMA 293: 1737–1745, 2005*
10. Charytan D, Kuntz RE: The exclusion of patients with chronic kidney
prove the care and outcomes of this very vulnerable population disease from clinical trials in coronary artery disease. Kidney Int 70:
of older patients with CKD. 2021–2030, 2006
11. Coca SG, Krumholz HM, Garg AX, Parikh CR: Underrepresentation of
renal disease in randomized controlled trials of cardiovascular disease.
JAMA 296: 1377–1384, 2006*
TAKE HOME POINTS
12. Shepherd J, Blauw GJ, Murphy MB, Bollen EL, Buckley BM, Cobbe
• Most established cardiovascular risk factors are also predictive in older SM, Ford I, Gaw A, Hyland M, Jukema JW, Kamper AM, Macfarlane
patients with CKD PW, Meinders AE, Norrie J, Packard CJ, Perry IJ, Stott DJ, Sweeney
• Sparse information is available on the efficacy of interventions in older BJ, Twomey C, Westendorp RG; PROSPER study group. PROspective
patients with CKD Study of Pravastatin in the Elderly at Risk: Pravastatin in elderly indi-
• The best evidence is available for secondary cardiovascular prevention viduals at risk of vascular disease (PROSPER): a randomised controlled
with statins, although the optimal lipid targets are not known trial. Lancet 360: 1623–1630, 2002
• BP and diabetes control also seem to confer benefit in older patients 13. Tonelli M, Isles C, Curhan GC, Tonkin A, Pfeffer MA, Shepherd J,
with CKD Sacks FM, Furberg C, Cobbe SM, Simes J, Craven T, West M: Effect of
• Most of the evidence is from observational studies, with associations pravastatin on cardiovascular events in people with chronic kidney
that do not necessarily reflect causal relationships disease. Circulation 110: 1557–1563, 2004*
• Underuse of curative and preventive therapies is prevalent in older 14. Holdaas H, Fellström B, Jardine AG, Holme I, Nyberg G, Fauchald P,
patients with CKD Grönhagen-Riska C, Madsen S, Neumayer HH, Cole E, Maes B, Am-
bühl P, Olsson AG, Hartmann A, Solbu DO, Pedersen TR; Assessment
of LEscol in Renal Transplantation (ALERT) Study Investigators: Effect
of fluvastatin on cardiac outcomes in renal transplant recipients: a
DISCLOSURES multicentre, randomised, placebo-controlled trial. Lancet 361: 2024 –
None. 2031, 2003
15. Wanner C, Krane V, März W, Olschewski M, Mann JF, Ruf G, Ritz E;
German Diabetes and Dialysis Study Investigators: Atorvastatin in
patients with type 2 diabetes mellitus undergoing hemodialysis.
REFERENCES N Engl J Med 353: 238 –248, 2005
16. Hermanson B, Omenn GS, Kronmal RA, Gersh BJ: Beneficial six-year
*Key References outcome of smoking cessation in older men and women with coronary
1. Williams MA, Fleg JL, Ades PA, Chaitman BR, Miller NH, Mohiuddin artery disease. Results from the CASS registry. N Engl J Med 319:
SM, Ockene IS, Taylor CB, Wenger NK; American Heart Association 1365–1369, 1988
Council on Clinical Cardiology Subcommittee on Exercise, Cardiac 17. Critchley JA, Capewell S: Mortality risk reduction associated with
Rehabilitation, and Prevention: Secondary prevention of coronary smoking cessation in patients with coronary heart disease: a system-
heart disease in the elderly (with emphasis on patients ⱖ75 yr of age): atic review. JAMA 290: 86 –97, 2003
an American Heart Association scientific statement from the Council 18. Menon V, Greene T, Wang X, Pereira AA, Marcovina SM, Beck GJ,
on Clinical Cardiology Subcommittee on Exercise, Cardiac Rehabili- Kusek JW, Collins AJ, Levey AS, Sarnak MJ: C-reactive protein and
tation, and Prevention. Circulation 105: 1735–1743, 2002* albumin as predictors of all-cause and cardiovascular mortality in
2. Go AS, Chertow GM, Fan D, McCulloch CE, Hsu CY: Chronic kidney chronic kidney disease. Kidney Int 68: 766 –772, 2005
disease and the risks of death, cardiovascular events, and hospitaliza- 19. Winkelmayer Winkelmayer WC, Schaeffner ES, Chandraker A, Kramar
tion. N Engl J Med 351: 1296 –1305, 2004 R, Rumpold H, Sunder-Plassmann G, Födinger M: A J-shaped associ-

4 Geriatric Nephrology Curriculum American Society of Nephrology


ation between high-sensitivity C-reactive protein and mortality in kid- 27. Apple FS, Murakami MM, Pearce LA, Herzog CA: Predictive value of
ney transplant recipients. Transplant Int 20: 505–511, 2007 cardiac troponin I and T for subsequent death in end-stage renal
20. Ridker PM, Danielson E, Fonseca FA, Genest J, Gotto AM Jr, Kastelein disease. Circulation 106: 2941–2945, 2002
JJ, Koenig W, Libby P, Lorenzatti AJ, MacFadyen JG, Nordestgaard 28. Freda BJ, Tang WH, Van Lente F, Peacock WF, Francis GS: Cardiac
BG, Shepherd J, Willerson JT, Glynn RJ; JUPITER Study Group: Ro- troponins in renal insufficiency: review and clinical implications. J Am
suvastatin to prevent vascular events in men and women with elevated Coll Cardiol 40: 2065–2071, 2002
C-reactive protein. N Engl J Med 359: 2195–2207, 2008 29. Iliou MC, Fumeron C, Benoit MO, Tuppin P, Calonge VM, Moatti N,
21. Winkelmayer WC, Kramar R, Curhan GC, Chandraker A, Endler G, Föd- Buisson C, Jacquot C: Prognostic value of cardiac markers in ESRD:
inger M, Hörl WH, Sunder-Plassmann G: Fasting plasma total homocys- Chronic Hemodialysis and New Cardiac Markers Evaluation (CHANCE)
teine levels and mortality and allograft loss in kidney transplant recipients: study. Am J Kidney Dis 42: 513–523, 2003
a prospective study. J Am Soc Nephrol 16: 255–260, 2005 30. Chertow GM, Normand SL, McNeil BJ: “Renalism”: inappropriately
22. Menon V, Sarnak MJ, Greene T, Wang X, Pereira AA, Beck GJ, Kusek low rates of coronary angiography in elderly individuals with renal
JW, Selhub J, Collins AJ, Levey AS, Shlipak MG: Relationship between insufficiency. J Am Soc Nephrol 15: 2462–2468, 2004
homocysteine and mortality in chronic kidney disease. Circulation 113: 31. Berger AK, Duval S, Krumholz HM: Aspirin, beta-blocker, and angio-
1572–1577, 2006 tensin-converting enzyme inhibitor therapy in patients with end-stage
23. Sunder-Plassmann G, Winkelmayer WC, Fodinger M: Approaching the renal disease and an acute myocardial infarction. J Am Coll Cardiol 42:
end of the homocysteine hype? Am J Kidney Dis 51: 549 –553, 2008* 201–208, 2003
24. Suki Suki WN, Zabaneh R, Cangiano JL, Reed J, Fischer D, Garrett L, 32. Winkelmayer WC, Charytan DM, Brookhart MA, Levin R, Solomon DH,
Ling BN, Chasan-Taber S, Dillon MA, Blair AT, Burke SK: Effects of Avorn J: Kidney function and use of recommended medications after
sevelamer and calcium-based phosphate binders on mortality in he- myocardial infarction in elderly patients. Clin J Am Soc Nephrol 1:
modialysis patients. Kidney Int 72: 1130 –1137, 2007 796 – 801, 2006
25. Charytan DM, Setoguchi S, Solomon DH, Avorn J, Winkelmayer WC: 33. Winkelmayer WC, Levin R, Setoguchi S: Associations of kidney func-
Clinical presentation of myocardial infarction contributes to lower use tion with cardiovascular medication use after myocardial infarction.
of coronary angiography in patients with chronic kidney disease. Kid- Clin J Am Soc Nephrol 3: 1415–1422, 2008*
ney Int 71: 938 –945, 2007 34. Wetmore JB, Shireman TI: The ABCs of cardioprotection in dialysis
26. Sosnov J, Lessard D, Goldberg RJ, Yarzebski J, Gore JM: Differential patients: a systematic review. Am J Kidney Dis 53: 457– 466, 2008*
symptoms of acute myocardial infarction in patients with kidney disease: 35. Sarnak MJ, Levey AS: Cardiovascular disease and chronic renal dis-
a community-wide perspective. Am J Kidney Dis 47: 378 –384, 2006 ease: a new paradigm. Am J Kidney Dis 35: S117–S131, 2000

American Society of Nephrology Geriatric Nephrology Curriculum 5


REVIEW QUESTIONS: CARDIOVASCULAR DISEASE b. Patients on hemodialysis
IN THE ELDERLY WITH KIDNEY DISEASE c. Older individuals
d. All of the above
1. What individuals have frequently been excluded from partic-
ipation in large cardiovascular efficacy trials?
3. Most cardiovascular risk factors in the general population ap-
a. Older individuals
pear to be also operational in older patients with chronic kid-
b. Patients with advanced kidney disease or on dialysis
ney disease
c. Both
a. True
d. Neither
b. False
2. The efficacy of statins in the secondary prevention of cardio- c. Uncertain. While several cardiovascular risk factors have
vascular events has been shown in which populations? been confirmed in older patients with CKD, others were
a. Patients with mild to moderate chronic kidney disease not probably because of limited statistical power

6 Geriatric Nephrology Curriculum American Society of Nephrology


Chapter 13: Vascular Disease in the Elderly
Nobuyuki Bill Miyawaki* and Paula E. Lester†
*Division of Nephrology and †Division of Geriatrics, Winthrop University Hospital, Mineola, New York

PHYSIOLOGIC EFFECTS OF AGING ON stiffening of medium and large arteries lined with
BLOOD VESSEL ELASTICITY AND atheromatous plaques. The progressive accumula-
COMPLIANCE tion of atherosclerotic plaque continues with aging
and often remains silent until lesions reach a critical
The aging process is commonly associated with in- stenotic threshold or rupture, leading to dimin-
creased vascular rigidity and decreased vascular ished organ perfusion. Arterial stiffening also leads
compliance. This process reflects the accumulation to an increase in pulse wave velocity and an en-
of smooth muscle cells and connective tissue in the hancement in central aortic systolic pressure.3 The
walls of major blood vessels. Endothelial cells and increased waveform velocity allows the backward
smooth muscle cells constitute most of vessel wall reflective wave to return earlier back to the heart.
cellularity and the remainder of the wall is com- The resulting increases of the left ventricular myo-
posed of extracellular matrix including collagen cardial load and the loss of coronary perfusion at
and elastin. Although aging has minimal effect on the onset of diastole may potentiate myocardial
the muscular tunica media layer thickness, aging ischemia.3
leads to profound progressive thickening of the tu- Common ailments in the elderly including dia-
nica intima layer comprised of endothelial cells and betes, dyslipidemia, hypertension, obesity, and cig-
an extracellular matrix. In addition, with aging, arette smoking can accelerate the process. Although
there is a thinning and separation of individual elas- the exact mechanisms have yet to be elucidated,
tin lamellae, as well as an increase in the collagen multiple factors including genetics, advanced glyca-
matrix.1 tion endproducts, and oxidative stress likely con-
The age-related vascular rigidity and decreased ar- tribute to the pathogenesis of accelerated athero-
terial compliance leads to progressive increase in sys- sclerosis.
tolic BP, with 25% of patients over 75 yr of age suffer-
ing from isolated systolic hypertension.2 Healthy
elderly patients without hypertension also show a EFFECTS OF AGING, ATHEROSCLEROSIS,
modest increase in peripheral vascular resistance and AND HYPERTENSION ON THE KIDNEYS
only a modest related increase in systolic BP.2 Dilation
and stiffening of the proximal aorta and its major Renal blood flow declines with aging at a magnitude
branches including the brachiocephalic, carotid, and of 6 ml/min per year, with a proportionately larger
subclavian arteries occur to a greater extent than the reduction in cortical blood flow than medullary
peripheral arteries with aging. This also blunts the ca- flow.2 GFR also declines with aging albeit at a slower
rotid baroreceptor sensitivity and increase the risk of rate of 1 ml/min per year. It is unclear if this change
end organ damage to the kidneys, heart, and brain.1,2 is caused by the aging process alone or is also attrib-
It is important to keep in mind that the presence of utable to hypertension and arteriosclerosis. It is es-
pre-existing hypertension and accelerated atheroscle- timated that as much as 26% of all end-stage kidney
rosis intensify the vascular pathology discussed to a far disease (ESKD) in the United States is related to
greater extent. hypertensive arteriolar nephrosclerosis. This num-

Correspondence: Nobuyuki Bill Miyawaki, MD, Winthrop Univer-


EFFECTS OF ATHEROSCLEROSIS ON sity Hospital, Division of Nephrology, 200 Old Country Road,
AGING Suite 135, Mineola, NY 11501. Phone: 516-663-2169; Fax: 516-
663-2179; E-mail: [email protected]

Atherosclerosis is characterized by the progressive Copyright 䊚 2009 by the American Society of Nephrology

American Society of Nephrology Geriatric Nephrology Curriculum 1


ber is expected to grow as the population of the United States Intrarenal Arterioles and Interlobular Arteriolar
continues to age.4 Furthermore, it is estimated that 10% of all Disease: Arterial Hyalinosis
ESKD is caused by renal artery stenosis.5,6 The ESKD incidence Aging, hypertension, and arteriosclerosis all affect the arte-
from renovascular disease has increased in parallel with the rioles in the kidney parenchyma causing afferent and efferent
advancing age of the general population in the United States.6 arteriolar atrophy.4 Microcirculatory adaptive changes to the
Numerous causes of vascular insult to the kidneys exist as glomerular mass also occur, including impaired autoregula-
listed in Figure 1. When classified anatomically by arterial ves- tion with preglomerular vasodilatation. This is associated with
sel size, the vascular diseases can be subdivided to those affect- structural and functional hypertrophy of the intact nephrons
ing (1) the microscopic vessels in the kidney, (2) arterioles, and leading to glomerular hypertension, albuminuria, and further
(3) the main renal arteries. We will focus here on the predom- glomerular sclerosis.4 The histologic changes include hyper-
inant forms of vascular insult to the kidneys seen with aging: plastic elastic arteriosclerosis present in the interlobular arter-
hypertensive nephrosclerosis, arteriolar hyalinosis, and ath- ies, intimal thickening, reduplication of the lamina elastica in-
erosclerotic renal artery stenosis. terna, and mild hyalinization.4 The development of
hyperplastic elastic and/or hyaline arteriolosclerosis leads to a
Microscopic Vascular Disease in the Kidney With reduction in renal blood flow along with progressive ischemic
Aging: Nephrosclerosis changes observed in the cortical glomeruli and renal tubules.4
As systemic hypertension worsens, an increase in renal vascu-
lar resistance and a progressive decrease in effective renal Main Renal Artery Disease: Atherosclerotic Renal
plasma flow are observed. Although the GFR is relatively main- Artery Stenosis
tained, serious renal impairment may occur in as many as 20% Renal artery stenosis (RAS) may be defined as the presence of
of essential hypertensive patients. cross-sectional arterial luminal narrowing, which may or may
With aging, the total number of functioning glomeruli de- not have any significant hemodynamic effects. Atherosclerotic
crease by 30 to 50% as the percentage of sclerotic or abnormal renal artery stenosis leads to an occlusion at the ostium and the
glomeruli increases.4 Under ischemic insult from vascular dis- proximal third of the renal arteries. Variability in criteria used
ease, the glomeruli in renal cortex show wrinkling of the cap- to define RAS based on anatomical luminal narrowing exist
illary basement membrane followed by thickening, collapse, but atherosclerotic RAS often becomes clinically significant
and global sclerosis. The initial ischemic changes are reversible. only when the arterial cross-sectional luminal diameter is re-
However, with the appearance of hyaline in the capsular space, duced by 70%.6 Hemodynamically significant stenosis leads to
the process proceeds to complete glomerular obsolescence. In parenchymal ischemia, atrophy, and loss of kidney function.
the deeper juxtamedullary areas, direct channels between af- The exact prevalence of atherosclerotic RAS is unclear, but
ferent and efferent arterioles are formed that leads to the for- it has been consistently shown that the risk increases with older
mation of aglomerular arterioles. These vascular lesions are age. Based on autopsy series estimates, when renal artery ste-
usually distributed in a focal, irregular fashion throughout the nosis is defined as a 50% or greater reduction in luminal diam-
renal cortex and result in ischemic changes that follow a similar eter, it is present in 27% of patients in their sixties and 62% of
pattern of distribution. However, if the vascular lesions are patients 70 years of age and older.6 The presence of aortic and
severe and generalized, the entire kidney may undergo isch- peripheral arterial disease increases the chance of coexisting
emic atrophy. atherosclerotic renal arteries.
There are several histologic changes seen with aging. Sec- This ischemic insult leads to histologic changes in mul-
ondary tubular atrophy, thickening of tubular basement mem- tiple levels. Patchy tubular necrosis and tubular atrophy are
branes, and interstitial fibrosis occur. Renal tubules decrease in prominently seen, in addition to focal areas of collagenous
number. Additionally, the proximal tubules decrease in vol- filling of Bowman’s space and patchy peritubular leukocyte
ume and length, whereas the distal tubules form diverticula and mononuclear infiltrates. Glomerular atrophy with
because of alterations in intrarenal vasculature. wrinkling of glomerular capillary tuft and thickening and
duplication of Bowman’s capsule is also found. Of note, the
Vascular Insult to Kidneys often concurrent extension of atherosclerotic disease into
Hypertensive nephrosclerosis the major branches of intrarenal arterioles can lead to the
Atherosclerotic renal artery stenosis ischemic glomerular and tubular changes noted above even
Fibromuscular dysplasia induced renal artery stenosis in the absence of hemodynamically significant main renal
Renal artery thrombosis artery stenosis.
Polyarteritis nodosa In response to the chronic vascular ischemic change, corti-
Microscopic polyangiitis cal blood flow is redistributed to the deeper corticomedullary
Microangiopathic hemolytic anemia circulation that further attenuates the GFR.6 Clinical correlate
Scleroderma
data suggest that approximately 30 to 60% of patients with
Atherosclerotic embolic disease to the kidneys
atherosclerotic ischemic renal disease progressively worsen
Figure 1. Causes of vascular injury to kidneys. their stenosis and kidney function in a relatively short duration

2 American Society of Nephrology American Society of Nephrology


of 6 months to 3 years.6 Total occlusion may take place in CT angiogram are poor in visualization of intrarenal arterioles.
excess of 10% of patients during the same duration of follow- Renal Doppler ultrasound with concurrent measurement for
up.6 resistive index (RI) can be used to indirectly assess the blood
flow within the renal parenchyma. An RI reading of 0.8 or
Diagnosis of Atherosclerotic Renal Artery Stenosis. greater may signify the presence of intrarenal arteriolar disease.
The presence of atherosclerotic RAS can lead to progressive In patients with progressive CKD without a clear explanation,
renal insufficiency from ischemic nephropathy as outlined a renal biopsy may be required for definitive diagnosis of isch-
above. It also can lead to or exacerbate the underlying essential emic nephrosclerosis if the clinical situation warrants.
hypertension. No clinical signs are pathognomonic. However,
such symptoms and signs as the onset of hypertension after the Treatment of Atherosclerotic Renal Artery Stenosis.
age 50 yr, abdominal bruit, the presence of atrophic kidney, Not all cases of documented RAS need to be corrected because
recurrent flash pulmonary edema or the occurrence of acute some patients will have no clinical sequela. However, RAS may
renal failure after initiation of angiotensin converting enzyme lead to ischemic nephropathy with progressive CKD or it may
inhibitor (ACEi) or angiotensin receptor blocker (ARB) may exacerbate hypertension. Variable outcomes of percutaneous
suggest the presence of renal artery stenosis. Additionally, pro- transluminal renal angioplasty (PTRA) with stents for the
gressive renal insufficiency in a patient with diffuse atheroscle- treatment of hypertension from RAS are reported depending
rosis may raise the level of clinical suspicion. on patient selection and the criteria used to show its efficacy.
If adequate clinical suspicion warrants the work-up, the di- Nonetheless, it is rare for PTRA to cure hypertension.7 Better
agnosis must be confirmed by anatomical or functional corre- outcomes can be expected from treatment of ischemic ne-
lation. Several diagnostic modalities with their respective ben- phropathy from atherosclerotic RAS with PTRA. Improve-
efits and limitations are listed in Figure 2. Although an ment and/or stabilization of progressive renal failure is seen in
angiogram of the renal arteries is still considered as the gold approximately two thirds of the patients undergoing this pro-
standard, it has low utility as a screening tool because of its cedure.7 The elderly can benefit equally from PTRA, and age
invasive nature and the risks of cholesterol emboli. Renal ar- alone should not discourage the procedure.8 Ultimately, a de-
tery Doppler and renal scintigraphy are limited by lower sen- cision to intervene must always be carefully weighed against
sitivity. Computed tomographic (CT) angiogram requires the the procedure related risks, including cholesterol emboli, renal
use of intravenous contrast, which has inherent risks of con- arterial subintimal dissection, and renal artery thrombosis, be-
trast nephropathy. Magnetic resonance arteriography (MRA) cause aggregate complication rates may be in excess of 9%.7,9
was once considered as the panacea to this diagnostic dilemma
but recent concerns of nephrogenic systemic fibrosis in pa-
tients with CKD have diminished the enthusiasm for this mo- THERAPEUTIC IMPLICATIONS
dality.
Elderly patients require a specialized approach in the manage-
Diagnosis of Intrarenal Arteriolar Disease and Ischemic Nephro- ment of vascular disease. Careful drug dosing is needed be-
sclerosis. cause of their often diminished GFR. The carotid baroreflex
The aforementioned imaging modalities including MRA and response is often blunted, and vasodilator antihypertensive

Imaging Modality Benefits Limitations


Doppler Ultrasound • Non-invasive • Operator dependent with potential
• No intravenous contrast compromised accuracy
• Lower sensitivity than MRA
MRA with IV gadolinium • High resolution near- • Risk of NSF
angiogram • Not suitable with AICD/PPM
CT angiogram • High sensitivity and • Risk of CIN
specificity for RAS • Radiation exposure
• 95% negative predictive
value
Angiogram of Renal Arteries • Gold standard • Invasive with potential risks of
• CO2 angiography may be cholesterol emboli
used in patients with CKD
Nuclear Renal Scan • Positive predictive value as • Lower sensitivity, especially in
(ACEI Renal Scintigraphy) high as 92% under ideal patients with CKD
conditions • May need further confirmation by
• Minimally invasive above modality
Figure 2. Imaging modalities available for evaluation of renal artery stenosis.10

American Society of Nephrology American Society of Nephrology 3


medications may lead to dizziness and orthostatic hypoten- • It is estimated that 10% of all ESKD is caused by renal artery stenosis
• Improvement and/or stabilization of progressive kidney disease is seen
sion. Physicians also must be aware of the manifestations of in approximately two thirds of the patients undergoing PTRA; the
vascular disease when forming the differential diagnosis on elderly can benefit equally from PTRA and age alone should not dis-
hypertension and CKD. Additionally, clinicians must assess courage the procedure
elderly patients for their understanding and ability to comply
with the treatment regimen because older patients may not be
taking the correct doses of medications because of cognitive DISCLOSURES
impairment, visual challenges, swallowing difficulty, or inabil- None.
ity to afford the medication.

CONCLUSIONS REFERENCES

Aging alters the physiology of arterial vasculature. Arterial cir- *Key References
1. O’Rourke MF: Arterial aging: pathophysiological principles. Vasc Med
culation stiffens and becomes less compliant, proximal arteries (Lond) 12: 329 –341, 2007*
dilate, and carotid baroreceptors become less sensitive. GFR 2. Rowe JW: Clinical consequences of age-related impairments in vas-
also declines with aging. Coexisting hypertension and athero- cular compliance. Am J Cardiol 60: 68G–71G, 1987
sclerotic disease exaggerate the normal vascular aging process, 3. Townsend RR: Analyzing the radial pulse waveform: narrowing the gap
endangering the longevity of end organ function including the between blood pressure and outcomes. Curr Opin Nephrol Hypertens
16: 261–266, 2007
kidneys. Hypertensive and atherosclerotic renal diseases ac- 4. Bauer JH, Reams GP, Wu Z: The aging hypertensive kidney: patho-
count for a large proportion of progressive CKD and ESKD in physiology and therapeutic options. Am J Med 90: 21S–27S, 1991*
the United States. It is important for the nephrologists to focus 5. Mailloux LU, Napolitano B, Bellucci AG, Vernace M, Wilkes BM, Mos-
on the preventative measures of atherosclerosis including an sey RT: Renal vascular disease causing end-stage renal disease, inci-
aggressive treatment of dyslipidemia, diabetes, and hyperten- dence, clinical correlates, and outcomes: a 20-year clinical experience.
Am J Kidney Dis 24: 622– 629, 1994
sion along with cigarette smoking cessation. Furthermore, a 6. Greco BA, Breyer JA: Atherosclerotic ischemic renal disease. Am J
careful evaluation of suspected atherosclerotic disease may be Kidney Dis 29: 167–187, 1997*
warranted. 7. Bloch MJ, Pickering T: Renal vascular disease: medical management,
angioplasty, and stenting. Semin Nephrol 20: 474 – 488, 2000*
8. Bloch MJ, Trost DA, Whitmer J, Pickering TG, Sos TA, August P: Ostial
renal artery stent placement in patients 75 years of age or older. Am J
TAKE HOME POINTS
Hypertens 14: 983–988, 2001
• Effect of aging on baroreceptor reflex may profoundly increase the 9. Bonelli FS, McKusick MA, Textor SC, Kos PB, Stanson AW, Johnson
orthostatic side effect profile of antihypertensive vasodilators CM, Sheedy PF 2nd, Welch TJ, Schirger A: Renal artery angioplasty:
• As much as 26% of all ESKD in the United States is related to hyper- technical results and clinical outcome in 320 patients. Mayo Clin Proc
tensive arteriolar nephrosclerosis 70: 1041–1052, 1995*
• As much as 62% of patients over 70 may suffer from RAS and the risk 10. Ramchandani P, Jones LP, Langer JE, Toririan DA, Perini RF, Divgi CR:
increases with aging Renal imaging. NephSAP 7: 218 –287, 2008

4 American Society of Nephrology American Society of Nephrology


REVIEW QUESTIONS: VASCULAR DISEASE IN THE d. Renal artery stenosis likely accounts for 10% of all end-
ELDERLY stage kidney disease cases
e. Magnetic Resonance Imaging (MRI) with IV gadolinium
1. Age related changes in vasculature may lead to the following:
is safe for patients with advanced chronic kidney disease
a. An increases in systolic BP and vascular rigidity only if
and allows for high resolution imaging otherwise unavail-
there is an underlying history of hypertension
able
b. A continued decline in the total number of functional glo-
meruli, contributing to losses in GFR
3. Advancing age will:
c. Hypertensive arteriolar nephrosclerosis responsible for
a. Diminish the likelihood that renal artery stenting will be
75% of all end-stage kidney disease in the United States
beneficial in the treatment of ischemic nephropathy and
d. Labile BP and increased orthostatic effects of vasodilators
should be discouraged
caused by carotid receptor hypersensitivity
b. Enhance the side-effect profile of vasodilator antihyper-
2. In a patient with suspected renal artery stenosis, which of the tensive medications, such as dizziness and orthostatic hy-
following statements is considered true: potension
a. Renal artery stenosis should be identified on an imaging c. Often lead to increased risks of renal artery stenosis; the
study and always be corrected by stents or surgery when- presence of abdominal bruit and acute pulmonary edema
ever present are pathognomonic for renal artery stenosis and should
b. Hypertension is usually curative after correction of renal always be evaluated
artery stenosis with stents d. Increase the risks for renal artery stenosis and hyperten-
c. All complication risks associated with renal artery stenting are sion from fibromuscular dysplasia
trivial

American Society of Nephrology American Society of Nephrology 5


Chapter 14: Bone Disease and Calcium Abnormalities
in Elderly Patients With CKD
Harmeet Singh
Division of Nephrology, Brody School of Medicine, Greenville, North Carolina

An 81-yr-old African-American woman with end- issue problematic to define. Osteoporosis in CKD is
stage kidney disease (ESKD) who has been on he- only a part of the constellation of metabolic bone
modialysis for 10 yr seeks a consultation with you. problems. Therefore, its diagnosis and manage-
Her primary physician obtained a dual-energy x- ment may differ from general population. Bones
ray absorptiometry (DEXA) scan and asked her to are more severely affected in CKD than that from
discuss the results with her nephrologist for further normal aging. In a patient with renal osteodystro-
management. She is diagnosed with osteoporosis phy, there is the potential for low BMD to coexist
based on her T-scores of ⫺5.1 (AP spine), ⫺4.1 (left with an enormous range of functional abnormali-
femoral neck), and ⫺5.4 (left total hip). Her last ties. These range from high turnover bone lesions in
dialysis laboratory results show serum calcium is patients with uncontrolled hyperparathyroidism to
10.7 mg/dl, intact parathyroid hormone (PTH) is severely reduced bone remodeling activity in pa-
207 pg/ml, phosphorus is 5.7 mg/dl, and alkaline tients with adynamic bone disease. This is in con-
phosphatase (ALP) is 122 IU/L. She receives pari- trast to the non-CKD patient with osteoporosis
calcitol 5 mg intravenously on hemodialysis three where bone remodeling is not severely affected.
times per week.
Chronic kidney disease (CKD)-related bone dis-
ease is known as renal or uremic osteodystrophy. It IMPACT ON QUALITY OF LIFE
is associated with derangements in bone and min-
eral metabolism that leads to abnormal regulation Patients with CKD-MBD and osteoporosis are as-
of calcium, phosphorous, vitamin D, and PTH. It sociated with increased risk of fractures and are at a
encompasses a spectrum of conditions that are clas- high risk of cardiovascular disease.2 The overall in-
sified based on bone biopsy findings including os- cidence of hip fractures among dialysis patients is
teitis fibrosa (high turnover disease), mixed uremic about four-fold higher than that expected for gen-
osteodystrophy, osteomalacia (low turnover dis- eral population. The risk is increased in both men
ease), and adynamic bone disease. KDIGO (kidney and women.3 Fractures may limit ambulation, lead-
disease: improving global outcomes) has proposed ing to loss of independence and chronic pain,
to define CKD-related bone and mineral metabolic thereby decreasing quality of life. Mortality risk in
abnormalities in the context of a systemic disorder dialysis patients with hip fracture is twice that of
called CKD–mineral and bone disorder (CKD- patients without hip fracture.4 Women who are 65
MBD).1 yr of age and older and have moderate renal dys-
Osteoporosis is a condition characterized by low function (eGFR ⬍ 60 ml/min per 1.73 m2) are also
bone mass leading to reduced bone strength and an at an increased risk of hip fractures.5 In addition to
increased risk of fractures. Hip, spine, and wrist are the traditional risk factors, several risk factors for
most commonly affected. The WHO definition of low BMD have been identified in the CKD popula-
osteoporosis is based on bone mineral density tion such as renal osteodystrophy, ethnicity, trans-
(BMD) measurements. The NIH consensus state- plant status, and duration of dialysis.6
ment refers to osteoporosis as a skeletal disorder
characterized by compromised bone strength pre-
Correspondence: Harmeet Singh, Division of Nephrology, Brody
disposing to increased risk of fracture.
School of Medicine, 2355 W. Arlington Boulevard, Greenville, NC
Osteoporosis and renal osteodystrophy may co- 27834. E-mail: [email protected]
exist in elderly patients with CKD, which makes the
Copyright 䊚 2009 by the American Society of Nephrology

American Society of Nephrology Geriatric Nephrology Curriculum 1


EVALUATION diagnostic test for determining the type of bone disease asso-
ciated with CKD is iliac crest bone biopsy with double tetracy-
Assessment of bone strength in an elderly patient with CKD is cline labeling and bone histomorphometric analysis. Bone his-
complex. The initial evaluation of CKD-related bone and min- tology shows information about turnover, mineralization, and
eral disorders should include serum biomarkers and noninva- volume and may be helpful in guiding therapy. Bone biopsy is
sive imaging. Bone histomorphometric analysis might be not performed routinely in clinical practice because it is inva-
needed in some cases. Bone strength is represented by two sive, available in limited centers, and requires special expertise
main features: bone density and bone quality. Bone density can and an experienced pathologist for interpretation. However, it
be measured using several different radiologic techniques, but should be considered in patients with Stage 5 CKD who have
bone quality is difficult to assess because it depends on archi- (1) fractures with minimal or no trauma (pathologic frac-
tecture, turnover, and mineralization. Correlation of BMD tures); (2) intact plasma PTH levels between 100 and 500
with bone histology is poor. pg/ml (11.0 to 55.0 pmol/L; in CKD Stage 5) with coexisting
conditions such as unexplained hypercalcemia, severe bone
Serum Biomarkers pain, or unexplained increases in bone alkaline phosphatase
Surrogate markers of bone metabolism such as serum intact activity; and (3) suspected aluminum bone disease, based on
PTH, calcium (preferably ionized), phosphorus, alkaline clinical symptoms or history of aluminum exposure.2 Bone
phosphatases, and bicarbonate levels should be obtained ini- biopsy should be strongly considered before starting bisphos-
tially. High intact PTH levels may correlate with high turnover phonate therapy in patients with Stage 5 CKD.
bone disease. The optimal target level for intact PTH in CKD is
not known.
TREATMENT
Noninvasive Imaging
Several imaging tools are available to assess bone health includ- Whether the standard agents used for osteoporosis in general
ing DEXA scan, quantitative computed tomography (qCT), population can be applied to patients with CKD is unclear. The
and heel ultrasound. The value of BMD in evaluation of CKD- management of osteoporosis includes addressing modifiable
related bone disease is not well established. risk factors and using pharmacologic agents.
DEXA scan is most commonly used to assess bone mass. It (1) Smoking cessation is strongly recommended. Excess al-
can only detect overall density but not quality of the bone. cohol consumption should be avoided.
According to WHO definition, a T-score of ⫺2.5 is defined as (2) Exercise: Moderate weight-bearing physical activity has
osteoporosis. The extension of this definition to groups other been associated with improvement in bone density and reduc-
than postmenopausal Caucasian women is controversial. Be- tion in risk of hip fractures. However, in older women, the
cause of vascular and extraskeletal calcifications in CKD, the benefit is modest.
interpretation may be affected by artifacts. However, it is com- (3) Calcium and vitamin D requirements: Vitamin D defi-
monly used because of low cost, accuracy, and wide availabil- ciency is common in older adults especially during winter. It
ity. The distal radius may be the preferred site in CKD pa- may be related to reduced synthesis, inadequate intake, dietary
tients.7 KDOQI recommends use of DEXA to measure BMD in restrictions, and inactive lifestyle in dialysis patients. 25(OH-
patients with fractures and in those with known risk factors for )Vitamin D deficiency is associated with increased iPTH levels,
osteoporosis. reduced BMD, and increased rate of hip fractures. The extra-
renal effects of vitamin D are also receiving increasing atten-
Quantitative CT tion. Low 25(OH)vitamin D levels may be associated with in-
Information regarding use of qCT in assessing BMD in pa- creased risk of cardiovascular events in patients with peritoneal
tients with CKD is limited. It has the advantage of distinguish- dialysis (PD)9 and higher risk of myocardial infarction in
ing outer dense cortical bone from inner spongy trabecular men.10 The optimal serum 25(OH)vitamin D concentration
bone. Hyperparathyroidism leads to sclerotic thickening of needed to maintain bone health has not been established. Cal-
trabecular bone with increased BMD but stimulates resorption cium and vitamin D supplementation has been reported to
in cortical bone with significant reductions in BMD. Thinning result in small improvement in hip bone density. It did not
of the cortical bone results in increased risk of fractures. A significantly reduce hip fractures but increased the risk of kid-
small study has reported association of radius cortical param- ney stones.11 In CKD Stages 3 and 4, KDOQI recommends
eters with fractures in hemodialysis patients. qCT is more ex- maintaining 25(OH)vitamin D levels above 30 ng/ml by sup-
pensive than DEXA and results in greater exposure to radia- plementation with ergocalciferol. In CKD Stages 3 to 5, total
tion.8 elemental calcium intake including dietary and calcium based
binders should not exceed 2000 mg/d. The National Osteopo-
Bone Biopsy rosis Foundation recommends a daily calcium intake of 1200
It remains the gold standard for diagnosis of renal osteodys- mg and vitamin D intake of 800 to 1000 IU/d for adults 50 and
trophy and assessment of bone architecture. The most accurate older.

2 Geriatric Nephrology Curriculum American Society of Nephrology


(4) Role of bisphosphonates: Bisphosphonates are effective hypercalcemia and increased uric acid levels.16 Safety data in dial-
in treating osteoporosis, but their use in CKD Stages 4 and 5 is ysis patients or severe renal insufficiency are not available.
controversial. No data are available that proves that bisphos- The above case shows difficult real-life situations faced by
phonates reduce risk of fractures in dialysis patients with os- clinicians where no guidelines are available to recommend de-
teoporosis. Use of bisphosphonates in CKD has been linked to finitive treatment strategies. If this patient develops bone pain
nephrotic syndrome, acute renal failure, and progressive renal and pathologic fractures, bone biopsy could be considered.
disease. It is imperative to make a correct diagnosis before Some authors have suggested careful use of low-dose bisphos-
treatment is initiated because bisphosphonates are not indi- phonates in this situation, although data are extremely limited.
cated in adynamic or osteomalacic bone disease. Bone biopsy is A bone biopsy should be obtained to rule out adynamic disease
recommended before using bisphosphonates. Alterations in before considering treatment with bisphosphonates. Conser-
calcium, phosphorous, vitamin D deficiency, and hyperpara- vative measures including mild weight-bearing exercise,
thyroidism should be addressed before starting bisphospho- smoking cessation, and avoiding alcohol should be encour-
nates. Bisphosphonates have been used in the setting of renal aged. Fall prevention risk assessment and counseling should be
transplant to prevent bone loss in the posttransplant period. In provided.
small studies, alendronate, risedronate, clodronate, and iban-
dronate have been shown to be safe to use in CKD. Some au-
thors recommend using bisphosphonates for short periods of CONCLUSION
time (2 to 3 yr) in the CKD population, although there is no
evidence that it will result in reduction of fractures.12 Age-related bone loss is an important part of assessment in the
(5) Calcitonin: Calcitonin binds to osteoclasts and inhibits growing aging dialysis and CKD population. Osteoporosis and
bone resorption. It has a low side effect profile and can be given renal osteodystrophy may coexist in the elderly CKD population,
intranasally. It could help protect bone mass when used along making diagnosis and management complicated. The DEXA scan
with calcium and vitamin D supplementation, especially in the is most commonly used to diagnose osteoporosis. Management
posttransplant population.13 of osteoporosis includes addressing modifiable risk factors and
(6) Estrogen/progestin therapy has fallen out of favor be- pharmacologic approaches. Patients with CKD are at greater risk
cause of increased risk of breast cancer, stroke, and thrombo- for osteoporosis than general population. Osteoporotic fractures
embolism. It may be an option in women who are not able to have significant morbidity and negative impact on quality of life.
tolerate other forms of treatment. More data are needed to address the optimal management of
(7) Selective estrogen receptor modulators (SERM): In a mineral and bone disorders in the elderly.
subgroup analysis, raloxifene was associated with a greater in-
crease in spine BMD, a reduction in vertebral fractures, and no
effect on nonvertebral fractures compared with placebo. It was TAKE HOME POINTS
safe to use in women with osteoporosis and mild to moderate
• Bone disease in elderly persons with CKD is complicated by co-exis-
CKD over the 2- to 3-yr observation period.14 Some SERMs tence of renal osteodystrophy and osteoporosis
may increase the risk of deep venous thrombosis (DVT) and • Patients with CKD-MBD and osteoporosis are at increased risk of
pulmonary embolism. Therefore, these agents should be fractures and higher risk of cardiovascular disease
avoided in women with active or history of DVT. They may • Hip fractures in dialysis patients is associated with doubling of mortality
• Bone biopsy is the gold standard for diagnosis of renal osteodystrophy
also increase the risk of death caused by stroke in postmeno-
and assessment of bone architecture
pausal women with coronary heart disease. More information • Although bisphosphonates are increasingly being used in the post renal
is needed on long-term safety and efficacy of these agents in transplant setting for prevention of bone loss, there is no evidence that
CKD. it results in reduction of incidence of fractures
(8) Cinacalcet: It is a calcimimetic that increases the sensi-
tivity of calcium-sensing receptors in the parathyroid gland to
calcium, thereby playing a role in regulation of PTH levels. It DISCLOSURES
helps in improving bone histology, reducing bone turnover, None.
and reducing fibrosis in patients with secondary hyperparathy-
roidism. Some studies indicated that it may lower the risk of
parathyroidectomy, fracture, and cardiovascular hospitaliza- REFERENCES
tion.15 Side effects include gastrointestinal intolerance and hy-
pocalcemia, which may lead to seizures. *Key References
(9) Anabolic agents: A new class of anti-osteoporosis drugs are 1. Moe S, Drüeke T, Cunningham J, Goodman W, Martin K, Olgaard K,
Ott S, Sprague S, Lameire N, Eknoyan G; Kidney Disease: Improving
now available that stimulate bone formation. A subgroup analysis
Global Outcomes (KDIGO): Improving global outcomes (KDIGO). Def-
of patients with mild to moderate CKD included in the fracture inition, evaluation, and classification of renal osteodystrophy: a posi-
prevention trial showed that teriparatide significantly increased tion statement from Kidney Disease: Improving Global Outcomes
lumbar spine and femoral neck BMD. Adverse effects included (KDIGO). Kidney Int 69: 1945–1953, 2006*

American Society of Nephrology Geriatric Nephrology Curriculum 3


2. NKF: K/DOQI clinical practice guidelines for bone metabolism and 10. Giovannucci E, Liu Y, Hollis BW, Rimm EB: 25-hydroxyvitamin D and
disease in chronic kidney disease. Am J Kidney Dis 42(Suppl 3): risk of myocardial infarction in men: a prospective study. Arch Intern
S1–S202, 2003* Med 168: 1174 –1180, 2008
3. Alem AM, Sherrard DJ, Gillen DL, Weiss NS, Beresford SA, Heck- 11. Jackson RD, LaCroix AZ, Gass M, Wallace RB, Robbins J, Lewis CE,
bert SR, Wong C, Stehman-Breen C: Increased risk of hip fracture Bassford T, Beresford SA, Black HR, Blanchette P, Bonds DE, Brunner
among patients with end-stage renal disease. Kidney Int 58: 396 – RL, Brzyski RG, Caan B, Cauley JA, Chlebowski RT, Cummings SR,
399, 2000 Granek I, Hays J, Heiss G, Hendrix SL, Howard BV, Hsia J, Hubbell FA,
4. Mittalhenkle A, Gillen DL, Stehman-Breen CO: Increased risk of mor- Johnson KC, Judd H, Kotchen JM, Kuller LH, Langer RD, Lasser NL,
tality associated with hip fracture in the dialysis population. Am J Limacher MC, Ludlam S, Manson JE, Margolis KL, McGowan J, Ock-
Kidney Dis 44: 672– 679, 2004 ene JK, O’Sullivan MJ, Phillips L, Prentice RL, Sarto GE, Stefanick ML,
5. Ensrud KE, Lui LY, Taylor BC, Ishani A, Shlipak MG, Stone KL, Cauley Van Horn L, Wactawski-Wende J, Whitlock E, Anderson GL, Assaf AR,
JA, Jamal SA, Antoniucci DM, Cummings SR; Osteoporotic Fractures Barad D; Women’s Health Initiative Investigators: Women’s Health
Research Group: Osteoporotic Fractures Research Group. Renal func- Initiative Investigators. Calcium plus vitamin D supplementation and
tion and risk of hip and vertebral fractures in older women. Arch Intern the risk of fractures. N Engl J Med 354: 669 – 683, 2006
Med 167: 133–139, 2007 12. Miller PD: Is there a role for bisphosphonates in chronic kidney dis-
6. Stehman-Breen CO, Sherrard DJ, Alem AM, Gillen DL, Heckbert SR, ease? Semin Dial 20: 186 –190, 2007*
Wong CS, Ball A, Weiss NS: Risk factors for hip fracture among 13. El-Agroudy AE, El-Husseini AA, El-Sayed M, Mohsen T, Ghoneim MA:
patients with end-stage renal disease. Kidney Int 58: 2200 –2205, 2000 A prospective randomized study for prevention of postrenal transplan-
7. Yamaguchi T, Kanno E, Tsubota J, Shiomi T, Nakai M, Hattori S: tation bone loss. Kidney Int 67: 2039 –2045, 2005
Retrospective study on the usefulness of radius and lumbar bone 14. Ishani A, Blackwell T, MORE Investigators: The effect of raloxifene
density in the separation of hemodialysis patients with fractures from treatment in postmenopausal women with CKD. J Am Soc Nephrol 19:
those without fractures. Bone 19: 549 –555, 1996 1430 –1438, 2008
8. Jamal SA, Gilbert J, Gordon C, Bauer DC: Cortical pQCT measures 15. Cunningham J, Danese M, Olson K, Klassen P, Chertow GM: Effects of
are associated with fractures in dialysis patients. J Bone Miner Res the calcimimetic cinacalcet HCl on cardiovascular disease, fracture,
21:543–548, 2006 and health-related quality of life in secondary hyperparathyroidism.
9. Wang AY, Lam CW, Sanderson JE, Wang M, Chan IH, Lui SF, Sea MM, Kidney Int 68: 1793–1800, 2005
Woo J: Serum 25-hydroxyvitamin D status and cardiovascular out- 16. Miller PD, Schwartz EN, Chen P, Misurski DA, Krege JH: Teriparatide
comes in chronic peritoneal dialysis patients: a 3-y prospective cohort in postmenopausal women with osteoporosis and mild or moderate
study. Am J Clin Nutr 87:1631–1638, 2008 renal impairment. Osteoporos Int 18: 59 – 68, 2007

4 Geriatric Nephrology Curriculum American Society of Nephrology


REVIEW QUESTIONS: BONE DISEASE AND CALCIUM 4. Which of the following statements is false
ABNORMALITIES IN ELDERLY PATIENTS WITH CKD a. Calcitonin has been shown to protect bone mass in post
transplant patients when used along with bisphospho-
1. Incidence of hip fractures among dialysis patients compared nates, calcium and vitamin D
with general population is about b. SERMs increase the risk of DVT and thrombosis therefore
a. Two-fold should not be used in patients with active or history of
b. Four-fold DVT
c. Six-fold c. Some studies indicate that Cinacalcet may lower the risk of
d. Eight-fold parathyroidectomy, fracture and cardiovascular hospital-
ization
2. Risk of mortality in dialysis patients from hip fracture com- d. A subgroup analysis of the patients with mild to moderate
pared with those without hip fracture is CKD included in fracture prevention trial showed that
a. Not affected treipartide significantly increased lumbar spine density
b. Doubled and femoral neck BMD
c. Tripled
d. Quadroupled 5. Bisphophonates are increasingly being used in the posttrans-
plant setting. Bisphosphonates
3. Bone strength is a function of bone density and quality; the a. Reduce incidence of fractures in the posttransplant period
best test for the assessment of bone quality is b. Reduce posttransplant bone loss
a. DEXA scan c. Can be safely used in failed transplant patients who return
b. Quantitative CT to dialysis
c. Bone biopsy d. Can potentally benefit patients with adynamic bone dis-
d. Serum intact PTH ease

American Society of Nephrology Geriatric Nephrology Curriculum 5


Chapter 15: Anemia in the Elderly With CKD
Julio C. Vijil Jr
Section of Nephrology, University of Illinois at Chicago, Chicago, Illinois

Anemia is a common sequela of chronic kidney dis- lower Hg levels must always be considered when mak-
ease (CKD). As GFR declines, the risk of anemia ing the diagnosis of anemia. These include altitude of
increases, particularly when the GFR is ⬍60 ml/min residence, smoking status, normal fluctuations in
per 1.73 m2.1 Anemia is also common in the geriat- plasma volume, and other patient-centered factors.
ric population, with a prevalence of ⬎10% in pa- These are not routinely considered when Hg concen-
tients greater than 65 yr old (Figure 1).2 This poses a tration is reported by clinical laboratories, which typ-
significant challenge in the diagnosis and manage- ically only correct for age and sex. In addition, refer-
ment of elderly patients with anemia who have con- ence ranges for Hg may vary from laboratory to
current CKD. The following review discusses the laboratory across all subjects. Thus, results of Hg test-
definition, diagnosis, management, and outcomes ing must always be interpreted in the full context of
of anemia in elderly patients with CKD. the individual patient.

DEFINITION OF ANEMIA IN THE ELDERLY CAUSES OF ANEMIA IN THE ELDERLY

Defining normal hemoglobin levels in the elderly Because of the increased prevalence of lower Hg levels
population is important for purposes of establish- in the geriatric population, as well as increased risk of
ing a diagnosis and monitoring treatment effects comorbidities leading to anemia, determining the
and outcomes. In 1968, an expert panel from the cause of anemia can be challenging, especially in the
World Health Organization recommended normal face of CKD. Determining whether CKD is the cause
hemoglobin (Hg) levels be ⬎13.0 g/dl for men and of anemia, whether there is another explanation, or
⬎12.0 g/dl for nonpregnant women.3 Although the perhaps whether there are several etiologies of anemia
accuracy of these definitions has been called into has important implications for treatment and thus
question, more recent population-based studies, should be investigated rigorously before simply begin-
including NHANES III and the Kaiser-Scripps da- ning therapy with an erythropoietic stimulating agent
tabase, have been relatively consistent with the pro- (ESA). Below is a discussion of the diagnosis of anemia
posed levels.2,4 in the elderly with CKD.
As previously mentioned, anemia is quite common
in the elderly population. Using the WHO definition
of anemia, NHANES III data have shown that the CKD AND ERYTHROPOIETIN PRODUCTION
prevalence of anemia in subjects ⱖ65 yr old is
⬎10.6%, significantly higher than that of the general Anemia attributable to CKD was suspected in ap-
population. However, when present in the elderly, proximately 8% of all anemia cases in the elderly,
anemia tends to be mild. From the NHANES III data, according to NHANES III. Furthermore, some
⬍3% of subjects 65 yr of age and older had an Hg studies have shown lower than expected erythro-
⬍11.0 g/dl. Whether these modestly lower values of poietin (Epo) levels for the degree of anemia in ge-
Hg correlate with poorer outcomes is not known.
Attention should be paid to racial and ethnic dif-
ferences and other patient-centered factors in anemia Correspondence: Julio C. Vijil Jr, MD, MPH, Department of
Medicine, University of Illinois at Chicago, Section of Nephrology,
prevalence in the elderly as well. Data from NHANES MC 793, 820 South Wood Street, Room 417W, Chicago, IL
III showed that, among individuals 65 yr of age and 60612. Phone: 312-996-6736; Fax: 312-355-0418; E-mail: jvijil1@
older, African Americans have a higher prevalence of uic.edu

anemia (Figure 2).2 Other factors that contribute to Copyright 䊚 2009 by the American Society of Nephrology

American Society of Nephrology Geriatric Nephrology Curriculum 1


Figure 1. Prevalence of anemia in the United States
by age (NHANES III). Prevalence of WHO-defined ane-
mia (hemoglobin ⬍13.0 g/dl for males and ⬍12.0 g/dl
for nonpregnant females) in the United States, by age
and sex. NHANES III, Third US National Health and
Nutrition Examination Survey; WHO, World Health Or-
ganization. Data from first two phases of NHANES III,
1988 –1994. (Reprinted from Steensma DP, Tefferi A:
Anemia in the elderly: how should we define it, when
does it matter, and what can be done? Mayo Clin Proc
82: 958 –966, 2007).16

riatric patients. Some have theorized that the elderly have a eGFR alone cannot be used reliably to exclude anemia of CKD
decreased ability to produce Epo in response to hypoxia or that in the geriatric population.
there is decreased hypoxia-sensing in the kidney with increas-
ing age and thus a higher Epo requirement to maintain the
same Hg level.5,6 However, one study has shown that normal,
NUTRITIONAL ANEMIA
healthy elderly individuals have the capacity to produce ade-
quate Epo in response to phlebotomy.7 Whether this translates
Inadequate nutrition is a common problem in the elderly. Iron
into an ability to maintain Epo production and Hg levels in the
deficiency, both nutritional and because of chronic blood loss,
setting of chronic blood loss or other perturbations in Hg con-
accounts for most cases of nutritional anemia, representing
centration is not known. Thus, to date, there is no solid evi-
about one third of anemia cases in the elderly in the NHANES
dence that the elderly have a decreased ability to produce Epo
III cohort. Diagnosis of iron deficiency should be made with
when kidney function is normal.
free iron, total iron binding capacity, and ferritin levels. As is
One possible explanation for the observation of lower than
the case in the general CKD population with iron deficiency,
expected Epo levels is that CKD may be present to some degree
ferritin levels are not reliably low because of chronic inflam-
in some elderly subjects despite the results of GFR estimating
mation and thus should not be used to exclude iron deficiency
equations. This may be because of the fact that the equations to
anemia. Similarly, mean corpuscular volume (MCV) is not
estimate GFR are suspect in the elderly as the population of
always low in CKD, despite iron deficiency anemia, so a nor-
subjects in the studies used to create these equations did not
mal MCV should not exclude the diagnosis. Based on early
include very elderly patients. Second, the correlation of esti-
studies, an iron/total iron binding capacity (TIBC) ratio ⱕ16%
mated GFR to renal endocrine function is not as well studied in
or a serum ferritin ⬍12 ng/ml is necessary to make the diag-
the elderly. For the general population, anemia caused by CKD
nosis of iron deficiency anemia.8,9
can present as early as eGFR of about 60 ml/min per 1.73 m2.
Other causes of nutritional anemia must be investigated in
Whether this relationship between eGFR and Epo production
the elderly also. Particularly, folate and B12 deficiencies are
is similar for the elderly population is not known. Therefore,
more frequently present than in the general population. One
study found that ⬎10% of the elderly had borderline or low B12
levels.10 As was the case with iron deficiency, one must be care-
ful not to exclude the diagnosis of B12 or folate deficiency solely
by MCV when concurrent CKD is present.

MALIGNANCY

The elderly are at increased risk for malignancy. Several types


can cause anemia and must be considered whenever an elderly
patient presents with anemia. The two most important catego-
Figure 2. Prevalence of anemia in the United States by race and
ries are gastrointestinal (GI) malignancies and hematologic
ethnicity. Prevalence of WHO-defined anemia in US patients 65 yr
of age or older, by self-declared racial or ethnic group. NHANES
malignancies. GI malignancies lead to chronic blood loss and
III, Third US National Health and Nutrition Examination Survey; iron deficiency anemia, so stool testing for occult blood should
WHO, World Health Organization. Data from NHANES III, 1988 – be considered. However, colonoscopy should be considered
1994. (Reprinted from Steensma DP, Tefferi A: Anemia in the even if the stool for occult blood is negative in the presence of
elderly: how should we define it, when does it matter, and what an unexplained iron deficiency anemia in an elderly patient
can be done? Mayo Clin Proc 82: 958 –966, 2007).16 with CKD.

2 American Society of Nephrology American Society of Nephrology


Hematologic malignancies in general can cause anemia be- agents to raise Hg levels is documented, the benefits of raising
cause of disruption of hematopoiesis in the bone marrow. Two the Hg are still somewhat murky.
that have a particular predilection for elderly patients are my- Most studies documenting the benefit of anemia treatment
elodysplastic syndrome (MDS) and multiple myeloma. Both have been conducted in ESRD patients, whereas there are fewer
diseases have peaks in the seventh decade of life and can be studies in predialysis patients with CKD. Most of these studies
insidious in onset, with only subtle laboratory findings to point have had a reasonable number of subjects ⱖ65 yr of age, so
to the diagnosis. Several studies have shown that MDS is prob- results may be generalizable to the geriatric population to some
ably underdiagnosed in this population and may account for a degree. Improvement in health-related quality of life
significant number of cases of anemia in the elderly that do not (HRQOL), cognition, left ventricular mass, and decreased
have an identifiable cause.11 Multiple myeloma can also be need for transfusions have been documented in the general
easily missed in the workup of a patient with anemia. Thus, an ESRD population, and it is reasonable to believe that the same
elderly patient with CKD and anemia, in most cases, should be benefits would be conferred on the elderly ESRD population.
screened for the presence of paraproteins. The benefits of anemia treatment in predialysis CKD are more
controversial. Although a few studies have shown higher
HRQOL, lower left ventricular mass, and a possible decrease in
APPROACH TO DIAGNOSIS OF ANEMIA IN THE progression of CKD in those subjects treated for anemia, the
ELDERLY evidence is not strong, and no study has shown a survival ad-
vantage to treatment.
Initial evaluation of anemia in the elderly should include a When reviewing the benefits of anemia therapy, one ques-
review of the complete blood count, including red blood cell tion that remains is: What level of Hg should be targeted? At
morphology and parameters such as MCV. Worrisome find- this point, the target Hg level for individuals with CKD is not
ings on the peripheral blood smear should prompt consider- agreed on, particularly for the elderly. Two recent studies have
ation of referral to Hematology. Abnormal MCV should be cast doubt on the goal of targeting normal Hg levels. The Cor-
worked up appropriately (e.g., high MCV warrants folate, B12, rection of Hemoglobin and Outcomes in Renal Insufficiency
and homocysteine levels, consideration of alcohol or medica- (CHOIR) trial was a randomized study of predialysis patients
tions, liver or thyroid disease). with CKD and anemia. One group was targeted to an Hg of
A normal MCV should not completely rule out either high 13.5 g/dl, whereas another group was targeted to an Hg of 11.3
or low MCV causes of anemia, although their diagnosis is less g/dl.14 Average subject age was about 66 yr old. Achieved Hg
likely. Renal function should be estimated to determine the for the higher Hg group was actually lower, just below 13 g/dl.
possibility of anemia caused by renal disease. Concomitant There was increased risk of one of the primary or secondary
iron deficiency must always be evaluated in the face of CKD outcomes in the higher Hg group compared with the lower Hg
and worked up if present. An appropriately elevated Epo level group, with no benefit in HRQOL (Figure 3). The Cardiovas-
with normal iron studies should prompt referral to Hematol- cular Risk Reduction in Early Anemia Treatment with Epoetin
ogy to investigate for hematologic malignancies. If the workup Beta (CREATE) also studied higher Hg level in predialysis pa-
of an elderly CKD patient’s anemia has not yielded a diagnosis tients with CKD.15 No benefits were shown in the group tar-
at this point, consideration should be given to obtaining an geted to higher Hg, although excess risk of events was not sta-
Epo level because, as previously discussed, relative Epo defi- tistically significant in this study. Mean age of subjects in
ciency may be present in the elderly even with near “normal” CREATE was about 59 yr old.
eGFR. This excess mortality with the use of ESAs is of particular
importance in the elderly population with CKD. This is be-
cause the elderly are at higher risk of having the comorbidities
TREATMENT AND OUTCOMES associated with the vascular events that occurred more fre-
quently in the higher Hg groups. Thus, when treating anemia
After treating any underlying nutritional deficiencies noted of CKD in the elderly, one should not aim for high Hg levels,
earlier, and with persistent anemia, one should consider ther- especially if the patient has a history of any kind of vascular
apy for anemia of CKD. The mainstays of treatment are ESAs disease, pro-thrombotic state such as malignancy, or poorly
and iron. ESAs have been shown to be effective in raising Hg controlled hypertension. Based on the results of the two studies
levels appropriately in the elderly.12 Concomitant iron defi- mentioned previously, the evidence supports restoring Hg lev-
ciency being frequently present in this group, iron supplemen- els to a level between 10.0 and 12.0 g/dl.
tation is commonly part of treatment. Several formulations of
oral iron are available for consideration. In some patients with
CKD, oral iron is ineffective in raising available iron stores, CONCLUSION
especially in the elderly who frequently show poorer GI ab-
sorption of iron.13 Such patients may benefit from the intro- The diagnosis and workup of anemia in an elderly patient with
duction of intravenous iron. Although the ability of these CKD is, in the end, the same as for any patient. However,

American Society of Nephrology American Society of Nephrology 3


Figure 3. Primary and secondary endpoints for higher versus lower Hg groups in the CHOIR Study. Kaplan–Meier estimates of the
probability of the primary composite endpoint and secondary endpoints of individual components— hospitalization for congestive
heart failure (CHF) without renal replacement therapy (RRT), myocardial infarction, stroke, and death. (Reprinted from Singh AK, Szczech
L, Tang KL, et al.: Correction of anemia with epoetin alfa in chronic kidney disease. N Engl J Med 355: 2085–2098, 2006).14

epidemiologic information and special diagnostic consider- • ESAs and iron supplementation are effective in treating anemia of CKD
in this population
ations must be taken into account since the elderly are more • Hg should be targeted to a level between 10.0 and 12.0 g/dl
likely to have a concomitant cause of anemia in the face of
CKD. Treatment with ESAs is effective, and iron supplemen-
tation, initially oral but intravenous if ineffective, should be
DISCLOSURES
considered in the regimen given the frequency of iron defi-
None.
ciency in these individuals. When choosing a target Hg level, it
is important to remember that higher Hg levels may be associ-
ated with increased cardiovascular and thrombotic events. The
REFERENCES
elderly are at risk for conditions that raise the risk of these
events, so anemia must be treated judiciously. A target Hg of *Key References
10.0 to 12.0 g/dl is an acceptable target based on available evi- 1. Nurko S: Anemia in chronic kidney disease: causes, diagnosis, treat-
dence. ment. Cleve Clin J Med 73: 289 –297, 2006*
2. Guralnik JM, Eisenstaedt RS, Ferrucci L, Klein HG, Woodman RC:
Prevalence of anemia in persons 65 years and older in the United
States: evidence for a high rate of unexplained anemia. Blood 104:
TAKE HOME POINTS
2263–2268, 2004
• Anemia in the elderly with CKD is often multifactorial, with iron defi- 3. World Health Organization: Nutritional anemia: report of a WHO
ciency being common Scientific Group. World Health Organ Tech Rep Ser 405: 5–37, 1968
• Special consideration must be given to diagnoses common in this 4. Beutler E, Waalen J: The definition of anemia: what is the lower limit
population, including GI blood loss and hematologic causes (MDS, of normal of the blood hemoglobin concentration? Blood 107: 1747–
multiple myeloma) 1750, 2006

4 American Society of Nephrology American Society of Nephrology


5. Ble A, Fink JC, Woodman RC, Klausner MA, Windham BG, Guralnik hospitalized cognitively different geriatric patients. Gerontology 46:
JM, Ferrucci L: Renal function, erythropoietin, and anemia of older 323–327, 2000
persons: the InCHIANTI study. Arch Intern Med 165: 2222–2227, 2005 12. Agarwal A, Silver MR, Walczyk M, Liu W, Audhya P: Once-monthly
6. Ershler WB, Sheng S, McKelvey J, Artz AS, Denduluri N, Tecson J, darbepoetin alfa for maintaining hemoglobin levels in older pa-
Taub DD, Brant LJ, Ferrucci L, Longo DL: Serum erythropoietin and tients with chronic kidney disease. J Am Med Dir Assoc 8: 83–90,
aging: a longitudinal analysis. J Am Geriatr Soc 53: 1360 –1365, 2005 2007
7. Goodnough LT, Price TH, Parvin CA: The endogenous erythropoietin 13. Saltzman JR, Russell RM: The aging gut. Nutritional issues. Gastroen-
response and the erythropoietic response to blood loss anemia: the terol Clin North Am 27: 309 –324, 1998
effects of age and gender. J Lab Clin Med 126: 57– 64, 1995 14. Singh AK, Szczech L, Tang KL, Barnhart H, Sapp S, Wolfson M, Reddan
8. Bainton DF, Finch CA: The diagnosis of iron deficiency anemia. Am J D; CHOIR Investigators: Correction of anemia with epoetin alfa in
Med 37: 62–70, 1964 chronic kidney disease. N Engl J Med 355: 2085–2098, 2006*
9. Jacobs A, Worwood M: Ferritin in serum. Clinical and biochemical 15. Drüeke TB, Locatelli F, Clyne N, Eckardt KU, Macdougall IC, Tsakiris
implications. N Engl J Med 292: 951–956, 1975 D, Burger HU, Scherhag A; CREATE Investigators: Normalization of
10. Loikas S, Koskinen P, Irjala K, Löppönen M, Isoaho R, Kivelä SL, hemoglobin level in patients with chronic kidney disease and anemia.
Pelliniemi TT: Vitamin B12 deficiency in the aged: a population-based N Engl J Med 355: 2071–2084, 2006*
study. Age Ageing 36: 177–183, 2007 16. Steensma DP, Tefferi A: Anemia in the elderly: how should we define
11. Beloosesky Y, Cohen AM, Grosman B, Grinblat J: Prevalence and it, when does it matter, and what can be done? Mayo Clin Proc 82:
survival of myelodysplastic syndrome of the refractory anemia type in 958 –966, 2007*

American Society of Nephrology American Society of Nephrology 5


REVIEW QUESTIONS: ANEMIA IN THE ELDERLY b. In elderly patients, estimated GFR correlates better with
WITH CKD Epo production than in nonelderly patients
c. One study has shown that normal, healthy, elderly subjects
1. According to NHANES III data, the prevalence of anemia in could not produce adequate Epo in response to phlebot-
persons ⱖ65 yr of age is approximately: omy
a. 1% d. To date, there is insufficient evidence to conclude that the
b. 5% elderly have a diminished capacity to produce Epo in re-
c. 10% sponse to anemia
d. 20%
4. Which potential benefit of ESA therapy for predialysis CKD
patients has NOT been demonstrated in any study to date?
2. The most common cause of anemia in the elderly is:
a. Decreased mortality
a. Nutritional anemia (including iron deficiency)
b. Decreased left ventricular hypertrophy
b. Chronic kidney disease
c. Decrease in progression of CKD
c. Occult malignancy
d. Increase in health-related quality of life
d. Hematologic abnormalities (including myelodysplastic
syndrome) 5. Results of the CHOIR and CREATE trials support a target Hg
level less than:
3. Which of the following is true regarding erythropoietin (Epo) a. 10 g/dl
production in the elderly? b. 11 g/dl
a. In patients with normal kidney function, Epo production c. 12 g/dl
is decreased in the elderly compared with the nonelderly d. 13 g/dl

6 American Society of Nephrology American Society of Nephrology


Chapter 16: Disorders of Serum Sodium Concentration
in the Elderly Patient
Michael F. Michelis
Division of Nephrology, Lenox Hill Hospital, New York, New York

HYPONATREMIA tory that can produce information implicating


drugs that facilitate sodium excretion or stimulate
Disorders of serum sodium concentration are the AVP secretion. In addition, a thorough physical ex-
most common electrolyte abnormalities seen in the amination, including orthostatic BP changes, is es-
geriatric population.1 Furthermore, the develop- sential for the accurate determination of the pa-
ment of serum sodium abnormalities is associated tient’s body fluid status. This determination may be
with increased morbidity and mortality in affected especially difficult in elderly patients who may have
patients.2 Often, however, the severity of the pri- chronic changes in skin turgor that are more asso-
mary process contributing to the development of ciated with aging than they are with the state of
the abnormal serum sodium is responsible for the hydration.
unsatisfactory outcome. The most common disor-
Laboratory Diagnosis and Therapy
der of serum sodium concentration in the geriatric
When the history and physical are completed, as-
population is hyponatremia. Factors contributing
sessment of laboratory data are essential (Figure 1).
to the development of hyponatremia in the elderly
First, a plasma osmolality must be obtained to
include age-associated decreases in GFR and free
ensure that one is dealing with hypo-osmolar hypo-
water clearance, as well as sodium losses from de-
natremia. Normo-osmolar hyponatremia as can
creased activity of the renin-angiotensin-aldoste-
occur with certain compounds such as mannitol,
rone system and increased activity of natriuretic
hormones. The latter, however, may reflect the which can also cause hyperosmolality in normon-
early development of fluid retention as may occur atremic patients, and hyperosmolar hyponatremia
with excessive sodium intake from processed foods caused by hyperglycemia must be ruled out. Fur-
or subclinical cardiac disease. In addition, studies thermore, other serum studies are important to de-
have suggested increased vasopressin activity in cide whether levels of substances such as blood urea
nitrogen and uric acid are elevated and thereby con-
some elderly patients.
sistent with volume depletion or whether these lev-
Patients in outpatient settings exhibit hypona-
els seem to be diluted as would be seen with water
tremia in about 5% of those tested, with occurrence
excess occurring in the syndrome of inappropriate
rates increasing to as high as 20% in hospitalized
antidiuretic hormone secretion (SIADH). Finally,
geriatric patients and 30% in patients seen in inten-
urine sodium concentration and urine osmolality
sive care units.3 The most common cause of hypo-
must be measured to ascertain the endocrine and
natremia in this population involves abnormalities
renal responses to the hypo-osmolar state.
of secretion of the pituitary hormone arginine va-
In dehydrated patients, levels of urine sodium
sopressin (AVP), also called antidiuretic hormone
can be considered low (i.e., sodium conservation)
(ADH).
when the spot urine sodium concentration is ⬍20
mEq/L; in older patients, where sodium conserva-
Clinical Diagnosis
tion may be limited,4 levels up to 30 mEq/L may also
Successful evaluation of serum sodium abnormali-
ties in the elderly depends on obtaining a careful
history especially noting reports of weight loss that Correspondence: Michael F. Michelis, Director, Division of Ne-
can be associated with neoplastic disease, changes phrology, Lenox Hill Hospital, 100 East 77th Street, New York, NY
in the level of daily activities that can be associated 10075. E-mail: [email protected]

with endocrine abnormalities, and medication his- Copyright 䊚 2009 by the American Society of Nephrology

American Society of Nephrology Geriatric Nephrology Curriculum 1


HYPONATREMIA (DIFFERENTIAL DIAGNOSIS)

Confirm Hypoosmolality

Volume Assessment (Physical Examination)


and Urine Sodium Measurement

Hypovolemia Euvolemia Edema

Urine Na Urine Na
>30 mEq/L <20 mEq/L
Urine Na Urine Na Congestive Heart Failure
<20-30 mEq/L >20-30 mEq/L SIADH
Cirrhosis
Diuretic Use - Water Replacement
Vomiting Addison’s Disease Nephrotic Syndrome
Endocrine Deficiency
Diarrhea Polycystic Kidneys Hypoalbuminemia
Pancreatitis Bicarbonaturia
Diuretic Use Figure 1. Hyponatremia (differential diagnosis).

be considered indicative of some degree of conservation of generally treated by addressing the primary underlying abnor-
sodium. These urine sodium levels will often be associated with mality and using diuretic regimens.
urine osmolality values at least 1.5 times that of the plasma, In elderly hyponatremic patients who appear euvolemic
suggesting attempts at water conservation in response to fluid and have elevated urine sodium concentrations (⬎20 or 30
deficits. Such patients generally respond to the replacement of mEq/L) and elevated urine osmolalities suggesting inappropri-
intravascular volume with normal saline. Patients who appear ate water retention, a diagnosis of SIADH is often made.6 Here
dehydrated but who have elevated urine sodium levels (⬎30 it is important to rule out endocrine abnormalities such as
mEq/L) often have urine osmolalities closer to that of the hypoadrenalism and hypothyroidism. In addition, a careful
plasma levels and should be considered to have renal salt wast- search for drugs that stimulate AVP or facilitate the effects of
ing in the face of intravascular volume contraction. They AVP in the kidney should be undertaken. If these consider-
should also be treated with normal saline while attempts to ations are eliminated from diagnostic possibilities, the patient
diagnose the underlying abnormality are made. Various for- should be evaluated for other causes of SIADH. Table 1 lists
mulas such as the Adrogué-Madias formula (Figure 2) have some of the commonly associated central nervous system dis-
been used in an attempt to predict the increase in serum so- orders, tumors, and drugs associated with inappropriate AVP
dium that can occur when various concentrations of sodium secretion. Certain drug therapies such as SSRIs may pose a
replacement therapy are employed.5 If more or less than 1 L of special risk for the development of hyponatremia in the el-
replacement fluid is used, the change in serum sodium will derly, especially those who are older and smaller in body size.7
vary directly in proportion to the amount of fluid adminis-
tered. Table 1. Common causes of SIADH
When patients appear to be fluid overloaded or edematous, Central nervous system disease
they may have one of several edema-forming states, including Trauma
congestive heart failure, nephrotic syndrome, and cirrhosis. Stroke
Infection
The poor renal perfusion associated with any of these states is
Tumor
generally associated with urine sodium concentrations in the
Intracranial bleeding
lower ranges (⬍20 to 30 mEq/L) and a tendency to urine os- Neoplasms
molalities closer to that seen in the plasma. These disorders are Lung
Pancreas
ADROGUÉ-MADIAS FORMULA Prostate
Throat
Lymphoma
Infusate sodium concentration – Drugs
Patient serum concentration
Change in serum sodium concentration with 1L of infusate = SSRIs
Total body water + 1L Carbamazepine
Figure 2. The Adrogué-Madias Formula for the prediction of the Opiates
change in serum sodium that can occur following intravenous Cyclophosphamide
sodium replacement therapy. Mirtazapine

2 Geriatric Nephrology Curriculum American Society of Nephrology


If primary causes of the syndrome SIADH cannot be imme- conservative manner. Current recommendations are that se-
diately eliminated, patients should be treated with therapies rum sodium should not increase ⬎12 mEq/L over a 24-h pe-
that can reverse the hyponatremia.8 In patients with a history riod and no more than 18 mEq/L over a 48-h period.
with confounding factors such as the possible prior use of di-
uretic agents, a trial of normal saline administration may be Complex Clinical Syndromes
used. In patients with severe hyponatremia, i.e., serum sodium Finally, complicated situations have been described in which
concentrations ⬍110 mEq/L, it may be appropriate to use patients are hyponatremic and seem to have excess AVP activ-
multiple 100-ml intravenous aliquots of hypertonic (3%) sa- ity as well as sodium depletion syndromes.10 On attempts at
line to improve the serum sodium levels to avoid more serious correction with saline, including hypertonic saline, patients
complications of hyponatremia such as seizures or profound may exhibit a response in which volume correction shuts off
coma. In patients with serum sodium concentrations in the AVP secretion and causes a profound water diuresis with rapid
111- to 120-mEq/L range, therapies may include lesser increases in the serum sodium concentration and a tendency
amounts of hypertonic saline, and greater reliance on fluid toward too rapid correction. When this occurs, reversal ther-
restriction and agents that interfere with AVP effect, such as apy may be required with the administration of free water
demeclocycline and the new AVP receptor antagonists. With and/or AVP (usually as the AVP agonist desmopressin). In
lesser degrees of hyponatremia, i.e., serum sodium 121 to 129 addition, patients on drugs that stimulate AVP secretion may
mEq/L, fluid restriction, demeclocycline, and AVP receptor have the effects of these drugs wear off during the period that
antagonists can be used. therapies are undertaken, also resulting in unexpectedly rapid
Other approaches to hyponatremia include saline infusions correction.
and furosemide being used to replace existing hypotonic intra-
vascular fluid with fluid of a higher sodium concentration. In Conclusion
hyponatremic patients with central nervous system disease, ce- Disorders of serum sodium, especially hyponatremia, are com-
rebral salt wasting should be ruled out. This disorder is associ- mon in the geriatric population and often are related to excess
ated with natriuretic peptide responses as well as AVP effect, vasopressin secretion. A trial of saline administration may be
sodium loss, and signs of volume deficit and should be treated considered before standard approaches to therapy that now
with volume replacement. These hyponatremic patients often include vasopressin blocking drugs. Complex clinical syn-
exhibit unusually high urine sodium concentrations. dromes often occur and may be related in part to drugs asso-
ciated with inappropriate vasopressin secretion.
New Therapies
Over the last several years, a new class of drugs has been studied
that specifically block the effect of AVP on the collecting tu- HYPERNATREMIA
bule. One such drug that has been approved by the FDA for
clinical use, conivaptan, belongs to the group of drugs called Hypernatremia is a common occurrence in elderly patients.2
vasopressin receptor antagonists (vaptans), because they block The issues relating to the development of an increased serum
the binding of AVP to the AVP V2 receptor on the renal tu- sodium can involve deficient free water intake, abnormal reg-
bule.9 Conivaptan has been approved for use in hospitalized ulation of AVP secretion, decreased renal responsiveness to
patients in an intravenous preparation, and it has been ap- AVP secretion, and finally free water losses that are associated
proved for therapy of both euvolemic hyponatremia (e.g., SI- with inadequate replacement.
ADH) and hypervolemic hyponatremia (e.g., heart failure).
Conivaptan is rapidly acting and can be given as an intrave- Clinical Diagnosis
nous bolus, which can be followed by 1 d to as many as 4 d of The evaluation of elderly patients with hypernatremia starts
intravenous infusion. Frequent monitoring of the results of intra- with a careful history. Details in the history that are important
venous therapy should be done during the phase of active correc- involve decreases in weight, decreases in intake either because
tion, with serum sodium measurements every 4 to 6 h during the of lack of interest or availability, and history from caretakers
first day, perhaps two to three times a day during the second day, about types of fluids and types of nutritional additives used
and less frequently during the subsequent 2 d. With lesser degrees and/or laxative use. Physical examination would involve the
of hyponatremia, it is still unclear what the specific indications for assessment of volume status, with particular attention to the
therapy are, and what the benefits of changes in the level of hypo- degree of BP decrease in cases where severe hypernatremia is
natremia may be, but it has been suggested that central nervous associated with significant hypotension.
system function and gait stability may be adversely affected by
even mild to moderate degrees of chronic hyponatremia. Laboratory Diagnosis and Therapy
Hypernatremia can be defined as a serum sodium concentra-
Rate of Correction tion greater than the normal level, which is usually considered
Acute hyponatremia should be treated more aggressively, up to 145 mEq/L. In general, patients with hypernatremic
whereas chronic hyponatremia should be addressed in a more states can be segregated by their measured urine osmolality.

American Society of Nephrology Geriatric Nephrology Curriculum 3


The ratio of urine osmolality to plasma osmolality is useful in Table 2. Cerebral lesions that can affect hypothalamic
defining AVP responses. Also, as in the hyponatremic patient, function
assessment of spot urine sodium concentration will be helpful Trauma
to discern the type of hypernatremic state that has developed Infection
(Figure 3). Tumors
Variable but low urine osmolalities and variable urine so- Hystiocytosis
dium concentrations can be seen with central diabetes insipi- Vascular abnormalities
dus depending on the extent of the lesion, sodium intake, and
intravascular volume status. Central diabetes insipidus may be rogenic disorders should be initially approached therapeutically
associated with varying degrees of AVP deficiency, which can by avoiding, if present, the agent or agents associated with resis-
be the result of a variety of intracerebral lesions (Table 2). tance to the effects of AVP. Additional therapy for nephrogenic
Water deprivation tests with clinical follow-up and AVP diabetes insipidus can include decreases in the intravascular vol-
administration can be used to help diagnose central versus ume with a mildly restricted sodium diet and thiazide diuretics.
nephrogenic diabetes insipidus. Measurement of plasma AVP
levels can also be useful to differentiate central from nephro- Rate of Correction
genic diabetes insipidus.1 Patients with more isotonic urine In general, water deficits are calculated using a standard for-
and urine sodium concentrations that tend to be elevated, ⬎20 mula (Figure 4), and physicians are generally advised to correct
to 30 mEq/L, can have disorders associated with osmotic di- half of the deficit over the first 24 h.
uresis, diuretic therapy, and other instances of renal dysfunc- Patients in shock must be treated aggressively with rapid
tion where endocrine and renal responses cannot offset fluid volume replacement with normal saline. Once the patient
and sodium losses, which are mandated by osmolar excretion shows a normal BP, hypotonic replacement can be performed
or renal disease. Urine osmolalities that are clearly higher than at a more moderate rate. This replacement may consist of hy-
measured plasma osmolality and urine sodium levels ⬍20 to potonic saline, dextrose, and water with careful monitoring of
30 mEq/L generally are associated with endocrine and renal serum glucose or oral water administration. Rare cerebral le-
responses to extreme fluid losses. This can be seen with de- sions that damage or eliminate osmoreception can result in
creased thirst or water availability or gastrointestinal losses unusual clinical syndromes in which volume expansion results
such as vomiting and diarrhea and lack of water replacement in significant water diuresis and hypernatremia.11 As with hy-
particularly in warm environments as may be seen in a nursing ponatremic states, water deficits are more rapidly replaced in
home situation. Hypernatremia in the elderly is commonly more acute conditions and more slowly replaced when the pa-
associated with fluid loss, limitation in urinary concentrating tients are relatively asymptomatic and the condition has ex-
ability, intravascular volume depletion, and inadequate re- isted for some time. Too rapid dilute replacement therapy can
placement caused by decreased thirst, limited mentation, lim- be associated with the development of cerebral edema.12 Cen-
ited fluid availability, and/or inadequate fluid prescription. tral nervous system function should be monitored carefully.
Central nervous system disorders associated with lack of
AVP secretion are treated with intravenous or oral dilute fluid Conclusion
and, as needed, AVP replacement therapy. Patients with neph- Physiologic changes associated with aging and lack of mobility
may facilitate the development of hypernatremia in elderly pa-
HYPERNATREMIA (DIFFERENTIAL DIAGNOSIS) tients. Careful history and laboratory evaluation are essential
to determine the correct diagnosis and can include measure-
ment of plasma vasopressin levels. Calculation of water deficits
Urine Osmolality
will assist in developing a sound therapeutic plan and avoid
improper rates of correction.
CALCULATION FOR H2O DEFICIT

(70 kg Patient x 50% Total Body Water Factor for Older Patient) = 35L

NL TBW 35L X NL SNa 140 mEq/L = Present Body Water


Present SNa (e.g. 160 mEq/L)

Present Body Water = 30.6L


Deficit = 4.4L
Hypotonic Isotonic or Mildly Hypertonic
Urine Hypotonic Urine Urine

UNa Variable UNa >20-30 mEq/L UNa <20-30 mEq/L


NL = Normal
Complete or Partial Osmotic Diuresis GI Losses TBW = Total Body Water
Central Diabetes Insipidus Renal Disease Skin Losses SNa = Serum Sodium
Nephrogenic Diabetes Insipidus Diuretics Burns
Decreased Thirst
Figure 4. The calculation for H2O deficit in an elderly hypernatre-
Figure 3. Hypernatremia (differential diagnosis). mic patient is outlined.

4 Geriatric Nephrology Curriculum American Society of Nephrology


TAKE HOME POINTS The Kidney, 8th Ed., edited by Brenner BM, Philadelphia, Saunders
Elsevier, 2008, pp 459 –504*
• Differential diagnosis of hyponatremia should be performed with con- 2. Arinzon Z, Feldman J, Peisakh A, Zuta A, Berner Y: Water and sodium
firmation of hypoosmolality by means of measurement of plasma os- disturbances predict prognosis of acute disease in long term care frail
molality elderly. Arch Geront Geriatr 40: 317–326, 2005
• After volume assessment is attempted, spot urine sodium levels should 3. Miller M: Hyponatremia and arginine vasopressin dysregulation:
then be evaluated to separate salt conserving and salt losing states mechanisms, clinical consequences, and management. J Am Geriatr
• Urine osmolality will further aid in assessing whether appropriate re- Soc 54: 345–353, 2006*
sponses to the plasma dilution are occurring 4. Epstein M, Hollenberg NK: Age as a determinant of renal sodium
• Therapy should proceed in a manner formulated to avoid too rapid or conversation in normal man. J Lab Clin Med 87: 411– 417, 1976
overcorrection 5. Adrogué HJ, Madias NE: Hyponatremia. N Engl J Med 342:1581–
• Hypernatremia may be preventable in controlled situations such as a 1589, 2000*
nursing home by avoiding warm environments and excess protein in 6. Ellison DH, Berl T: The syndrome of inappropriate antidiuresis. N Engl
the diet, and provision of adequate hydration J Med 356: 2064 –2072, 2007*
• Normal saline may be required as initial therapy for hypotensive, hy- 7. Fabian TJ, Amico JA, Kroboth PD, Mulsant BH, Corey SE, Begley AE,
povolemic, hypernatremia Bensasi SG, Weber E, Dew MA, Reynolds CF 3rd, Pollock BG: Parox-
• Standard measurements including plasma osmolality, urine osmolality, etine-induced hyponatremia in older adults. A 12-week prospective
and spot urine sodium concentration will help in the differential diag- study. Arch Intern Med 164: 327–332, 2004
nosis 8. Verbalis JG, Goldsmith SR, Greenberg A, Schrier RW, Sterns RH:
• Too rapid correction or overcorrection should be avoided Hyponatremia treatment guidelines 2007: expert panel recommenda-
tions. Am J Med 120: S1–S21, 2007*
9. Metzger BL, DeVita MV, Michelis MF: Observations regarding the use
of the aquaretic agent conivaptan for treatment of hyponatremia. Int
DISCLOSURES Urol Nephrol 40: 725–730, 2008
None. 10. Mohmand HK, Issa D, Ahmad Z, Cappuccio JD, Kouides RW, Sterns
RH: Hypertonic saline for hyponatremia: risk of inadvertent overcor-
rection. Clin J Am Soc Nephrol 2: 1110 –1117, 2007*
11. DeRubertis FR, Michelis MF, Davis BB: Essential hypernatremia. Arch
REFERENCES Intern Med 134: 889 – 895, 1974
12. Ayus JC, Brennan S: Hypernatremia. In: Fluid, Electrolyte, and Acid-
*Key References Base Disorders, 2nd Ed., edited by Arieff AI, DeFronzo RA, New York,
1. Verbalis JG, Berl T: Disorders of water balance. In: Brenner & Rector’s Churchill Livingstone, 1995, pp 304 –317*

American Society of Nephrology Geriatric Nephrology Curriculum 5


REVIEW QUESTIONS: DISORDERS OF SERUM a. Change to normal saline infusion
SODIUM CONCENTRATION IN THE ELDERLY b. Stop the hypertonic saline infusion
PATIENT c. Administer a quantity of free water and consider vasopres-
sin therapy
1. Hyponatremia is common in elderly patients. Which of the
d. Increase the frequency of serum sodium measurements
factors listed is not a common cause for its development?
e. Closely observe the patient for CNS changes
a. Central nervous system disease
b. Neoplasm 5. Which factor would not contribute to the development of hy-
c. Living in warm climates pernatremia in elderly patients?
d. Antidepressant drug therapy a. Sodium intake
e. Endocrine disorders b. Immobility
c. Careful control of environmental temperature
2. Diagnostic evaluation of hyponatremic patients include mea- d. Drug therapy
surement of: e. Decreased alertness
a. Serum osmolality
b. Urine sodium level 6. What fluid or fluids would be first choice for a patient with
c. Urine osmolality hypernatremia and hypotension?
d. Serum BUN and uric acid level a. 1/2 normal saline
e. All of the above b. Dextrose and water alternating with normal saline
c. Normal saline
3. SIADH can result from the use of SSRIs. What would be ap- d. Dextrose and water alternating with 1/2 normal saline
propriate therapy in such a patient? e. Dextrose and water
a. Discontinue the drug
7. A patient with hypernatremia has an elevated plasma vaso-
b. Water restriction
pressin level, hypertonic urine, and urine sodium level ⬍20
c. Use a diuretic which works on the Loop of Henle
mEq/L. The most likely diagnosis would be:
d. Use a vasopressin blocking drug
a. Fluid losses via the skin or GI tract
e. All of the above
b. Complete central diabetes insipidus
4. A patient has a serum sodium level of 116 mEq/L. Hypertonic c. Osmotic diuresis with increased dietary protein
saline infusion is started. In 6 h, the serum sodium level is 126 d. Excess diuretic use
mEq/L. Which treatment would be least appropriate? e. Abrupt discontinuation of SSRI use

6 Geriatric Nephrology Curriculum American Society of Nephrology


Chapter 17: Fluid Balance Disorders in the Elderly
Myron Miller
Department of Medicine, Johns Hopkins University School of Medicine, and Division of Geriatric Medicine,
Department of Medicine, Sinai Hospital of Baltimore, Baltimore, Maryland

Disorders of fluid balance are common in the el- duction to the nighttime period becomes evident.
derly and often are caused by age-related alterations With further increase in age, there is often reduc-
in urinary tract function, which can present clini- tion in the ratio of day to night urine flow to the
cally as urinary frequency, nocturia, and inconti- point that nighttime flow rates become equal to or
nence. Among the factors predisposing to the de- exceed daytime rates. Despite the change in circa-
velopment of these clinical disturbances are aging dian pattern of urine excretion, total urine produc-
changes in the renal and hormonal systems that tion per 24 h is not affected.3
control water and sodium excretion along with
changes in the reservoir function of the bladder1
(Table 1). This chapter will examine the effect of ARGININE VASOPRESSIN SECRETION
aging on the systems involved in urine formation
and consider how these changes interact with those Arginine vasopressin [AVP; antidiuretic hormone
of bladder function and lead to urinary frequency, (ADH)] is the major hormone responsible for the
nocturia, and incontinence. regulation of urine formation. The magnocellular
In young, healthy persons there is a circadian supraoptic and paraventricular nuclei of the hypo-
pattern to urine production in which nighttime thalamus where AVP is synthesized do not seem to
urine flow rate is less than daytime flow rate. In undergo age-related degenerative changes with ei-
association with the normal aging process, there is ther morphologic features of cell destruction or de-
an increase in nocturnal production of urine, so cline in cell hormone synthetic ability or hormone
that nighttime urine flow rate equals or exceeds content.
daytime production rate.2 When of sufficient mag- There is some controversy regarding the influ-
nitude to result in nocturia, this change has been ence of normal aging on daytime blood AVP levels.
termed nocturnal polyuria syndrome. A number of studies have indicated that, under
The establishment of a circadian rhythm of urine basal conditions, daytime plasma AVP concentra-
flow takes place during childhood, generally be- tion is not affected by increasing age. In contrast,
tween the ages of 2 to 5 yr. In normal children with several other studies have reported increased basal
a mean age of 7 yr, daytime rate of urine production plasma AVP levels in healthy elderly persons. Fur-
is two to three times that of the nighttime period. ther adding to controversy are reports that healthy
Delay or failure of the circadian rhythm to develop elderly subjects have daytime plasma AVP concen-
during childhood is associated with the presence of trations that are significantly lower than in young
nocturnal enuresis. In adulthood, the ratio of day- subjects.
time to nighttime urine production is usually In healthy adults, there is a diurnal release of
greater than 2:1, so that about 25% or less of daily AVP into the circulation with peak blood concen-
urine output occurs during sleep. Typical urine tration occurring during the hours of sleep. This
production rates are approximately 70 to 80 ml/h rhythm seems to be linked to the wake-sleep cycle
during the waking period and 30 to 40 ml/h during
sleep. This circadian pattern seems to be linked to
the day-night sleep pattern. The circadian pattern Correspondence: Myron Miller, MD, Director, Division of Geriat-
ric Medicine, Department of Medicine, Sinai Hospital of Balti-
of urine flow is paralleled by similar rhythms of re- more, 2401 West Belvedere Avenue, Baltimore, MD 21215.
nal plasma flow and GFR. The circadian pattern is Phone: 410-601-6852; Fax: 410-601-9146; E-mail: myrmiller@pol.
maintained in healthy persons until around age 60 net; [email protected]
yr when a shift to a greater proportion of urine pro- Copyright 䊚 2009 by the American Society of Nephrology

American Society of Nephrology Geriatric Nephrology Curriculum 1


Table 1. Physiologic changes of aging associated with the thick ascending loop of Henle is impaired and can contrib-
increased urine production ute to both sodium loss and to impaired tonicity in the renal
Age-associated impaired renal concentrating capacity medulla. After the administration of an acute water load with
Resistance to AVP action, i.e., “acquired partial nephrogenic consequent expansion of intravascular volume, there is an ex-
diabetes insipidus⬙ aggerated natriuresis in elderly individuals compared with
Impaired sodium conservation younger subjects. An excess of sodium excretion with increas-
Decreased nocturnal AVP secretion ing age has been described in patients with mild hypertension.
Decreased renin-angiotensin-aldosterone secretion
The ability of the aged kidney to conserve sodium in response
Increased atrial natriuretic hormone secretion
to salt restriction is sluggish. Restriction of dietary sodium in-
take to 10 mEq/d resulted in a half-life for reduction of urinary
rather than to time of day. The circadian rhythm becomes es- sodium excretion of 17.6 h in young individuals in contrast to
tablished during childhood and delay in its maturation is often 30.9 h in old subjects. The tendency to sodium wasting in el-
associated with enuresis. With advanced age, there seems to be derly persons can lead to natriuresis with an accompanying
blunting of the nocturnal phase of AVP secretion so that day- obligatory osmotic diuresis, especially when individuals are in
time and nighttime blood levels of the hormone are similar.4,5 the recumbent position.
In contrast, elderly persons may have greater response of
AVP secretion to several stimuli than younger persons. The
plasma AVP response to intravenous hypertonic saline infu- RENIN-ANGIOTENSIN-ALDOSTERONE SYSTEM
sion in healthy elder subjects (age 54 to 92 yr) is almost double
that of the younger subjects, suggesting that aging results in The ability of the kidney to conserve sodium is partly depen-
increased osmoreceptor sensitivity. Studies using water depri- dent on the actions of the renin-angiotensin-aldosterone sys-
vation, intravenous metoclopramide, or cigarette smoking as tem, a system affected by the normal aging process.6,7 Healthy
stimuli for AVP secretion have also shown a similar age-related older individuals (age 62 to 70 yr) who consume a normal
enhancement of response. In contrast, the stimulation of AVP amount of dietary sodium have lower levels of both plasma
release by BP reduction or upright posture seen in young per- renin activity and aldosterone in the supine position than
sons did not occur in elderly subjects, indicating an age-related young healthy persons age 20 to 30 yr. After the combined
impairment of volume/pressure mediated AVP release most stimuli of low dietary sodium intake and upright posture,
likely related to impaired baroreceptor function. plasma renin activity and aldosterone concentration increase
in both age groups, but the mean values achieved are signifi-
cantly lower in the elderly persons group. Decreased conver-
RENAL WATER LOSS sion of inactive to active renin may be at least partially respon-
sible for the reduction in active renin concentration in elderly
In association with normal aging, there is a decline in renal persons. The age-related decrease in plasma renin results in
concentrating capacity. After 24 h of dehydration, there is a decreased aldosterone production by the adrenal gland. Age
progressive decline in maximal urine osmolality with increas- itself does not affect the ability of the adrenal gland to synthe-
ing age. This effect of aging persists even after correction for the size and secrete aldosterone because stimulation of the adrenal
age-related decline in GFR. The decline in renal concentrating by corticotropin infusion results in similar increases of both
ability of the aging kidney is not caused by an inadequate re- plasma cortisol and aldosterone in young and elderly subjects.
sponse of AVP to the stimulus of water deprivation but rather It is likely that the age-related reduction in aldosterone con-
to impaired renal tubular response to AVP. Thus, aging can be centration is one of the factors associated with impaired renal
considered to lead to the development of an acquired form of sodium conserving ability in elderly persons. There does not
partial nephrogenic diabetes insipidus. Age-associated in- seem to be an effect of age on renal tubular responsiveness to
creased excretion of urea can also contribute to increased urine aldosterone.
excretion through its osmotic effect. Under conditions of ade-
quate intake of fluid, these changes may result in no apparent
clinical consequence other than mild increase in urine flow ATRIAL NATRIURETIC HORMONE
rate, but the elderly person who has restricted fluid intake may
be unable to adequately reduce urine formation and therefore ANH may be a significant factor in mediating the altered renal
be at increased risk of dehydration. sodium handling of the elderly. Through its action on the kid-
ney, it produces natriuresis and obligatory diuresis. A number
of studies have shown that ANH concentration in the blood is
RENAL SODIUM LOSS increased with aging. Mean basal plasma ANH levels were five-
fold higher in elderly male nursing home residents than in
Normal aging is associated with impaired ability of the kidney young normal men.
to retain sodium. In very old persons, sodium reabsorption in In addition, the intravenous infusion of normal saline re-

2 Geriatric Nephrology Curriculum American Society of Nephrology


sulted in an exaggerated rise in plasma ANH concentration in gency increases to 40 to 49% in men from the fifth to seventh
the elderly persons group. Several other studies involving decades and to 40 to 50% in women in their fifth decade and
healthy elderly persons have also established that basal plasma beyond. In institutionalized elderly persons with incontinence,
ANH concentration is increased and that stimuli that can in- 72% were found to have detrusor instability. The clinical con-
crease intracardiac pressure result in a greater rise of plasma sequence of detrusor instability is decline in functional bladder
ANH in elderly persons than in the young. There is evidence volume so that small urine volumes result the sensation of full
that the renal response to ANH may also be greater in elderly bladder and the initiation of bladder contractions leading to an
persons. The natriuretic response to a bolus injection of ANH increase in the frequency of voiding during the day and at night
has been observed to be larger in older persons with a mean age (Table 3).
of 52 yr than in younger persons with a mean age of 26 yr.
ANH has been shown to interact with the renin-angioten-
sin-aldosterone system. High levels of ANH suppress renal re- NOCTURNAL POLYURIA SYNDROME
nin secretion, plasma renin activity, plasma angiotensin II,
and, as a consequence, plasma aldosterone. Even minimal in- The constellation of increased nocturnal urine production and
creases of ANH within the physiologic range produced by consequent nocturia with its associated effects on disruption of
slow-rate intravenous infusion of ANH are capable of inhibit- sleep has been termed the nocturnal polyuria syndrome.3,9,10
ing angiotensin II-induced aldosterone secretion, providing Several definitions have been used in characterizing individu-
support for the functional impact of the increased ANH levels als as having nocturnal polyuria: (1) urine output during sleep
in elderly persons. ANH can also suppress plasma aldosterone ⱖ33% of total 24-h volume; (2) nighttime urine flow rate ⱖ0.9
by a direct effect on aldosterone secretion. Thus, ANH may be ml/min; and (3) 7:00 p.m. to 7:00 a.m. urine volume ⱖ50% of
an important contributor to age-related impaired renal so- total 24-h volume. A number of clinical states have been asso-
dium conserving ability, and concomitant water loss, both ciated with the development of nocturnal polyuria (Table 4).
through its direct natriuretic effect and through ANH-induced In persons with nocturnal polyuria, there is an absence of
suppression of aldosterone secretion.8 diurnal change in plasma AVP concentration. Both urinary
The above studies clearly establish that normal aging is ac- frequency and hourly urine volume are greater at night in older
companied by increased urine excretion during the nighttime persons with nocturnal polyuria syndrome and, in these indi-
period of sleep. There are multiple physiologic changes in renal viduals, functional bladder capacity is lower at night with an
water and sodium conserving mechanisms and in the hor- approximate volume of 200 ml. Patients with nocturia have
monal systems governing water and sodium regulation. The significantly lower urine osmolality at night. Both plasma AVP
effect of these changes is to alter the diurnal rhythm of urine and angiotensin II are lower at night in patients with nocturnal
excretion in the direction of nighttime diuresis (Table 2). polyuria, and plasma ANP levels have been found to be high-
er.4,5 Both the nocturnal decrease in AVP and the nocturnal
rise in ANP seem to be responsible for the high nighttime urine
AGING EFFECTS ON BLADDER RESERVOIR flow and resultant nocturia. In association with reduction in func-
FUNCTION tional bladder volume, the clinical consequence is nocturia.

In healthy adults, 24-h urine production is usually in the range Nocturnal Polyuria in Multiple System Atrophy
of 1000 to 1500 ml and bladder capacity generally ranges from Multiple system atrophy (MSA) is a central nervous system
approximately 400 to 750 ml. This is associated with a daytime degenerative disease that most commonly occurs in older
voiding frequency of four to five times in men and five to six persons. It affects many areas of the central nervous system
times in women and with rare voiding during normal periods and has central autonomic insufficiency or Shy-Drager syn-
of sleep. There is a suggestion that voiding frequency increases drome as one its components. Patients with central auto-
with age, especially in men. With advancing age, there is a nomic insufficiency have been observed to excrete large
progressive increase in the prevalence of urgency, most com- amounts of urine when recumbent at night, and this noc-
monly caused by detrusor instability. The prevalence of ur-
Table 3. Association between nocturnal urine flow rate,
Table 2. Typical parameters related to circadian water bladder volume, and nocturnal urinary frequency in healthy
excretion in healthy young and elderly adults* young and elderly adults*
Young Elderly Young Elderly
Day Night Day Night Nighttime urine flow rate (ml/h) 35 70
Urine volume (ml/h) 75 35 50 70 Time in bed (h) 8 8
Urine osmolality (mosm/kg) 700 830 510 450 Nighttime urine production (ml) 280 560
Plasma AVP (pg/ml) 1.1 2.0 1.9 1.3 Bladder capacity (ml) 400 200
Plasma ANH (pg/ml) 19 17 40 55 Number of voids during sleep period 0 2
*Data show typical values based on literature and/or author’s experience. *Data show typical values based on literature and/or author’s experience.

American Society of Nephrology Geriatric Nephrology Curriculum 3


Table 4. Clinical states associated with nocturnal polyuria Table 5. Factors predisposing patients with Alzheimer’s
syndrome disease to nocturia and urinary incontinence
Nocturnal polyuria syndrome Decreased urine concentrating ability
Primary enuresis in children Decreased plasma AVP and loss of circadian rhythm
Normal aging Increased nocturnal urine flow rate
Multiple system atrophy Decreased functional bladder volume (detrusor instability)
Alzheimer’s disease Diminished perception of bladder fullness
Spinal cord injury Diminished appropriate toileting behavior
Other causes of increased nocturnal urine production Diminished mobility
Edematous states
CHF
Renal disease Other clinical conditions that may be associated with
Hepatic disease increased nocturnal urine production include edematous
Venous insufficiency
states such as congestive heart failure, renal and hepatic
Osmotic diuresis
disease, and peripheral venous insufficiency, where edema
Diabetes mellitus
Diuretics
fluid may be mobilized when individuals are in the recum-
Impaired renal concentrating ability bent position. Osmotic diuresis, as occurs in patients with
Renal disease poorly controlled diabetes mellitus and after administration
Hypokalemia of diuretic drugs, can also lead to increased nocturnal urine
Hypercalcemia flow. Although uncommon in elderly persons, high fluid
Increased fluid intake intake states, as may be seen in patients with primary poly-
Iatrogenic dipsia or in individuals who ingest large fluid volumes in the
Primary polydipsia belief that doing so is beneficial, can cause nocturnal poly-
uria.
turnal dieresis is associated with marked decrease in noctur-
nal AVP production with a resultant diabetes insipidus-like
nocturnal diuresis. Treatment of MSA patients who have THERAPY OF NOCTURNAL POLYURIA SYNDROME
nocturnal polyuria with the AVP analogs DDAVP (desmo-
pressin) or lysine vasopressin has resulted in reduction of noc- Hormonal
turnal polyuria and improvement in the orthostatic hypotension The demonstration of reduced daytime and/or nocturnal AVP
that is part of the syndrome and worsened by the polyuria-caused secretion in patients with nocturnal polyuria suggests that hor-
intravascular volume depletion. mone replacement therapy with AVP or its potent antidiuretic
analogs may be beneficial in reversing the syndrome.11–13
Nocturnal Polyuria in Alzheimer’s Disease Short-term treatment with 20 ␮g of intranasal DDAVP for 2
The synthesis of AVP by hypothalamic neurons is not affected wk results in a 25 to 40% reduction of nocturnally voided urine
in patients with Alzheimer’s, but AVP release into the circula- with a corresponding reduction in nocturnal frequency. Pro-
tion is impaired so that AVP concentration in the blood has longed treatment for 2 mo with 40 ␮g of DDAVP daily has
been reported to be lower in patients with Alzheimer’s disease shown a sustained response and a decrease in nocturnal ur-
than in comparably aged persons with normal cognitive func- gency and incontinence.
tion. In patients with Alzheimer’s disease, there is loss of noc- Intranasal doses of DDAVP ranging from 10 to 40 ␮g
turnal AVP secretion and an accompanying reversal of day- daily were capable of producing a 10 to 50% reduction in
night urine production and nocturnal polyuria. Both the nocturnal urine volume in Alzheimer’s disease patients with
decrease in daytime AVP release and the loss of nocturnal se- nocturnal incontinence. Long-term treatment of these pa-
cretion put patients with Alzheimer’s disease at increased risk tients with 10 ␮g DDAVP daily has been observed to result
for fluid loss and dehydration. Clinically, this can be expressed in restoration of nighttime continence in some individuals
by the high prevalence of nocturia and both daytime and night- (Table 6).
time urinary incontinence in patients with Alzheimer’s disease. The use of DDAVP in the treatment of the elderly patient
Detrusor instability is commonly present in patients with Alz- with nocturia and nocturnal polyuria may be associated with
heimer’s disease. The combination of high nighttime urine adverse effects because of water retention, especially when
production, decreased functional bladder volume, and cogni- DDAVP is administered in the intranasal form, with as many
tive impairment can readily explain why ⬎80% of institution- as 20% of treated patients developing hyponatremia. Thus, use
alized patients with Alzheimer’s disease have urinary inconti- of DDAVP for treatment of nocturnal polyuria should be lim-
nence. When there is impaired mobility as well, the underlying ited to the oral preparation. In addition, DDAVP effect may be
factors are likely to result in near 100% presence of inconti- limited by age or disease-associated impairment of renal con-
nence (Table 5). centrating response.

4 Geriatric Nephrology Curriculum American Society of Nephrology


Table 6. Treatment of nocturnal polyuria syndrome • DDAVP may be an effective intervention in management of nocturia
and incontinence in some elderly persons
Hormone replacement: DDAVP (desmopressin)
Intranasal: 20–40 ␮g
Oral: 0.1–0.4 mg
DISCLOSURES
Anticholinergic agents (detrusor antispasmodics)
None.
Oxybutynin: 2.5–5 mg, bid to qid
Oxybutynin xl: 5–30 mg once daily
Tolteradine: 2–4 mg once daily
Solifenacin: 5–10 mg once daily
REFERENCES
Darifenacin: 7.5–15 mg once daily
Trospium: 20 mg, bid *Key References
1. Miller M: Water balance in older persons. In: Contemporary Endocri-
Imipramine: 1 mg/kg body weight orally at 8:00 p.m.
nology: Endocrinology of Aging, edited by Morley JE, van den Berg L,
Totowa, NJ, Humana Press, 1999, pp 31– 41*
CONCLUSIONS 2. Nakamura S, Kobayashi Y, Tozuka K, Tokue A, Kimura A, Hamada C:
Circadian changes in urine volume and frequency in elderly men.
J Urol 156: 1275–1279, 1996
Aging is associated with changes in the renal and hormonal 3. Miller M: Nocturnal polyuria in the elderly: pathophysiology and clin-
systems involved in the conservation of water and sodium. As a ical implications. J Am Geriatr Soc 48: 1321–1329, 2000*
consequence, there is increase in nocturnal urine production. 4. Asplund R, Aberg H: Diurnal variation in the levels of antidiuretic
Concurrent age-related changes occur in the reservoir func- hormone in the elderly. J Intern Med 299: 131–134, 1991
5. Ouslander JG, Nasr SZ, Miller M, Withington W, Lee CS, Wiltshire-
tion of the bladder. The combination of increased urine pro- Clement M, Cruise P, Schnelle JF: Arginine vasopressin levels in
duction at night and decrease in functional bladder volume nursing home residents with nighttime urinary incontinence. J Am
results in the development of nocturnal urinary frequency and Geriatr Soc 46: 1274 –1279, 1998*
predisposes to the development of urinary incontinence. Di- 6. Bauer JH: Age-related changes in the renin-aldosterone system: phys-
minished AVP secretion in patients with Alzheimer’s disease iological effects and clinical implications. Drugs Ageing 3: 238 –245,
1993
puts this population at even greater risk for nocturia and in- 7. Cugini P, Lucia P, Di Palma L, Re M, Canova R, Gasbarrone L, Cianetti
continence and the risk is further magnified by diminished A: Effect of aging on circadian rhythm of atrial natriuretic peptide,
perception of bladder fullness and/or impaired mobility. The plasma renin activity and plasma aldosterone. J Gerontol 47: B140 –
availability of easily administered hormone replacement in the B19, 1992*
form of intranasal or oral DDAVP opens the possibility that for 8. Kikuchi Y: Participation of atrial natriuretic peptide (hANP) levels and
arginine vasopressin (AVP) in aged persons with nocturia. Jap J Urol
some patients there may be an effective intervention available 86: 1651–1659, 1995
to reduce nocturia and urinary incontinence and the clinical 9. Weiss JP, Blaivas JG, Stember DS, Brooks MM: Nocturia in adults:
consequences associated with these disorders. etiology and classification. Neurourol Urodynam 17: 467– 472, 1998
10. Matthiesen TB, Rittig S, Nørgaard JP, Pedersen EB, Djurhuus JC:
Nocturnal polyuria and natriuresis in male patients with nocturia and
lower urinary tract symptoms. J Urol 156:1292–1299, 1996
TAKE HOME POINTS
11. Cannon A, Carter PG, McConnell AA, Abrams P: Desmopressin in the
• Renal and hormonal changes of normal aging can result in increased treatment of nocturnal polyuria in the male. Br J Urol 84: 20 –24, 1999
nocturnal urine production in healthy elderly persons 12. Asplund R, Aberg H: Desmopressin in elderly subjects with increased
• Functional bladder volume is often decreased in elderly persons nocturnal diuresis. a two-month treatment study. Scand J Urol Neph-
• Increased nocturnal urine production and decreased functional bladder rol 27: 77– 82, 1993
volume predispose to nocturia and urinary incontinence 13. Asplund R, Sunberg B, Bengtsson P: Oral desmopressin for nocturnal
• Decreased nocturnal AVP production puts patients with Alzheimer’s polyuria in elderly subjects: a double blind, placebo-controlled ran-
disease at high risk for nocturia and incontinence domized exploratory study. Br J Urol 83: 591–595, 1999*

American Society of Nephrology Geriatric Nephrology Curriculum 5


REVIEW QUESTIONS: FLUID BALANCE DISORDERS d. Initial laboratory studies to include serum electrolytes and
IN THE ELDERLY calcium, first voided morning urine specimen for osmola-
lity
1. All of the following are normal changes of aging that can lead e. Referral to a urologist for possible cystoscopy
to increased urine production except which one:
a. Acquired resistance to the action of antidiuretic hormone 3. An 83-yr-old woman with dementia attributed to Alzheimer’s
b. Increased atrial natriuretic hormone secretion disease is seen for a history of worsening urinary incontinence.
c. Increased nocturnal antidiuretic hormone secretion All of the following findings are likely to be present except
d. Impaired renal sodium conservation which one:
e. Decreased renin-angiotensin-aldosterone secretion a. Urine volume from 7:00 p.m. to 7:00 a.m. is greater than
urine volume from 7:00 a.m. to 7:00 p.m.
2. A 73-yr-old male gives a history of nocturia twice nightly for b. Maximum bladder capacity is likely to be ⬎500 ml
the past 3 yr. Your initial evaluation should include all of the c. Plasma level of atrial natriuretic hormone (ANH) will be
following except which one: higher than the level in normal persons in the 25 to 50 yr
a. Digital rectal examination age range
b. Asking the patient to complete and bring in a 3-d voiding d. Urine osmolality in a first voided morning urine specimen
diary is likely to be ⬍700 mOsm/kg
c. Taking a careful history of all prescription and nonpre- e. A risk of a therapeutic trial with DDAVP is the develop-
scription medications taken ment of hyponatremia

6 Geriatric Nephrology Curriculum American Society of Nephrology


Chapter 18: Acute Kidney Injury in the Elderly
Mitchell H. Rosner
Division of Nephrology, University of Virginia Health System, Charlottesville, Virginia

Acute kidney injury (AKI), as defined by the precip- proposed that parenchymal loss in the aging kidney
itous decline in GFR, is frequently encountered in directly confers a higher susceptibility to acute
the elderly. The effect of advancing age in decreas- damage, this is not supported by experimental data
ing renal reserve and the associated comorbid con- in which the reduction in renal mass surprisingly
ditions of elderly patients increase the risk for the protected against ischemia/reperfusion injury in a
development of AKI. Although studies describing 5/6 nephrectomy model.7 Thus, cellular and molec-
the incidence of AKI in this population are difficult ular alterations that occur with aging may be more
to compare because the definitions of AKI vary dra- important than simply a loss in nephron numbers.
matically from study to study, it is clear that the Lameire et al.8 showed that, in combination with
elderly are at the highest risk for the development of dehydration, a disturbance in autoregulatory de-
AKI. Indeed, Feest et al.1 showed that there is a fense mechanisms that would normally preserve
three- to eight-fold, progressive, age-dependent in- GFR and renal blood flow can, in the elderly kidney,
crease in the frequency of development of commu- lead to ischemia and AKI. One hypothesis links
nity-acquired AKI in patients older than 60 yr of blunted nitric oxide (NO) production in the elderly
age. The mean age of patients with AKI has in- kidney to an increased risk for AKI.9 For example,
creased by 5 to 15 yr over the past 25 yr.2 Groen- studies in a rat model of gentamicin-induced AKI
eveld et al.3 showed that the age-related yearly inci- show that an increased glomerular NO production
dence of AKI rose from 17 per million in adults seems to protect renal function through its vasodi-
under age 50 yr to 949 per million in the 80 to 89 yr latory effects.10 When old and young animals are
age group. Although all causes of AKI are encoun- treated with equivalent doses of gentamicin, older
tered in this age group, the frequency of prerenal animals show more severe AKI that correlates with
and postrenal etiologies is especially prevalent in a blunted stimulation in NO production.10 Miura et
the elderly.4 Furthermore, elderly patients are more al.11 have also hypothesized that, in addition to al-
frequently subjected to invasive procedures and ex- teration autoregulation, aging tubular cells may be
posure to multiple (and possibly nephrotoxic) more vulnerable to ischemic damage because cellu-
medications and to radiocontrast agents, all of lar antioxidant defenses decline with age as well as
which increase the risk for AKI. the fact that tubular cells have alterations in metab-
olism that render them more susceptible to injury
(such as an accelerated rate of ATP depletion caused
STRUCTURAL AND FUNCTIONAL by mitochondrial alterations).
ALTERATIONS IN THE AGED KIDNEY

In the absence of a specific disease, the kidney un- CAUSES OF AKI IN THE ELDERLY
dergoes age-dependent structural and functional
alterations leading to a significant decrease in renal In the elderly, AKI is often iatrogenic and multifac-
mass, functioning nephron numbers, and baseline torial. Elderly patients show the same spectrum for
kidney function.5 Under normal conditions, these the causes of AKI as the general population. How-
changes can be functionally compensated for by ad-
aptations in renal hemodynamics to maintain a suf-
Correspondence: Mitchell H. Rosner, Division of Nephrology,
ficient GFR. However, in the setting of pathophys- University of Virginia Health System, Box 800133, Charlottesville,
iologic challenges, the older kidney lacks sufficient VA 22908. Phone: 434-924-2187; Fax: 434-924-5848; E-mail:
functional reserve and is more likely to develop [email protected]

clinically relevant damage.6 Although it has been Copyright 䊚 2009 by the American Society of Nephrology

American Society of Nephrology Geriatric Nephrology Curriculum 1


ever, specific differences in the incidences and presentation Hemodynamically Mediated AKI
make this group unique. For example, elderly patients are Direct interference with the protective autoregulatory mecha-
more likely to have received multiple concurrent insults that nisms of renal blood flow and GFR may precipitate AKI. Drugs
result in AKI. commonly prescribed to the elderly and known to impair renal
autoregulation or to interfere with the vasodilatory capacity
Prerenal AKI include nonsteroidal anti-inflammatory agents (NSAIDs), an-
Prerenal AKI is the second most common cause of AKI in the giotensin-converting enzyme inhibitors (ACEi). and angioten-
elderly, accounting for nearly one third of cases.12 The main sin receptor antagonists (ARB).
cause of prerenal AKI is decreased perfusion to the kidney. Well-known risk factors for NSAID-induced AKI include
Although many of the causes of renal hypoperfusion can be age ⬎60 yr, atherosclerotic cardiovascular disease, pre-existing
reversed with adequate fluid replacement, others progress to chronic kidney disease (CKD), and renal hypoperfusion states
acute tubular necrosis (ATN). Interestingly, the evolution to (sodium depletion, diuretic use, cirrhosis, congestive heart
ATN occurs more frequently in the elderly than in younger failure, hypotension, volume depletion).15 In one study,
patients.4 Decreased perfusion to the kidney can develop from NSAIDs accounted for 15.3% of all cases of drug-induced AKI
decrease in cardiac output or effective circulating volume (sep- but accounted for ⬎25% of cases in those age ⬎65 yr.16 It
sis, cirrhosis, nephrotic syndrome) or hypovolemia (gastroin- should be recognized that there is little evidence of NSAIDs
testinal losses, bleeding, diuretic use, poor oral intake). impairing renal function in otherwise healthy elderly individ-
Dehydration is a common occurrence in the elderly, affect- uals.
ing nearly 1% of hospital admissions in the elderly.13 Risk fac- In the elderly, the frequency of AKI secondary to ACEi has
tors for dehydration include acute febrile illnesses, polyphar- been estimated to vary between 6 to 38%.17 In part, this may be
macy (diuretics, laxatives, drugs that decrease appetite or level to the greater incidence of significant renovascular disease in
of consciousness), and being bedridden. These patients often the elderly (either bilateral renal artery stenosis, or unilateral
present in AKI with significant hypernatremia and, if un- stenosis in a solitary functioning kidney).
treated, the condition has a very high mortality rate.14
It may be difficult to make the diagnosis of hypovolemic Acute Tubular Necrosis
AKI in the elderly because the clinical signs and symptoms of Acute tubular necrosis is the most frequent cause of AKI in the
dehydration (such as tachycardia, skin tenting) are unreliable. elderly, with an incidence ranging from 25 to 87%.18 The in-
Furthermore, the traditional urinary parameters for differen- sults that lead to this condition include nephrotoxins (amino-
tiating prerenal from intrinsic renal failure may simply reflect glycosides, radiocontrast agents), pigment-induced (rhabdo-
age-related disturbances in tubular handling of sodium and myolysis), and ischemia (sepsis, surgery). In older patients
water as well as drug effects (diuretics). Thus, a high degree of with ATN, several chronic premorbid conditions such as con-
suspicion for prerenal etiologies must be entertained, and a gestive heart failure, hypertension, and diabetes predispose to
cautious trial of fluid therapy may be warranted. Because of the the development of severe tubular injury. Elderly patients
high incidence of urinary incontinence, a foley catheter should more frequently undergo significant cardiovascular surgery
be placed to closely monitor urine output. (aortic aneurysm repair, bypass surgery) that is associated with
a high risk for ATN. Elderly patients are more susceptible to
serious infections and the development of sepsis and multisys-
tem organ failure. In this setting, the development of AKI re-
RENAL AKI quiring dialysis has an attendant mortality of ⬎80%.19
Prevention of ATN requires careful attention to baseline
Numerous intrarenal causes of AKI can affect the elderly, of GFR. In the elderly, serum creatinine values may appear nor-
which ATN is the most common. An exhaustive description of mal yet be associated with a significant decline in GFR. This
these causes is beyond the scope of this chapter; however, those reliance on serum creatinine as a marker of renal function can
causes that are more specifically relevant to the elderly popu- easily lead to inappropriate dosing of antibiotics and other
lation are discussed below. nephrotoxins. Thus, use of either the Cockroft-Gault or
Modification of Diet in Renal Disease (MDRD) equation to
Renovascular Diseases estimate renal function is mandatory. When available, preven-
This group of etiologies includes any cause of acute obstruc- tative strategies such as intravenous hydration before radio-
tion of the renal vasculature. Thus, cholesterol embolization contrast administration should be undertaken.20
after intravascular procedures or surgery or rarely acute renal
artery thrombosis may lead to this syndrome of AKI. Elderly Acute Interstitial Nephritis
patients, by virtue for their increased burden of atherosclerotic Elderly patients are at increased risk secondary to the large
vascular disease and increased need for invasive procedures, number of medications that they may be taking. This may in-
are at heightened risk. clude herbal supplements and other over-the-counter medica-
tions. Given the complex comorbidities of elderly patients, it

2 Geriatric Nephrology Curriculum American Society of Nephrology


may be difficult to make the diagnosis of acute interstitial ne- imaging tests. As stated above, given the high prevalence of
phritis (AIN) as the cause of AKI, and renal biopsy may be obstructive causes for AKI in this group, ultrasonography is
required. mandatory. Microscopic examination of the urine for cellular
elements, casts, and crystals is also mandatory and can lead to a
Glomerulonephritis proper diagnosis in a rapid, efficient manner. The finding of
Elderly patients have a higher incidence of p-anti-neutrophil pigmented granular casts in the urine is indicative of tubular
cytoplasmic antibody (ANCA) and anti-glomerular basement damage and supports the diagnosis of ATN. The appearance of
membrane (GBM) associated with rapidly progressive glomer- red blood cell casts in the urine indicates active glomerular
ulonephritis (RPGN).21 Although the same principles apply to damage (glomerulonephritis) and warrants consideration of
treatment of older adults with RPGN as with younger individ- whether a renal biopsy should be performed. Urine electro-
uals, caution in the use of immunosuppressant medications lytes, urine eosinophils, and serologic testing for glomerulone-
such as corticosteroids, cytotoxic drugs, and plasmapheresis is phritis should be used when appropriate. There are no specific
warranted given a much higher risk of opportunistic infections caveats for the elderly patient in this regard.
and complications. The relative risk of death is 5.3 times higher
in patients ⬎60 yr compared with younger patients after ag-
gressive immunosuppression in the treatment of RPGN.22
However, because of the potential for reversing AKI in some TREATMENT OF THE ELDERLY WITH AKI
forms of glomerulonephritis, one should not hesitate to per-
form a renal biopsy if indicated. The procedure does not carry In general, the treatment of AKI in the elderly follows the same
a higher risk in the elderly.23 However, the presence of signifi- principles as for the general population. However, the decision
cant glomerulosclerosis and arteriosclerosis in biopsy tissue to initiate dialytic support in the very elderly with multiple
may render the interpretation of histologic findings more dif- comorbidities and a very poor prognosis may be difficult. This
ficult. is especially true for those individuals with significant baseline
renal impairment where the likelihood of renal recovery may
be low. The decision to initiate dialysis in these patients re-
POSTRENAL AKI quires a coordinated discussion with family members, consult-
ing physicians and other care providers.
In two major series, the incidence of postrenal obstructive AKI
was 7.9 and 9% in patients over 65 and 70 yr, respectively.1,4
The obstruction may be either intrinsic or extrinsic and can
occur at any level of the urinary tract. Among the causes of PREVENTION OF AKI IN THE ELDERLY
lower urinary tract obstruction, the most common in males is
prostatic enlargement caused by benign prostatic hypertrophy Given the morbidity and mortality associated with AKI, pre-
or carcinoma. The second most common cause in males is ventative strategies are clearly important. Tables 1 and 2 list
urethral stricture disease often secondary to trauma. In fe- both general preventative strategies and exposure-specific
males, the most common cause of postrenal failure is ureteral strategies that can be used in patients at risk for AKI.
obstruction caused by pelvic malignancy (invasive carcinoma
of the cervix). An uncommon cause of obstruction seen in the
elderly is caused by an inflammatory aortic aneurysm and can Table 1. General approaches for the prevention of AKI
be identified through proper imaging studies. Avoidance of nephrotoxins
All elderly patients presenting with AKI require urethral Recognition of potential nephrotoxic agents
catheterization and ultrasonography to identify possible ob- Recognition of high risk patients and clinical settings
structive etiologies. False-negative ultrasonography is usually Avoidance of concomitant use of multiple nephrotoxins
only seen in those with very early obstruction or in those pa- Use of lowest dose and for shortest time possible
If applicable, monitoring of drug dose
tients with significant retroperitoneal fibrosis that encases the
Frequent monitoring of renal function
ureters and renal pelvis preventing dilation.
Maintain euvolemia
Minimization of nosocomial infection
Extracellular fluid expansion
LABORATORY EVALUATION OF AKI IN THE (maintain good urine output, stable hemodynamics)
ELDERLY Avoid agents that impair renal blood flow autoregulation
(NSAIDS, ACE inhibitors, ARBs)
The laboratory evaluation of AKI in the elderly patient is no Pharmacologic interventions if applicable
different than for other patients. Attention to the history and Use of computer surveillance systems
physical examination is critical in narrowing the differential Identify high risk patients and medications
diagnosis to potential etiologies and to specific laboratory or Determine correct dose for GFR

American Society of Nephrology Geriatric Nephrology Curriculum 3


Table 2. Examples of specific renal protective strategies TAKE HOME POINTS
Exposure Strategy
• Elderly patients are at higher risk for the development of AKI
Radiocontrast agents Intravenous hydration (normal saline) • Specific hemodynamic, metabolic, and molecular changes lead to in-
Intravenous sodium bicarbonate (?) creased susceptibility to injury in the aged kidney
N-acetylcysteine • Certain causes of AKI are more common in the elderly: postrenal
Vitamin C obstructive disease, ischemic ATN, and hemodynamically mediated
Iso-osmolar contrast AKI
• Multiple etiologies are often operative in the development of AKI
Aminoglycoside Once-daily dosing
• Diagnostic and therapeutic issues in AKI are no different for the elderly
antibiotics
patient as for the general population
Monitoring of drug levels • The outlook for renal recovery is likely impaired in the elderly patient
Tumor lysis (uric acid) Allopurinol/rasburicase
Intravenous hydration/urine alkalinization
Ethylene glycol Ethanol/fomepizole
ingestion DISCLOSURES
Hemodialysis None.
Rhabdomyolysis Intravenous hydration/urine alkalinization⫾
mannitol
Methotrexate Intravenous hydration/urine alkalinization
Acyclovir Intravenous hydration REFERENCES
Calcineurin inhibitors Monitor drug levels ⫾ calcium-channel
blockers *Key References
Amphotericin B Use of lipid formulation 1. Feest TJ, Round A, Hamad S: Incidence of severe acute renal failure in
adults: results of a community-based study. BMJ 306: 481– 483, 1993*
2. Turney JH, Marshall DH, Brownjohn AM, Ellis CM, Parsons FM: The
RECOVERY OF RENAL FUNCTION AND PROGNOSIS evolution of acute renal failure. QJM 74: 83–104, 1990
OF AKI IN THE ELDERLY 3. Groeneveld AB, Tran DD, van der Meulen J, Nauta JJ, Thijs LG: Acute
renal failure in the intensive care unit: predisposing, complicating
A recent systematic review and meta-analysis of recovery of factors affecting outcome. Nephron 59: 602– 610, 1991
4. Macı́as-Núñez JF, López-Novoa JM, Martı́nez-Maldonado M: Acute
kidney function after AKI in the elderly has shown that recov-
renal failure in the aged. Semin Nephrol 16: 330 –338, 1996*
ery after AKI is approximately 28% less likely to occur when 5. Choudhury DRD, Levi M: Effect of aging on renal function ad disease.
the patient is older than 65 yr.24 Whether these results are In: Brenner and Rector’s The Kidney, 7th Ed., edited by Brenner B,
caused by the effects of advanced age on the kidney itself or the Philadelphia, PA, Saunders, 2004, pp 2305–2315
increased number of comorbidities (including baseline CKD) 6. Esposito C, Plati A, Mazzullo T, Fasoli G, De Mauri A, Grosjean F,
Mangione F, Castoldi F, Serpieri N, Cornacchia F, Dal Canton A: Renal
in the elderly is not certain. Long-term recovery is also less
function and functional reserve in healthy elderly individuals. J Neph-
likely and it is believed that AKI in elderly more often results in rol 20: 617– 625, 2007*
CKD.24 The lower likelihood of renal recovery in the elderly 7. Vercauteren SR, Ysebaert DK, De Greef KE, Eyskens EJ, De Broe ME:
may be due to the effects of aging to impair the capacity for Chronic reduction in renal mass in the rat attenuates ischemia/reper-
kidney repair.25 The capacity for renal epithelial cell prolifera- fusion injury and does not impair tubular regeneration. J Am Soc
Nephrol 10: 2551–2561, 1999*
tion declines with aging as does the function of progenitor and
8. Lameire N, Hoste E, Van Loo A, Dhondt A, Bernaert P, Vanholder R:
stem cells that are critical for tubular repair.25 Pathophysiology, causes and prognosis of acute renal failure in the
Several other individual studies have not been able to show elderly. Ren Fail 18: 333–346, 1996*
that age is specifically associated with impaired renal recov- 9. Reckelhoff JF, Manning RD: Role of endothelium-derived nitric oxide
ery.18 Thus, in the individual patient, it may not be clear if age in control of renal microvasculature in aging male rats. Am J Physiol
265(Suppl): R1123–R1129, 1993
is an independent predictor of a poor prognosis and other
10. Rivas-Cabañero L, Rodriguez-Barbero A, Arévalo M, López-Novoa JM:
comorbid conditions may play a more important role in driv- Effect of Ng-nitro-L-arginine methyl ester on gentamicin-induced
ing the risk for poor outcomes.26 nephrotoxicity in rats. Nephron 71: 203–207, 1995
11. Miura K, Goldstein RS, Morgan DG, Pasino DA, Hewitt WR, Hook JB:
Age-related differences in susceptibility to renal ischemia in rats.
Toxicol Appl Pharmacol 87: 284 –296, 1987
CONCLUSIONS
12. Cheung CM, Ponnusamy A, Anderton JG: Management of acute renal
failure in the elderly patient: a clinician’s guide. Drugs Aging 25:
For numerous reasons, elderly patients are at higher risk for the 455– 476, 2008
development of AKI, and certain causes of AKI are more com- 13. Lavizzo-Mourey R, Johnson J, Stolley P: Risk factors for dehydration
monly seen in this group. In fact, there are more often multiple among elderly nursing home patients. J Am Geriatr Soc 36: 213–218,
1988
etiologies of AKI in this age group. Diagnostic approaches to
14. Weinberg AD, Pals JK, Levesque PG, Beal LF, Cunningham TJ, Mi-
AKI should focus on the most likely etiologies. Because of mul- naker KL: Dehydration and death during febrile episodes in the nurs-
tiple factors, the likelihood of complete renal recovery is im- ing home. J Am Geriatr Soc 42: 968 –971, 1994
paired in this group. 15. Blackshear JL, Davidman M, Stillman MT: Identification of risk for renal

4 Geriatric Nephrology Curriculum American Society of Nephrology


insufficiency from non-steroidal anti-inflammatory drugs. Arch Int Med biopsy in patients 65 years of age or older. An analysis of the results
143: 1130 –1138, 1983 of 334 biopsies. J Am Geriatr Soc 38: 669 – 674, 1990
16. Kleinknecht D, Landais P, Goldfarb B: Pathophysiology and clinical 22. Keller F, Michaelis C, Buttner P, Bennhold I, Schwarz A, Distler A: Risk
aspects of drug-induced tubular necrosis in man. Contrib Nephrol 55: factors for long-term survival and renal function in 64 patients with
145–158, 1987 rapidly progressive glomerulonephritis (RPGN). Geriatr Nephrol Urol
17. Hollenberg NK: Medical therapy of renovascular hypertension: effi- 4: 5–13, 1994
cacy and safety of captopril in 269 patients. Cardiovasc Rev Rep 4: 23. Rakowski TA, Winchester JF: Renal biopsy in the elderly patient. In:
852– 859, 1983 Geriatric Nephrology, edited by Michelis MF, Preuss HG, New York,
18. Lameire N, Verspeelt J, Vanholder R, Ringoir S: A review of the Field Rich, 1986, pp 37–39
pathophysiology, causes and prognosis of acute renal failure in the 24. Schmitt R, Coca S, Kanbay M, Tinetti ME, Cantley LG, Parikh CR: Recov-
elderly. Geriatr Nephrol Urol 1: 77– 89, 1991 ery of kidney function after acute kidney injury in the elderly: a systematic
19. Wardle EN: Acute renal failure and multiple organ failure. Nephrol review and meta-analysis. Am J Kidney Dis 52: 262–271, 2008
Dial Transplant 9(Suppl 4): 104 –107, 1994 25. Schmitt R, Cantley LG: The impact of aging on kidney repair. Am J
20. Pfaffle AE, Moses JW, Chan R, Gleim GW, Zabetakis PM, Stugenki K, Physiol Renal Physiol 294: F1265–F1272, 2008
Michelis M: Negative fluid balance and the occurrence of radiocon- 26. Lameire N, Nelde A, Hoeben H, Vanholder R: Acute renal failure in the
trast associated acute renal failure. Geriatr Nephrol Urol 4: 71–77, elderly. In: Nephrology and Geriatrics Integrated, edited by Oreopou-
1994 los DG, Hazzard WR, Luke R, Dordrecht, Kluwer Academic, 2000, pp
21. Preston RA, Stemmer CL, Materson BJ, Perez-Stable E, Pardo V: Renal 1–113

American Society of Nephrology Geriatric Nephrology Curriculum 5


REVIEW QUESTIONS: ACUTE KIDNEY INJURY IN ago was notable for a serum creatinine of 1.9 mg/dl (baseline
THE ELDERLY value). Which one of the following steps is indicated in the
further evaluation of acute kidney injury in this patient?
1. Which one of the following statements is true?
a. Renal biopsy
a. Serum creatinine in the elderly patient (age ⬎65 yr) is an
b. Urine electrolytes and calculation of fractional excretion
accurate reflection of GFR and can be used to predict the
of sodium
risk for acute kidney injury
c. Urgent initiation of hemodialysis
b. Aggressive immunosuppression for elderly patients with
d. Intravenous hydration with normal saline at a rate of 500
glomerulonephritis is associated with a five-fold higher
ml/h
risk of death than in younger patients
e. Placement of a foley catheter and urgent renal ultrasound
c. Acute interstitial nephritis is less commonly seen in elderly
patients than in younger cohorts.
3. Which statement regarding recovery of renal function after
d. The likelihood of complete renal recovery after acute kid-
acute kidney injury in the elderly is correct?
ney injury in the elderly is similar to that in the general
a. The prognosis for renal recovery in the elderly is similar to
population
that of the general population
e. The diagnostic ability of granular, pigmented casts for
b. Renal function never recovers to baseline after an episode
acute tubular necrosis is poor in the elderly patient
of acute kidney injury in the elderly
2. A 76-yr-old male presents to the emergency department with c. The capacity for renal epithelial cell proliferation declines
complaints of lethargy and fatigue. Laboratory work reveals a with aging as does the function of progenitor and stem
serum creatinine of 6.5 mg/dl, blood urea nitrogen of 104 mg/ cells that are critical for tubular repair
dl, and serum electrolytes are within normal limits. Recent d. Elderly patients with acute kidney injury have a 60%
history reveals that he just began furosemide 40 mg daily for higher rate than the general population of requiring long-
complaints of dyspnea on exertion. Prior laboratory work 1 wk term dialysis

6 Geriatric Nephrology Curriculum American Society of Nephrology


Chapter 19: Nocturia in Elderly Persons and Nocturnal
Polyuria
Dean A. Kujubu
Department of Medicine, UCLA School of Medicine, Kaiser Permanente Los Angeles Medical Center, Los Angeles,
California

Nocturia is defined by the International Conti- Conditions such as congestive heart failure, ne-
nence Society as the interruption of sleep one or phrotic syndrome, autonomic neuropathy, and ve-
more times at night to void.1 Although nocturia is nous insufficiency lead to interstitial edema forma-
relatively uncommon among younger adults, by 80 tion during the day. Mobilization of the
yr of age, the prevalence rises to 80 to 90% in both accumulated interstitial fluid while recumbent re-
men and women.2 The presence of nocturia dis- sults in nocturia. Obstructive sleep apnea is associ-
rupts sleep, leading to daytime somnolence, depres- ated with excessive atrial natriuretic peptide pro-
sive symptoms, cognitive dysfunction, and a re- duction. Neurologic dieases, such as Alzheimer’s
duced sense of well being and quality of life.3 disease and Parkinson’s disease, are associated with
Moreover, nocturia is associated with an increased alterations in the diurnal secretory pattern of neu-
risk morbidity and even mortality.4,5 rohormones, such as natriuretic peptides and anti-
diuretic hormone. Patients with chronic kidney dis-
ease are unable to maximally concentrate their
PATHYPHYSIOLOGY urine and often must void at night.
In many cases, the cause of nocturnal polyuria is
Although it is commonly assumed that nocturia in undefined. In idiopathic nocturnal polyuria, As-
the elderly is primarily a urologic problem, such plund and Aberg8 suggested that anti-diuretic hor-
thinking is inaccurate. The pathophysiology of noc- mone (ADH) levels, which are typically elevated
turia in the elderly involves the complex interplay of during sleep, are abnormally low in these individu-
several factors.6 Age-related changes in the urinary als. This finding is not universally seen, however,
system and in renal function occur. Sleep itself has particularly among women.9 Furthermore, a rela-
effects on renal function. Sleeping patterns and tive nocturnal deficiency of ADH fails to explain the
sleep architecture change with aging. Finally, dis- altered diurnal excretion patterns of sodium and
ease states and medications may affect the urinary nonelectrolyte solutes that occur among these indi-
system, sleep architecture, and renal function. viduals. In some individuals with nocturnal poly-
Common causes of nocturia in the elderly are listed uria, diurnal variation in GFR is absent or even re-
in Table 1. versed, such that creatinine and sodium excretion
rates are higher at night than during the day.10 Some
investigators suggest that these increases are associ-
SYNDROME OF NOCTURNAL POLYURIA ated with higher night-time BP or the “nondip-
ping” phenotype.11
Nocturnal polyuria is a syndrome where the usual
day to night ratio of urine production is altered.7 In
patients with nocturnal polyuria, ⬎33% of the total
daily urine output occurs at night, although the
daily total urine output remains normal. A careful Correspondence: Dean A. Kujubu, Program Director, Nephrol-
voiding diary, incorporating measurements of ogy Fellowship, Clinical Assistant Professor of Medicine, UCLA
voided volumes, is essential to make the diagnosis. School of Medicine, Kaiser Permanente Los Angeles Medical
Center, 4700 Sunset Boulevard, 2nd Floor, Los Angeles, CA
Common causes of nocturnal polyuria are listed in 90027. E-mail: [email protected]
Table 2.
Copyright 䊚 2009 by the American Society of Nephrology

American Society of Nephrology Geriatric Nephrology Curriculum 1


Table 1. Common causes of nocturia in the elderly Table 3. Evaluation of nocturia in the elderly
Bladder dysfunction History: including medications, comorbid conditions
Bladder outlet obstruction (caused by, for example, benign Physical examination: orthostatic vital signs, complete exam
prostatic hyperplasia) including abdominal and genitourinary exams
Severe detrusor dysfunction/large residual urine volume Laboratory studies: electrolytes, calcium, renal function, glucose,
Detrusor overactivity urinalysis
Urinary tract infection Other studies: post void residual volume, urinary flow studies,
Decreased functional bladder capacity voiding diary
Bladder tumor or bladder stones Other studies to be considered: urine culture, polysomnogram,
Pelvic floor laxity (caused by, for example, cystocoele, uterine 24-h ambulatory blood pressure monitor, urodynamic evaluation
prolapse)
Excessive nocturnal urine production deficits related to the sacral nerve roots, including sensory def-
Edema-forming states (e.g., congestive heart failure, nephrosis)
icits, poor sphincter tone, or absent anal wink reflex.
Obstructive sleep apnea
Initial laboratory testing should include assessment of renal
Neurodegenerative conditions (e.g., Parkinson’s disease,
Alzheimer’s disease)
function, blood glucose and electrolytes, serum calcium, and
Diabetes mellitus and diabetes insipidus urinalysis (incorporating microscopic examination of the
Hypokalemia and hypercalcemia (causing nephrogenic diabetes urine). If symptoms suggest infection, a urine culture should
insipidus) be obtained. Urinary flow rate and postvoid residual urine vol-
Drugs (diuretics, calcium channel blockers, caffeine, alcohol, ume should also be assessed. The patient should be requested
SSRI) to keep a careful voiding diary for 3 d. The volume and time of
Chronic kidney disease each void, as well as whether or not the voiding episode dis-
Autonomic neuropathy and venous stasis rupted sleep, should be noted. If bladder dysfunction or blad-
Excessive fluid intake der outlet obstruction is suspected, detailed urodynamic eval-
Idiopathic nocturnal polyuria (possibly caused by deficient
uation may be indicated. A polysomnogram may be indicated
nocturnal ADH secretion)
if symptoms suggest obstructive sleep apnea. A 24-h ambula-
tory BP recording can be considered to ascertain the presence
EVALUATION of nondipping at night.

The evaluation of a patient with nocturia is outlined in Table 3.


A careful history and physical examination provide clues to the TREATMENT OF NOCTURIA IN THE ELDERLY
etiology of nocturia. A weakened urinary stream, hesitancy,
and a sense of incomplete voiding suggest bladder outlet ob- Treatment for nocturia in the elderly is outlined in Table 4.
struction. Frequency, urgency, and bladder spasms suggest Simple maneuvers, such as reducing fluid intake for 6 h before
bladder irritation, perhaps caused by infection. Gross hematu- recumbency, are usually not successful. Compression stock-
ria might indicate a bladder tumor or stones. The presence of ings, phototherapy, and pelvic floor exercises can be tried.
concurrent diseases and the use of medications such as diuret- Phototherapy is thought to reset the normal circadian rhythm
ics, calcium channel blockers, and selective serotonin reuptake
inhibitors (SSRIs), and habits such as excessive intake of fluids, Table 4. Treatment of nocturia in the elderly
alcohol, and caffeine are also important to note. Nonpharmacologic
On physical examination, orthostatic vital signs should be Reducing fluid intake 6 h before recumbency
assessed. Evidence of edema-forming states, including venous Reduce caffeine and alcohol intake
insufficiency, should be sought. Abdominal examination and a Dried fruit
careful genitourinary examination should be performed to de- Compression stockings
Biofeedback, bladder/pelvic floor exercises
tect prostatic enlargement in men, pelvic floor laxity in
Phototherapy
women, bladder outlet obstruction—as manifest by a large
Continuous positive airway pressure (for obstructive
postvoid residual urine volume— or evidence of neurologic sleep apnea)
Neuromodulation
Table 2. Causes of the syndrome of nocturnal polyuria Pharmacologic
Congestive heart failure Alpha adrenergic blockers, 5-␣-reductase inhibitors
Obstructive sleep apnea Estrogen creams, hormone replacement
Nephrotic syndrome Nonsteroidal anti-inflammatory agents
Autonomic neuropathy Melatonin
Chronic kidney disease Imipramine
Venous insufficiency Anticholineric agents
Neurologic diseases (Parkinson’s disease, Alzheimer’s disease) Loop diuretics
Idiopathic Desmopressin

2 Geriatric Nephrology Curriculum American Society of Nephrology


that is disrupted in patients with nocturia. Continuous positive • Voiding diaries are essential for diagnosis
• Consider alternative diagnoses
airway pressure for obstructive sleep apnea improves nocturia • Management of nocturia is frequently nonsurgical
in anecdotal cases. • Several therapeutic options are available
Double-blind, placebo-controlled studies have been per-
formed examining various pharmacologic measures in the
treatment of nocturia. Most of the studies are small and are of DISCLOSURES
short-term duration. The administration of loop diuretics None.
timed 6 to 10 h before recumbency, which induces a mildly
hypovolemic state, can be tried but is usually not successful. REFERENCES
The most extensive studies have been performed using desmo-
pressin, a synthetic analog of anti-diuretic hormone. Multi- *Key References
center, double-blind, placebo-controlled trials of oral desmo- 1. van Kerrebroeck P, Abrams P, Chaikin D, Donovan J, Fonda D, Jack-
pressin in both men and women have shown a reduction in son S, Jennum P, Johnson T, Lose G, Mattiasson A, Robertson G,
Weiss J; Standardisation Sub-committee of the International Conti-
nocturnal voiding among patients with nocturnal polyuria
nence Society: The standardisation of terminology in nocturia: report
during a 10- to 12-mo follow-up.12 Adverse effects— head- from the standardisation sub-committee of the International Conti-
ache, nausea, dizziness, and peripheral edema—were seen in nence Society. Neurourol Urodyn 21: 179 –183, 2002
⬍5 to 10% of cases. Hyponatremia was seen in 14% of patients 2. Weiss J: Nocturia. J Urol 163: 5–12, 2000*
but was asymptomatic and mild in most cases. Although ad- 3. Coyne K, Zhou Z, Bhattacharyya S, Thompson C, Dhawan R, Versi E:
The prevalence of nocturia and its effect on health-related quality of
verse effects seen in these trials were few, several anecdotal
life and sleep in a community sample in the USA. BJU Int 92: 948 –954,
cases of severe symptomatic hyponatremia with desmopressin 2003
have since been reported especially when used in the elderly. In 4. Stewart R, Moore M, May F, Marks R, Hale W: Nocturia: a risk factor for
December 2007, the FDA issued a warning on the use of des- falls in the elderly. J Am Geriatrics Soc 40: 1217–1220, 1992
mopressin.13 5. Asplund R: Mortality in the elderly in relation to nocturnal micturation.
Br J Urol Int 84: 297–301, 1999
In those who fail pharmacologic therapy, electrical neuro-
6. Kujubu DA, Aboseif SR: An overview of nocturia and the syndrome of
modulation of the sacral nerves using either transcutaneous or nocturnal polyuria in the elderly. Nature Clin Prac Nephrol 4: 426 –
implantable electrodes has been used in patients with detrusor 435, 2008*
overactivity with some success.14 7. Asplund R: The nocturnal polyuria syndrome. Gen Pharmacol 26:
1203–12309, 1995*
8. Asplund R, Aberg H: Diurnal rhythm of antidiuretic hormone in elderly
CONCLUSIONS subjects with nocturia. Med Sci Res 19: 765–766, 1991*
9. Asplund R, Aberg A: Diurnal variation in the levels of antidiuretic
hormone in the elderly. J Int Med 229: 131–134, 1991
Nocturia in the elderly is a common condition associated with 10. Fujii T, Uzu T, Nishimura M, Takeji M, Kuroda S, Nakamura S, Inenaga
increased morbidity and mortality. Nocturia results from the T, Kimura G: Circadian rhythm of natriuresis is disturbed in nondipper
complex interplay of age-related changes in the urinary system, type of essential hypertension. Am J Kidney Dis 33: 29 –35, 1999
renal function, sleeping patterns, concurrent disease states, 11. Graugaard-Jensen C, Rittig S, Djurhuus J: Nocturia and circadian
habits, and medications. Appropriate treatment requires iden- blood pressure profile in healthy elderly male volunteers. J Urol 176:
1034 –1039, 2006
tifying the correct underlying pathophysiologic cause of noc- 12. Lose G, Mattiasson A, Walter S, Lalos O, van Kerrebroeck P, Abrams
turia in a given patient. The syndrome of nocturnal polyuria, a P, Freeman R: Clinical experiences with desmopressin for long-term
frequent cause of nocturia among the elderly, may result from treatment of nocturia. J Urol 172: 1021–1025, 2004*
decreased nocturnal secretion of antidiuretic hormone. Sev- 13. Alert: Information for healthcare professionals: desmopressin acetate
eral treatments options are available for nocturia in the elderly, (marketed as DDAVP nasal spray, DDAVP rhinal tube, DDAVP, DDVP,
Minirin, and Stimate nasal spray). Available online at: http://www.
depending on the underlying cause. fda.gov/CDER/DRUG/infoSheets/HCP/desmopressinHCP.htm.
14. Hassouna MM, Siegel SW, Nÿeholt AA, Elhilali MM, van Kerrebroeck
PE, Das AK, Gajewski JB, Janknegt RA, Rivas DA, Dijkema H, Milam
TAKE HOME POINTS DF, Oleson KA, Schmidt RA: Sacral neuromodulation in the treatment
• Nocturia is a common condition in individuals ⬎60 yr old and is asso- of urgency-frequency symptoms: a multicenter study on efficacy and
ciated with morbidity and mortality safety. J Urol 163: 1849 –1854, 2000

American Society of Nephrology Geriatric Nephrology Curriculum 3


REVIEW QUESTIONS: NOCTURIA IN ELDERLY b. Nocturnal antidiuretic hormone secretion is decreased in
PERSONS AND NOCTURNAL POLYURIA all cases
c. Oral desmopressin is both ineffective and potentially dan-
1. Which of the following is most correct regarding nocturia in
gerous
the elderly?
d. Continuous positive airway pressure (CPAP) may help
a. The majority of cases are due to urologic issues
obstructive sleep apnea but has no effect on nocturnal
b. It disrupts quality of life but has no effects on morbidity or
polyuria
mortality
c. Its prevalence increases to 80 to 90% among those 80 yr old
4. All of the following are causes of the syndrome of nocturnal
and older
polyuria EXCEPT:
d. Chronic kidney disease is associated with decreased urine
a. Congestive heart failure
output
b. Venous insufficiency
2. Which of these is NOT associated with nocturia in the elderly? c. Autonomic dysfunction
a. Detrusor overactivity d. Acute use of nonsteriodal anti-inflammatory drugs
b. Hypercalemia
c. Neurodegenerative conditions 5. All of the following treatment options have been used for noc-
d. Syndrome of inappropriate antidiuretic hormone turia in the elderly EXCEPT:
a. Neuromodulation
3. Which of the following is most correct regarding the syndrome b. Phototherapy
of nocturnal polyuria? c. Biofeedback
a. Oral loop diuretics taken 6 to 10 h before recumbency d. Calcium channel blockers
reduce nocturnal urinary frequency

4 Geriatric Nephrology Curriculum American Society of Nephrology


Chapter 20: Hemodialysis in Elderly Patients
Seth Wright
Beth Israel Deaconess Medical Center, Boston, Massachusetts

When dialysis was a scarce resource worldwide, pa- WHAT IS THE LIFE EXPECTANCY OF
tients of advanced age were often excluded. How- ELDERLY HD PATIENTS?
ever, this has changed dramatically. Hemodialysis
(HD) of elderly patients has become routine, just as Unsurprisingly, mortality is considerably higher for
selected nonagenarians and centenarians now un- elderly patients on dialysis than for elderly patients
dergo invasive procedures such as coronary bypass who are not. For example, the actuarial life expect-
or valve repair. As a consequence of the aging of the ancy of a 75-yr-old patient on dialysis is approxi-
dialysis population, new issues have emerged. HD mately 3 yr, as opposed to 11 yr for one not on
in elderly and very elderly patients has important dialysis.1 A Canadian database study from the late
differences from younger patients, and an under- 1990s found that patients older than 75 had survival
standing of these issues is critical in effectively guid- at 1 and 5 yr of 69.0 and 20.3%, respectively, after
ing their care. HD initiation.4 For the very old, such as those 90 yr
Before summarizing the available data, there is and older when starting dialysis, survival is ⬍50%
an important caution that applies to many studies at 1 yr.3
describing elderly patients. Until recently, many However, it is important to note that survival is
studies used a cut-off of 65 yr old (or even younger) influenced strongly by comorbidities such as vascu-
as a definition of elderly, grouping all patients over lar disease and cardiac disease. Once these are taken
that age. However, the medical and social issues are into account, the age of elderly patients is not an
clearly very different for a patient in her late 60s in independent risk factor for increased mortality in
comparison to another in her late 90s. Attention to some models.5 This is consistent with the approach
the age of the study group is important when con- that the characteristics of an individual patient are
sidering how study results might apply to an indi- more important than numerical age. Dialysis
vidual patient. should not be withheld on the basis of age alone if
otherwise appropriate.

EPIDEMIOLOGY WHEN SHOULD HD BE INITIATED IN THE


ELDERLY AND WHAT MORTALITY BENEFIT
Older patients are now the rule rather than the ex- DOES IT CONFER?
ception in HD. According to USRDS data from
2006, nearly one half of incident dialysis patients in The ideal timing of dialysis initiation for slowly pro-
the United States are senior citizens, with the me- gressive chronic kidney disease (CKD) in the elderly
dian age at dialysis initiation at 64.4 yr old.1 Fur- is not known, and this is likely to remain uncertain
thermore, the elderly are the fastest-growing group given the impossibility of randomization. For now,
of incident dialysis patients, meaning that this me- there are no criteria that are specifically tailored for
dian age will continue to increase.2,3 Nearly all of elderly patients. In practice, elderly patients are ac-
these elderly patients employ HD, with only 3 to 5% tually started at a higher level of remaining kidney
using peritoneal dialysis (PD). function than younger patients, although selection
The rapid growth in this population will pre-
sumably be accompanied by a rise in per-patient Correspondence: Seth Wright, Beth Israel Deaconess Medical
dialysis expenditures, because costs for HD in a per- Center, 330 Brookline Avenue, DA 517, Boston, MA 02215. E-
son over 65 yr average 10 to 35% more than for a mail: [email protected]

person under 65. Copyright 䊚 2009 by the American Society of Nephrology

American Society of Nephrology Geriatric Nephrology Curriculum 1


bias and errors in estimating GFR presumably confound this.6 WHAT FACTORS INFLUENCE QUALITY OF LIFE FOR
In addition, as time goes on, the trend is for elderly patients to ELDERLY PATIENTS ON HD?
be initiated at earlier and earlier stages of kidney dysfunction.3
This will likely complicate comparisons between studies be- Independent of mortality, quality of life is an important factor
cause some of these new patients being started on dialysis in decisions regarding HD in elderly patients. One survey
would have died before starting dialysis in prior years. found that most elderly patients would elect dialysis if offered
There are no randomized trials to assess the benefit of HD in but that relief of symptoms was a major goal rather than sur-
the elderly once initiated. It is reasonable to conjecture that the vival alone.15 Overall, patients on HD rate their quality of life as
benefit of dialysis might be limited when the risk of death from poorer than age-matched controls.16 This difference may be
other causes is high. One registry study found that older pa- particularly dramatic for the elderly, who are at more risk of
tients with advanced CKD were more likely to die than to have intradialytic hypotension17 and may be more exhausted by a
dialysis, although it is unclear how many declined dialysis.7 dialysis session. These issues do not mean that dialysis is con-
Another study in the United Kingdom followed a cohort of traindicated in elderly patients, but they are factors that need to
elderly patients who were offered dialysis when estimated GFR be weighed in decisionmaking.
fell below 15 ml/min per 1.73 m2. The patients who accepted Traveling to an HD unit can be extremely time consuming,
dialysis lived longer than those who declined it, but notably especially if independent driving is no longer an option. Fur-
this difference was not found in those who had the highest thermore, patients who depend on public or ambulance trans-
comorbidities.8 Selection bias and small sample size prohibit portation rather than a car have poorer quality-of-life mea-
general conclusions from this study. sures18 and survival. Home HD addresses this issue but is
relatively rare.
Overall, the prevalence of functional impairment is high in
elderly dialysis patients. Patients on HD have a higher inci-
HOW DOES HD COMPARE WITH PD IN ELDERLY dence of cognitive impairment than similarly aged patients
PATIENTS? with less severe CKD.19,20 Almost all have some sort of func-
tional disability.21 Of particular concern is a lack of indepen-
As noted, only about 4% of elderly dialysis patients in the dent mobility, which is associated with a higher death rate.3
United States use PD. The rationale for this is not clear, because Hospitalization rates are high, with approximately 2 hospi-
the proportion is much higher in other countries. Quality of talizations/yr; however, this is not appreciably different from
life and mortality seem to be about the same between HD and that seen in younger dialysis patients,1 although the duration of
PD in this age group in several studies,9,10 although selection stay may be longer.
bias may confound this and others have found that subgroups
(such as diabetics) may do better with HD.11 A recent study
reported poorer long-term survival among older patients on WITHDRAWAL FROM HD
PD than on HD. However, this study defined old rather liber-
ally as ⬎60 yr of age, and differences between modalities only Sometimes HD in elderly patients is initiated with the under-
appeared several years after initiation.12 standing that it is a time-limited trial. Even if this is not the
Overall, differences are not likely to be large. PD is likely an case, elderly HD patients are in general quite ill, and consider-
underused modality13 and should still be offered freely to el- ation should be given to the possibility of a short lifespan. It is
derly patients. This topic is discussed in more detail in a sepa- worth noting that being on HD is associated with poor out-
rate chapter. comes after cardiopulmonary resuscitation: in one series, only
8% of dialysis patients receiving CPR survived to hospital dis-
charge and only 3% were alive 6 mo later.22
About 30% of patients aged 75 and over withdraw from
WHAT IS THE OPTIMAL VASCULAR ACCESS their dialysis at the end of life.1 On the other hand, withdrawal
STRATEGY? may not be required even when undergoing palliative care.
Medicare hospice benefits apply to patients who continue on
There are several concerns regarding vascular access in the el- HD as long as the imminent cause of death is nonrenal, al-
derly, particularly that applying guidelines designed for the though the details of availability vary locally.23,24
general population may lead to unnecessary procedures.14 As
in younger patients, the observation remains that catheters are
definitely associated with higher mortality than other types of TAKE HOME POINTS
access. Unfortunately, the data on other aspects of vascular
• The elderly dialysis population is growing, and nearly all use HD as a
access in elderly patients are conflicting, with studies not modality
reaching consensus on the preferred location or type of access. • HD is not substantially different from PD in outcomes and both should
This topic is discussed in more detail in a separate chapter. be offered

2 Geriatric Nephrology Curriculum American Society of Nephrology


• The mortality benefit of initiating HD is likely reduced in patients with and costs in the North Thames Dialysis Study of elderly people on
the most comorbidities dialysis: a prospective cohort study. Lancet 356: 1543–1550, 2000
• Life expectancy is short for many elderly patients after dialysis initiation, 10. Selgas R, Cirugeda A, Fernandez-Perpén A, Sánchez-Tomero JA, Bar-
but this varies widely and is based more on comorbidities than on ril G, Alvarez V, Bajo MA: Comparisons of hemodialysis and CAPD in
numerical age patients over 65 years of age: a meta-analysis. Int Urol Nephrol 33:
259 –264, 2001
11. Winkelmayer WC, Glynn RJ, Mittleman MA, Levin R, Pliskin JS, Avorn
J: Comparing mortality of elderly patients on hemodialysis versus
DISCLOSURES peritoneal dialysis: a propensity score approach. J Am Soc Nephrol
None. 13: 2353–2362, 2002
12. McDonald SP, Marshall MR, Johnson DW, Polkinghorne KR: Relation-
ship between dialysis modality and mortality. J Am Soc Nephrol 20:
155–163, 2009
REFERENCES 13. Harris SA, Lamping DL, Brown EA, Constantinovici N; North Thames
Dialysis Study (NTDS) Group: Clinical outcomes and quality of life in
*Key References elderly patients on peritoneal dialysis versus hemodialysis. Perit Dial
1. U.S. Renal Data System: USRDS 2008 Annual Data Report: Atlas of Int 22: 463– 470, 2002
Chronic Kidney Disease and End-Stage Renal Disease in the United 14. O’Hare AM, Bertenthal D, Walter LC, Garg AX, Covinsky K, Kaufman
States. Bethesda, MD, National Institute of Diabetes and Digestive JS, Rodriguez RA, Allon M: When to refer patients with chronic kidney
and Kidney Diseases, 2008* disease for vascular access surgery: should age be a consideration?
2. Coresh J, Astor BC, Greene T, Eknoyan G, Levey AS: Prevalence of Kidney Int 71: 555–561, 2007
chronic kidney disease and decreased kidney function in the adult US 15. Ahmed S, Addicott C, Qureshi M, Pendleton N, Clague JE, Horan MA:
population: Third National Health and Nutrition Examination Survey. Opinions of elderly people on treatment for end-stage renal disease.
Am J Kidney Dis 41: 1–12, 2003 Gerontology 45: 156 –159, 1999
3. Kurella M, Covinsky KE, Collins AJ, Chertow GM: Octogenarians and 16. Evans RW, Manninen DL, Garrison LP Jr, Hart LG, Blagg CR, Gutman
nonagenarians starting dialysis in the United States. Ann Intern Med RA, Hull AR, Lowrie EG: The quality of life of patients with end-stage
146: 177–183, 2007* renal disease. N Engl J Med 312: 553–559, 1985
4. Jassal SV, Trpeski L, Zhu N, Fenton S, Hemmelgarn B: Changes in 17. Davenport A: Intradialytic complications during hemodialysis. Hemo-
survival among elderly patients initiating dialysis from 1990 to 1999. dial Int 10: 162–167, 2006
CMAJ 177: 1033–1038, 2007 18. Moist LM, Bragg-Gresham JL, Pisoni RL, Saran R, Akiba T, Jacobson
5. Couchoud C, Labeeuw M, Moranne O, Allot V, Esnault V, Frimat L, SH, Fukuhara S, Mapes DL, Rayner HC, Saito A, Port FK: Travel time to
Stengel B; for the French Renal Epidemiology and Information Net- dialysis as a predictor of health-related quality of life, adherence, and
work (REIN) registry: A clinical score to predict 6-month prognosis in mortality: the Dialysis Outcomes and Practice Patterns Study (DOPPS).
elderly patients starting dialysis for end-stage renal disease. Nephrol Am J Kidney Dis 51: 641– 650, 2008
Dial Transplant (in press)* 19. Kurella M, Chertow GM, Luan J, Yaffe K: Cognitive impairment in
6. U.S. Renal Data System: USRDS 2000 Annual Data Report: Atlas of chronic kidney disease. J Am Geriatr Soc 52: 1863–1869, 2004
Chronic Kidney Disease and End-Stage Renal Disease in the United 20. Murray AM, Tupper DE, Knopman DS, Gilbertson DT, Pederson SL, Li
States. Bethesda, MD, National Institute of Diabetes and Digestive S, Smith GE, Hochhalter AK, Collins AJ, Kane RL: Cognitive impair-
and Kidney Diseases, 2000 ment in hemodialysis patients is common. Neurology 67: 216 –223,
7. O’Hare AM, Choi AI, Bertenthal D, Bacchetti P, Garg AX, Kaufman JS, 2006
Walter LC, Mehta KM, Steinman MA, Allon M, McClellan WM, Land- 21. Cook WL, Jassal SV: Functional dependencies among the elderly on
efeld CS: Age affects outcomes in chronic kidney disease. J Am Soc hemodialysis. Kidney Int 73: 1289 –1295, 2008
Nephrol 18: 2758 –2765, 2007* 22. Moss AH, Holley JL, Upton MB: Outcomes of cardiopulmonary resus-
8. Murtagh FE, Marsh JE, Donohoe P, Ekbal NJ, Sheerin NS, Harris FE: citation in dialysis patients. J Am Soc Nephrol 3: 1238 –1243, 1992
Dialysis or not? A comparative survival study of patients over 75 years 23. Wright AA, Katz IT: Letting go of the rope–aggressive treatment,
with chronic kidney disease stage 5. Nephrol Dial Transplant 22: hospice care, and open access. N Engl J Med 357: 324 –327, 2007
1955–1962, 2007 24. Thompson KF, Bhargava J, Bachelder R, Bova-Collis R, Moss AH:
9. Lamping DL, Constantinovici N, Roderick P, Normand C, Henderson L, Hospice and ESRD: knowledge deficits and underutilization of pro-
Harris S, Brown E, Gruen R, Victor C: Clinical outcomes, quality of life, gram benefits. Nephrol Nurs J 35: 461– 466, 2008

American Society of Nephrology Geriatric Nephrology Curriculum 3


REVIEW QUESTIONS: HEMODIALYSIS IN ELDERLY 3. Regarding quality of life and disabilities for the elderly on he-
PATIENTS modialysis:
a. Hemodialysis is associated with a high incidence of cogni-
1. Hemodialysis initiation in the elderly: tive dysfunction
a. Generally occurs at a later stage of kidney disease (lower b. Approximately 15% of elderly hemodialysis patients have
eGFR) than in younger patients some sort of functional disability
b. Has leveled off in rate, after an increase over the last 10 yr c. Travel time to dialysis centers is not associated with de-
c. Is being started earlier (i.e., at higher eGFRs) than in the creased quality-of-life ratings
past d. Hospitalization rate is approximately double that of non-
d. Is governed by specific KDOQI guidelines covering this elderly dialysis patients
age group
4. Which of the following is true for elderly patients?
2. Elderly patients on hemodialysis: a. Hemodialysis is associated with shorter survival than peri-
a. If over 90 yr old, are likely to survive less than 1 yr toneal dialysis in observational studies
b. If 75 or older, on average live 3 yr b. More than 95% of elderly patients use hemodialysis as
c. Have life expectancy governed more strongly by comor- their dialysis modality in the United States
bidities than by numerical age c. Unlike in younger patients, catheters are not associated
d. Have life expectancies much less than elderly patients not with worsened mortality compared with other access types
on dialysis d. Dialysis must be discontinued for a patient to be eligible
e. c and d for hospice
f. All of the above

4 Geriatric Nephrology Curriculum American Society of Nephrology


Chapter 21: Vascular Access for Hemodialysis in the
Elderly
Seth Wright and John Danziger
Beth Israel Deaconess Medical Center, Boston, Massachusetts

All types of hemodialysis access are potentially that promote fistulas—the rate of fistula placement
problematic in elderly persons. Fistulas or grafts in the elderly has been rising. It is uncertain whether
can be difficult to initiate because of atherosclerosis this approach will improve long-term patient sur-
and prior vascular damage and additionally pose a vival or morbidity. There is some evidence that
risk of causing distal ischemia. In patients with more aggressive fistula placement is being accom-
heart failure, the extra cardiac load of an arterio- panied by a greater failure rate.2 Despite this general
venous shunt might raise concern. Last, in some trend, it is important to keep in mind that KDOQI
cases, the maturity time for a fistula may exceed the also advises tailoring the vascular access options for
lifespan of the patient, and therefore, placing one the individual patient.3 As will be detailed below,
exposes the patient to a procedure for no benefit. this is particularly important for the elderly patient
Although catheters are simpler to initiate, elderly in whom the risk/benefit balance of interventions
patients are also at higher risk of death from infec- may be less well defined.
tion or other complications associated with them Before exploring the issues of vascular access for
than are younger patients. elderly patients in more detail, there is one impor-
tant caveat. The definition of “elderly” is not con-
sistent among various studies, making comparisons
difficult. Most studies have used 65 yr old as a cut-
EPIDEMIOLOGY AND BACKGROUND
off. However, this is somewhat unsatisfying, be-
cause the issues facing a patient in his or her 60s may
Despite these disadvantages, most elderly patients
be quite different from those facing an octogenar-
treated for end-stage kidney disease (ESKD) in the
ian. Only recently have some studies begun to ex-
United States are on hemodialysis and therefore re-
amine differences specifically in the “older old.”
quire one of these options. This situation has arisen
because there are limited alternatives. For one, renal
transplantation in patients over 65 is relatively un-
WHAT IS THE PREFERRED ACCESS IN AN
common. Furthermore, peritoneal dialysis is rarely
OLDER PATIENT?
used in the elderly in the United States, representing
only about 4% of the dialysis patients over 70. This
There are no randomized trials to answer this ques-
tiny rate is far less than in other countries and likely
tion definitively. Older patients have shorter life ex-
represents underuse, but it is nevertheless continu-
pectancies and more comorbidities, so one might
ing to decrease.
speculate that there is less opportunity to realize a
The vast majority of these accesses are central
benefit from a fistula. However, the limited data
venous catheters: for patients over 75, the rate of
available from observational studies show a mortal-
catheter use is nearly 85% at the time of dialysis
ity pattern that favors fistulas in the elderly, just as
initiation. One study examining dialysis patients 65
in younger patients. Specifically, in patients over 67
to 75 and ⬎75 yr of age found that even 3 mo after
yr old, fistulas are associated with lower death rates
starting dialysis, two thirds were still using a cathe-
ter.1
KDOQI recommendations for vascular access Correspondence: Seth Wright, Beth Israel Deaconess Medical
do not differentiate between younger and older pa- Center, 330 Brookline Avenue, DA 517, Boston, MA 02215. E-
tients, so under the current “fistula first” ap- mail: [email protected]

proach—including the advent of quality measures Copyright 䊚 2009 by the American Society of Nephrology

American Society of Nephrology Geriatric Nephrology Curriculum 1


than grafts or catheters, with catheters having by far the worst who has a short life expectancy or who is averse to invasive
outcome (HR 1.70).4 This implies that fistulas are preferred, procedures.
but it is difficult to draw firm conclusions because these are
observational data. It is likely that fistula placement is also
acting as a marker that identifies healthier patients. Further- IF IT IS TO BE PLACED, WHERE SHOULD THE
more, these data reflect only the situation in which a fistula has FISTULA BE LOCATED?
already been placed, has matured, and is being used for access.
The lack of prospective data makes it difficult to offer definitive This is also a matter of current debate. In younger patients,
practice guidelines. With that as a caveat, however, existing there is a preference for a distal-to-proximal fistula placement,
clinical data can still provide guidance to help choose the ap- with the intent being to preserve long-term dialysis options.
propriate access for elderly patients. However, an elderly patient with multiple comorbidities may
not live long enough for the exhaustion of dialysis sites to be
relevant. Because there have been some studies showing that
the most distal location (radiocephalic) has a lower success rate
WHAT IS THE NATURAL HISTORY OF A NEWLY in the elderly than brachiocephalic fistulas (e.g., 81 versus 66%
CREATED FISTULA IN AN OLDER PATIENT? patency at 1 yr), the possibility of starting with an upper-arm
fistula has been raised.6,8 However, there are conflicting studies
One approach to better understand this question is to examine that find no difference between locations in elderly patients,7
the natural history of each kind of access. Some studies suggest and in the absence of consensus, no firm conclusion can be
that age is a risk factor for a nonmaturing fistula. The RE- drawn.
DUCE-FTM I study found that age ⬎65 doubled the risk of A reasonable approach is to have a low threshold for at-
nonmaturity within 6 mo of placement,5 perhaps not surpris- tempting an upper arm fistula in an elderly patient if the radio-
ingly given the higher rates of atherosclerotic disease in older cephalic site is complicated and life expectancy is not likely to
patients. A recent large meta-analysis reached similar conclu- be limited by the need for multiple dialysis sites.
sions, suggesting that elderly patients were 70% more likely to
have failure of fistulas at 1 yr compared with younger patients.6
This meta-analysis has been criticized for including studies ARE GRAFTS ASSOCIATED WITH DIFFERENT
with variable age cut-offs, although in practice these findings OUTCOMES THAN FISTULAS?
do not seem to have been driven by any particular study.
However, other studies suggest that there is no difference in In the general dialysis population, fistulas are currently pre-
fistula survival in the elderly. A recent large retrospective co- ferred over grafts on the basis of patency and complication
hort study of patients referred for fistulas, dividing patients issues. However, in elderly patients specifically, one observa-
into ages ⬍65, 65 to 79, and ⱖ80 yr, found no difference be- tional study failed to find a difference between fistulas and
tween any of the age groups in fistula patency at 1 yr.7 It should grafts in rates of intervention or mortality.9 Another observa-
be noted that the overall fistula success rate in this study was tional study found a modest mortality benefit associated with
low, with only 50% of patients actually using their access at 1 yr fistulas over synthetic grafts (mortality HR 1.16) but not over
without major revision, but nevertheless, this suggests that age autologous vein grafts.4
is not an important factor. Therefore, it seems that fistulas are still the first choice in
How can we resolve these conflicting studies? Because there elderly patients but that grafts may be an acceptable alternative
was no randomization, all of these studies only included pa- when obstacles arise. The combination of an overall shortened
tients in whom fistulas were placed. Furthermore, there may expected life survival in elderly compared with younger pa-
have been variable aggressiveness on the part of surgeons and tients further challenges the notion of fistula before graft. In
referring physicians that may have affected outcomes. In sup- addition, the ability to use grafts soon after placement mini-
port of the latter is the observation that the death rates reported mizes the duration of catheter exposure, potentially decreasing
were lower than for dialysis patients in general, suggesting the infectious risks. These considerations might suggest a strat-
some selection bias. egy of a more liberal use of grafts in elderly patients. However,
Overall, it seems that under some circumstances fistulas such an approach remains untested.
work just as well in selected elderly patients as in younger pa-
tients. However, this is not a universal finding, and the balance
of the evidence suggests that elderly patients contemplating a WHAT MEDICATIONS IMPROVE THE PATENCY RATE
fistula should be prepared for a somewhat lower success rate OF GRAFTS OR FISTULAS?
and a longer time to maturity. Independent of differences be-
tween age groups, even in the best circumstances there is a There are no trials that address this in elderly patients in par-
moderate likelihood that a fistula will not be working 1 yr after ticular. In the general population, contrary to what might be
placement, which may affect a recommendation to a patient assumed, the use of antiplatelet agents or anticoagulants for

2 Geriatric Nephrology Curriculum American Society of Nephrology


other medical purposes has not been found to be associated be assumed to be superior to grafts, although this has not yet
with improved access patency and is likely applicable to the been shown definitively in the elderly, and either could be ac-
elderly as well.10 Furthermore, given the high risks associated ceptable depending on the individual. There is likely reduced
with anticoagulation in the elderly, there is little support for fistula maturity and patency compared with younger popula-
anticoagulation solely for maintenance of a vascular access. tions, but this may not apply to selected patients. Given the lack
of randomized trials, there is not—and likely will not be—a
consensus on how the algorithm of developing access should
IS CONGESTIVE HEART FAILURE A be different for elderly patients. The data above suggest some
CONTRAINDICATION TO A GRAFT OR FISTULA? important differences from younger patients but are not yet
established enough to make general recommendations. Over-
Looking at all patients, the rate of high-output congestive heart all, the most important principle is to keep in mind that indi-
failure (CHF) is increased in patients with fistula blood flows vidualizing vascular access is also part of the current national
⬎2 L/min, particularly with an upper arm fistula.11 However, recommendations as well.
in dialysis patients ⬎65 yr old and with diagnosed heart failure,
the death rate from cardiac causes was not higher in those with
a fistula compared with a catheter. Although this is observa-
tional, this suggests that in general the diagnosis of heart failure TAKE HOME POINTS
should not prevent a fistula from being considered.12 • No randomized trials exist to definitively resolve the question as to the
best type of dialysis access in the elderly
• Choices of dialysis access must be made in consideration of many
WHAT IS THE ROLE OF A CENTRAL VENOUS factors
• In appropriately selected patients, arteriovenous fistulae may be ex-
CATHETER?
pected to work as well in the elderly as in a younger population
• Given the shorter expected life span in those elderly patients who have
As noted, catheters are associated with higher mortality in el- multiple comorbidities, arteriovenous grafts may be reasonable
derly patients, and from a medical standpoint should be • Anticoagulants and antiplatelet agents are not recommended in the
avoided when there are alternatives. However, observationally, elderly if the sole purpose is to improve access patency
they are by far the most common access type in this group,
indicating a gap between this principle and actual practice.
One issue is that there seem to be considerable obstacles in DISCLOSURES
transitioning from catheters to more permanent types of ac- John Danziger is a recipient of a Baxter Renal Discoveries Grant.
cess. One study showed that the chance of graduating from a
catheter to a fistula or graft within 90 d is half as great in pa-
tients over 70 yr old compared with those under 50.1
This may be related to the increased difficulty of developing REFERENCES
a usable alternative in these patients. However, this could also
be driven by provider or patient opinion that there is limited *Key References
1. Wasse H, Speckman RA, Frankenfield DL, Rocco MV, McClellan WM:
usefulness of placing an arteriovenous fistula in the face of
Predictors of delayed transition from central venous catheter use to
other life issues. Given the clear advantage of the fistula over permanent vascular access among ESRD patients. Am J Kidney Dis 49:
the catheter even in the elderly, this medical rationale should 276 –283, 2007
be reserved for cases where the certainty of short survival is 2. Patel ST, Hughes J, Mills JL Sr: Failure of arteriovenous fistula matu-
high. ration: an unintended consequence of exceeding dialysis outcome
quality initiative guidelines for hemodialysis access. J Vasc Surg 38:
It is important to keep in mind that, from the patient per-
439 – 445, 2003
spective, catheters have the advantage of being the least inva- 3. NKF: KDOQI Clinical Practice Guidelines and Clinical Practice Recom-
sive procedure. This may be relevant to decisions regarding mendations. Available online at http://www.kidney.org/professionals/
aggressiveness of care. For example, a catheter may be the pre- KDOQI/guideline_upHD_PD_VA/index.htm. Accessed December 5,
ferred initial access for an older patient who wishes to have a 2008*
4. Xue JL, Dahl D, Ebben JP, Collins AJ: The association of initial hemo-
time-limited trial of dialysis before deciding whether to con-
dialysis access type with mortality outcomes in elderly Medicare ESRD
tinue indefinitely. patients. Am J Kidney Dis 42:1013–1019, 2003*
5. Lok CE, Allon M, Moist L, Oliver MJ, Shah H, Zimmerman D: Risk
equation determining unsuccessful cannulation events and failure to
CONCLUSIONS maturation in arteriovenous fistulas (REDUCE FTM I). J Am Soc Neph-
rol 17: 3204 –3212, 2006
6. Lazarides MK, Georgiadis GS, Antoniou GA, Staramos DN: A meta-
A high proportion of elderly persons use catheters rather than analysis of dialysis access outcome in elderly patients. J Vasc Surg
grafts or fistulas despite the observation that catheters are as- 45:420 – 426, 2007*
sociated with worse outcomes in this population. Fistulas may 7. Weale AR, Bevis P, Neary WD, Boyes S, Morgan JD, Lear PA, Mitchell

American Society of Nephrology Geriatric Nephrology Curriculum 3


DC: Radiocephalic and brachiocephalic arteriovenous fistula out- access outcomes and medication use: a USRDS study. Semin Dial 19:
comes in the elderly. J Vasc Surg 47:144 –150, 2008* 535–539, 2006
8. Berardinelli L, Vegeto A: Lessons from 494 permanent accesses in 348 11. Basile C, Lomonte C, Vernaglione L, Casucci F, Antonelli M, Losurdo
haemodialysis patients older than 65 years of age: 29 years of expe- N: The relationship between the flow of arteriovenous fistula and
rience. Nephrol Dial Transplant 13(Suppl 7):73–77, 1998 cardiac output in haemodialysis patients. Nephrol Dial Transplant 23:
9. Chan MR, Sanchez RJ, Young HN, Yevzlin AS: Vascular access out- 282–287, 2008
comes in the elderly hemodialysis population: a USRDS study. Semin 12. Dhingra RK, Young EW, Hulbert-Shearon TE, Leavey SF, Port FK: Type
Dial 20: 606 – 610, 2007 of vascular access and mortality in U.S. hemodialysis patients. Kidney
10. Yevzlin AS, Conley EL, Sanchez RJ, Young HN, Becker BN: Vascular Int 60: 1443–1451, 2001

4 Geriatric Nephrology Curriculum American Society of Nephrology


REVIEW QUESTIONS: VASCULAR ACCESS FOR c. Survival of fistulas has been consistently shown to be
HEMODIALYSIS IN THE ELDERLY shorter in elderly persons
d. The use of anticoagulants or antiplatelet agents in this
1. KDOQI guidelines regarding vascular access in the elderly include: population is not encouraged for the sole purpose of
a. A recommendation that fistulas be avoided in elderly pa- maintaining fistula patency
tients with peripheral vascular disease
b. A recommendation that fistulas not be placed in patients 3. Hemodialysis catheter use in the elderly
with a life expectancy under 1 yr a. Is associated with an increased risk of infection and hospi-
c. An exception to the target goals of fistula placement rates talization in this age group, but not mortality, when com-
(⬎65%) for elderly persons pared with other access
d. No specific recommendations b. Is the most common type of access upon initiation, and
transition to other types of access is at lower rates than
2. Regarding fistula placement in the elderly: in younger patients
a. Radiocephalic fistulas have consistently shown the same out- c. Represents the preferred access for patients carrying the
comes as upper extremity fistulas and should be preferred diagnosis of heart failure
b. Using a graft rather than a fistula has been universally shown d. Is rarely preferred by patients if all options are explained,
to be associated with higher mortality in elderly persons because there are few advantages

American Society of Nephrology Geriatric Nephrology Curriculum 5


Chapter 22: Peritoneal Dialysis in Elderly Patients
Seth Wright and John Danziger
Beth Israel Deaconess Medical Center, Boston, Massachusetts

The issue of whether—and how—to implement of patients older than 65 yr. However, if there was
peritoneal dialysis (PD) in elderly patients is in- not a contraindication, one third of elderly patients
creasingly important given the rapid growth of this elected to start PD rather than HD.4 Elderly patients
dialysis population. PD has some particular advan- on PD can do quite well: the 2- and 5-yr survival of
tages and disadvantages in the elderly. Further- patients over 65 yr of age in Hong Kong was re-
more, these advantages and disadvantages are not ported to be 88 and 56%, respectively.5
always fully understood by medical providers. Not In comparing PD and HD, one should keep in
only is a better understanding of PD in elderly pa- mind that data quality is limited by the inability to
tients relevant for patient autonomy, medical out- randomize patients. The larger prospective cohort
come, and comfort, but there are systemic implica- studies such as the NTDS have subject numbers in
tions for cost and education as well. the hundreds, whereas registry studies are larger but
presumably are confounded by selection bias. Fur-
thermore, all comparisons are complicated by vary-
BACKGROUND AND EPIDEMIOLOGY ing definitions of what age constitutes “elderly.”
Most use ⬎65 as a cut-off, whereas others use 70 or
In the United States, PD is used less frequently in higher.
elderly patients than in younger patients, and the
rate is declining. In recent USRDS data, 12% of pa-
tients ages 20 to 55 were on PD, whereas only 4% of PHYSIOLOGY OF PD IN ELDERLY
dialysis patients ⬎75 yr of age used this modality.1 PATIENTS
This contrasts strikingly with other countries. In
France, PD is dominant in elderly patients, with The fundamental physiology of PD is not age de-
more than one half of all PD patients being ⬎70 yr pendent; a rich capillary plexus brings blood into
old. In Hong Kong, 80% of all dialysis patients are the peritoneum and filtrate flows across the perito-
on PD, with a median age of 62. The United King- neal membrane into the dialysate. However, there
dom and Canada are intermediate, with 17 and are several physiologic considerations unique to el-
12% of incident elderly dialysis patients treated derly patients that may affect clinical outcomes.
with PD.2 Emerging data suggest that peritoneal mesothelial
The cost of PD is generally less than that of he- cells change during the aging process and may be
modialysis (HD).3 Because elderly patients are the more prone to inflammation. Whether this ob-
fastest-growing segment of the dialysis population, served proinflammatory profile in elderly patients
their relatively infrequent use of PD has financial actually has clinical significance remains untested at
implications. this point.
The reasons for the wide variation in use of PD in In addition to possible age-related changes in the
elderly patients are multifactorial, including finan- peritoneal membrane, elderly patients have a
cial, resource availability, and cultural issues. How- higher incidence of intestinal pathology, including
ever, a particular concern is that unfamiliarity of diverticulosis, bowel perforation, and constipation.
providers with the use of PD in elderly patients All of these can affect the underlying physiology of
leads to a self-perpetuating cycle of underuse. This
is especially of note because, given the opportunity, Correspondence: Seth Wright, Beth Israel Deaconess Medical
many elderly would elect PD. It is not always an Center, 330 Brookline Avenue, DA 517, Boston, MA 02215. E-
option; in one study, it was considered contraindi- mail: [email protected]

cated for medical or social reasons in about one half Copyright 䊚 2009 by the American Society of Nephrology

American Society of Nephrology Geriatric Nephrology Curriculum 1


the membrane, as well as the functionality of the peritoneal higher and lower peritonitis rates, as well as showing both a
catheter. In addition, many elderly patients have undergone negative and a neutral effect of having visiting assistance. Over-
previous abdominal surgeries, which will increase the risk of all, the data do not show a consistent difference. For example,
adhesions and potential abdominal wall leaks. a recent study found that, in comparing incident patients over
65 versus under 65, all of whom were performing their own
care, there was no difference in the probability of being peri-
GENERAL CONSIDERATIONS IN CHOOSING PD tonitis free for 12 mo (76.6 versus 76.5%).9 When peritonitis
VERSUS HD does occur, several studies have found different proportions of
causative organisms than are seen in younger patients. How-
Both PD and HD pose challenges for elderly patients. PD re- ever, these studies are not consistent in their findings, and
quires an appropriate living situation, some degree of mobility there is no recommendation for different empiric treatment on
and vision, a peritoneum not disrupted by prior surgeries, and this basis. Interestingly, there is evidence that exit site and tun-
the ability to learn and independently perform a daily medical nel infections are less common in elderly patients, perhaps be-
technique. Although many elderly can accomplish this, for cause of less vigorous activity.6
very elderly patients (⬎80 yr old), ⬎60% may require assis-
tance.6 However, PD has the advantage of being physiologi-
cally gentle and can be performed at home. IS THERE A DIFFERENCE IN QUALITY OF LIFE
HD requires less technical participation, but there can be a BETWEEN HD AND PD FOR ELDERLY PATIENTS?
substantial cost in time spent in the procedure and traveling to
dialysis units, especially if transportation assistance is needed. In younger patients, PD is often associated with a better qual-
The dialysis procedure can also be particularly exhausting for ity-of-life rating than HD, although this is likely in part because
elderly patients. Furthermore, vascular access can be difficult of self-selection. There are limited data specifically in older
in this population. patients, but overall, elderly patients report the same quality of
life whether on HD or PD.10 Of course, PD may be particularly
DOES SELECTING PD VERSUS HD AFFECT appropriate for individual patients who place a high value on
MORTALITY IN ELDERLY PATIENTS? independence or who would prefer to dialyze at night.

Because of the inability to randomize, this question has not


been definitively answered, and studies have had conflicting IS THERE A NUTRITIONAL CONSEQUENCE TO PD?
results. However, in general, mortality is similar. In one case,
the NTDS prospectively followed 125 patients in the United In general, elderly patients on any kind of dialysis have poor
Kingdom over the age of 70 who were initiating dialysis. Al- nutritional status. The data comparing PD to HD are scant, but
though this was not randomized, nearly one half (44%) started there is no clear evidence that nutritional status is poorer in the
PD, indicating that assignment to PD was fairly liberal. The elderly PD patients despite potential protein losses in the efflu-
HD and PD groups had the same rates of hospitalization and ent.6
death.7
From databases, the REIN study in France found that el-
derly PD patients as a whole did as well as HD patients, al- WHAT IS AVAILABLE TO ASSIST ELDERLY PATIENTS
though perhaps slightly worse once unplanned HD starts (who ON PD? WHO CAN HELP?
were presumably more ill) were discounted. Finally, one US
database review found that death rates for nondiabetic patients There are a number of devices designed to accommodate phys-
over 65 without other comorbidities were slightly lower on PD ical disabilities, some of which might apply to the elderly. For
than on HD, whereas the reverse was true for diabetics with or example, there are devices to help “spike” PD fluid bags auto-
without comorbidities in the same age group.8 These data need matically, and there is a desk-mounted clamp that allows one-
to be generalized with caution because these were registry stud- handed operation of the transfer set. There are also magnifying
ies and the effect sizes were not large. devices that can be adapted to assist with the process.
Overall, it seems that on the basis of mortality outcome However, often the elderly will require personal assistance
alone, there is no compelling reason to either avoid or select as well, especially if there are cognitive deficits. Commonly,
PD in elderly patients. this support is provided by a family member, and this can
impose a considerable burden on the caregiver. One study
found that caregivers spent an average of 56 h/wk in care of
ARE THERE DIFFERENCES IN INFECTION RATES elderly PD patients. Although this was less than the time re-
BETWEEN ELDERLY AND YOUNG PATIENTS ON PD? ported for caregivers of elderly HD patients requiring assis-
tance, several markers of quality of life were lower for the PD
There have been conflicting data, with studies showing both caregivers.11

2 Geriatric Nephrology Curriculum American Society of Nephrology


This burden can perhaps be simplified by the common • There is no substantial mortality difference between PD and HD in
elderly patients
practice of using automated cyclers, which do not have a higher • Quality of life is not reported to be different between elderly patients
technique failure rate in the elderly. In addition, in some cir- on PD compared with HD
cumstances, a community nurse may visit twice daily to con- • Infection rates are not higher in elderly versus younger PD patients
nect and disconnect an automated cycler, a practice known as • Assistants or commercially available devices may make PD more acces-
assisted automated PD. Although the availability of this varies, sible to some elderly patients

it could dramatically expand the range of candidates for PD.


One center in Denmark with aggressive PD enrollment re- DISCLOSURES
ported good success in starting patients over 65 on PD, with John Danziger is a recipient of a Baxter Renal Discoveries Grant.
approximately one half overall using assisted automated PD.
This included nearly 50% unplanned starts, i.e., patients who
REFERENCES
required initiation of dialysis less than 9 d after PD catheter
placement. The ability to start dialysis initially on PD is impor-
*Key References
tant, because patients tend to stay with their initial mode of 1. Finkelstein FO, Afolalu B, Wuerth D, Finkelstein SH: The elderly pa-
dialysis.12 tient on CAPD: helping patients cope with peritoneal dialysis. Perit
When a patient is not at home but instead in an assisted Dial Int 28: 449 – 451, 2008
living facility, there may still be PD options, although these are 2. Brown EA: Should older patients be offered peritoneal dialysis? Perit
Dial Int 28:444 – 448, 2008*
rare. Staff turnover limits the training for PD among nursing
3. Shih YC, Guo A, Just PM, Mujais S: Impact of initial dialysis modality
staff, but it is available in some facilities. and modality switches on Medicare expenditures of end-stage renal
disease patients. Kidney Int 68: 319 –329, 2005
4. Jager KJ, Korevaar JC, Dekker FW, Krediet RT, Boeschoten EW;
CONCLUSIONS Netherlands Cooperative Study on the Adequacy of Dialysis (NE-
COSAD) Study Group: The effect of contraindications and patient
preference on dialysis modality selection in ESRD patients in The
Contrary to general assumption, there is no dramatic differ- Netherlands. Am J Kidney Dis 43:891– 899, 2004*
ence in clinical outcomes in elderly patients who are on PD 5. Li PK, Law MC, Chow KM, Leung CB, Kwan BC, Chung KY, Szeto CC:
versus those on HD. Furthermore, quality of life seems to be at Good patient and technique survival in elderly patients on continuous
least as good. Although the available information may be af- ambulatory peritoneal dialysis. Perit Dial Int 27(Suppl 2):S196 –S201,
2007
fected by selection bias, these conclusions remain even in co- 6. Dimkovic N, Oreopoulos DG: Chronic peritoneal dialysis in the el-
horts that have a high use of peritoneal dialysis, indicating that derly. Semin Dial 15: 94 –97, 2002*
they may nevertheless be generalizable. 7. Lamping DL, Constantinovici N, Roderick P, Normand C, Henderson L,
Nevertheless, rates of PD in the elderly are very low in the Harris S, Brown E, Gruen R, Victor C: Clinical outcomes, quality of life,
United States, implying underuse. The elderly already tend to and costs in the North Thames Dialysis Study of elderly people on
dialysis: a prospective cohort study. Lancet 356: 1543–1550, 2000
have less predialysis care and a greater need for urgent dialysis 8. Vonesh EF, Snyder JJ, Foley RN, Collins AJ: The differential impact of
starts, both factors that tend to increase the initial use of HD risk factors on mortality in hemodialysis and peritoneal dialysis. Kidney
over PD. It is important for nephrologists to recognize this Int 66:2389 –2401, 2004*
disparity and guard against assumptions on the basis of age 9. Szeto CC, Kwan BC, Chow KM: Peritonitis risk for older patients on
that would prevent advocating for PD. Particularly for patients peritoneal dialysis. Perit Dial Int 28: 457– 460, 2008
10. Harris SA, Lamping DL, Brown EA, Constantinovici N; North Thames
valuing the PD lifestyle, PD should be offered to the elderly Dialysis Study (NTDS) Group: Clinical outcomes and quality of life in
among their dialysis options. elderly patients on peritoneal dialysis versus hemodialysis. Perit Dial
Int 22: 463– 470, 2002
11. Belasco A, Barbosa D, Bettencourt AR, Diccini S, Sesso R: Quality of
life of family caregivers of elderly patients on hemodialysis and peri-
TAKE HOME POINTS
toneal dialysis. Am J Kidney Dis 48: 955–963, 2006
• PD is rare and probably underused in elderly patients in the United 12. Povlsen JV, Ivarsen P: Assisted peritoneal dialysis: also for the late
States referred elderly patient. Perit Dial Int 28: 461– 467, 2008

American Society of Nephrology Geriatric Nephrology Curriculum 3


REVIEW QUESTIONS: PERITONEAL DIALYSIS IN 3. Peritoneal dialysis in the elderly is:
ELDERLY PATIENTS a. The preferred modality in some countries
b. Desirable, but felt to be contraindicated nearly 90% of the
1. Peritoneal dialysis in the elderly: time by medical or social assessment
a. Is associated with a higher mortality than hemodialysis c. Is rarely selected by elderly patients to whom it is offered
b. Is associated with a higher rate of peritonitis than in because of the logistical difficulties it entails
younger patients, especially gram-negative organisms d. Has been shown to be a poor choice as a means of initiating
c. Should be considered primarily for patients who have no dialysis unless the initiation is planned several months in
vascular access options for hemodialysis advance
d. Unlike in younger patients, is not associated with im-
proved quality of life ratings when compared with hemo- 4. Peritoneal dialysis is:
dialysis a. Available by law in the United States in all nursing homes
where hemodialysis is offered
2. The proportion of elderly dialysis patients using peritoneal b. Associated with better ratings of caregiver quality of life
dialysis in the United States: than is seen in caregivers of patients on hemodialysis, al-
a. Is about 12% and has been stable over the last 10 yr though this may largely be because of self-selection
b. Is about 10%, which is similar to most other countries c. May be associated with lower rates of tunnel infection than
c. Is about 5%, but rising as adaptive technologies and home in younger patients
care make it more accessible despite functional impair- d. Associated with substantially poorer nutritional status
ment than hemodialysis in elderly patients, presumably because
d. Is about 4% and falling of protein losses in the effluent

4 Geriatric Nephrology Curriculum American Society of Nephrology


Chapter 23: Assisted Peritoneal Dialysis in Elderly
Persons
M. J. Oliver*† and R. R. Quinn*
*Division of Nephrology, Department of Medicine, Sunnybrook Health Sciences Centre and the University of
Toronto, Toronto, Canada; and †Department of Health Policy, Management, and Evaluation, University of Toronto,
Toronto, Canada

Peritoneal dialysis (PD) utilization is on the decline perience, 8% of patients were assisted by family
in many regions around the world. There are mul- members and 14% received regular visits by home
tiple potential contributors, but the role of barriers care nurses. For the remainder of this chapter, we
to self-care PD in an aging population is likely an will focus on the role of assistance provided by vis-
important factor to consider. Providing home care iting home care nurses in the management of the
assistance to support elderly persons on PD may elderly persons with end-stage kidney disease
help to overcome these barriers and increase the (ESKD). This specific form of assistance is
number of individuals that can be safely treated in HCAPD.2
the home. This chapter will cover the following is- HCAPD can be a valuable form of therapy for a
sues: (1) what is home care assisted PD (HCAPD); number of reasons. First, the offer of home care
(2) who is a candidate for HCAPD; (3) what are the support can provide reassurance to elderly persons
logistical considerations when providing HCAPD; and their families when considering PD as a treat-
(4) how do patient outcomes on HCAPD compare ment option. In this way, assistance may act as an
with other dialysis modalities; and (5) is HCAPD a incentive to some individuals to choose PD regard-
cost-effective therapy? less of whether it is actually required. Second,
HCAPD is a form of ongoing training or mentoring
that allows patients to gain confidence performing
DEFINING ASSISTANCE PD-related tasks in a supervised setting. Cognitive
impairment is common in the elderly dialysis pop-
Providing assistance to PD patients involves identi- ulation and may make it difficult for patients to
fying and training an individual other than the pa- learn what they need to know to perform PD inde-
tient to perform dialysis-related tasks. These tasks pendently during a traditional training program.
may include connecting the patient to a cycler, set- With ongoing home care support and education by
ting up the cycler, disconnecting from a cycler, or nurses, some patients eventually reach a point over
performing continuous ambulatory peritoneal di- weeks to months to feel comfortable enough to
alysis (CAPD) exchanges. Spouses or other family graduate to self-care PD. Third, HCAPD can act as
members, paid caregivers, or visiting health care a bridge therapy in individuals performing self-care
professionals in the home can provide assistance. PD who develop an intervening illness that makes it
Assisted PD can also be provided by staff in other temporarily difficult to continue on PD. These in-
settings including rehabilitation centers, retirement dividuals can be provided with home care support
homes, nursing homes, and complex continuing for as long as is required. The support can then be
care facilities. The reliance of PD patients on others withdrawn as indicated when the patient recovers.
for assistance in the home is often underappreci- In those that develop permanent barriers to self-
ated and is borne out by studies showing that mar-
ried individuals are more likely to receive PD and
that patients who live alone are less likely to be Correspondence: Matthew Oliver, MD, MHS, FRCPC, Sunny-
treated by this modality. Evidence from the French brook Health Sciences Centre, Room A239, 2075 Bayview Ave-
nue, Toronto, Ontario M4N 3M5, Canada. Phone: 416-480-4755;
PD Registry suggested that one in five patients re- Fax: 416-480-4245; E-mail: [email protected]
quired some form of assistance.1 In the French ex-
Copyright 䊚 2009 by the American Society of Nephrology

American Society of Nephrology Geriatric Nephrology Curriculum 1


care dialysis, the provision of home care assistance can mean medical team, regardless of available assistance in the home.
averting PD technique failure. If assistance was not available, Conditions that are frequently cited by the medical team as
these patients would be forced to switch to in-center hemodi- contraindications to PD in our ongoing study include morbid
alysis (HD). Fourth, HCAPD can function as a form of respite obesity, extensive abdominal scarring, ileostomies, colosto-
care for families or friends who are providing assistance to mies, ileal conduits, gastric tubes, abdominal aneurysms, her-
loved ones. If they are experiencing provider burnout and need nias, colitis, large polycystic kidneys, or other miscellaneous
a break, or would like to travel, home care nurses can tempo- abdominal conditions.3–5 Contraindications are somewhat
rarily provide much needed support to the patient. Elderly subjective and likely vary among programs and physicians. It is
persons may not choose PD because they do not want to bur- not clear whether contraindications as defined above are more
den their families, and the availability of HCAPD can alleviate common in elderly persons.
this concern. In addition, family members that wish to main- Barriers on the other hand make self-care PD difficult and
tain employment but assist with PD when they are available do seem to be more common in elderly persons.3–5 They can be
can be supported with additional visits from a home care nurse classified as physical, cognitive, psychologic, or social. Physical
during the work week. Finally, HCAPD can provide chronic barriers include decreased vision, strength, manual dexterity,
assistance to elderly persons that have permanent mental or or mobility. Cognitive barriers include dementia, psychiatric
physical disabilities. This subgroup is the most resource inten- conditions, learning disabilities, or language barriers. Psycho-
sive and requires the most care but represents only a portion of logic barriers include fear of lack of supervision, fear of isola-
the spectrum of patients treated with HCAPD. This is impor- tion in the home, or feeling generally overwhelmed by the pos-
tant to remember when considering the economic and logisti- sibility of performing of home dialysis. In our incident ESKD
cal impact of providing home care assistance to patients in a population with a mean age of 68 yr, the most common barri-
dialysis program. ers were decreased strength (22%), decreased manual dexterity
(22%), decreased vision (23%), immobility (8%), decreased
hearing (7%), and anxiety (16%). Because barriers are more
IDENTIFYING CANDIDATES FOR ASSISTED PD common in elderly persons, HCAPD patients are significantly
older than self-care PD patients are. For example, in the French
In our program, all new dialysis patients and imminent starts registry, the mean age of assisted patients was 72.6 yr old com-
are identified, assessed by a multidisciplinary team (physician, pared with 51.1 yr old for self-care PD patients.1
nurse, and social worker), and discussed at a weekly multidis- If there are no contraindications to therapy and patient bar-
ciplinary meeting. This process facilitates the identification of riers have been discussed, the patient’s residence is reviewed
important contraindications and barriers to PD therapy, in- and the supports available in the home to assist with dialysis are
cluding those that may not be well documented in the medical identified. Social barriers are residences that are either unstable
history (Figure 1). We define contraindications as a condition (no permanent residence, unclean or unsafe residence) or ones
that make the patient ineligible for PD, in the judgment of the that do not easily permit PD often because of rules and regu-
lations (retirement homes, nursing homes). Considering all
these factors, the team makes a final judgment as to whether a
PD eligilble with no
barriers patient is eligible for PD. Acutely ill patients and patient with
PD ineligible
9% complex social circumstances often require discussion at mul-
because of barrier(s) tiple meetings until these issues are resolved. If a patient
22% chooses PD, barriers are often clarified during the training pe-
riod when the patient attempts to perform self-care. If home
care assistance is required to overcome them, it is arranged.

LOGISTICS OF PROVIDING ASSISTANCE


PD ineligible
because of medical
contraindication(s) To provide HCAPD, the region where it will be available must
13% first be defined, and a critical mass of nurses in that region
PD eligible with
must be trained. The density of the population, the number of
barriers patients requiring assistance that live within a region, and the
56% travel distance between patients are important to consider.
Figure 1. This figure shows that only 13% of incident dialysis Our program was created in the city of North York, Canada,
patients at Sunnybrook Health Sciences Centre had contraindica- which is a borough in the Greater Toronto Area. North York
tions to PD therapy, whereas barriers were very frequent, even in has a population of approximately 600,000 people, and, at the
those considered eligible for PD. The median age of patients time, our program served a prevalent PD population of about
evaluated was 73 yr. 80 patients. We worked with a single home care agency to train

2 Geriatric Nephrology Curriculum American Society of Nephrology


20 to 25 nurses to provide up to two visits per days, 7 d/wk to tients tend to be older and have a higher burden of comorbid
patients that required assistance. Training was conducted by illness (age range in published studies 72.6 to 77.1 yr; Charlson
the home care agency educator, the PD nurses in our regional comorbidity index score 7.0 and 4.3 for HCAPD and self-care
dialysis center, and the PD vendor. Nurses were taught to as- PD, respectively).1,6,7 Risk adjustment is important in studies
sess PD patients, disconnect patients from cyclers, set-up cy- comparing HCAPD to other treatment modalities to account
clers, and perform CAPD exchanges. Assistance was provided for these differences in case-mix severity but has been incom-
liberally at the inception of the program so that recently pletely done to date.
trained nurses were able to develop and maintain their skills. Povlsen and Ivarsen7 found lower patient and technique
Visiting nurses reviewed patients with the PD staff in the re- survival with HCAPD compared with self-care PD after adjust-
gional program every week by telephone. Our PD nurses did ment for age, comorbidity (median number of conditions),
not regularly supervise the visiting nurses in the patient’s and a limited set of baseline laboratory variables (urea, creati-
home, although studies from the French registry suggest it may nine, albumin). The patient survival for autonomous PD and
reduce peritonitis rates.1 assisted PD was approximately 83 and 70%, respectively (data
There are additional considerations. Whenever possible, we not reported, estimated from survival curves). In our unad-
tried to match patients with a single nurse that would provide justed analysis, HCAPD was associated with a death rate of 0.12
consistent care. We also had to make sure that home care visits per patient-year, which is on par with other modalities.1 The
were not missed and that patients and families knew whom to limited evidence available to date makes it difficult to draw any
contact in the event that this did occur. It was also helpful if conclusions about patient and technique survival among pa-
most patients were capable of disconnecting themselves from a tients on HCAPD or how it compares to traditional, self-care
cycler in the event of an emergency or a missed visit. This PD.
provides some flexibility for nursing staff as well because they It is unclear whether rates of peritonitis are higher on
can set up the cycler whenever is convenient during the day and HCAPD. Verger et al.1 performed an unadjusted analysis that
return for an abbreviated visit at night to connect the patient showed higher peritonitis-free survival in HCAPD patients
for the evening. At the start of the program, we elected to use compared with individuals assisted by family members (70 ver-
registered nurses exclusively for home care visits. Now, regis- sus 54%; P ⫽ 0.04). However, this difference disappeared if
tered practical nurses and health care aids are also employed to visiting home care nurses had closer supervision. The actual
provide assistance at the discretion of the visiting registered peritonitis rates reported in the study were not statistically dif-
nurse. They primarily provide care to medically stable patients ferent between the two groups, although the trend was toward
who have permanent physical barriers to PD (e.g., cannot lift a higher rate in HCAPD patients (one episode every 36 mo
bags on to cycler). versus one episode per 45 mo).
There is also conflicting evidence concerning rates of hos-
pitalization in HCAPD. In a study by Lobbedez et al.,6 31 of 36
PATIENT OUTCOMES ON ASSISTED PD assisted PD patients were hospitalized within 15 mo of starting
dialysis. This resulted in a hospitalization rate of 4.8 admis-
The introduction of HCAPD has meant that we have expanded sions per patient-year and a total of 45.6 hospital days per
eligibility for PD.1 By offering HCAPD, we are treating indi- patient-year. We reported much lower rates of hospitalization
viduals in the home that would not have been candidates for in HCAPD patients (1.4 admissions per patient-year; mean of
PD otherwise. As a result, it is important to track outcomes in 23.5 hospital days per patient-year) that were similar to self-
these patients to ensure that this therapy is being provided care PD and in-center HD.1
safely. Relevant outcomes include patient survival, technique In summary, the evidence that is currently available does
survival, peritonitis rates, and rates of hospitalization (Table not allow any firm conclusions about the risk of important
1). adverse events in HCAPD patients or how they compare with
Well-designed studies that are adequately powered to ex- traditional dialysis modalities. Well-designed, adequately
amine outcomes in HCAPD patients are currently lacking. powered studies are required to address this issue. In the mean-
However, preliminary evidence suggests that this population time, assisted PD patients should be considered a higher-risk
may have a relatively high rate of adverse events. HCAPD pa- population and should be monitored carefully.

Table 1. Summary of outcomes in HCAPD reported in the


literature ECONOMIC CONSIDERATIONS
Outcome Result
Technique survival 58–86% at 1 yr There has been a concern among some that HCAPD is not a
Peritonitis rate, one episode One episode per 28–36 mo cost-saving therapy. It has been argued that the incremental
Hospitalizations 1.4–4.8 per patient-year cost associated with providing home care visits eliminates the
Hospital days 23.5–45.6 per patient-year savings traditionally associated with PD compared with in-
Patient survival 83% at 1 yr center HD.8 A recent review of European HCAPD programs

American Society of Nephrology Geriatric Nephrology Curriculum 3


estimated that the cost of assisted PD ranged from €5356 to TAKE HOME POINTS
€18,200 for automated peritoneal dialysis (APD) [or continu-
• Home care assistance may increase the number of elderly persons that
ous cycling peritoneal dialysis (CCPD)] and from €7280 to are candidates for PD by helping to overcome barriers to self-care
€23,400 for CAPD.9 These estimates were based on the reim- • A rigorous multidisciplinary assessment can help to identify important
bursement formula in each country for assisted PD rather than barriers to self-care PD in elderly persons and to determine the need
the actual cost of providing care. More importantly, these cal- for home care assistance
culations assumed that all HCAPD patients received the max- • There are a number of logistical issues to consider when starting a
program of HCAPD including the training of a critical mass of home
imum number of home care visits available them. In practice, care nurses in each region and providing them with supervision and
this is not the case. In fact, we showed the average assisted support
patient only required 5.8 visits/wk in our program, resulting in • Well-designed, adequately powered studies are needed to further
an incremental cost of approximately $12,000 dollars per PD evaluate important patient outcomes on HCAPD and to determine the
patient, per year (assuming a cost of $50 per home care visit). cost-effectiveness of this therapy relative to traditional modalities

This would suggest that HCAPD is still cost saving compared


with in-center HD. An additional consideration is that ex-
panding HD capacity involves a significant capital investment DISCLOSURES
to build or renovate new dialysis units that is not required to R. R. Quinn is supported by a Canadian Institutes for Health Research
expand PD capacity. This has generally not taken into account (CIHR)–Institute for Health Services and Policy Research fellowship.
in costing studies, but is a relevant consideration for healthcare
payers. Better information about the relative effectiveness of
HCAPD compared with traditional dialysis therapies is
needed. If outcomes on HCAPD are shown to be equivalent or REFERENCES
better, a properly conducted cost-effectiveness analysis would
be a valuable contribution to the literature. However, cost ef- *Key References
fectiveness is not the only consideration when deciding 1. Verger C, Duman M, Durand PY, Veniez G, Fabre E, Ryckelynck JP:
Influence of autonomy and type of home assistance on the prevention
whether to fund HCAPD. Countries may choose to fund it as a
of peritonitis in assisted automated peritoneal dialysis patients. An
chronic disease management strategy because it is viewed as a analysis of data from the French Language Peritoneal Dialysis Registry.
way to maintain the independence of elderly persons receiving Nephrol Dial Transplant 22: 1218 –1223, 2007*
dialysis and to reduce the reliance on in-center HD. 2. Oliver MJ, Quinn RR, Richardson EP, Kiss AJ, Lamping DL, Manns BJ:
Home care assistance and the utilization of peritoneal dialysis. Kidney
Int 71: 673– 678, 2007*
3. Little J, Irwin A, Marshall T, Rayner H, Smith S: Predicting a patient’s
CONCLUSION
choice of dialysis modality: experience in a United Kingdom renal
department. Am J Kidney Dis 37: 981–986, 2001
Assistance allows elderly persons with barriers to self-care PD 4. Jager KJ, Korevaar JC, Dekker FW, Krediet RT, Boeschoten EW;
an opportunity to be offered a home-based dialysis modality. Netherlands Cooperative Study on the Adequacy of Dialysis (NE-
The level of assistance required to adequately support elderly COSAD) Study Group: The effect of contraindications and patient
preference on dialysis modality selection in ESRD patients in The
PD patients and their families is highly variable. Patients do not
Netherlands. Am J Kidney Dis 43: 891– 899, 2004
always require the maximal number of visits, and support may 5. Prichard SS: Treatment modality selection in 150 consecutive patients
often be temporary. We outlined several important logistical starting ESRD therapy. Perit Dial Int 16: 69 –72, 1996
considerations when starting a program of HCAPD. Successful 6. Lobbedez T, Moldovan R, Lecame M, de Ligny BH, El Haggan W,
programs need to develop a rigorous process for identifying Ryckelynck JP: Assisted peritoneal dialysis. Experience in a French
renal department. Perit Dial Int 26: 671– 676, 2006
appropriate candidates, train and monitor a critical mass of
7. Povlsen JV, Ivarsen P: Assisted peritoneal dialysis: also for the late
home care nurses, and track the outcomes of patients treated referred elderly patient. Perit Dial Int 28: 461– 467, 2008*
with this form of therapy. It is likely that HCAPD is a cost- 8. Mendelssohn DC: A skeptical view of assisted home peritoneal dial-
saving strategy relative to traditional, in-center HD, but fur- ysis. Kidney Int 71: 602– 604, 2007
ther work is need to better detail the economic considerations 9. Dratwa M: Costs of home assistance for peritoneal dialysis: a Euro-
pean survey. Kidney Int 73(Suppl 108): S72–S75, 2008
for programs and payors.

4 Geriatric Nephrology Curriculum American Society of Nephrology


REVIEW QUESTIONS: ASSISTED PERITONEAL 3. The majority of individuals with ESRD starting chronic dialy-
DIALYSIS IN ELDERLY PERSONS sis are:
a. Ineligible for PD because of contraindications to PD
1. Patients may be provided assistance for peritoneal dialysis
b. Ineligible for PD because of barriers to self-care PD
from:
c. Eligible for PD and have no barriers to self-care PD
a. Spouses or common-law partners
d. Eligible for PD but have barriers to self-care PD
b. Sons or daughters
e. Other
c. Paid caregivers
d. Home care nurses
4. The cost of home care assisted PD:
e. All of the above
a. Is primarily determined by the use of CAPD in the PD
2. The best person or group to identify patients who are candi- population
dates for home care assisted peritoneal dialysis is to b. Is primarily determined by the maximum number of visits
a. Physician available to the PD population
b. Predialysis nurse c. Is primarily determined the mean number of visits pro-
c. Multidisciplinary team vided to the PD population
d. Home care nurse d. Is greater than in-center HD
e. Funding agency e. Does not permit the use of registered nurses

American Society of Nephrology Geriatric Nephrology Curriculum 5


Chapter 24: Renal Transplantation in the Older Adult
Erica L. Hartmann
Abdominal Organ Transplant Program, Department of Internal Medicine-Nephrology, Wake Forest University Health
Sciences, Winston-Salem, North Carolina

BACKGROUND who received a transplant from a deceased donor.


Wolfe et al.2 evaluated ⬎46,000 patients listed be-
Many patients once thought too high risk for renal tween 1991 and 1997, comparing survival by age
transplantation, including those with advancing groups, but the older age group was limited to pa-
age, are now routinely transplanted. Although the tients who were 60 to 70 yr of age because trans-
number of patients younger than 50 yr of age added plantation was infrequent in people over age 70
to the waiting list for kidney transplantation has during the study period. Older transplant recipients
remained relatively static during the last decade, in were shown to have an improved cumulative sur-
the age group over 65 yr, both the number of pa- vival rate after the first posttransplant year. Their
tients newly placed on the waiting list and the num- projected life span increased by 4 yr, and the long-
ber of patients transplanted have tripled in the same term risk of death decreased by 61%.
time frame.1 Accordingly, persons over age 65 made Although waiting times and comorbidity have
up approximately 16% of both the waiting list and increased since the study of Wolfe et al., it seems
of kidney transplant recipients in 2006 compared that renal transplantation in older adults still has
with only 8% in 1997. Despite tremendous growth, acceptable results. In a more recent study using na-
this only represents a small fraction of kidney trans- tional registry data, Rao et al.3 showed that even
plants in an age distribution that makes up one half patients over the age of 70 derive a survival benefit
of incident dialysis patients. with transplantation. In the long term (18 mo after
transplant), the relative risk (RR) of death was 56%
lower for transplant patients versus waiting list pa-
PATIENT AND GRAFT SURVIVAL WITH tients (P ⬍ 0.0001). The survival benefit was im-
RENAL TRANSPLANTATION pressive in those with diabetes or hypertension as a
cause of their kidney failure; in these patients, the
Kidney transplantation is the treatment modality of RR of death with a transplant dropped by 47 and
choice for people with ESRD. Several single-center 44% (P ⬍ 0.0001).
studies have shown that kidney transplantation is
safe and successful in older recipients. In suitable
older candidates, renal transplantation improves MORTALITY IN OLDER RENAL
patient survival over remaining on dialysis. Al- TRANSPLANT CANDIDATES/RECIPIENTS
though renal transplantation more than doubles
the expected lifetime of an older person with ESRD The continued growth in the overall number of
(Table 1), the remaining life years only reach about people registered on the renal transplant waiting list
one half of that expected in people of the same age. has led to a parallel increase in waiting times for
Kidney allograft survival in older transplant recipi- older patients, increasing the odds that older pa-
ents is often comparable to that of younger patients tients will die before receiving a transplant. Older
when the survival analysis is censored for death (i.e.,
a patient who dies with a functioning graft is not
viewed as a graft failure). Correspondence: Erica L. Hartmann, Medical Director, Abdomi-
nal Organ Transplant Program, Assistant Professor, Department
Longitudinal registry studies of mortality have of Internal Medicine-Nephrology, Wake Forest University Health
examined the survival benefit of transplantation by Sciences, Winston-Salem, NC 27157. Phone: 336-716-4830; Fax:
comparing long-term dialysis patients who re- 336-716-4318; E-mail: [email protected]
mained on the transplant waiting list with those Copyright 䊚 2009 by the American Society of Nephrology

American Society of Nephrology Geriatric Nephrology Curriculum 1


Table 1. Life expectancy by age group concrete predictors exist to inform clinicians which patients
Age Group General US will not benefit from transplant.
Dialysis Transplant
(yr) Population
60–64 21 4.6 12.7
65–69 17.2 3.9 10.6 PATIENT SELECTION FOR TRANSPLANTATION
70–74 13.8 3.3 8.9
75–79 10.8 2.8 7.4 Interested older ESRD patients who lack medical or surgical
Source: USRDS 2007 Annual Data Report. contraindications should be referred for a renal transplanta-
tion evaluation. Each transplant program develops its own
candidates who wait years for a deceased donor are at a marked written criteria for determining a patient’s suitability for place-
disadvantage compared with younger patients because of their ment on the waiting list or transplantation. Therefore, one
poorer baseline health at onset of ESRD, rapid changes in should become familiar with centers that have greater expertise
health on dialysis, and higher mortality on the waiting list. or comfort in transplanting older people.
Patient age and duration of dialysis affect a transplant candi- Developing more selective criteria for candidate listing
date’s risk of death. A patient over age 64 who is expected to among older candidates has been advocated by some in the
wait 5 yr for a deceased donor organ has about a 50% chance of transplant community to reduce the number of patients on the
dying before being transplanted. Furthermore, older patients list with marginal potential gain from transplant. Unfortu-
have higher rates of postoperative mortality; therefore, the nately, there currently are no valid screening criteria that accu-
time to which patient survival exceeds remaining on dialysis rately predict which patients will have minimal gain from renal
for older transplant recipients is significantly longer than for transplantation.
younger recipients. The most recent US clinical practice guidelines5 do not set
an absolute age limit for evaluating potential renal transplant
candidates. As more high-risk patients are placed on the wait-
COMORBIDITY IN OLDER RENAL TRANSPLANT ing list, the process of evaluating and maintaining candidacy
CANDIDATES consumes increasing resources. The standard pretransplant
evaluation is heavily weighted on cardiovascular risk assess-
Along with the aging of the dialysis population has come a ment and malignancy screening. The current approach ac-
higher degree of medical complexity in renal transplant candi- knowledges that social support and patient expectations must
dates. Not surprisingly, a higher burden of comorbidity mea- be addressed6; however, guidelines are not concrete. Physi-
sured by the Charlson Comorbidity Index has been associated cians are encouraged to use their best judgment when deciding
with increased perioperative and long-term mortality in older candidacy for older candidates and to assess potential candi-
renal transplant recipients in the United States and Canada. A dates on an individual basis.
US registry study4 of nearly 40,000 kidney transplant recipients The Canadian consensus guidelines on eligibility for kidney
(approximately 9000 ⱖ60 yr of age) examined the impact of transplantation7 recommend that careful consideration of the
increased age and comorbidities on 1-yr patient survival by current waiting time for a deceased donor kidney and the older
including the type of donor organ (younger versus older de- patient’s expected survival beyond that period should be con-
ceased donors) and early organ function (immediate versus sidered when deciding whether or not to proceed with wait
delayed function) in the analysis. The overall 1-yr mortality listing or transplantation in older patients. Although there are
rate was 10.5%, which substantially increased in recipients of little data to support the assessment of physical function or
an older donor kidney with comorbidities, from 16% in dia- nutrition in the decision-making process, the Canadian guide-
betics to 42.3% labeled as having chronic obstructive pulmo- lines extrapolate data from other disease states that suggest that
nary disease. Despite an increased burden of comorbidities, it poor functional capacity or malnutrition might be associated
seems that the overall survival of older transplant recipients with an increased risk of death on the waiting list or after trans-
has improved from 1990 to 1999. Regardless of multiple stud- plantation. Additionally, cognitive function should be evalu-
ies evaluating the impact of comorbidities on outcomes, no ated because it affects the ability to adhere to a complex post-
transplant regimen. The trajectory of cognition in older adults
Table 2. Special issues to consider in older transplant after transplantation is unclear. Although many of the assess-
candidates and recipients ments mentioned are not routine, there are some data support-
Cardiovascular disease ing incorporating such assessments in the future.
Infection/malignancy risk
Psychosocial factors
Immune function WAITING LIST STATUS
Physical function
Cognition It is important to recognize that, although the total number of
Patient expectations/quality of life candidates on the kidney transplant waiting list continues to

2 Geriatric Nephrology Curriculum American Society of Nephrology


grow, the number of active patients on the kidney transplant challenge is to establish which patients will benefit the most from
waiting list has stabilized, and the number of inactive patients these kidneys; however, benefit analyses suggest that older pa-
on the waiting list has rapidly expanded.1 The number of pa- tients have the most to gain when receiving an ECD reduces the
tients over age 65 yr on inactive status (registered on the wait- waiting time to transplant. The time spent waiting for a standard
ing list but will not be called in for transplant) quadrupled deceased donor kidney varies widely by region of the country.
from 2002 to 2006, representing 17% of the inactive waiting list
at the end of 2006. This finding coincides with an Organ Pro-
curement and Transplantation Network (OPTN) policy im- OLDER LIVING DONORS
plemented in November 2003 that allows inactive candidates
to accrue points for waiting time for the entire duration of time Because many older recipients will not live long enough to
they are listed. Although this policy acknowledges that a per- receive a deceased donor kidney transplant, another donor op-
son’s health is a dynamic process, it underscores the need for tion that deserves special mention is the use of older living
the primary nephrologist to be aware of a patient’s status on kidney donors. Living donation in the elderly has grown to the
the waiting list so that they may communicate with the trans- point that people over age 60 now receive over one third of live
plant center when an individual’s health status changes be- donor transplants. Recipients of older living donor kidneys
cause older patients may have a limited window for successful have lower graft survival than younger living donor recipients
transplantation. Likewise, timely communication with the but have superior patient and graft survival compared with all
transplant center may help prevent unsuitable candidates from deceased donor options.
being transplanted. Both the US and Canadian guidelines em-
phasize that the medical fitness of older waitlisted patients be
DECISION ABOUT DONOR TYPE
reviewed on a regular basis.
Kidneys from younger and healthier donors are associated
with lower death rates but require longer waiting times. Schold
DONOR OPTIONS and Meier-Kriesche8 examined whether certain patient groups
would benefit from being transplanted sooner with an ECD
The median waiting time for a standard deceased donor kidney versus delaying transplant in favor of possibly receiving a better
for a recipient over age 65 is nearly 4 yr, which is compounded donor kidney. In separate analyses, the authors estimated the
by the reality that an older person’s medical suitability may expected survival for transplanted patients with short (⬍2 yr)
decline faster on the waiting list than younger patients. Better versus longer durations (2 to 4 yr) of pretransplant dialysis. For
donor organs are associated with lower death rates after renal patients over 65 yr old, the life expectancy for ECD recipients
transplantation. The majority of patients over age 65 yr receive with ⬍2 yr on dialysis was 5.6 yr, which was comparable to
standard donor kidneys, followed by live donors, and last, ex- both Standard Criteria Donor (SCD) (5.3 yr) and living donor
panded donors. In an effort to expand the donor pool, two recipients (5.5 yr) after 4 yr of pretransplant dialysis. This is yet
trends have emerged: the increased utilization of both older another study emphasizing the need for timely transplantation
deceased and older living donors for older recipients. in older transplant candidates because the survival benefit of a
standard or live donor kidney transplant seems to be negated
by longer waiting times on dialysis before receiving a kidney
EXPANDED CRITERIA DONORS transplant. Shifting of ECD kidneys to older patients shortly
after they are placed on the waiting list and SCD kidneys to
The 2002 Expanded Criteria Donor (ECD) policy was a formal younger patients may therefore be the most efficient resource
attempt by the transplant community to maximize the use of de- utilization, meeting the goal of expanding life expectancies for
ceased donor kidneys that had been recovered for transplantation, both age groups.
in part by decreasing the discard rate of older donor kidneys. An
expanded criteria donor is any deceased donor over age 60 or a
deceased donor over age 50 with two of the three following crite- POSTTRANSPLANT MANAGEMENT
ria: serum creatinine ⬎1.5 mg/dl at the time of procurement,
death from cerebrovascular accident, or history of hypertension. Pattern of Acute Rejection, Infections, and Malignancy
Despite a 70% increased risk of graft failure compared with a ref- A unique aspect of transplanting older patients includes the
erence group of younger ideal donor kidneys, recipients of ECD aging of the immune system, which is clinically manifested as a
kidneys are still expected to have a survival benefit over remaining lower risk of acute rejection (supported by most studies) and
on dialysis. The ECD policy has increased access to transplanta- an increased risk of infections and death from infectious causes
tion, primarily for patients over age 50 yr, who receive the major- compared with younger transplant recipients. The risk for in-
ity of ECD organs. Although the ECD policy is viewed as success- fections is linear in older waitlisted patients compared with an
ful, it must be recognized that some believe that such policies exponential slope in older transplant recipients, which sug-
unfairly direct less desirable kidneys to older people. An ongoing gests a magnification caused by transplant immunosuppres-

American Society of Nephrology Geriatric Nephrology Curriculum 3


sion. The slope for posttransplant malignancies is linear in TAKE HOME POINTS
older transplant patients; however, it is steeper than that seen • There is a large and growing representation of older candidates on the
in similar dialysis patients on the waiting list. kidney transplant waiting list, both active and inactive status
Although older patients have lower rates of acute rejection, • The proportion of newly transplanted patients over age 65 has doubled
the impact of acute rejection may be more significant in this over the past decade and is expected to continue growing
• There are no concrete predictors to aid clinicians with the selection of
patient population. Because older patients more often receive
older people for transplantation; however, a higher burden of comor-
older donor kidneys, it is believed that these grafts may not be bidity has been associated with increased perioperative and long-term
able to recover from the insult of acute rejection, especially mortality in older renal transplant recipients
considering the aging milieu of the recipient that may be asso- • Time spent on dialysis before renal transplantation is an important
ciated with poorer repair processes. Additionally, older pa- modifiable risk factor for both patient and graft survival
• Older donor kidneys (both deceased “expanded criteria” and older
tients have less reserve and may not tolerate aggressive acute
living donors) are increasingly being used to help meet the demand for
rejection therapies because of the heightened risk for infection. kidney transplantation by older people with renal failure
• Expanded criteria donor (ECD) organs may decrease the waiting time
Immunosuppression to transplantation; older adults who are transplanted with ECD soon
The observations that older transplant recipients have lower after being placed on the waiting list have similar outcomes to older
recipients of younger donor kidneys.
rates of acute rejection, higher rates of infections, and an exag-
• Older transplant recipients are at higher risk of developing infectious
gerated malignancy risk have led many transplant physicians to complications and malignancies, however, their risk of acute rejection
use less aggressive immunosuppressive strategies in older appears lower
transplant recipients. Options include lower drug doses or • In the long term, the risk for chronic allograft nephropathy is higher
concentrations, rapid steroid withdrawal or minimization, • The optimal immunosuppressive regimen in older renal transplant re-
cipients in unknown; however, clinical studies support using less immu-
non–lymphocyte depleting antibody induction regimens, or
nosuppression overall
avoidance of induction agents altogether. Often, lower con-
centrations of calcineurin inhibitors (cyclosporine or tacroli-
mus) are targeted in recipients of expanded criteria donor kid-
DISCLOSURES
neys to minimize the vasoconstrictive effect because many of
None.
these donor kidneys already have a certain degree of vascular
disease at the time of implantation. At this time, there are in-
sufficient data in the transplant literature to provide the clini-
cian with the optimal immunosuppressive regimen for the REFERENCES
older transplant recipient.
*Key References
1. 2007 Annual Report of the U.S. Organ Procurement and Transplanta-
Chronic Allograft Failure tion Network and the Scientific Registry of Transplant Recipients:
Older recipient age is a significant risk factor for the develop- Transplant Data 1997–2006. Rockville, MD, Department of Health and
ment of interstitial fibrosis and tubular atrophy (aka, chronic Human Services, Health Resources and Services Administration,
allograft nephropathy). Older donor age is synergistic with Healthcare Systems Bureau, Division of Transplantation, 2007*
2. Wolfe RA, Ashby VB, Milford EL, Ojo AO, Ettenger RE, Agodoa LY,
older recipient age in terms of the risk for chronic graft failure,
Held PJ, Port FK: Comparison of mortality in all patients on dialysis,
especially beyond 36 mo after transplant. patients on dialysis awaiting transplantation, and recipients of a first
cadaveric transplant. N Engl J Med 341: 1725–1730, 1999*
3. Rao PS, Merion RM, Ashby VB, Port FK, Wolfe RA, Kayler LK: Renal
CONCLUSIONS transplantation in elderly patients older than 70 years of age: results
from the Scientific Registry of Transplant Recipients. Transplantation
83: 1069 –1074, 2007
The aging of the ESRD population is leading to an increased de-
4. Kauffman HM, McBride MA, Cors CS, Roza AM, Wynn JJ: Early mor-
mand for renal transplantation by older adults. In carefully se- tality rates in older kidney recipients with comorbid risk factors. Trans-
lected recipients, patient and graft survival are acceptable. Older plantation 83: 404 – 410, 2007
transplant recipients benefit the most when transplanted sooner. 5. Kasiske BL, Cangro CB, Hariharan S, Hricik DE, Kerman RH, Roth D,
Older donor kidneys (from both deceased and living donors) are Rush DN, Vazquez MA, Weir MR: The evaluation of renal transplanta-
tion candidates: clinical practice guidelines. Am J Transplant 1(Suppl
increasingly used for transplantation in this population. Older
2): 3–95, 2001*
recipients have lower rates of acute rejection but are at higher risk 6. Danovitch G, Savransky E: Challenges in the counseling and manage-
for the complications of overimmunosuppression, such as infec- ment of older kidney transplant candidates. Am J Kidney Dis 47(4
tions and malignancies. Older transplant recipients are also more Suppl 2): S86 –S97, 2006*
likely to die with a functioning graft or lose their allograft to 7. Knoll G, Cockfield S, Blydt-Hansen T, Baran D, Kiberd B, Landsberg D,
Rush D, Cole E: Canadian Society of Transplantation: consensus guide-
chronic allograft fibrosis and tubular atrophy compared with
lines on eligibility for kidney transplantation. CMAJ 173: S1–S25, 2005
younger recipients. The optimal immunosuppressive approach is 8. Schold JD, Meier-Kriesche HU: Which renal transplant candidates
unclear, but most transplant physicians use less immunosuppres- should accept marginal kidneys in exchange for a shorter waiting time
sion overall in the older transplant recipient. on dialysis? Clin J Am Soc Nephrol 1: 532–538, 2006*

4 Geriatric Nephrology Curriculum American Society of Nephrology


REVIEW QUESTIONS: RENAL TRANSPLANTATION IN 2. A 68-yr-old male with polycystic kidney disease wants a renal
THE OLDER ADULT transplant so he can live longer. Which of the following state-
ments is true?
1. A 71-yr-old male has been dialysis-dependent for 3 yr. He
a. Renal transplantation improves patient survival over re-
seeks your advice about his candidacy for renal transplanta-
maining on dialysis
tion and potential donor options. He previously thought he
b. Better donor organs are associated with lower death rates
was “too old” for a transplant. He has a 60 yr old friend who is
c. ECDs are most beneficial when they reduce the waiting
a potential donor. You determine that he is medically fit for
time to transplant for older recipients
transplant. Which of the following is a reasonable assessment
d. All of the above
of his risk?
a. He is at higher risk of acute and chronic rejection because
3. Which of the following transplant immunosuppressive strat-
of his age
egies is preferable in older renal transplant candidates?
b. He is more likely to die on dialysis than receive a young
a. No induction immunosuppression
deceased donor transplant; he should consent to receiving
b. Reduced mycophenolate mofetil (Cellcept) dose
an older live or expanded criteria donor organ
c. The optimal regimen is unknown
c. He is not a suitable candidate for transplantation because
d. Steroid avoidance
of his advanced age and prolonged time on dialysis

American Society of Nephrology Geriatric Nephrology Curriculum 5


Chapter 25: Interactions of Dialysis Teams With
Geriatricians
Nicole Stankus* and Kellie Campbell†
*Section of Nephrology, University of Chicago, Chicago, Illinois; and †Section of Geriatrics, University of Chicago,
Chicago, Illinois

In 2006, ⬎500,000 patients received dialysis for RECOMMENDED INTERACTIONS


end-stage kidney disease (ESKD), a 3.4% growth BETWEEN IDT AND GERIATRICIANS
since 2005. Of 110,854 incident patients, almost
50% were elderly (38,185 were over 65 and 15,657 The management of conditions associated with
were over 80 yr of age). Geriatric patients accounted ESKD may be at odds with competing medical con-
for 64% of all deaths (of 87,654 patients who died, ditions or patient preference. The heterogeneity of
35,950 were over 65 and 20,362 were over 80).1 elderly patients’ comorbid conditions and cognitive
Medicare’s spending on ESKD has reached and functional abilities makes individualization of
$22.7 billion, or 6.4% of total spending. To im- care essential. Referral of all elderly patients before
prove dialysis patient outcomes, the new Centers the initiation of dialysis therapies would be prefer-
for Medicare and Medicaid Services (CMS) Con- able but is not always possible. IDT needs to prior-
ditions of Coverage for chronic ESKD were is- itize geriatric consultations in certain situations.
sued in 2008, replacing the original ones from
1976. CMS focuses on the uniqueness of each pa-
tient and requires individualized multidisci- LONG-TERM RENAL REPLACEMENT
plinary care based on patient participation.2 THERAPY GOALS
There is considerable overlap in the populations
served by nephrologists and geriatricians. Close in- Selection of Dialysis Setting (In-Center
volvement of the geriatrician as a member of a mul- Versus at Home or Self-Care)
tidisciplinary care team would be the next obvious All dialysis patients should be offered home care
step in the integration of care for dialysis patients. options.
However, there are no published models that de- Self-care abilities of the elderly may be significantly
scribe how interactions between dialysis teams and limited and the geriatrician should be involved in as-
geriatricians might be facilitated. sessing not only the patient (vision, dexterity, mem-
ory) but also the extent and involvement of the home
support systems. One should be cautious when choos-
COMPREHENSIVE DIALYSIS PATIENT ing a home care modality in frail patients with limited
ASSESSMENT AND PLAN support.
Some elderly patients may initially do very well
Each dialysis facility has an interdisciplinary with home dialysis, but the loss of a spouse/friend/
team (IDT) that includes the following: (1) the child could be an indication to discuss with the ger-
patient (or designee); (2) primary dialysis nurse; iatrician whether a transition to an in-center dialy-
(3) nephrologist; (4) social worker; and (5) reg- sis would be a safer option.
istered dietitian. Assistance with dialysis treatments at home by
This team is responsible for providing each pa- the trained dialysis staff, especially in peritoneal di-
tient with an individualized and comprehensive as-
sessment of his/her needs and using it to develop a Correspondence: Nicole Stankus, Section of Nephrology, Uni-
treatment plan and expectations for care.3 Geriatri- versity of Chicago, 5841 South Maryland Avenue, MC 5100,
cians can be a valuable resource for the dialysis team Chicago, IL 60637. E-mail: [email protected]

(Table 1). Copyright 䊚 2009 by the American Society of Nephrology

American Society of Nephrology Geriatric Nephrology Curriculum 1


Table 1. Interdisciplinary care of older dialysis patient
Comprehensive Assessment Team Leader Comprehensive Care Plan
Physical health Nephrologist Yearly history and physical
Comorbidities, medications, immunizations Geriatrician With updated comorbidities and their medical
management
Mental and psycho-social health Social worker Interventions are individualized to:
Cognitive status, mental health Geriatrician Meet patient’s needs and achieve psychosocial and
rehabilitation goals
Ability to meet basic needs and follow Inform about advance directives
dialysis prescription
Coping with and adjusting to dialysis Perform annual survey of physical and mental
functioning (KDQOL-36)
Rehabilitation goals SW refers to:
Home environment Community mental health services
Legal issues (advance directives) Transportation providers
Financial capabilities and resources In-home support services
Access and eligibility to available federal,
state, or local resources
Activities of daily living (ADLs) Registered Nurse Encouragement and referrals for PT/OT/rehabilitation
to maintain or return to optimal functioning at
home and in the community
Social worker
Instrumental abilities of daily living (IADLs) Geriatrician
Nutrition Registered dietitian Achieve an effective nutrition:
Nutritional parameters to be addressed: Geriatrician Chewing and swallowing
Nutritional and hydration status Route of nutrition
Anthropometric data such as height, Use of supplements
weight changes, volume status
Information from the person that cooks Communication with the long-term care facilities to
and provides meals for the patient provide continuity of nutritional care for patients
Table 1 reflects current CMS Conditions of Coverage outlining the comprehensive assessment and plan of care for dialysis patients. Each team member has to
be intimately familiar with overall assessment and plan of care, but specific parts of assessment are under direct responsibility of the team leader (underlined).
Geriatricians integrate naturally into this framework (highlighted) in every specific component, because many of these tasks are routinely performed by
geriatricians as part of a comprehensive geriatric assessment. Goals of care for older patients change over time. Geriatricians are skilled in modifying treatment
plans not only to address the acuity, severity, or emergence of new medical problems, but also to align both long-term and short-term goals with patient
preferences and remaining life expectancy.

alysis, may become more available in the United States in the Integration of the comprehensive geriatric assessment in a
future, as it is practiced in other countries. It would be a very dialysis patient’s records is important for coordination of care.
viable option for patients with self-care limitations who wish to
continue with home dialysis. Decision About Withdrawal of Dialysis
Discontinuing dialysis and entering hospice care is usually a
Transplantation Referral joint decision between nephrologist and geriatrician. Initial
Age is no longer a single limiting factor for kidney transplantation, conversation with the family and patient is best held outside of
and reasons for a nonreferral must be documented by the IDT for the dialysis facility, with the focus on the overall prognosis and
every dialysis patient. In most elderly patients with significant co- palliative care options.
morbidities, nonreferral is quite obviously a correct choice. The nephrologist should reach out to geriatricians early on,
Even a highly functioning older dialysis patient would ben- if an elderly patient is developing increasing difficulties and
efit from a comprehensive geriatric assessment as a part of the discomfort during dialysis treatments, because it takes collab-
transplant evaluation. Functional abilities can dramatically de- orative effort to make a difficult decision to stop a life-sustain-
crease after initiation of dialysis and timely interventions by ing therapy. Institution of time-limited trials may be appropri-
geriatricians could help maintain physical and mental capabil- ate for some patients.
ities of waitlisted older patients.

Combined Medical Records ONGOING COORDINATION OF CARE


Sharing of the comprehensive assessment and care plans yearly
with patient’s geriatrician can improve care by improving BP Management
communication. Monthly reassessment of medication needs, fluid gains, and

2 American Society of Nephrology American Society of Nephrology


adjustments in target weight require not only knowledge of the INDIVIDUALIZED COMPREHENSIVE PLAN OF CARE
patient’s pre/intra/post-BP but also INTERdialytic BP.
Utilization of office BP and home-health through the geri- The plan is developed and implemented by the IDT and in-
atrician’s offices may be essential to ensure interdialytic BP cludes measurable and expected outcomes and estimated
stability and avoid causing dizziness and falls in the elderly timetables to achieve them. For all patients, and especially
patients. older ones, their needs, wishes, and goals have to be consid-
ered.
Monthly Laboratory Testing The written patient plan of care is based on the comprehen-
Printed results of dialysis laboratories should be provided to sive assessment and includes (1) problem(s) identified at as-
each patient monthly. Patients should be advised to bring sessment/reassessment; (2) measurable goals/outcomes; (3)
those results to their regular appointments with the geriatri- planned interventions for achieving goals; and (4) time tables
cian. This will help to ensure that the dialysis patient gets goal- and reassessment date(s).
driven individualized care. When a specified target is not met, the IDT could involve a
Encourage geriatricians to limit in-office blood draws and geriatrician to help define further actions and decide whether
rather submit requests for additional testing to the dialysis cen- the goal for the patient should differ from the standards and is
ters. This improves patient satisfaction, helps minimize unnec- adjusted appropriately for an older and/or frail patient.
essary blood draws, and preserves veins for access. A copy of the Plan of Care should be sent to the geriatrician
and, if specific changes in goals/timelines are suggested by the
Dialysis Access Type and Maintenance geriatrician, the IDT should reassess whether targets selected
Every patient, regardless of age, should be encouraged to ob- for an older patient are reasonable and appropriate for the
tain permanent dialysis access. The efficacy of the hemodialysis severity and acuity of the problems and also ensure that
(HD) patient’s vascular access and the peritoneal dialysis (PD) planned interventions are coordinated to avoid duplication of
patient’s peritoneal catheter correlates with the quality (ade- services.
quacy) of their dialysis treatments and is vital to their overall The IDT needs to enlist geriatricians’ help in educating pa-
health status. tients to understand their role in managing the prescribed diet
Nephrologists must communicate with and ensure support and medications. Without patient involvement in their own
of the geriatrician to assure the most expeditious permanent care, it is impossible to attain goals of care and achieve the best
dialysis access placement; this helps to overcome apprehension outcomes.
in older patients, who often are resistant to surgical proce-
dures.
CONCLUSIONS
Patients With Functional or Cognitive Decline
Changes in activities of daily living (ADLs)/instrumental activ- In summary, geriatricians are perfectly suited to help in the
ities of daily living (IADLs), KDQOL/SF-36 scores, patient care of older dialysis patients. They are well trained in compre-
falls, and cognitive decline should trigger an evaluation by the hensive assessments and are familiar with working in interdis-
geriatrician. Functional decline has impact on mortality but ciplinary teams. Dialysis IDTs need to take a leadership role
can also decrease patient independence and quality of life (QOL). to establish and maintain communications with the geriatri-
Geriatricians are able to offer interventions to help improve cians and use their expertise in providing planned and goal-
those conditions that might be amenable to intervention, such driven care. Caring for an older dialysis patient provides a
as improving IADLs. They are also able to recommend services unique opportunity to develop models of specialty–specialty
for patients and families that can improve the QOL for both interactions.
patients and families.
The presence of cognitive impairment can impact decision
making for the patient but can also interfere with the dialysis
TAKE HOME POINTS
procedure itself. Geriatricians are trained to help with behav-
ioral problems associated with impaired cognition that can • Geriatricians should be viewed as an integral part of the multidisci-
plinary care team for older dialysis patients and be included in devel-
negatively affect the ability to comply with the dialysis pre-
opment of comprehensive care plan and institution of appropriate
scription. interventions, thereby meeting CMS guidelines and avoiding duplica-
Geriatricians can help to identify depression in elderly pa- tion of care
tients on dialysis, distinguish this from dementia, and institute • Consultations with geriatricians are especially important when concerns
appropriate treatment plans. Depression is common in these arise about functional, cognitive, or psychosocial status of older pa-
tients that might interfere with dialysis treatments
patients. Medical management of depression allows patients to
• Geriatricians should be involved in major decisions, such as permanent
achieve their best quality of life, improves their compliance dialysis access placement, referral for transplantation, and discontinu-
with medication and dialysis prescription, and helps them ation of dialysis, because these are pivotal changes in the life of an
maintain their independence. elderly patient and impact their overall quality of life

American Society of Nephrology American Society of Nephrology 3


DISCLOSURES 2. Department of Health and Human Services: Conditions for Coverage
for End Stage Renal Disease Facilities; Final Rule. Department of
None.
Health and Human Services, Centers for Medicare and Medicaid
Programs, 42 CFR. Available online at http://cms.hhs.gov. Accessed
April 9, 2000
REFERENCES 3. Department of Health and Human Services, Centers for Medicare and
Medicaid Programs: Conditions for Coverage for End Stage Renal
1. US Renal Data System: 2008 Annual Data Report. Available online at Disease Facilities; Interpretive Guidance. Part 494. Available online at
www.usrds.org http://cms.hhs.gov. Accessed April 9, 2000

4 American Society of Nephrology American Society of Nephrology


REVIEW QUESTIONS: INTERACTIONS OF DIALYSIS a. Registered dietician
TEAMS WITH GERIATRICIANS b. Nephrologist
c. Primary care physician
1. Geriatricians would be least helpful in performing which of the
d. Social worker
following comprehensive assessments:
a. Ability to follow dialysis prescription
3. A geriatric consultation should be initiated for an older dialysis
b. Assessment of cognitive status
person when:
c. Discussion of advance directives
a. Functional decline occurs
d. Physical exam including review of medical comorbidities
b. The patient is considered for renal transplantation
and immunizations
c. Patient expectations or goals of care change
2. Current dialysis interdisciplinary teams do NOT include d. All of the above
which of the following:

American Society of Nephrology American Society of Nephrology 5


Chapter 26: Comprehensive Geriatric Assessment:
A Multidimensional Process Designed to Assess an
Elderly Person’s Functional Ability, Physical Health,
Cognitive and Mental Health, and Socio-Environmental
Situation
Steven R. Gambert
Department of Medicine, Johns Hopkins University School of Medicine, Baltimore, Maryland; and University of
Maryland School of Medicine, Baltimore, Maryland

Elderly persons have special needs and problems that dialysis are optimal choices for periodic geriatric assess-
ment.
often escape recognition and treatment until late in • A comprehensive geriatric assessment provides a systematic
their course. When considering the older individual, it approach to the collection of patient data; allows a patient’s
is important to be able to determine what findings health status to be evaluated for existing and also potential
constitute normal age-related changes compared with problems; emphasizes functional capacity; and does not aim
age-prevalent illness or an atypical and nonspecific to cure all problems but rather identifies and suggests ways
to maximize quality life years.
presentation of disease. Because there are often several • The primary goal is to help the patient regain lost
reasons for a given finding, one should not be too function and maintain as much independence as possi-
quick to conclude the “reason” for a specific issue; ble. It is important to note that physical and mental
there may also be more than one reason for a particu- illnesses affecting the elderly often interact and result in
lar presenting problem. a loss of functional ability much more than any one
problem in itself. The elderly are particularly vulnerable
A well-formulated problem list is the best way to as they have less “reserve capacity.”
unlock the possible causes of each finding whether
it is a key historical fact, physical finding, laboratory There is a great variation in the way that geriatric as-
test abnormality, or some diagnostic test result. A sessments are conducted from single physician evalu-
“change” in what has been a stable problem or an ations with referral to other skilled professionals as
acute functional decline should also alert the physi- necessary to full teams of professionals evaluating all
cian. Because many older persons have multiple in- patients. In the latter case, a team meeting is usually
teracting problems, a team of skilled professionals is held to discuss individual findings with the end prod-
often needed to provide a comprehensive evalua- uct being a unified team care plan. The team may in-
tion to address current and future needs. The neph- clude representatives from medicine, nursing, den-
rologist has the advantage of already having experi- tistry, dietary, social work, audiology/speech, physical
ence caring for persons with chronic illness and and occupational medicine, and psychology/psychia-
multiple interacting problems. Dialysis also in- try, among others. In the case of persons with renal
volves working with a team of skilled professionals, failure who may or may not already be receiving dial-
much like the model used in geriatric medicine. The ysis with a carefully designed care plan, care plans de-
following is a summary of key issues relating to veloped after the Geriatric Assessment must be care-
comprehensive geriatric assessment and its possible fully coordinated to avoid conflicting plans, messages,
benefits.
• The periodic comprehensive geriatric assessment should
be part of a coordinated care plan with the patient’s other
health care providers. Correspondence: Steven R. Gambert, Department of Medicine,
• Individuals of significantly advanced age, those suffer- Johns Hopkins University School of Medicine and University of
Maryland School of Medicine, Baltimore, MD 21201. E-mail:
ing from multiple and interacting medical problems,
[email protected]
those on multiple medications, and individuals meeting
criteria for “frailty” benefit most; elderly persons on Copyright 䊚 2009 by the American Society of Nephrology

American Society of Nephrology Geriatric Nephrology Curriculum 1


or treatments. Proper and thorough communication is key to a ASSESSING FRAILTY
successful outcome.
In recent years, there has been an increasing awareness that a
subpopulation of elderly individuals are frail.4 Frailty is a clin-
GOALS OF GERIATRIC ASSESSMENT ical syndrome of weight loss, fatigue, and weakness and is con-
sidered to be a prognostic factor for poor outcomes.
The goals of geriatric assessment are as follows: (1) provide a long- Frailty criteria include weight loss, weakness, exhaustion,
term solution for “hard to manage” patients with multiple physi- slowed walking speed, and low activity. The definition of frailty
cian and/or emergency room visits and hospital admissions with is met if three of these five are present.
poor follow-up; (2) focus on preventive medicine rather than Studies have shown a relationship between frailty and in-
acute medicine; (3) focus on improving/maintaining functional flammatory markers, although these remain experimental at
ability and not necessarily a “cure”; (4) aid in the diagnosis of this time.5 Frailty has been associated with an increased risk for
health-related problems; (5) develop plans for treatment and fol- falls, worsening mobility, worsening ADL disability, hospital-
low-up care; (6) establish plans for coordination of care; (7) de- ization, increased recovery time from surgery and illness, and
termine the need for and site of long-term care as appropriate; (8) death.
determine optimal use of healthcare resources; and (9) prevent
readmission to the hospital.
The following are considered necessary components of a GERIATRIC ASSESSMENT AND THE PATIENT WITH
comprehensive geriatric assessment: (1) diagnoses and prob- CHRONIC KIDNEY DISEASE AND/OR DIALYSIS
lems (past and current) based on history, physical examina-
tion, and appropriate diagnostic testing; assessment to include Chronic kidney disease (CKD) can accelerate the otherwise
oral health, vision, hearing, and gait/posture1; (2) detailed re- normal aging process and result in a decline in functional sta-
view of all prescription and over-the-counter medications in- tus and increased dependency on others for components of
cluding supplements/herbs with attention to side effect profile care. In most cases, patients with CKD have multiple medical
and potential drug– drug/drug–food interactions; (3) nutri- problems in addition to CKD, particularly diabetes mellitus
tional assessment including dietary recall/diary, anthropomet- and hypertension, each with their own set of medical compli-
ric measurements, and laboratory testing as appropriate; (4) cations and requirements. Polypharmacy is the rule; a coordi-
urinary continence/incontinence history; (5) assessment of nated approach is essential to avoid fragmented care and con-
bowel function; (6) assessment of cognitive ability and capacity flicting treatments. Geriatric assessment programs may help to
for informed decision making2; (7) mental/emotional health; identify issues and play a role in optimizing healthcare for this
(8) functional abilities (activities and instrumental activities of at-risk population.
daily living [ADLs and IADLs, respectively]) as independent,
some help required, or dependent3; (9) assessment regarding
ability to adhere to prescribed care plan; (10) spiritual needs/
CONCLUSIONS
beliefs; (11) existing community support structure; (12) cur-
rent and anticipated living environment; (13) legal issues in-
When considering the older person, it is important to consider
cluding living wills, durable power of attorney, and end-of-life
what observations are a result of normal age-related changes com-
issues; and (14) fall risk assessment.
pared with age-prevalent illness; diseases often present with non-
ADLs are further subdivided into physical or basic ADLs and
specific and/or atypical findings as well. A comprehensive geriat-
IADLs. Basic ADLs include mobility, bathing, dressing, toileting,
ric assessment can not only help identify issues that need to be
transferring, continence, and feeding. IADLs include the ability to
treated but also help to recognize potential problems that deserve
use the telephone, shop, prepare food, housekeeping, laundry,
early consideration and preventive strategies. Older persons with
transportation, taking medication, and managing money.
CKD are at “double jeopardy” and clearly are ideal candidates for
There have been many studies evaluating the benefits of
periodic geriatric assessment.
geriatric assessment programs both in the ambulatory and in-
patient setting. Clearly the financial cost-benefit impact varies
with the type of program established, the personnel involved,
and the ability to track outcomes. TAKE HOME POINTS
In general, individuals who participate in a geriatric assess- • A periodic geriatric assessment is of value, particularly to “frail” older
ment program benefit from (1) greater diagnostic accuracy; (2) persons and those with multiple medical problems and/or have had
improved functional ability; (3) improved cognitive ability and frequent hospitalizations/visits to the emergency room
• Benefits are many to both the patient and physician: a thorough assess-
emotional state; (4) reduced morbidity; (5) reduced adverse
ment not only can identify existing issues to be dealt with but also helps to
drug events; (6) reduced mortality; (7) decreased use of nurs- anticipate future problems that would benefit from proactive steps
ing home and readmission to hospitals; and (8) greater satis- • Assessment should encompass all aspects of the patient’s care and
faction with care. should not be limited to medical issues

2 Geriatric Nephrology Curriculum American Society of Nephrology


• A team approach is the best way to make sure that all issues are 2. Folstein MF, Folstein SE, McHugh PR: “Mini-Mental State”. A practical
identified and an appropriate plan developed method for grading the cognitive state of patients for the clinician.
• Older persons who meet criteria for “frailty” are at particularly high risk J Psychiat Res 12: 189 –198, 1975
of morbidity and mortality and have more significant consequences 3. Lawton MP, Brody EM: Assessment of older people: self-maintaining
from medical interventions including surgery and instrumental activities of daily living. Gerontologist 9: 179 –186,
1969
4. Fried LP, Walston J: Approach to the frail elderly patient. In: Kelly’s
Textbook of Internal Medicine, 4th Ed., edited by Humes HD, Phila-
REFERENCES
delphia, Lippincott Williams and Wilkins, 2000, pp 3069 –3073
5. Leng S, Chaves P, Koenig K, Walston J: Serum IL-6 and hemoglobin as
1. Tinetti, ME: Preventing falls in elderly persons. N Engl J Med 348: physiologic correlates in the geriatric syndrome of frailty: a pilot study.
42– 49, 2003 J Am Geriatr Soc 50: 1268 –1271, 2002

American Society of Nephrology Geriatric Nephrology Curriculum 3


REVIEW QUESTIONS: COMPREHENSIVE GERIATRIC c. Fall risk assessment
ASSESSMENT: A MULTIDIMENSIONAL PROCESS d. Evaluation of a cardiac murmur
DESIGNED TO ASSESS AN ELDERLY PERSONⴕS
3. All of the following are criteria for a diagnosis of frailty except:
FUNCTIONAL ABILITY, PHYSICAL HEALTH,
a. Weight loss
COGNITIVE AND MENTAL HEALTH, AND SOCIO-
b. Weakness
ENVIRONMENTAL SITUATION
c. Anemia
d. Slow walking speed
1. All of the following are goals of Geriatric Assessment except: 4. Which of the following is considered to be an ADL:
a. Develop plans for treatment and follow-up care a. Use of a telephone
b. Establish plans for coordination of care b. Ability to prepare food
c. Determine optimal use of health care resources c. Toileting
d. Evaluate family history for causes of chronic kidney dis- d. Ability to manage money
ease
5. Which of the following is considered to be an IADL:
2. All of the following are considered to be necessary Compo- a. Mobility
nents of a Comprehensive Geriatric Assessment except: b. Bathing
a. Urinary continence/incontinence history c. Transferring
b. Existing community support structure d. Ability to take one’s medications

4 Geriatric Nephrology Curriculum American Society of Nephrology


Chapter 27: Rehabilitation Services for Elderly Dialysis
Patients
Sarbjit Vanita Jassal
Division of Nephrology, University Health Network, Toronto, Canada

Based on recent data, Canadians starting dialysis rehabilitation involves a process by which patients
between the ages of 75 and 79 yr will have an average both learn new ways to restore function but also meth-
life expectancy of 3.2 yr.1 In the United States, pa- ods which help them adapt to the new disability.
tients 65 to 79 yr of age starting dialysis have a re- Geriatric rehabilitation depends highly on a
ported life expectancy of 25 mo.2 During this time, model of interdisciplinary care. In this model, dif-
patients may experience transient or permanent ferent team members have both overlapping and
loss of personal independence, which, in turn, has a complementary skills. Unlike acute care units
negative impact on their quality of life and financial where the physician often heads the team, rehabili-
situation and a significant impact on health care tation teams do not depend on leadership from one
utilization.3 In general, older patients have complex particular discipline. Rather, treatment decisions
medical histories and a higher incidence of chronic are led by the team member most involved with the
ailments such as arthritic pain, vision loss, or fa- patient. Team members include physicians, nurses,
tigue. Often patients adapt by walking more slowly social workers, occupational therapists, physiother-
or taking more rests and developing fixed routines apists, speech therapists, psychologists, and phar-
with little variability or limiting activities. Over macists. Nurses play a key role because they spend
time, these symptoms and adaptations have a cu- the most time with the patient. Through their
mulative effect on functional status, possibly lead- daily interactions with patients, nurses empower
ing to dependency. In the renal literature, prevalent patients to assume self-care and responsibility
patients on hemodialysis seem to have high levels of and evaluate their psychosocial needs. Often
functional loss.4 Preliminary studies show that this nurses help reduce muscle deconditioning by en-
is exacerbated by acute hospitalization.5 The impact couraging patients to perform self-care activities
of a lower functional status may be reduced by of- outside of their formal therapy sessions. After
fering rehabilitation to dialysis patients. In the non- multiple team members assess the impact of dis-
dialysis literature such programs are common and ease on functionality from a variety of perspec-
seem to limit the impact of functional disability on tives, they identify, through discussion, which of
patients, their families, and the healthcare system.6,7 the disciplines is best suited to developing solu-
In this chapter, I will review the benefits and con- tions before working in a collaborative manner.
cepts of geriatric rehabilitation, the role of the As an example, one patient with difficulty walk-
nephrologists, and highlight some common com- ing indoors may work with a physiotherapist to
plications. increase muscle strength, whereas another may
benefit from occupational therapy sessions to
learn to overcome visual limitations.
WHAT IS GERIATRIC REHABILITATION?

Rehabilitation can be defined as a process by which


form and function is restored after injury or illness,
such that life can be lived to the fullest capacity Correspondence: S. V. Jassal, Associate Professor, University of
compatible with the degree of abilities and disabil- Toronto, Staff Physician, University Health Network, Director, Ge-
riatric Dialysis Rehabilitation Program, Toronto Rehab Institute,
ities.8 This definition recognizes two important char- 8NU-857, 200 Elizabeth Street, Toronto M5G 2C4, Canada.
acteristics of rehabilitation in the older population— Phone: 416-340-3196; Fax: 416-340-4999;
first that restoration of function is most likely to occur E-mail: [email protected]

in those with a recent loss of function and, second, that Copyright 䊚 2009 by the American Society of Nephrology

American Society of Nephrology Geriatric Nephrology Curriculum 1


Table 1. Summary of studies evaluating inpatient rehabilitation in dialysis patients
Percent Percent
Comorbidity Mean Average
Publication N Patients With Discharged Comments
Details Age (yr) LOS (d)
Diabetes Home
Li (2008)24 164 51 CCI ⫽ 7.8 ⫾ 2.5 74.5 48.5 68 Majority had a high burden of comorbid diseases.
24% patients required transfer to an acute care
institution, of whom 40% did not return to rehab
Forrest (2005)19 40 Not reported Not reported 62.3 12.1 80.0 Prior medically complicated conditions in 8 patients
(20%)
Forrest (2004)18 34 Not reported Not reported 68.7 16.0 72.2 Mostly admitted post-procedure. Prior acute
hospitalization in 27%
Frank (2002)20 5 Not reported Not reported 76.4 — 20.0 Used Berg Balance Scores and mobility for
functional assessment
Garrison (1997)21 3 100 100 50.0 18.0–34.0 100.0 Small sample size, postamputation only
Czyrny (1994)17 19 Not Reported Not Reported 59.0 27.0 79.0 Postamputation
Cowen (1995)16 28 Not reported Not reported 61.5 17.5 89.0 Reported converted functional impairment measure
scores
Greenspun (1986)22 4 100 Not reported 55.2 44.2 n/a Postamputation
LOS, length of stay; CCI, Charlson Comorbidity Index.

PUBLISHED BENEFITS OF REHABILITATION IN and high levels of functional dependence. In our unit, dialysis
DIALYSIS CARE is offered on site on a daily dialysis schedule. Staffing levels
have been increased in both the dialysis suite and on the ward
Numerous programs have reported using rehabilitation in to accommodate the higher burden of comorbidity and higher
their dialysis units. Most develop exercise programs designed dependency levels.24
to build muscle strength through exercise regimens during di-
alysis or within the home.9 –14 Such programs have been largely
successful but are often confounded by high drop-out rates
ISSUES UNIQUE TO REHABILITATION OF ELDERLY
and the high costs of providing staff to supervise the exercises.
DIALYSIS PATIENTS
Cardiac rehabilitation is recommended for dialysis patients
who have recently survived myocardial infarction, had bypass
Comorbidity Burden
surgery, or those with chronic stable angina. Dialysis patients
Elderly dialysis patients have a high incidence of comorbidity
have been shown to benefit from actively participating in car-
with recent studies in prevalent hemodialysis patients showing
diac rehab, with outcome studies showing a 35% reduced risk
the mean number of medical conditions to be around 10.5 ⫾
for cardiac mortality.15 However, cardiac rehab seems to be
3.5.25 The high number of comorbid illnesses place a heavy
underused in dialysis patients, with only 10% of dialysis pa-
medical burden on the rehabilitation unit staff and the phar-
tients, compared with 23% of the general population, under-
macy. Elderly dialysis patients have a high rate of transfer out
going cardiac rehabilitation after coronary artery bypass graft
to acute care for new or recurrent acute illnesses. Of those who
(CABG) surgery.15 Reasons behind this apparent paradox have
not been clearly identified.
In the older individual, building muscle can be challenging, Number of patients

because patients do not have the strength or capacity to per- 120

form strenuous or repetitive exercises. Sensory changes, such 100


as in hearing or vision, or decreases in cognitive function also 80
limit the ability to learn new exercises. Geriatric renal rehabil-
60
itation has not been widely formalized and, in many cases, is
available only on a case-by-case basis. Published results of out- 40

comes with dialysis rehabilitation are available but often lim- 20


ited by small study size and the inclusion of young patients 0
(⬍70 yr of age) with few comorbid conditions (Table 1).16 –22 Home Assisted Care Long-term care Other

Success rates, as measured by the proportion of patients re- Figure 1. Graph showing discharge disposition of patients ad-
turning home, vary from 20 to 100%. In the largest and most mitted to the Toronto Dialysis Geriatric Rehabilitation Program.
recent report, approximately 70% of patients23,24 met their Patients who did not return to rehabilitation after an acute inter-
personalized goals and returned home (Figure 1). At the time current illness or who transferred to palliative care are shown as
of admission, patients had a significant burden of comorbidity having been discharged to Other.

2 American Society of Nephrology American Society of Nephrology


are transferred, almost 40% are too unwell to return to reha- and if so, how often and with what tools. Clearly clinical events
bilitation.24 such as hospitalization, falls, or a change in social status (e.g.,
moving home or the death of a spouse or carer) should prompt
Dialysis Scheduling a functional reassessment by either the primary physician or
Short daily dialysis is recommended where possible. It is well members of the allied health team.
tolerated and may lead to improved nutrition and better par- Nephrologists should also work closely with the rehabilita-
ticipation in therapy sessions. Patients report less fatigue and tion team to evaluate and, if necessary, reset health targets for
fewer symptoms associated with rapid fluid shifts. Scheduled older dialysis patients. Although nephrologists routinely adjust
dialysis and rehabilitation therapy sessions lead to improved ultrafiltration/target weight and blood sugar targets, some pa-
outcomes and shorter length of hospital stay.19 tients may require temporary relaxation in these goals partic-
ularly during periods of functional loss. Relaxation of dietary
Enforced Immobility During Dialysis restrictions may allow improved nutrition and allow patients
Patients may benefit from occupational therapy and physio- to meet their calorie requirements during a period of repair
therapy assessment during the dialysis session. Customized and recovery. Minor adjustments in volume status can have a
seating aids may improve seating balance while simple physio- significant effect on fatigue. In our experience, debilitated in-
therapy exercises may reduce the impact of remaining rela- dividuals often report or manifest symptoms during their
tively immobile for longer sessions. physiotherapy or occupational therapy sessions. Adjustment
of target weight based on these observations or symptoms can
impact function, with small changes (e.g., an increase or de-
THE SICK ROLE: THE IMPORTANCE OF crease of 200 ml of ultrafiltration) being effective in some cases.
ENCOURAGEMENT (We have maximized this through the use of short daily dialysis
regimens.) Some flexibility around blood sugar control may
Many healthcare workers perceive dialysis patients as being also be helpful for individuals who are prone to labile diabetes.
heavily dependent and unable to participate in exercise. In con- Although tight blood sugar control is always a long-term ob-
trast, patients are interested in maintaining their functional inde- jective, we have found it necessary to relax blood sugar goals
pendence and taking part in exercise. In a study to identify the during the initial rehabilitation period. By having a tolerance
barriers to exercise, one important factor identified was that nei- for a higher mean blood sugar, the patient may feel more able
ther nurses nor doctors encourage elderly patients to be active.26 to focus on other aspects of care. Success in these other aspects
Studies to see whether changing staff attitudes can alter patient of personal functioning, empowers them to then address self-
perceptions or activity levels are underway. care issues around blood sugar management. Sadly, the need to
fulfill benchmark targets may limit the longer-term use of in-
dividualized care plans in patients with borderline functional
ROLE OF THE NEPHROLOGIST IN DIALYSIS- independence.
RELATED GERIATRIC REHABILITATION Nephrologists play a key role in helping to rationalize med-
ications. Many dialysis patients experience polypharmacy. In-
The nephrologist role is to work with the team and to take a patient rehabilitation care is an ideal setting for discontinua-
lead position on identification and referral of patients who may tion of medications such as gastric acid suppressants, sedatives,
benefit from rehabilitation; customization of the individual and laxatives, the aim being to reduce unnecessary drugs and
medical goals and targets (goals should be adapted to allow the minimize drug interactions.
patient to best achieve personal independence and function Pain management and detection and treatment of depres-
and yet maintain long-term health); and reduction of polyp- sion are two important aspects of rehabilitation care. Many
harmacy and rationalization of medication. patients undergoing rehabilitation report chronic pain. Ar-
One of the more difficult roles a nephrologist must play is thritic knees or hands can limit the use of aids and therefore
the identification of patients who would benefit from rehabil- impact both functionality and recovery. The presence of
itation. Few nephrologists are formally trained in rehabilita- chronic uncontrolled pain may lead to depression and of
tion medicine, and there is little literature to advise on screen- course vice versa, meaning that pain management strategies
ing or referral protocols. Nevertheless, it is the nephrologist should include antidepressants if appropriate.
who follows the chronic predialysis and dialysis patient most
closely and who is involved in their long-term care planning.
The nephrologist is therefore best placed to identify functional IDENTIFYING AND MANAGING COMMON
decline and question if rehabilitation would improve function- PROBLEMS IN DIALYSIS REHABILITATION
ality. A practical first step is to perform a full geriatric assess-
ment after major events such as dialysis initiation and at set Pain
intervals thereafter. Unfortunately, this field is relatively new Pain management is a major issue in ESRD. The topic is large
and it remains unclear whether all patients should be screened, and complex because of the altered metabolism of many drugs,

American Society of Nephrology American Society of Nephrology 3


and the reader is encouraged to read further on the topic.27–29 alysis initiation. Unfortunately, symptoms, such as fatigue or
Key summary principles are discussed below. poor sleep and loss of appetite, can be attributed both to dial-
Appropriate drugs include acetaminophen ⫾ codeine and ysis dependency and to depression, and therefore, depression
opiates. Nonsteroidal anti-inflammatory drugs (NSAIDs) can is best detected by having a high index of suspicion. Patients
be used in anuric patients with arthritic symptoms. Ideally respond well to antidepressants. In our unit, the preference is
these should only be used in short courses at low dose because for the newer agents such as citalopram and sertraline in par-
of the lower drug clearance and high risk of gastrointestinal ticular because they have a lower incidence of drug– drug in-
side effects. teractions. As mentioned previously, mirtazapine is particu-
Preferred opiates include hydromorphone, fentanyl, and larly useful in patients with poor appetite and/or sleep
methadone. Morphine,meperidine, and detroporoxyphene problems. Venlafaxine is our preferred choice for patients with
should be avoided because of accumulation of the drug. Cau- significant anxiety symptoms.
tion must be exercised with transdermal administration of fen-
tanyl because it seems to have a variable absorption depending Confusional States
on the location of the patch. In addition, absorption may vary Cognitive impairment is common in dialysis patients30 and
depending on the temperature of the skin in that area. (Per- may significantly impact the success rates with rehabilitation.
sonal note: We avoid the use of fentanyl in our unit.) Associated delirium or agitation is not uncommon and may
If prescribing opiates, aim to use regular doses of long-act- improve after medication rationalization or treatment of any
ing agents at set times, e.g., twice a day; doses should be sup- intercurrent illness. Careful assessment of sensory functions,
plemented with additional breakthrough medications, partic- such as hearing or vision, may be beneficial. Patients with re-
ularly initially because the use of breakthrough medications duced vision or hearing may not be aware of “normal” external
helps determine how much uptitration is needed. Medication stimuli and what may be a simple startle reaction may be mis-
doses can be reassessed every 3 to 5 d. The regular dose of a interpreted as agitation. Agitated patients may respond well to
long-acting agent should be increased to be equal to the total routine. If possible, dialysis scheduling should facilitate dialysis
average dose of opiate used since the last titration. Therefore, if in the same station at the same time each day for vulnerable
the patient is taking 3 mg hydromorphone long acting twice patients.
daily and is, on average, taking an additional 1-mg break-
through dose five times a day, one would increase the total
long-acting hydromorphone doses to 6 mg twice a day (for a
CONCLUSIONS
total of 12 mg opiate/24 h) in the hope that pain will be suffi-
ciently well controlled to not require breakthrough drugs.
Widespread development of programs offering rehabilitation
Down titration is best done by a gradual reduction in the opiate
to dialysis patients is likely increasingly worthwhile as the av-
doses administered twice daily.
erage age of the dialysis patients increases. Programs seem ef-
The use of adjuvant therapies such as heat packs, transder-
fective in minimizing the disability associated with aging and
mal electrical nerve stimulation therapy, acupuncture, and an-
chronic disease. Nephrologists play an important role in iden-
tidepressants nortriptyline (in preference to amitriptyline) and
tification of patients; setting appropriate medical goals; and
gabapentin is encouraged.
managing common problems such as pain control. Specific
training in care of the elderly patient may be of benefit to neph-
Sleep Disorders
rologists.
Sleep disorders and chronic fatigue are common symptoms in
dialysis patients. Sleep hygiene programs include regular sleep
scheduling, keeping the patient out of bed and the bedroom
until bedtime, a snack before bedtime, and instruction on TAKE HOME POINTS
mental imagery or deep breathing relaxation techniques. • There is a high burden of dependency and disability in the older dialysis
Sleeping during dialysis, although common, should be dis- population
couraged. A patient with a poor sleep pattern may be suffering • Recent onset dependence and disability may be reversed through
from concomitant depression, and assessment is advised. If targeted rehabilitation programs
• Rehabilitation outcomes are improved if nephrologists work in conjunc-
associated with depression, the use of antidepressants with a
tion with rehabilitation specialist teams; important areas for collabora-
mildly sedative effect may be beneficial. Mirtazapine is often tion include identification of appropriate candidates, medical goal ad-
used because it causes relatively short-term drowsiness (6 to justment, and pain and medication management
8 h) and also acts as an appetite stimulant. • Scheduled dialysis sessions are associated with better rehabilitation
outcomes; short daily dialysis sessions are best tolerated (opinion)
Depression
Depression is common in both dialysis patients and those un-
dergoing rehabilitation. Frail elderly dialysis patients are there- DISCLOSURES
fore at particularly high risk, especially around the time of di- None.

4 American Society of Nephrology American Society of Nephrology


REFERENCES 15. Kutner NG, Zhang R, Huang Y, Herzog CA: Cardiac rehabilitation and
survival of dialysis patients after coronary bypass. J Am Soc Nephrol
17: 1175–1180, 2006
*Key References 16. Cowen TD, Huang CT, Lebow J, DeVivo MJ, Hawkins LN: Functional
1. Jassal SV, Trpeski L, Zhu N, Fenton SSA, Hemmelgarn BR: Changes in outcomes after inpatient rehabilitation of patients with end-stage
survival over the years 1990 –1999 for elderly patients initiating dialy- renal disease. Arch Phys Med Rehabil 76: 355–359, 1995
sis. CMAJ 177: 1033–1038, 2007 17. Czyrny JJ, Merrill A: Rehabilitation of amputees with end-stage renal
2. Kurella M, Covinsky KE, Collins AJ, Chertow GM: Octogenarians and disease. Functional outcome and cost. Am J Phys Med Rehabil 73:
nonagenarians starting dialysis in the United States. Ann Intern Med 353–357, 1994
146: 177–183, 2007 18. Forrest GP: Inpatient rehabilitation of patients requiring hemodialysis.
3. USRDS Data report. http://www.usrds.org/adr_2007.htm 2007 Arch Phys Med Rehabil 85: 51–53, 2004
4. Cook WL, Jassal SV: Functional dependencies among the elderly on 19. Forrest G, Nagao M, Iqbal A, Kakar R: Inpatient rehabilitation of
hemodialysis. Kidney Int 73: 1289 –1295, 2008* patients requiring hemodialysis: improving efficiency of care. Arch
5. Lo D, Chiu E, Jassal SV: A prospective pilot study to measure changes Phys Med Rehabil 86: 1949 –1952, 2005*
in functional status associated with hospitalization in elderly dialysis- 20. Frank C, Morton AR: Rehabilitation of geriatric patients on hemodial-
dependent patients. Am J Kidney Dis 52: 956 –961, 2008 ysis; a case series. Geriatr Today 5: 136 –139, 2002
6. Forster A, Young J, Lambley R, Langhorne P: Medical day hospital 21. Garrison SJ, Merritt BS: Functional outcome of quadruple amputees with
care for the elderly versus alternative forms of care. Cochrane Data- end-stage renal disease. Am J Phys Med Rehabil 76: 226 –230, 1997
base Syst Rev CD: 001730, 2008 22. Greenspun B, Harmon RL: Rehabilitation of patients with end-stage
7. Clark GS, Siebens HC: Rehabilitation of the geriatric patient. In: Re- renal failure after lower extremity amputation. Arch Phys Med Rehabil
habilitation Medicine, edited by DeLisa JA, Philadelphia, Lippincott, 67: 336 –338, 1986
1993, pp 642– 665 23. Jassal SV, Chiu E, Li M: Geriatric hemodialysis rehabilitation care. Adv
8. Eisenberg MG: Dictionary of Rehabilitation. New York, Springer, 1995 Chronic Kidney Dis 15: 115–122, 2008
9. Johansen KL, Painter PL, Sakkas GK, Gordon P, Doyle J, Shubert T: 24. Li M, Porter E, Lam R, Jassal SV: Quality improvement through the
Effects of resistance exercise training and nandrolone decanoate on introduction of interdisciplinary geriatric hemodialysis rehabilitation
body composition and muscle function among patients who receive care. Am J Kidney Dis 50: 90 –97, 2007*
hemodialysis: a randomized, controlled trial. J Am Soc Nephrol 17: 25. Cook WL, Jassal SV: Prevalence of falls among seniors maintained on
2307–2314, 2006 hemodialysis. Int Urol Nephrol 37: 649 – 652, 2005
10. Painter P, Johansen KL: Improving physical functioning: time to be a 26. Kontos PC, Miller KL, Brooks D, Jassal SV, Spanjevic L, Devins GM, De
part of routine care. Am J Kidney Dis 48: 167–170, 2006 Souza MJ, Heck C, Laprade J, Naglie G: Factors influencing exercise
11. Painter P, Carlson L, Carey S, Paul SM, Myll J: Low-functioning hemo- participation by older adults requiring chronic hemodialysis: a quali-
dialysis patients improve with exercise training. Am J Kidney Dis 36: tative study. Int Urol Nephrol 39: 1303–1311, 2007
600 – 608, 2000 27. Arnold RM, Verrico P, Davison SN: Opioid use in renal failure #161. J
12. Johansen KL, Shubert T, Doyle J, Soher B, Sakkas GK, Kent-Braun JA: Palliat Med 10: 1403–1404, 2007
Muscle atrophy in patients receiving hemodialysis: effects on muscle 28. Davison SN: Pain in hemodialysis patients: prevalence, cause, severity,
strength, muscle quality, and physical function. Kidney Int 63: 291– and management. Am J Kidney Dis 42: 1239 –1247, 2003*
297, 2003 29. Davison SN: Chronic pain in end-stage renal disease. Adv Chronic
13. Painter P: The importance of exercise training in rehabilitation of Kidney Dis 12: 326 –334, 2005
patients with end-stage renal disease. Am J Kidney Dis 24: S9, 1994 30. Murray AM, Tupper DE, Knopman DS, Gilbertson DT, Pederson SL, Li S,
14. Painter P: Why exercise can make a difference. Nephrol News Issues Smith GE, Hochhalter AK, Collins AJ, Kane RL: Cognitive impairment in
20: 52, 2006 hemodialysis patients is common. Neurology 67: 216 –223, 2006

American Society of Nephrology American Society of Nephrology 5


REVIEW QUESTIONS: REHABILITATION SERVICES over the past few months. The rehabilitation team review the
FOR ELDERLY DIALYSIS PATIENTS literature for clinical evidence about the effectiveness of inpa-
1. A 72-yr-old lady presents with acute chest pain to the emer- tient geriatric rehabilitation in elderly, dialysis patients. Which
gency room. She has been living independently in her own of the following statements is true:
home for some years without difficulty. She is noted to have a. Strong, grade A (randomized controlled trial) evidence
chronic kidney disease on initial bloodwork. Over the follow- that rehabilitation is effective in elderly dialysis patients
ing few days, her cardiac condition deteriorates. Because of b. Some Grade B evidence (from observational studies) that
marked fluid overload and her background CKD, she needs to rehabilitation is effective in elderly dialysis patients
start dialysis emergently. One month after initiating dialysis, c. No evidence in either direction (either supporting effec-
she is noted by the dialysis staff to be unsteady on her feet when tiveness or no effectiveness) in elderly dialysis patients
coming to dialysis. Since discharge, she is known to have had d. Strong evidence (randomized controlled trial) that reha-
multiple falls at home and is currently complaining of pain in bilitation is not effective in elderly dialysis patients
her left shoulder after a fall. She has no fracture but has signif- 3. A 68-yr-old man is admitted to an acute medical ward with
icant bruising and pain and has limited movements of the arm. symptoms consistent with pneumonia. He is seen by the
Her BP is 160/85 mmHg predialysis and 140/78 mmHg post- physio as part of the discharge planning process and is noted to
dialysis. Her medications include a renal vitamin, aspirin 81 be having difficulty walking and transferring because of mus-
mg OD, ramipril 10 mg OD, metoprolol 50 mg po twice daily, cle weakness. His bone mineralization profile shows he has
atorvastatin 20 mg qHS, lorazepam 10 mg qHS, quinine sulfate mild secondary hyperparathyroidism with elevated PTH lev-
300 mg OD on dialysis days for cramps, allopurinol 100 mg els. He starts undergoing physiotherapy and rehabilitation
OD, hydroxyzine 25 mg three times daily for itch, omeprazole with the physio on an adhoc basis. He is a dialysis patient and
20 mg OD, zopiclone 7.5 mg qHS, and acetaminophen 1000 receives dialysis in the outpatient dialysis suite on Monday,
mg QID prn for pain. Which of the following statements is Wednesday, and Friday mornings where possible. He is a
true: pleasant man who is cooperative and readily amenable to
a. Her falls are likely only happening on postdialysis days and changing his dialysis time to later in the afternoon to suit the
therefore related to hypotension; the most appropriate ac- staff. There is concern that he does not appear to be making
tion is adjustment of target weight any progress in his walking. Which of the following statements
b. The most appropriate first step is to manage her pain by is true:
introduction of an opiate on a regular schedule a. Rehabilitation provided on an acute medical ward is inef-
c. The most appropriate first step is to prescribe a walker to fective
help with her unsteadiness b. Changing patients to a fixed dialysis schedule may reduce
d. Her falls are likely multifactorial and, taken together with the length of stay
her unsteadiness and polypharmacy, she may benefit from c. Dialysis patients are likely to require twice as long for re-
referral to a rehabilitation specialist for evaluation, and habilitation as nondialysis patients
treatment. d. The muscle weakness is unlikely to resolve with exercises
2. An 85-yr-old hemodialysis patient is referred for rehabilita- as it is related to a myopathy associated with secondary
tion after the family notice a decline in his functional status hyperparathyroidism

6 American Society of Nephrology American Society of Nephrology


Chapter 28: Integrated Care of the Elderly With ESKD
Jocelyn Wiggins
University of Michigan, Ann Arbor, Michigan

Nephrologists worldwide are increasingly taking illness. Nephrologists were also providing ongoing
care of older patients. Mean age at the start of renal treatment for comorbid chronic illnesses such as
replacement therapy is 62.3 yr for men and 63.4 yr diabetes and heart disease. In 1993, they went on to
for women. Peak incident counts of treated end- confirm similar statistics in their chronic peritoneal
stage kidney disease (ESKD) occur in the 70 to 79 dialysis patients.10 This would suggest that the
age group at ⬎15,000 patients per year. Peak inci- nephrologist needs to be prepared to take on the full
dent rates of treated ESKD occur in the 70- to complexity of care for their older patients, particu-
79-yr-old age group at 1543 per million popula- larly their chronic dialysis population. In older pa-
tion.1 Patients in this older age group are likely to tients, this would include health maintenance
have multiple comorbidities. The average 75 yr old screening and immunizations. Although malignan-
suffers from 3.5 chronic diseases.2 Chronic kidney cies are more common in both the dialysis popula-
disease (CKD) in the elderly rarely occurs in isola- tion and in the posttransplantation population
tion from other chronic medical conditions and is than in the general population, life expectancy, age,
often a marker for those conditions. Many symp- and cost effectiveness need to be considered by the
toms in older patients are caused by multiple defi- nephrologists before ordering screening tests.
cits and not by a single disease. These diseases and What can the geriatrician offer this very sick
their treatments are likely to interact and compli- population? Geriatricians are trained to perform
cate one another. Murray3 has reported that up to comprehensive geriatric assessment. This involves
70% of dialysis patients 55 yr of age and older have evaluation of all medical problems, but also covers
chronic cognitive impairment of a level severe several other domains— cognition, affect, func-
enough to impact on their compliance and ability to tional level, sensory, socio-economic needs, and en-
make informed decisions.4,5 Prevalence of depres- vironmental needs. Most geriatricians practice in a
sion is reported to be as high as 45% in the older multidisciplinary group that includes social work-
dialysis population.6 – 8 Metabolic bone disease is ers, pharmacists, physical therapists, and nurse
complicated by age-related osteoporosis. The car- practitioners.
diovascular consequences of CKD are complicated This group can help the patients with organizing their
by structural heart disease such as valvular insuffi- medical regimen and transportation to the dialysis unit.
ciency and atrial fibrillation. Neurodegenerative They can provide support and counseling to the patient
disease impacts on the patient’s mobility and cog- and family for coping with chronic disease.
They can advise on cognitive function and whether the
nitive function. Osteoarthritis and neuropathy patient should still be signing their own consents.
limit their physical activity. As age and disease ad- They can take care of the intercurrent illnesses and chronic
vance, frailty becomes an issue. All of these things conditions.
combine to make the care of the older dialysis pa- They can review and manage polypharmacy.
tient much more complex than that of a younger They can help manage pain.
They can coordinate the other specialty care.
individual. Drug interactions and inappropriate Many practices now provide a transitional care clinic for
dosing becomes an increasing issue as the number rapid follow-up on discharge to prevent bounce back to
of comorbidities and medications increases. the hospital.
In 1992, Nespor and Holley9 did a small study of
in-center hemodialysis patients in Pittsburgh.
Eighty percent of these patients did not have a fam- Correspondence: Jocelyn Wiggins, University of Michigan, 1150
ily physician and relied on their nephrologist for all W. Medical Center Drive, 1560 MSRB II, Ann Arbor, MI 48109.
of their medical care. Ninety-one percent sought E-mail: [email protected]

treatment from their nephrologist for minor acute Copyright 䊚 2009 by the American Society of Nephrology

American Society of Nephrology Geriatric Nephrology Curriculum 1


They can help evaluate whether the living environment is still appropriate. DISCLOSURES
They can identify community resources to help the patient remain
None.
independent in the community.
They can assist in nutrition management and access to appropriate food.
Geriatricians can also help with advanced care planning, guiding the
patient through appointing an advocate and laying out their wishes
with respect to end of life care. REFERENCES
Many geriatricians also have experience in palliative care and can manage
the patient if and when they choose to withdraw from dialysis.
*Key References
1. U.S. Renal Data System: USRDS 2007 Annual Data Report: Atlas of
Chronic Kidney Disease and End-Stage Renal Disease in the United
States. Bethesda, MD, National Institutes of Health, National Institute
CONCLUSIONS of Diabetes and Digestive and Kidney Diseases, 2007
2. US Census Bureau: American Factfinder. Available online at www.
Cooperative care can be handled in two ways. Patients can be census.gov. Accessed November 2008
3. Murray AM: Cognitive impairment in the aging dialysis and chronic
referred to a geriatric center for comprehensive geriatric eval- kidney disease populations: an occult burden. Adv Chronic Kidney Dis
uation and for ongoing primary care. Alternatively a geriatric 15: 123–132, 2008*
physician can be asked to do consultative rounds in the dialysis 4. Murray AM, Tupper DE, Knopman DS, Gilbertson DT, Pederson SL, Li
unit and participate in team meetings. There are no studies that S, Smith GE, Hochhalter AK, Collins AJ, Kane RL: Cognitive impair-
support either of these models in the ESKD population or data ment in hemodialysis patients is common. Neurology 67: 216 –223,
2006
to show that you can improve quality of care, hospitalizations, 5. Madero M, Gul A, Sarnak MJ: Cognitive function in chronic kidney
or mortality statistics. However, the busy nephrologist may disease. Semin Dial 21: 29 –37, 2008
wish to consider using a geriatrician to help with the care of 6. Kimmel PL: Depression in patients with chronic renal disease: what we
their most complex and frail elderly patients. know and what we need to know. J Psychosom Res 53: 951–956, 2002
7. Watnick S, Kirwin P, Mahnensmith R, Concato J: The prevalence and
treatment of depression among patients starting dialysis. Am J Kidney
Dis 41: 105–110, 2003
TAKE HOME POINTS 8. Kimmel PL, Cohen SD, Peterson RA: Depression in patients with
chronic renal disease: where are we going? J Ren Nutr 18: 99 –103,
• Average age for starting dialysis is 62.3 for men and 63.4 for women 2008*
• Older patients have more comorbidities 9. Nespor SL, Holley JL: Patients on hemodialysis rely on nephrologists
• Cognitive impairment and depression are common in this group and and dialysis units for maintenance health care. ASAIO J 38: M279 –
complicates treatment M281, 1992*
• Cooperative care with a geriatrician can help the nephrologists with the 10. Holley JL, Nespor SL: Nephrologist-directed primary health care in
nonkidney issues chronic dialysis patients. Am J Kidney Dis 21: 628 – 631, 1993*

2 Geriatric Nephrology Curriculum American Society of Nephrology


REVIEW QUESTIONS: INTEGRATED CARE OF THE a. 10%
ELDERLY WITH ESKD b. 30%
c. 50%
1. The average age for initiating dialysis is between d. 80%
a. 45 to 50 e. 100%
b. 50 to 55
c. 55 to 60 3. A geriatrician can help with all of the following except:
d. 60 to 65 a. Cognitive evaluation
e. 65 to 70 b. Depression evaluation
c. Library fines
2. What percent of dialysis patients rely on their nephrologists d. Polypharmacy
for all of their medical care? e. Advance Care Planning

American Society of Nephrology Geriatric Nephrology Curriculum 3


Chapter 29: Nutrition and the Kidney in the Elderly
Patient
John E. Morley
Division of Geriatric Medicine, St. Louis University School of Medicine and GRECC, VA Medical Center, St. Louis,
Missouri

Both older persons and persons with kidney failure 3. Overall, the best treatments are exercise (espe-
are highly prone to develop nutritional deficiencies cially resistance exercise), anabolic hormones (e.g.,
(Table 1). The major nutritional problem experi- testosterone and selective androgen receptor mole-
enced by both groups is weight loss associated with cules), and vitamin D replacement. Creatine to-
protein energy malnutrition. Renal failure patients gether with exercise improves muscle strength in
with low body mass index, weight loss, low albu- older persons. It also reduces cramps. There are no
min, and low cholesterol all have increased morbid- data in renal failure.
ity and mortality.1 These are classical components The decreased testosterone and vitamin D asso-
of the malnutrition, inflammation, and atheroscle- ciated with kidney disease make sarcopenia more
rosis (MIA) syndrome in ESRD. Similarly, in older likely to occur in renal failure. Insulin resistance in
persons, weight loss is associated with increased renal failure further decreases muscle anabolism
mortality. and increases fat accumulation in muscle. With ag-
ing, there is a physiologic anorexia of aging with
older males reducing their caloric intake by a third
WEIGHT LOSS and females by a quarter over their lifespan.3 There
are multiple causes of this physiologic anorexia. Ag-
The causes of weight loss in older persons are as ing is associated with a decline in taste and olfac-
follows2: cachexia, anorexia, sarcopenia, and dehy- tion. With aging there is a decrease in adaptive re-
dration. laxation of the fundus of the stomach, resulting of a
Sarcopenia is the loss of muscle mass that occurs quicker filling of the antrum and early satiation.
with aging. It is associated with varying degrees of This is associated with slower gastric emptying that
muscle power (dynapenia). Severe sarcopenia (de- occurs with large gastric volumes in older persons.
fined as appendicular skeleton lean mass corrected The satiation hormone, cholecystokinin, is in-
for height that is 2 SD below the normal value for creased with aging, and it is more effective at reduc-
young persons) occurs in 5 to 13% of persons over ing food intake in older persons. The reduction in
the age of 70 yr. Sarcopenia is associated with in- testosterone in older males leads to an increase in
creased disability, and its medical costs have been leptin that can reduce food intake and increase en-
calculated to be $18.4 billion per year in the United ergy metabolism.
States. On the whole, fat older persons who main- Renal failure is classically associated with an-
tain muscle mass do fairly well, but those who are orexia because of circulating uremic toxins. In ad-
fat but have lost muscle mass (the “fat frail” or sar- dition, urea in the mouth produces gingivitis, de-
copenic obese) have worse outcomes as far as dis- creasing the enjoyment of eating. Male kidney
ability and mortality than do the thin sarcopenic. failure patients have low testosterone, increasing
There are many causes of sarcopenia (Table 2). the potential of higher leptin levels increasing an-
These include genetic factors, weight at birth, poor orexia.
energy and protein intake, low levels of activity, de-
creased motor units, insulin resistance, decreased
anabolic hormones, low vitamin D, increased cyto- Correspondence: John E. Morley, MB, BCh, Division of Geriatric
Medicine, St. Louis University School of Medicine, 1402 S. Grand
kines, and peripheral vascular disease. The poten- Boulevard, M238, St. Louis, MO 63104. E-mail: [email protected]
tial treatments for sarcopenia are outlined in Table
Copyright 䊚 2009 by the American Society of Nephrology

American Society of Nephrology Geriatric Nephrology Curriculum 1


Table 1. Nutritional alterations in kidney disease Table 3. Treatment of sarcopenia
1. Weight Loss Exercise
—Sarcopenia Aerobic
—Protein energy malnutrition Resistance
Anorexia Vibration platform exercises
Malabsorption Nutrients
Hypermetabolism Essential amino acids
Cachexia Creatine
—Dehydration Anabolics
2. Vitamin abnormalities Testosterone
—Decreased folate Selective androgen receptor molecules
—Decreased pyridoxine Ghrelin analogs
—Increased homocysteine Proteolysis Inhibitor
—Decreased niacin Angiotensin converting enzyme (ACE) inhibitors
—Decreased vitamin C Orexigenics
—Decreased 25(OH) vitamin D Megestrol
—Increased vitamin A Dronabinol
3. Trace mineral abnormalities
—Decreased zinc variety of medicines interfere with taste and produce anorexia.
—Decreased selenium The common reversible causes of weight loss in older persons
—Decreased iron or its bioavailability
are given in Table 4. Nutritional deficiencies may be related to
—Increased copper
(1) the inability to access groceries— decline in mobility, poor
—Increased magnesium
4. Electrolytes
vision, and loss of driving privileges and (2) lack of an enthu-
—Hyperkalemia siasm to cook and a loss of ability to prepare food. Food prep-
—Hyponatremia aration requires cognitive styles, dexterity, and ability to stand
5. Carnitine deficiency and/or functional impairment for long periods, all of which are often lacking in elderly dialysis
6. Lipids patients.4
—Hypertriglyceridemia In renal failure, nausea, vomiting, and diarrhea can lead to
—Hypercholesterolemia weight loss. Confusion, depression, and anxiety can lead to
7. Carbohydrates decreased food intake. Generalized bone pain may limit food
—Insulin resistance intake.
—Hyperglycemia
Approximately 8% of renal failure patients on dialysis have
cachexia. Cachexia or wasting disease consists of loss of both
The most common cause of pathologic anorexia with aging muscle and fat, anorexia, hypermetabolism, decreased intesti-
is depression. Therapeutic diets are bland and further aggra- nal mobility, low albumin, low cholesterol, and elevated acute
vate anorexia. Chronic pain often limits the desire to eat. A phase proteins.5 In general, in renal failure patients, it is caused
by elevated inflammatory cytokine levels. Cytokines decrease
Table 2. Causes of sarcopenia*
cholesterol and cause third spacing of albumin and prealbu-
Lack of physical activity min, explaining the very low levels of these circulating sub-
Lack of adequate protein ingestion stances often seen in renal failure. In renal failure, superim-
Anabolic hormone deficiency
posed acute and chronic illnesses can further aggravate the
Testosterone
Dehydroepiandrosterone insulin growth factor-1 (DHEA) Table 4. MEALS-ON-WHEELS mnemonic for treatable
Growth hormone, including its muscle isoform causes of weight loss in older persons
Vitamin D deficiency
Medications (e.g., digoxin, theophylline, cimetidine)
Cytokine excess (interleukin-6, tumor necrosis factor-␣)
Emotional (e.g., depression)
Motor neuron loss
Alcoholism, elder abuse, anorexia tardive
Insulin resistance
Late life paranoia
Low birth weight
Swallowing problems
Genetics
Oral factors
Myostatin
Nosocomial infections (e.g., tuberculosis)
Ciliary neurotrophic factor (CNTF) and its receptor
Wandering and other dementia-related factors
Vitamin D receptor (VDR Bsm1)
Hyperthyroidism, hypercalcemia, hypoadrenalism
Angiotensin converting enzyme
Enteral problems (e.g., gluten enteropathy)
Androgen receptor gene (CAG-repeats)
Eating problems
Cyclin-dependent kinase inhibitor 1A
Low salt, low cholesterol, and other therapeutic diets
*For those wishing more details regarding the genetic factors, please see
reference 2.
Stones (cholecystitis)

2 Geriatric Nephrology Curriculum American Society of Nephrology


cachexia. Peritoneal dialysis is associated with potentially large maintain adequate erythropoiesis, as measured by flow cytom-
losses of protein and albumin. Blood loss that is common in etry (⬎10% hypochromic subpopulation), intravenous iron is
older persons with renal failure further contributes to protein required. Excess iron replacement should be avoided because
energy malnutrition. The role of toxins, retained during renal of the possibility that it may increase the likelihood of certain
failure, at producing catabolism is uncertain. Acidemia sup- infections.
presses albumin synthesis and promotes negative nitrogen bal- Aluminum excess occurs mainly because of use of Al(OH)3
ance. as a phosphate binder. Aluminum excess has been correlated
Recent studies have suggested that optimal survival in pa- with cognitive dysfunction and bone disease. Thus, low phos-
tients with renal failure requires a protein intake of between 1.1 phate diets and use of nonaluminum phosphate binders are
to 1.4 g/kg protein per day.6 In end-stage renal failure patients, preferred for control of hyperphosphatemia.
an energy intake of 35 to 40 kcal/kg per day seems to be neces-
sary to maintain weight and nitrogen balance. In persons with
a lower caloric intake, amino acid supplementation may be
helpful. CARNITINE
Studies in older persons with ESRD have suggested that very
low protein diets may decrease the time to dialysis and days in Carnitine is a nutrient that is essential for the transport of long
hospitalization without altering mortality. As this is a quality- chain fatty acids into mitochondria. As such, it plays a key role
of-life issue, older persons should be informed of this possibil- in mitochondrial energy control. Serum carnitine deficiency
ity to allow them to make informed choices. In severely mal- occurs during hemodialysis. Carnitine in hemodialysis pa-
nourished anorectic patients on dialysis, intradialytic tients may reduce fatigue, increase exercise capacity, reduce
peripheral parenteral nutrition may be a reasonable ap- erythropoietin requirement, reduce cramps, and reduce hypo-
proach.7–9 tensive events during dialysis.12,13 Carnitine seems to be safe.
Available data consist of small trials and thus its use cannot be
recommended routinely. However, in severe muscle weakness,
MICRONUTRIENTS cramps, dialysis hypotension, fatigue, or anemia resistant to
erythropoietin, a therapeutic trial may be considered. Carni-
In general, because caloric intake is insufficient in persons on tine can be given orally as 0.5 g daily or infused intravenously
dialysis, it may be expected that micronutrient intake may be after dialysis (10 to 20 mg/kg, three times per week).
insufficient. Virtually no patients ingest the recommended fo-
late and pyridoxine intakes.10 Niacin and selenium intakes are
also low in persons on long-term dialysis. Vitamin C levels are TAKE HOME POINTS
reduced by hemodialysis.11 25(OH) Vitamin D levels are low in
ESRD. • Protein energy malnutrition is the most common nutritional problem in
older persons on dialysis; it is associated with poor outcomes
Elevated homocysteine levels in epidemiologic studies have • Folate and pyridoxine intakes are usually insufficient in older persons
been associated with cardiovascular disease, Alzheimer’s dis- with renal failure
ease, and osteoporosis. Renal failure itself causes elevation of • Outcomes, such as less falls and improved function can be obtained by
homocysteine. Folate has the best effect on reducing homocys- keeping 25(OH) vitamin D levels ⬎100 nmol/L.
teine in the deficiency patients. Addition of vitamin B12 may be
appropriate to prevent unmasking of latent vitamin B12 levels.
Elevated methylmalonic acid levels are diagnostic of vitamin DISCLOSURES
B12 deficiency. None.
Older persons with renal failure who are bruising easily will
benefit from vitamin C supplementation. Vitamin A levels are
elevated in end-stage kidney disease. Elevated vitamin A levels
cause increased production of PTH and bone disease. For this REFERENCES
reason, multivitamins with vitamin A need to be avoided in
*Key References
renal failure.
1. Kalantar-Zadeh K, Horwich TB, Oreopoulos A, Kovesdy CP, Younessi
As far as trace elements are concerned, zinc, selenium, and H, Anker SD, Morley JE: Risk factor paradox in wasting diseases. Curr
iron are the most likely to be deficient in end-stage kidney Opin Clin Nutr Metab Care 10: 433– 442, 2007
failure, whereas magnesium and copper are liable to be in ex- 2. Morley JE: Weight loss in older persons: new therapeutic approaches.
cess. Zinc deficiency in kidney failure may lead to dysgeusia, Curr Pharm Des 13: 3637–3647, 2007*
3. Morley JE: Anorexia and weight loss in older persons. J Gerontol A
anorexia, and hypogonadism. Zinc deficiency is particularly
Biol Sci Med Sci 58: 131–137, 2003
likely to occur in patients on diuretics. 4. Campbell KL, Ash S, Bauer JD: The impact of nutrition intervention on
Iron sulfate causes anorexia and gastrointestinal distress quality of life in pre-dialysis chronic kidney disease patients. Clin Nutr
and, as such, iron gluconate is preferred. If oral iron cannot 27: 537–544, 2008*

American Society of Nephrology Geriatric Nephrology Curriculum 3


5. Morley JE, Thomas DR, Wilson MM: Cachexia: pathophysiology and 9. Eyre S, Attman PO, Haraldsson B: Positive effects of protein restric-
clinical relevance. Am J Clin Nutr 83: 735–743, 2006* tion in patients with chronic kidney disease. J Ren Nutr 18: 269 –
6. Kopple JD: McCollum Award Lectures, 1996: protein-energy malnu- 280, 2008
trition in maintenance dialysis patients. Am J Clin Nutr 65: 1544 –1557, 10. Andrew NH, Engel B, Hart K, et al.: Micronutrient intake in haemodi-
1997 alysis patients. J Hum Nutr Diet 21: 375–376, 2008
7. Brunori G, Viola BF, Maiorca P, Cancarini G: How to manage elderly 11. Kalantar-Zadeh K, Kopple JD: Trace elements and vitamins in main-
patients with chronic renal failure: conservative management versus tenance dialysis patients. Adv Ren Replace Ther 10: 170 –182, 2003
dialysis. Blood Purif 26: 36 – 40, 2008 12 Savica V, Calvani M, Benatti P, Santoro D, Monardo P, Mallamace A,
8. Brunori G, Viola BF, Parrinello G, De Biase V, Como G, Franco V, Savica R, Bellinghieri G: Newer aspects of carnitine metabolism in
Garibotto G, Zubani R, Cancarini GC: Efficacy and safety of a very- uremia. Semin Nephrol 26: 52–55, 2006*
low-protein diet when postponing dialysis in the elderly: a prospective 13. Hurot J-M, Cucherat M, Haugh M, Fouque D: Effects of L-carnitine
randomized multicenter controlled study. Am J Kidney Dis 49: 569 – supplementation in maintenance hemodialysis patients: a systematic
580, 2007* review. J Am Soc Nephrol 13: 708 –714, 2002

4 Geriatric Nephrology Curriculum American Society of Nephrology


REVIEW QUESTIONS: NUTRITION AND THE KIDNEY a. Thin sarcopenic
IN THE ELDERLY PATIENT b. Obese
c. Obese sarcopenic
1. Optimal protein intake in older persons with kidney disease
d. BMI 21 to 23 kg/m2
are
a. 0.8 g/kg per day
3. Which of the following should be avoided in kidney
b. 0.8–1 g/kg per day
disease?
c. 1.1–1.4 g/kg per day
a. Vitamin A
d. 1.5–1.8 g/kg per day
b. Folate
e. 2.0 g/kg per day
c. Vitamin C
2. Which of the following have the worst outcomes as far as mor- d. Vitamin D
tality and disability are concerned? e. Zinc

American Society of Nephrology Geriatric Nephrology Curriculum 5


Chapter 30: Urinary Incontinence in the Elderly
George A. Kuchel* and Catherine E. DuBeau†
*UConn Center on Aging, University of Connecticut Health Center, Farmington, Connecticut; and †Division of
Geriatric Medicine, University of Chicago, Chicago, Illinois

EPIDEMIOLOGY is usually associated with uninhibited bladder con-


tractions, called detrusor overactivity (DO). How-
The prevalence of urinary incontinence (UI) in- ever, up to 40% of continent healthy older adults
creases with age. Moderate to severe UI affects 7% have DO on urodynamic testing,5 suggesting that
of women 20 to 39 yr of age, 17% 40 to 59 yr of age, urge UI requires not just DO but impaired central
23% 60 to 79 yr of age, and 32% ⱖ80 yr of age.1 The nervous system and other compensatory mecha-
prevalence in men is approximately one third that nisms as well. DO may be idiopathic, age-related,
of women, until it equalizes in the ninth decade. secondary to lesions in cerebral and spinal inhibi-
Nursing home residents have especially high rates tory pathways, caused by bladder outlet obstruc-
of UI, ranging from 60 to 78% in women and 45 to tion, or (less commonly) result from local bladder
72% in men. The primary impact of UI is on quality irritation (e.g., infection, stones, tumor). Recent ev-
of life, including self-concept, self-esteem, and the idence suggests that increased afferent signaling
burden of coping. Economic costs of UI continue to from the detrusor may also contribute to urgency,
rise, adding an estimated $6 billion annually to the DO, and urge UI. Frail older persons may have DO
cost of caring for older Americans.2 but without sufficient detrusor contractility to fully
empty the bladder, leading to an elevated postvoid-
ing residual volume (PVR). This condition is called
ETIOLOGY detrusor hyperactivity with impaired contractility
(DHIC).5
Whereas UI in younger and middle-aged persons is Stress UI is the symptom of leakage associated
nearly entirely caused by alteration in the lower uri- with increased intra-abdominal pressure. It occurs
nary tract (LUT) and its innervation, in older per- with impaired urethral sphincter support or dam-
sons, UI represents a geriatric syndrome with age impairing urethral closure, as may occur from
broadly based, patient level risk factors that include surgical scarring, radical prostatectomy, and some
age-related changes in physiology, comorbidity, spinal cord injuries. Although parity is an impor-
medications, and especially functional impair- tant risk factor for stress UI in younger and middle-
ments.3,4 Moreover, in older persons, UI can cause aged women, its contribution to stress UI in older
significant morbidity (such as falls and fractures) women is minimal to none.
and functional impairment. Additionally, many Mixed UI is the symptom of leakage with fea-
older and especially frailer persons require caregiv- tures of both urge and stress UI and is most com-
ers, and UI can lead to caregiver stress and institu- mon in younger-old women. Either urge or stress
tionalization of the frail elder. Risk factors for UI in UI symptoms may predominate.
older persons include impaired mobility, falls, UI with impaired bladder emptying is leakage
medications, depression, transient ischemic attacks associated with an elevated PVR, which can be
and stroke, dementia, congestive heart failure, fecal caused by bladder outlet obstruction, poor bladder
incontinence and constipation, and obesity.4
Correspondence: George A. Kuchel, MD, FRCP, Professor of
Medicine, Travelers Chair in Geriatrics & Gerontology, Director,
UConn Center on Aging, Chief, Divison of Geriatric Medicine,
TYPES OF INCONTINENCE University of Connecticut Health Center, 263 Farmington Avenue,
MC-5215, Farmington, CT 06030-5215. Phone: 860-679-3956;
Urge UI is the symptom of leakage associated with a Fax: 860-679-1307; E-mail: [email protected]
compelling, often sudden, urgency to void. Urge UI Copyright 䊚 2009 by the American Society of Nephrology

American Society of Nephrology Geriatric Nephrology Curriculum 1


contractility (detrusor underactivity), or both.6 The most causes) and nocturia (which can be caused by a disproportion-
common cause of obstruction in men is prostate disease, and in ate nocturnal polyuria or a primary sleep disorder).
women, it is either a large cystocele that kinks the urethra or
urethral scarring. Detrusor underactivity may be age related,
caused by detrusor smooth muscle damage, peripheral neu- EVALUATION
ropathy (diabetes mellitus, vitamin B12 deficiency, alcohol-
ism), or damage to the sacral cord and spinal bladder efferents Evaluation of UI in older persons should be multifactoral, ad-
by disc herniation, spinal stenosis, tumor, or degenerative neu- dressing comorbidity, function, and medications as potential
rologic disease. Neurologic diseases affecting the sacral spinal etiologic or contributing factors. The important first step is
cord can cause detrusor underactivity and/or neurally medi- active screening for UI, because 50% of affected persons do not
ated obstruction, depending on the exact level and extent of volunteer their symptoms to their providers. History should
damage. include UI onset, frequency, volume, timing, and associated
In general, over the age of 65, urge UI increases, stress UI factors or events. Patients and/or caregivers should be asked
decreases, the prevalence of mixed UI remains stable, and over- about UI-associated bother and quality-of-life impact. Simple
flow is rare. Urge UI is the most common form in older men, questions can help determine the type of UI symptoms: e.g.,
whereas stress UI is important after prostatectomy, especially “Do you lose urine during coughing, sneezing, or lifting?” (for
radical surgery. stress UI) and “Do you experience a such strong and sudden
Regardless of the type of UI symptom, leakage in older per- urge to urinate that you leak before reaching the toilet?” (for
sons may be caused or worsened by comorbid conditions, urge UI). In women, these questions are most helpful to diag-
medications, and functional impairment (Table). In the past, nose urge UI and slightly less so stress UI; if a woman denies
UI related to such factors has often been referred to as “tran- stress leakage, it is highly unlikely that she has physiologic
sient” or “functional” UI. However, for many older persons, stress UI.7
UI is a chronic and often progressive condition, and both Physical examination should include cognitive and func-
“transient” and “functional” contributing factors frequently tional assessments and focus on potential comorbid condi-
co-exist with other comorbidity and lower urinary tract (LUT) tions associated with UI. Rectal exam is used to assess for
abnormalities. Persons with UI usually have several other LUT masses, tone, and prostate nodules or firmness in men (not
symptoms (LUTS), including frequency (patient complaint of size). The neurologic evaluation should include evaluation of
too frequent voiding); nocturia (need to wake twice or more sacral cord integrity with perineal sensation, anal “wink” (anal
times at night to void); slow stream (perception of reduced sphincter contraction when the perirectal skin is lightly
urine flow); intermittent stream (urine flow which stops and scratched), and bulbocavernosus reflex (anal sphincter con-
starts during voiding); hesitancy (difficulty in initiating urine traction when either the clitoris or glans is lightly touched).
flow); straining (to either initiate, maintain or improve urine Vaginal mucosa should be evaluated for severe atrophy, and
flow); and sense of incomplete bladder emptying. These LUTS the pelvic exam should include evaluation for pelvic organ pro-
lack specificity, especially frequency (which can reflect in- lapse (cystocele, rectocele, uterine prolapse) with straining.4
creased fluid intake and/or increased diuresis from many Urinalysis is recommended for all patients, primarily to

Table 1. Examples of comorbidity causing or worsening UI in older persons


Aspect of Continence Affected Comorbidity
Ability to get to toilet Functional impairment, e.g., from arthritis, Parkinson’s, poor vision
Extrapyramidal effects of antipsychotic medications
Medications causing sedation or confusion, e.g., benzodiazepines
Poor access to toilets
Severe cognitive impairment: advanced dementia, severe depression
Fluid balance Excessive intake of caffeinated beverages, alcohol
Increased nocturnal diuresis from congestive failure, sleep apnea, venous stasis, or drugs
causing peripheral edema (e.g., amlopidine, thiazolinediones, gabapentin)
Diuretic medications
Urethral closure Marked obesity
Cough because of pulmonary disease, ACE inhibitors
Increased urethral tone from alpha adrenergic agonists, decreased tone from alpha blockers
Bladder contractile strength Medications impairing bladder contractility: calcium channel blockers, anticholinergics, opiates
Diabetes (advanced), vitamin B12 deficiency, lower spinal cord injury
Uninhibited bladder contractions CNS diseases, stroke, suprasacral spinal cord diseases
Diabetes
Local bladder irritation: stones, carcinoma

2 American Society of Nephrology American Society of Nephrology


look for hematuria (and glycosuria in diabetics). Pyuria and/or The two main behavioral therapies are bladder training and
bacteriuria likely represents asymptomatic bacteriuria—not pelvic muscle exercises, both of which are effective for urge,
cystitis—in women without dysuria, fever, or other signs of mixed, and stress UI and are often used in combination. Blad-
urinary tract infection, especially if UI is not acute. der training uses two principles: frequent voluntary voiding to
Although frail elderly may have a higher prevalence of ele- keep bladder volume low and urgency suppression using cen-
vated PVR, especially in association with DHIC, it is not clear tral nervous system and pelvic mechanisms. Prompted voiding
that evaluation of the PVR always would alter management. can be used instead of bladder training for persons with cogni-
Therefore, a prudent approach would be to limit PVR testing tive impairment and urge UI. Pelvic muscle exercises (PMEs)
to patients with either diabetes, previous urinary retention or strengthen the muscular components of urethral support and
elevated PVR, recurrent urinary tract infections (UTIs), severe are effective for urge, mixed, and stress UI.
constipation, complex neurologic disease (e.g., Parkinson’s), Pharmacologic treatment is largely limited to antimusca-
marked pelvic organ prolapse or prior anti-incontinence sur- rinic agents for urge UI, overactive bladder, and mixed UI.
gery (women), medications known to decrease detrusor con- Oral estrogen, alone or in combination with progestins, in-
tractility (e.g., anticholinergics), persistent or worsening urge creases stress and urge UI and should not be used. There is no
UI despite antimuscarinic treatment, or prior urodynamic consensus whether topical estrogen applied in the vagina im-
evaluation showing with poor contractility or outlet obstruc- proves UI, but it is helpful for uncomfortable vaginal atrophy
tion. Ultrasound or catheterization must be used because ab- and can decrease recurrent UTIs. There are currently five an-
dominal palpation is neither sensitive nor specific. timuscarinic agents for treatment of urge UI and overactive
A clinical stress test may be helpful in patients with stress UI bladder: oxybutynin (immediate and extended release, and
symptoms. The patient should have a full bladder and a relaxed topical patch), tolterodine (immediate and extended release),
perineum and buttocks, and the examiner should be posi- solifenacin, darifenacin, and trospium. These five antimusca-
tioned to observe or catch any leakage when the patient gives a rinics have similar efficacy, resulting in continence rates of
single vigorous cough.7 The test is most sensitive when the approximately 30% and reduce UI by an average of half an
patient is upright and insensitive if the patient cannot cooper- episode or more per day over placebo.8 Two additional agents
ate, is inhibited, or the bladder volume is low. Bladder diaries are expected to become available in 2009: an extended release
can be helpful to determine whether urine volume and timing version of trospium, and fesoterodine, which is a prodrug that
contribute to frequency and nocturia symptoms and can assist is metabolized to tolterodine.
evaluation of UI frequency, timing, and circumstances. The The most common adverse drug effect (ADE) from anti-
diary entails recording the time and volume of all continent muscarinics is dry mouth, which is not only bothersome but
voids and UI episodes, typically over 3 d. Routine urodynamic can cause dental caries, problems chewing, poorly fitting den-
testing is not necessary and may be misleading because of the tures, dysphagia, and sleeping difficulty. The highest rates of
high prevalence of DO in healthy, continent older persons. dry mouth occur with oxybutynin. Other ADEs include de-
Such testing should be reserved for patients considering inva- creased visual accommodation (causing blurred vision and
sive treatment; when either the etiology of UI is unclear and a possibly increased falls) and constipation (more likely with
precise diagnosis would change management; or when empiric oxybutynin, solifenacin, and darifenacin than tolterodine).
treatment has failed. Cystoscopy is necessary only for hematu- The major antimuscarinic ADE of concern in older and espe-
ria or otherwise unexplained pelvic pain. cially frail adults is cognitive impairment, yet the incidence,
prevalence, domains of impairment, magnitude, and impact of
this ADE from specific antimuscarinic UI drugs is largely un-
TREATMENT known. There is insufficient evidence at this time that one
agent is “safer” for all older patients or specifically those with
Correction of reversible precipitants and contributing factors dementia or central nervous system diseases, despite theoreti-
is critical. Specific treatment should be targeted to the patient’s cal arguments about the ability of specific agents to cross the
most bothersome symptom(s). Treatment should be stepped, blood– brain barrier. Most importantly, it is unclear that cog-
starting with noninvasive behavioral methods and then adding nitive risks outweigh potential treatment benefits. Other
medications (if urge UI still is problematic), and finally con- agents used for UI (e.g., flavoxate, propantheline, dicyclomine,
sideration of minimally invasive or more extensive surgery, if imipramine, hyoscyamine) have scant or poor efficacy data.
appropriate and acceptable. Specialist referral is recommended Vasopressin (DDAVP) should not be used to treat nocturia in
for patients with hematuria, pelvic pain, complicated neuro- older persons because of the high risk of hyponatremia.
logic disease (e.g., spinal cord injury), and previous pelvic surgery. Pessaries may benefit women whose stress or urge UI is
Lifestyle modifications that may be helpful include avoiding exacerbated by bladder or uterine prolapse. There are now sev-
extremes of fluid intake, caffeinated beverages, and alcohol; eral minimally invasive therapies available for those with urge
minimizing evening intake for nocturia; and smoking cessa- UI refractory to antimuscarinics, including botulinum toxin
tion for patients with stress UI. Weight loss has been shown to injection into the bladder wall and sacral neuromodulation.
decrease UI in morbidly obese women. Surgery provides the highest cure rates for women with stress

American Society of Nephrology American Society of Nephrology 3


UI. Many patients will require or choose to continue to use DISCLOSURES
pads and protective garments. Catheters should be reserved for None.
short-term decompression of acute retention, management of
outlet obstruction when medical or surgical treatment is not
possible, protection of wounds that need to be kept clean of REFERENCES
urine, and for terminally ill or severely impaired persons for
whom frequent clothing or garment changes would be very *Key References
uncomfortable. 1. Nygaard I, Barber MD, Burgio KL, Kenton K, Meikle S, Schaffer J,
Spino C, Whitehead WE, Wu J, Brody DJ: Prevalence of symptomatic
pelvic floor disorders in US women. JAMA 300: 1311–1316, 2008
2. Langa KM, Fultz NH, Saint S, Kabeto MU, Herzog AR: Informal care-
TAKE HOME POINTS giving time and costs for urinary incontinence in older individuals in
the United States. J Am Geriatr Soc 50: 733–737, 2002
• UI is highly prevalent in older persons and results in significantly de- 3. Inouye SK, Studenski S, Tinetti ME, Kuchel GA: Geriatric syndromes:
creased quality of life, morbidity, and high costs clinical, research, and policy implications of a core geriatric concept.
• In older persons, UI is not simply a LUT disorder but represents a J Am Geriatr Soc 55: 780 –791, 2007
geriatric syndrome with broadly based, patient level risk factors that 4. DuBeau CE: Beyond the bladder: management of urinary inconti-
include age-related changes in physiology, comorbidity, medications, nence in older women. Clin Obstet Gynecol 50: 720 –734,2007*
and functional impairments 5. Resnick NM, Yalla SV, Laurino E: The pathophysiology of urinary
• Older persons should be actively screened for UI, and an initial office- incontinence among institutionalized elderly persons. N Engl J Med
based evaluation based on history, exam, and urinalysis is sufficient to 320: 1–7, 1989
initiate treatment 6. Taylor JA, Kuchel GA: Detrusor underactivity: clinical features and
• UI treatment should be stepwise, progressing from behavioral and pathogenesis of an underdiagnosed geriatric condition. J Am Geriatr
medication therapy to more invasive approaches, as needed and ap- Soc 54: 1920 –1933, 2006*
propriate 7. Holroyd-Leduc JM, Tannenbaum C, Thorpe KE, Straus SE: What type
• Behavioral therapy (bladder training and pelvic muscle exercises) is of urinary incontinence does this woman have? JAMA 299: 1446 –
effective in reducing urge and stress UI 1456, 2008*
• Antimuscarinic medications for urge UI have similar efficacy, and drug 8. Shamliyan TA, Kane RL, Wyman J, Wilt TJ: Systematic review: ran-
choice should be guided by anticipated adverse effects and other domized, controlled trials of nonsurgical treatments for urinary incon-
factors (e.g., cost) tinence in women. Ann Intern Med 148: 459 – 473, 2008*

4 American Society of Nephrology American Society of Nephrology


REVIEW QUESTIONS: URINARY INCONTINENCE IN a. Initiate a course of pelvic muscle exercises
THE ELDERLY b. Stop the diuretic
c. Start a bladder relaxant
1. In older patients, uninhibited bladder contractions:
d. Refer to surgery if she is intent on cure
a. Although present may not be the cause of incontinence
b. Are rarely seen in asymptomatic patients
3. An 85-yr-old woman complains of urge incontinence episodes
c. Are primarily caused by CNS pathology
twice a day and nocturia three times at night. Her bladder diary
d. Are inevitable with dementia
reveals a daytime urine volume of 800 ml and a nocturnal
2. A 74-yr-old woman complains of leakage associated with an output of 700 ml. This patient’s nocturia:
abrupt urge to void that occurs at least twice daily but not at a. Is caused by DHIC (detrusor hyperactivity with impaired
night. She also notes leakage with coughing. She in general contractility)
good health, has hypertension treated with a thiazide diuretic, b. Should be treated with tolterodine at bedtime
and uses topical estrogen for vaginal dryness. The next step in c. May be related to pedal edema
management should be: d. Should be addressed with a bedside commode

American Society of Nephrology American Society of Nephrology 5


Chapter 31: Lower Urinary Tract Conditions in Elderly
Patients
Damon Dyche and Jay Hollander
William Beaumont Hospital, Royal Oak, Michigan

As our population ages, the number of patients pre- uroflow/urodynamic studies, and cystoscopy. Com-
senting to their primary care physicians with uro- mon transurethral treatment modalities include re-
logic problems is significantly increasing. Urologic section, laser ablation, and microwave or radiofre-
issues are the third most common type of complaint quency therapy.
in patients 65 yr of age or older and account for at There are two major approaches of medical ther-
least a part of 47% of office visits.1 One of the most apy for prostatic outflow obstruction: relaxing the
predominant urologic problems in elderly persons, prostate smooth muscle tissue or decreasing glan-
and the focus of this chapter, is lower urinary tract dular volume. ␣1-adrenergic blockade relaxes the
symptoms (LUTS). There are several disease pro- smooth muscle fibers of the prostatic stroma and
cesses that can lead to LUTS, as well as a number of can significantly improve urine flow. Because ␣
consequences. In this chapter, we will give a brief blockade can also have significant cardiovascular
overview of the major issues as they relate to elderly side effects, ␣1 selective medications were devel-
persons. oped to specifically target the urinary system. Com-
mon nonselective agents include terazosin and dox-
azosin; selective medications are tamsulosin and
BENIGN PROSTATIC HYPERPLASIA AND alfuzosin. 5-␣ reductase inhibitors block the con-
LUTS version of testosterone 3 DHT, which is a potent
stimulator of prostatic glandular tissue. This reduc-
The prostate surrounds the male urethra between tion in local androgen stimulation results in a pro-
the bladder neck and urinary sphincter like a gressive decrease in prostatic volume over a period
doughnut. When the doughnut enlarges, the of 6 mo to 1 yr. There is also a concomitant decrease
doughnut hole can close off and create an outflow in prostate-specific antigen (PSA) level by approx-
obstruction and/or irritative voiding symptoms. imately 50%, necessitating a doubling of the post-
Benign prostatic hyperplasia (BPH) is a condition treatment PSA to compare it with the pretreatment
that affects the majority of elderly men.2 Not all level. Common agents include finasteride and
cases of BPH need treatment. LUTS are assessed dutasteride. The combination of an ␣-blocker and a
with both subjective and objective studies. The 5␣-reductase inhibitor may work synergistically, al-
American Urological Association BPH symptom beit expensively, to improve LUTS.
score was designed to evaluate subjective com-
plaints. Patients are asked a series of questions re-
garding their urination in addition to a “bother PROSTATE CANCER
score.”3 Low scoring patients are advised of helpful
lifestyle changes, median range patients are given With increasing age, clinical prostate cancer be-
the option of medication, and patients with high comes more prevalent. It is estimated that about
scores (or those patients who are very bothered by 10% of patients ⬎65 yr of age have been diagnosed
their symptoms) are offered medication or trans- with prostate cancer. On autopsy studies, the inci-
urethral surgery. Surgery may be indicated for pa-
tients with recurrent/persistent infection, hematu-
Correspondence: Damon Dyche, William Beaumont Hospital,
ria, bladder stones, hydronephrosis, progressive
Department of Urology, 3601 West 13 Mile Road, Royal Oak, MI
renal failure, or acute urinary retention. Urologic 48073. E-mail: [email protected]
work-up is available and includes postvoid residual,
Copyright 䊚 2009 by the American Society of Nephrology

American Society of Nephrology Geriatric Nephrology Curriculum 1


dence is even higher, at almost 70% of cadavers 70 yr or older. intra-abdominal pressure. It can range from occasional leakage
There is a lot of controversy regarding prostate cancer screen- with strenuous exercise to leakage with ambulation, coughing,
ing because most prostate cancers are nonlethal and there may or sneezing. In women, stress incontinence is commonly asso-
be overtreatment in the United States. The American College ciated with a weakened urinary sphincter or inadequate pelvic
of Preventative Medicine (ACPM) reports there is insufficient floor muscle support that may be exacerbated by aging or mul-
evidence to support prostate cancer screening with digital rec- tiparity. Men may develop stress incontinence after prostate
tal exam (DRE) and PSA testing. The American Urologic As- surgery or other trauma to the urinary sphincter. Overflow
sociation recommends annual screening with DRE and PSA incontinence occurs when the patient has a maximally dis-
testing, beginning at age 50. Individuals with one or more first- tended bladder, leading to urine leakage. Common causes in-
degree relatives with a history of prostate cancer and African- clude prostatic obstruction/BPH, urethral stricture, or bladder
American men are at a higher risk of prostate cancer and contractility dysfunction that is neurologic in origin, such as
screening can be offered at 40 to 45 yr. Although the sensitivity diabetic neuropathy. Mixed incontinence is a combination of
and specificity of both screening tests are low, the combination urge and stress incontinence, commonly seen in elderly
of the two tests is synergistic and results in a better rate of women. Urge incontinence is very common in elderly men and
detection than either one alone. According to the American women.5 If patients with urge incontinence have a negative
Cancer Society, a PSA between 4 and 10 ng/ml represents a urinalysis and low postvoid residual, they can be empirically
25% risk of prostate cancer. Patients found to have an abnor- managed with anti-cholinergic medications. Stress inconti-
mal DRE and/or an elevated PSA typically undergo a prostate nence can also be managed medically or with behavioral mod-
biopsy, depending on the clinical scenario. Although LUTS are ifications. Surgical managements include bladder/urethral
much more commonly the result of BPH in men than prostate suspensions and artificial sphincters, which can be well toler-
cancer, the primary care physician must be familiar with PSA ated in elderly patients.
and the DRE. PSA screening in men over 80 yr may do more
harm than good, and a rectal exam alone should be sufficient.
NOCTURIA

POLYPHARMACY Nocturia is the act of awaking at night to urinate. Voiding two


or less times a night is likely normal for most elderly persons.
The average elderly patient is on two to six prescribed medica- More than two episodes of nocturia per night can disrupt sleep
tions, as well as one to three over-the-counter medications. and affect quality of life. It can be challenging to establish the
Many of these prescribed, over-the-counter, and herbal med- cause of nocturia, and a voiding diary is often helpful. Patients
ications can have urologic side effects. Many common medi- record the time of void and voided volume, along with the fluid
cations, including cold and flu therapies, have anti-cholinergic intake. Most problems can be avoided with simple lifestyle
properties that can exacerbate LUTS. Narcotics and sleep aids adjustments. Many elderly patients have nocturia because of
can result in constipation and restricted mobility, leading to increased fluid intake before bedtime or fluid mobilization.
voiding difficulties. Caffeine and alcohol are often associated This can be treated by limiting fluids and supine positioning
with urinary urgency and may result in leakage. The medica- for a few hours before bedtime. At times, nocturia can be
tion list of all elderly patients must be reviewed to insure lower caused by poor bladder emptying, and a postvoid residual
urinary tract signs or symptoms are not medication related.4 should be considered.

INCONTINENCE AND LUTS LOWER URINARY TRACT INFECTION

Urinary incontinence is not a normal part of aging. It can be The bladder’s basic line of defense against infection is a healthy
caused by a number of factors including medications, medical mucosa with low pressure storage and complete emptying of
comorbidities, and urologic pathology. A helpful mnemonic urine. Bacteria that may be introduced by poor hygiene, sexual
for the differential diagnosis of temporary or reversible incon- activity, and catheterization are normally flushed out of the
tinence is “DIAPERS”: delirium, infection/inflammation, bladder. When there is bladder obstruction or poor emptying,
atrophic vaginitis, polypharmacy, endocrine (diabetes), re- this defense mechanism is less effective and can lead to coloni-
stricted mobility, and stool impaction. There are four main zation or infection. The most common cause of urinary tract
types of urinary incontinence including urge, stress, overflow, infections (UTIs) in elderly patients are gram-negative organ-
and mixed. Urgency is a subjective feeling of a sudden need to isms. Residents of nursing facilities or patients who have had a
void, and it can be associated with incontinence. Causes in- long hospitalization are at an increased risk for multidrug-
clude infection, stones, medications, tumors, and neurologic resistant organisms such as pseudomonas and MRSA.
pathology. Stress incontinence is the failure of the sphincter to Bacteriuria can be either symptomatic or asymptomatic. All
remain closed during urine storage, because of an increase in patients with symptomatic bacteriuria should be treated with

2 Geriatric Nephrology Curriculum American Society of Nephrology


antibiotics. Patients will often experience burning with urina- der medications (i.e., anti-cholinergics) are helpful to mitigate
tion (dysuria), suprapubic discomfort, urinary frequency, and discomfort.
urgency. Elderly patients may have unique presentations in-
cluding incontinence, lethargy, anorexia, and altered mental
status.6 More severe cases develop fever, chills, nausea, and
vomiting. Patients with asymptomatic bacteriuria usually do INDICATIONS FOR URETHRAL CATHETERIZATION
not need treatment, depending on the clinical situation. Dia-
• Incontinence with open sacral/perineal wounds
betics, spinal cord injury patients, and patients that are immu- • Relieve urinary obstruction
nocompromised may not present with the normal symptoms • Accurate inputs and outputs in critical care
of an infection, and the decision to treat a positive culture is • Bladder dysfunction and urinary retention (i.e., neurogenic bladder)
left to the physician’s discretion. Risk factors for infection in- • Continuous bladder irrigation (bleeding or medication)
• Ease comfort of palliative or hospice care patient
clude prolonged catheterization, urinary tract anatomic abnor-
• Short-term use for surgery or procedure
malities, urinary retention (BPH), and comorbid diseases
(diabetes, immunosuppression). Urinary catheters should be In some patients, bladder-emptying dysfunction results in
avoided unless there is a clear indication (see below). chronic retention that may not be amenable to surgical or
medical treatment. In those patients, clean intermittent cathe-
terization is preferable to a chronic indwelling urinary cathe-
HEMATURIA ter. Severe chronic retention can result in obstructive uropathy
and hydronephrosis. Patients with retention or large postvoid
Blood in the urine is classified as either microscopic or gross, residuals should have renal function checked and hydrone-
depending on whether or not the patient has visibly red urine. phrosis ruled out if there is evidence of renal insufficiency.
Microscopic amounts of blood are commonly discovered on
urinalysis and can be from a variety of different etiologies in-
cluding infection, stones, renal disease, trauma, and cancer. If
the urine shows greater than three red blood cells per high- CONCLUSIONS
power field on two separate urinalyses, a hematuria work-up
should be undertaken. Basic laboratory work including a se- LUTS are ubiquitous to the elderly population. A simple his-
rum blood urea nitrogen (BUN) and creatinine should also be tory, physical exam, urinalysis, and postvoid residual are the
obtained. An elevated creatinine and proteinuria suggest med- basic components necessary to evaluate this patient popula-
ical renal disease. A work-up for hematuria includes imaging of tion. Most urinary conditions are self-limited and can be safely
the urinary tract, urine cytology, and cystoscopy. All three managed without the use of antibiotics or urinary catheters.
components are necessary because many small cancers are not With a basic understanding of lower urinary tract conditions,
visible on imaging. UTIs should be ruled out or treated before the primary physician will be able to safely manage many of
a hematuria work-up is started. these conditions and use prudent judgment in referral to a
urologist.

URINARY RETENTION AND CATHETERS


TAKE HOME POINTS
Indwelling Foley catheters are to be used as a temporary means
• LUTS are ubiquitous in the elderly population
of emptying the bladder for surgical patients, medical moni-
• Most urinary conditions are self-limited and can be safely managed
toring of urine output, or in those who cannot void on their without the use of antibiotics or urinary catheters
own. The Foley should be removed as soon as possible. In those • Clinical discretion is necessary to distinguish bacteriuria from urinary
patients who are in retention, the problem is often temporary. tract infection
The risk of bacterial colonization during prolonged indwelling • Hematuria has a variety of benign etiologies but if there is no obvious
source, urothelial cancer must be ruled out
catheterization is approximately 10% per day, meaning that
• Incontinence is not a normal part of the aging process and most causes
almost all patients that have a catheter for longer than a week are reversible
will be colonized. As stated above, unless patients are having • A patient’s bother score is the most useful tool when considering
symptoms, treatment is not necessary. If prolonged catheter- medication or surgery for BPH
ization is necessary, the catheter should be exchanged on a • A simple history, physical exam, urinalysis, and postvoid residual are the
basic components necessary to evaluate LUTS
monthly basis. Catheters can lead to urethral trauma, hematu-
ria, bladder spasms, infection, urethral erosion, and stricture
formation. Bladder spasms are caused by foreign body irrita-
tion of the bladder and usually resolve shortly after catheter DISCLOSURES
removal. In cases where a catheter is necessary, overactive blad- None.

American Society of Nephrology Geriatric Nephrology Curriculum 3


REFERENCES tom index for benign prostatic hyperplasia. J Urol 148: 1549 –1557,
1992
4. Reuben DB, et al.: Geriatrics at your fingertips. Available online at:
*Key References
http://www.geriatricsatyourfingertips.org*
1. Drach G: Fundamental issues in geriatric surgical care. American Uro-
5. Resnick NM, et al.: Geriatric incontinence and voiding dysfunction.
logic Association Plenary Session, Orlando, FL, May 2008
In: Campbell-Walsh Urology. 9th Ed, Philadelphia, Elsevier, 2008,
2. O’Donnell P: Geriatric Urology. New York, Little, Brown and Company,
pp 2305–2320*
1994*
6. Drach G, Forciea MA: Geriatric patient care: basics for urologists. AUA
3. Barry MJ, Fowler FJ Jr, O’Leary MP, Bruskewitz RC, Holtgrewe HL,
Update Series 24: 33, 2005
Mebust WK, Cockett AT: The American Urological Association symp-

4 Geriatric Nephrology Curriculum American Society of Nephrology


REVIEW QUESTIONS: LOWER URINARY TRACT b. Upper urinary tract imaging
CONDITIONS IN ELDERLY PATIENTS c. Urine cytology
d. All of the above
1. Operative intervention for benign prostatic hyperplasia is nec-
essary if: 4. True or false: a work up for hematuria is only necessary if there
a. AUA symptom score ⬎10 is gross hematuria.
b. The patient has a large prostate on exam a. True
c. Age ⬎80 yr b. False
d. The patient refuses medication
e. None of the above 5. Which of the following is not and indication for urethral cath-
eterization?
2. True or false: most cases of urinary incontinence are reversible.
a. Accurate intake/output reporting
a. True
b. Urinary tract infection
b. False
c. Urinary retention
3. A proper work up for hematuria will include d. Palliation
a. Cystoscopy e. Perioperative care

American Society of Nephrology Geriatric Nephrology Curriculum 5


Chapter 32: Urinary Tract Infections in Elderly Persons
Manisha Juthani-Mehta
Department of Internal Medicine, Section of Infectious Diseases, Yale University School of Medicine, New Haven,
Connecticut

ASYMPTOMATIC BACTERIURIA residents with long-term indwelling catheters, 100%


have ASB.5 Elderly persons with condom catheters
Because of physiologic changes related to aging and versus indwelling catheters have a lower incidence of
comorbid illnesses, asymptomatic bacteriuria ASB or urinary tract infections (UTIs).6
(ASB) is a common occurrence in older adults. Nei-
ther short-term nor long-term adverse outcomes Management
attributable to the high incidence and prevalence of For elderly persons, routine screening and treat-
ASB have been shown in this population. ment for ASB are not recommended. Based on rec-
ommendations of the Infectious Diseases Society of
Diagnosis America, screening for and/or treatment of ASB are
The diagnosis of ASB is based on the result of a not recommended for the following persons: (1)
urine culture from a urine specimen that minimizes diabetic women; (2) older persons living in the
contamination from a person without symptoms or community or institutionalized; (3) persons with
signs referable to urinary infection.1 For asymp- spinal cord injury; and (4) catheterized patients
tomatic women, bacteriuria is defined as two con- while the catheter remains in situ.2 Although a 3-d
secutive voided urine specimens with isolation of course of antibiotic therapy has been shown to de-
the same bacterial strain in quantitative counts crease the prevalence of bacteriuria at 6 mo,7 no
ⱖ105 colony forming units (cfu)/ml. For asymp- benefits in morbidity, mortality, and chronic uri-
tomatic men, bacteriuria is defined as a single, nary incontinence have been shown to date. Screen-
clean-catch voided urine specimen with one bacte- ing and treatment of ASB in older persons is only
rial species isolated in a quantitative count ⱖ105 recommended in the following two circumstances:
cfu/ml. For women and men, a single catheterized (1) before transurethral resection of the prostate
urine specimen with one bacterial species isolated and (2) before urologic procedures in which muco-
in a quantitative count ⱖ105 cfu/ml defines bacte- sal bleeding is anticipated.2 Although it has been
riuria.2 shown that nonurinary symptoms and signs are an
important factor in the prescription of antibiotics
Prevalence for ASB,8 there is no evidence to date to support this
Population studies throughout the world have shown practice.
a rise in the prevalence of asymptomatic bacteriuria
with age. Young women have a prevalence of ASB of 1 Prevention
to 2%. For women 65 to 90 yr of age, the prevalence of Few studies have been performed examining pre-
ASB ranges from 6 to 16%. The prevalence is highest vention strategies for ASB. In a randomized, dou-
for women over the age of 90, ranging from 22 to 43%. ble-blind, placebo-controlled trial in older female
ASB is very uncommon in young men, but for men community-dwelling and nursing home residents,
over the age of 65, the prevalence ranges from 5 to 21% cranberry juice reduced the frequency of bacteri-
and is highest in those men over the age of 90.3 Among uria plus pyuria in this population.9 However, be-
the institutionalized elderly, 25 to 50% of women and
15 to 35% of men have ASB. The prevalence is highest
Correspondence: Manisha Juthani-Mehta, Department of Inter-
among those most severely disabled. Up to 90% of nal Medicine, Section of Infectious Diseases, Yale University
institutionalized adults also have asymptomatic School of Medicine, PO Box 208022, New Haven, CT 06520.
pyuria (i.e., white blood cells in the urine in the ab- E-mail: [email protected]

sence of urinary tract–specific symptoms).4 For those Copyright 䊚 2009 by the American Society of Nephrology

American Society of Nephrology Geriatric Nephrology Curriculum 1


cause of several study design issues in this trial, the use of cran- Management
berry juice has not been fully advocated for the prevention of For community-dwelling older adults presenting to an acute
ASB. In a controlled trial of intravaginal estriol therapy in post- care hospital with presumed urosepsis, empiric therapy with a
menopausal women with recurrent UTI, the estriol group had third-generation cephalosporin is appropriate single agent
fewer episodes of ASB than the placebo group.10 However, this therapy until culture and susceptibility reports are available.
study was conducted specifically among women with a history Unless specific risk factors for a gram-positive infection are
of recurrent UTI. Intravaginal estriol therapy is not recom- identified (e.g., pressure sores, concomitant pneumonia), van-
mended for all postmenopausal women. In men, changes in comycin therapy is not empirically required.17 For outpatient
the bactericidal activity of prostate secretions has been re- oral therapy, nitrofurantoin, trimethoprim-sulfamethoxazole,
ported and proposed as one factor contributing to bacteriuria. and fluoroquinolones are appropriate first-line drugs for older
Last, avoiding long-term indwelling catheter use is optimal. If adults with UTIs.
possible, using a condom catheter provides more comfort to An appropriate first step in the evaluation of a UTI in insti-
the patient and fewer adverse outcomes.6 tutionalized older adults is performing a urinary dipstick.
Among nursing home residents with suspected UTIs, the neg-
ative predictive value of the urinary dipstick is 100%.18 Per-
forming this test in the nursing home setting obviates the need
URINARY TRACT INFECTIONS to outsource urine cultures and urinalyses. Clinical criteria for
empiric treatment of UTIs in institutionalized adults are also
UTIs are the second most common cause of infectious disease consensus based. In 2001, Loeb et al.19 recommended a mini-
hospitalization in adults 65 yr or older after lower respiratory mum set of clinical criteria necessary to initiate antibiotic ther-
tract infections.11 In 1998, UTIs were the most costly and re- apy for UTIs. As per these criteria, for residents who do not
source intensive condition, causing ⬎1.8 million physician of- have an indwelling catheter, minimum criteria for initiating
fice visits among Medicare beneficiaries. Total Medicare ex- antibiotics include acute dysuria alone or fever (⬎37.9°C
penditures for UTIs in all venues of care amounted to more [100°F] or 1.5°C [2.4°F] increase above baseline temperature)
than $1.4 billion, exclusive of medication costs.12 and at least one of the following: new or worsening urgency,
frequency, suprapubic pain, gross hematuria, costovertebral
Diagnosis angle tenderness, or urinary incontinence. For residents who
The diagnosis of UTI in community-dwelling older adults fol- have a chronic indwelling catheter, minimum criteria for ini-
lows a similar paradigm to the diagnosis of UTI in younger tiating antibiotics include the presence of at least one of the
adults, requiring significant bacteriuria (ⱖ105 cfu/ml) associ- following: fever (⬎37.9°C [100°F] or 1.5°C [2.4°F] increase
ated with genitourinary symptoms. In older adults that are above baseline temperature), new costovertebral angle tender-
cognitively intact and can report symptoms, the diagnosis of ness, rigors (shaking chills) with or without an identified cause,
UTI is easily made. However, among institutionalized older or new onset of delirium.
adults that are often cognitively impaired, distinguishing ASB
from UTI is often problematic. Prevention
In older institutionalized adults, multiple comorbid ill- Although cranberry capsule or juice administration is an appeal-
nesses may present with symptoms similar to UTI, and older ing prevention modality for UTIs because of its low side effect
adults with cognitive impairment may not be able to report profile and ease of administration, it has not been studied for the
their symptoms.13 Laboratory confirmation of UTIs with sig- purposes of preventing UTI in older adults. As such, no data exist
nificant bacteriuria (ⱖ105 cfu/ml on urine culture) and pyuria to date to show a benefit of long-term cranberry ingestion for the
(⬎10 white blood cells on urinalysis) is an agreed on minimum prevention of UTIs. Intravaginal estriol therapy in postmeno-
necessary but not sufficient criterion for diagnosis of UTI in pausal women with recurrent UTIs has been shown to decrease
this population.14 However, identifying symptoms that should the number of episodes of UTIs.10 Antibiotic prophylaxis is highly
be used for the diagnosis of UTI is challenging. Criteria for UTI effective at reducing the risk of recurrent UTIs in older women.
surveillance and diagnosis in nursing home residents have Continuous prophylaxis is recommended for women who expe-
been developed by infectious diseases consensus group recom- rience two or more symptomatic UTIs over a 6-mo period or
mendations.15 The only clinical features that have been associ- three or more over a 12-mo period, after an existing infection is
ated with bacteriuria plus pyuria in nursing home residents eradicated. Most experts recommend a 6-mo trial of a once
with clinically suspected UTIs are dysuria, change in mental nightly prophylactic agent, after which the regimen is discontin-
status (i.e., change in level of consciousness, periods of altered ued and the patient is observed for further infection. Some experts
perception, disorganized speech, or lethargy), and change in will advocate prophylaxis for up to 2 yr.20 Antimicrobial agents
character of urine (i.e., gross hematuria, change in color, or used for prophylaxis include trimethoprim-sulfamethoxazole, ni-
change in odor). Dysuria plus one of the other two clinical trofurantoin, and cefalexin.21
features predicts bacteriuria plus pyuria 63% of the time, sim- Other risk factors for recurrent UTIs have been identified in
ilar to what current consensus criteria have shown.16 older postmenopausal women; however, these risk factors have

2 Geriatric Nephrology Curriculum American Society of Nephrology


not been shown to be modifiable to date. In postmenopausal 5. Warren JW, Tenney JH, Hoopes JM, Muncie HL, Anthony WC: A
prospective microbiologic study of bacteriuria in patients with chronic
women, a history of UTI in the premenopausal period, incon- indwelling urethral catheters. J Infect Dis 146: 719 –723, 1982
tinence, presence of a cystocele, and postvoid residual urine 6. Saint S, Kaufman SR, Rogers MA, Baker PD, Ossenkop K, Lipsky BA:
predispose to UTIs. In institutionalized older adults, catheter- Condom versus indwelling urinary catheters: a randomized trial. J Am
ization, incontinence, antimicrobial exposure, and functional Geriatr Soc 54: 1055–1061, 2006
status are most strongly related to risk of recurrent UTIs.22 7. Boscia JA, Kobasa WD, Knight RA, Abrutyn E, Levison ME, Kaye D:
Therapy vs no therapy for bacteriuria in elderly ambulatory nonhospi-
Among older men, risk factors for UTIs include dementia, talized women. JAMA 257: 1067–1071, 1987
incontinence of bladder and bowel, and use of condom or in- 8. Walker S, McGeer A, Simor AE, Armstrong-Evans M, Loeb M: Why are
dwelling catheters.23,24 Condom catheters are preferable to in- antibiotics prescribed for asymptomatic bacteriuria in institutionalized
dwelling catheters; however, if an indwelling catheter is essen- elderly people? A qualitative study of physicians’ and nurses’ percep-
tial, staff should maintain a closed, dependent system to avoid tions. CMAJ 163: 273–277, 2000
9. Avorn J, Monane M, Gurwitz JH, Glynn RJ, Choodnovskiy I, Lipsitz LA:
introducing new organisms, be vigilant for the development of Reduction of bacteriuria and pyuria after ingestion of cranberry juice.
obstruction, and avoid trauma.25 Severe benign prostatic hy- JAMA 271: 751–754, 1994
pertrophy is often implicated as a risk for recurrent UTI. When 10. Raz R, Stamm WE: A controlled trial of intravaginal estriol in post-
possible, resection of the prostate can assist in reducing recur- menopausal women with recurrent urinary tract infections. N Engl
rent episodes. J Med 329: 753–756, 1993
11. Curns AT, Holman RC, Sejvar JJ, Owings MF, Schonberger LB:
Infectious disease hospitalizations among older adults in the United
States from 1990 through 2002. Arch Intern Med 165: 2514 –2520,
2005*
TAKE HOME POINTS 12. Litwin MS, Saigal CS, Beerbohm EM: The burden of urologic diseases
in America. J Urol 173: 1065–1066, 2005
• Asymptomatic bacteriuria is highly prevalent in elderly persons; screen- 13. Yoshikawa TT, Nicolle LE, Norman DC: Management of complicated
ing and treatment of asymptomatic bacteriuria is not recommended: urinary tract infection in older patients. J Am Geriatr Soc 44: 1235–
the only two circumstances in which they are recommended are (1) 1241, 1996
before transurethral resection of the prostate and (2) before urologic 14. Garner JS, Jarvis WR, Emori TG, Horan TC, Hughes JM: CDC defini-
procedures in which mucosal bleeding is anticipated tions for nosocomial infections, 1988. Am J Infect Control 16: 128 –
• Diagnosis of UTI in community-dwelling elderly persons is similar to 140, 1988
younger adults, requiring the presence of genitourinary symptoms; 15. McGeer A, Campbell B, Emori TG, Hierholzer WJ, Jackson MM,
however, among institutionalized adults, diagnosis is challenging: the Nicolle LE, Peppler C, Rivera A, Schollenberger DG, Simor AE: Defi-
only clinical features that have been associated with bacteriuria plus nitions of infection for surveillance in long-term care facilities. Am J
pyuria are dysuria, change in character of urine, and change in mental Infect Control 19: 1–7, 1991
status 16. Juthani-Mehta M, Quagliarello V, Perrelli E, Towle V, Van Ness P,
• Nitrofurantoin remains an effective outpatient oral therapy for UTI in Tinetti M: Clinical features to identify UTI in nursing home residents: a
elderly persons; for hospitalized elderly persons, a third-generation cohort study. J Am Geriatr Soc 2009, in press*
cephalosporin is an effective empiric treatment option for UTI 17. Ackermann RJ, Monroe PW: Bacteremic urinary tract infection in older
• There are few modifiable risk factors for asymptomatic bacteriuria or people. J Am Geriatr Soc 44: 927–933, 1996
UTI in elderly persons; cranberry capsules and intravaginal estriol re- 18. Juthani-Mehta M, Tinetti M, Perrelli E, Towle V, Quagliarello V:
main promising options Role of dipstick testing in the evaluation of urinary tract infection in
nursing home residents. Infect Control Hosp Epidemiol 28: 889 –
891, 2007
19. Loeb M, Bentley DW, Bradley S, Crossley K, Garibaldi R, Gantz N,
DISCLOSURES McGeer A, Muder RR, Mylotte J, Nicolle LE, Nurse B, Paton S, Simor
Reprinted in part from Clinics in Geriatric Medicine, Volume 23, Issue 3, AE, Smith P, Strausbaugh L: Development of minimum criteria for the
Manisha Juthani-Mehta, Asymptomatic Bacteriuria and Urinary Tract Infec- initiation of antibiotics in residents of long-term-care facilities: results
tion in Older Adults, pages 585–594, 2007, with permission from Elsevier. of a consensus conference. Infect Control Hosp Epidemiol 22: 120 –
124, 2001*
20. Juthani-Mehta M: Asymptomatic bacteriuria and urinary tract infection
in older adults. Clin Geriatr Med 23: 585–594, 2007*
21. Hooton TM: Recurrent urinary tract infection in women. Int J Antimi-
REFERENCES crob Agents 17: 259 –268, 2001
22. Stamm WE, Raz R: Factors contributing to susceptibility of postmeno-
*Key References pausal women to recurrent urinary tract infections. Clin Infect Dis 28:
1. Nicolle LE: Asymptomatic bacteriuria: when to screen and when to 723–725, 1999
treat. Infect Dis Clin North Am 17: 367–394, 2003 23. Nicolle LE, Henderson E, Bjornson J, McIntyre M, Harding GK, Mac-
2. Nicolle LE, Bradley S, Colgan R, Rice JC, Schaeffer A, Hooton TM: Donell JA: The association of bacteriuria with resident characteristics
Infectious Diseases Society of America guidelines for the diagnosis and survival in elderly institutionalized men. Ann Intern Med 106:
and treatment of asymptomatic bacteriuria in adults. Clin Infect Dis 40: 682– 686, 1987
643– 654, 2005* 24. Ouslander JG, Greengold B, Chen S: External catheter use and urinary
3. Nicolle LE: Asymptomatic bacteriuria in the elderly. Infect Dis Clin tract infections among incontinent male nursing home patients. J Am
North Am 11: 647– 662, 1997 Geriatr Soc 35: 1063–1070, 1987
4. Nicolle LE: Urinary tract infections in long-term-care facilities. Infect 25. Drinka PJ: Complications of chronic indwelling urinary catheters. J Am
Control Hosp Epidemiol 22: 167–175, 2001 Med Dir Assoc 7: 388 –392, 2006

American Society of Nephrology Geriatric Nephrology Curriculum 3


REVIEW QUESTIONS: URINARY TRACT INFECTIONS c. Has a spinal cord injury
IN ELDERLY PERSONS d. Has an indwelling catheter in place
1. It is appropriate to screen an elderly patient for asymptomatic 3. The presence of bacteriuria (⬎100,000 cfu/ml) plus pyuria
bacteriuria if the patient: (⬎10 WBC) is enough to make the diagnosis of UTI in an
a. Lives in a nursing home elderly patient. TRUE or FALSE?
b. Is older than 90 yr of age a. True
c. Has diabetes mellitus b. False
d. Is having a urologic procedure performed in which muco-
4. The urinary dipstick is most helpful in identifying a patient
sal bleeding is anticipated
that:
2. Treatment of asymptomatic bacteriuria is appropriate when a a. Does not have bacteriuria plus pyuria
patient: b. Deserves treatment for UTI
a. Has chronic incontinence c. Likely has bacteriuria
b. Is having a transurethral resection of the prostate d. Likely has pyuria

4 Geriatric Nephrology Curriculum American Society of Nephrology


Chapter 33: Falls in Elderly Patients With Kidney
Disease
John E. Morley
St. Louis University School of Medicine and GRECC, VA Medical Center, St. Louis, Missouri

Falls and associated fragility fractures are a major brain, e.g., pulmonary embolus, myocardial infarc-
cause of morbidity and mortality in older persons tion, anemia, stroke, seizures, dehydration, meta-
with kidney disease. In a longitudinal study from bolic abnormalities, and subdural hematoma (Ta-
one dialysis center for a median of 468 d, 47% fell.1 ble 4). Problems with lower limb strength and
The fall incident rate was 1.60 falls per year. Overall balance disorders are common in older dialysis pa-
studies suggest that the fall rate is much greater in tients and treatable with physical therapy. Drugs
dialysis patients than in the general population. In associated with falling are listed in Table 6. Studies
the general population over 75 yr of age, 30% of in older diabetics suggest that falls are reduced
persons fall each year, with one in five having an when HbA1C is not lowered below 7%.
injury. Hip fractures in persons on dialysis occur
three to four times more commonly than in the
general population. One-year mortality in dialysis BP ABNORMALITIES AND FALLS
patients who have a hip fracture is two to three
times of that in older community-dwelling persons Postural hypotension is a major cause of falls. It can
who have a fracture. A single fall in a dialysis patient occur without any dizziness. For this reason, BP
over 65 yr increases the risk of death after adjust- needs to be regularly measured in the standing po-
ment for comorbidities.2 In the first 2 yr after dial- sition. Orthostatic hypotension occurs more com-
ysis, renal transplant patients have a higher risk of monly in the morning, and in an individual with
fracture than patients on dialysis.3 Table 1 lists side severe orthostasis, it may only be present on one
effects of falls. half of the BP measurements. In a group of 23 el-
A community study of fall prevention in Con- derly on dialysis, orthostasis was present in 8 pa-
necticut showed that a simple education program tients before dialysis and 16 of 23 after dialysis.5
(focused on medication reduction and balance and Besides falls, orthostatic hypotension can lead to
gait training) could reduce falls and the need for syncope, myocardial infarction, stroke, and death.
fall-related medical services4 (www.fallsprevention. Causes of orthostatic hypotension include anticho-
org). Falls can be either caused by extrinsic (envi- linergic medications, anemia, prolonged recum-
ronmental) or intrinsic factors. Environmental fac- bency, dehydration, inadequate salt intake, protein
tors include wet, slippery floors, poor lighting, energy malnutrition, adrenal insufficiency, diabetic
uneven surfaces, and stairs. Descending stairs is a autonomic neuropathy, Parkinson’s disease, and
particular risk factor. There are multiple causes of multiple system atrophy (Shy-Drager syndrome).
falls caused by intrinsic factors as shown in Tables 2 Postprandial hypotension (a fall in BP of ⬎20
and 3.4 Specific dialysis-related causes of delirium mmHg) occurs in up to 25% of older persons and
include uremic encephalopathy, dialysis dementia, persons with diabetes. Its nadir is reached 1 to 2 h
Wernicke’s encephalopathy, and dialysis dysequi- after a meal. It is not necessarily associated with
librium. orthostasis. It has also been shown to be present
New onset falls are often caused by delirium. De- during dialysis in nondiabetic patients.6 Postpran-
lirium can present as purely the inability to pay at-
tention. Delirium should be considered as a cause
Correspondence: John E. Morley, MB, BCh, Division of Geriatric
of falling in any patient on dialysis who suddenly
Medicine, St. Louis University School of Medicine, 1402 S. Grand
starts falling. Delirium has multiple causes such as Boulevard, M238, St. Louis, MO 63104. E-mail: [email protected]
drugs, infection, active decrease in oxygenation to
Copyright 䊚 2009 by the American Society of Nephrology

American Society of Nephrology Geriatric Nephrology Curriculum 1


Table 1. Falls produce Table 3. Safe and sound mnemonic for renal disease–
Head injury associated falls
Lacerations Strength problems
Fractures Age
Hip Food associated hypotension
Vertebral Environmental factors
Other, e.g., Colles Atherosclerotic disease (syncope)
Soft tissue injuries Number of drugs
Fear of falling Diabetes
Decreased activity and functional decline Systolic blood pressure (low predialysis)
Will cost $55 billion by 2020 Orthostatic hypotension
Unsteady balance
No Vitamin D (⬍30 ng/ml)
dial hypotension is associated with falls, syncope, stroke, myo-
Dialysis disequilibrium or other causes of delirium
cardial infarction, and death. Treatment can consist of multi-
ple small meals with limited carbohydrate content. Because it is
caused by the release of a vasodilatory intestinal peptide, calci- Rarely is a diagnosis made when a person has a single syncopal
tonin gene–related peptide, it can be treated with somatostatin event. If a person has multiple syncopal events, they should be
analogs. Recent studies have shown that ␣-1-glucosidase in- given an event recorder that should be used.
hibitors, acarbose and miglitol, can markedly attenuate post-
prandial hypotension.
FEAR OF FALLING

LOSS OF CONSCIOUSNESS AND FALLS Many persons who fall or have disequilibrium develop a “fear
of falling.” Studies in older persons suggest that fear of falling
Persons who fall with loss of consciousness either have seizures puts persons at a marked increased risk of falls and other ad-
or syncope. More than one half of older persons with seizures verse outcomes.
have partial complex seizures (unusual behavior not necessar-
ily associated with toxic clonic seizures) explaining why it can
take nearly 2 yr to diagnose seizures in older persons. Older VITAMIN D
persons with syncope need to undergo carotid sinus massage
because those who develop bradycardia need a pacemaker. A 25(OH) vitamin D (calcidiol) level ⬍75 nmol/L (30 ng/ml)
has been identified as a cause of falls that responds to treatment
Table 2. Causes of falls with a reduction in falls.7 25(OH) Vitamin D deficiency is very
Extrinsic (environmental) common in renal failure patients.8 There is some evidence sug-
Uneven pavement gesting calcidiol is more effective than calcetriol.9
Poor lighting
No grab bar in toilet
Stairs cluttered FALLS IN DIALYSIS
Intrinsic
Low levels of 25(OH) vitamin D There is a paucity of studies examining factors associated with
Orthostasis
falls in dialysis patients. Cook et al.10 found that age, comor-
Postprandial hypotension
bidity, lower predialysis systolic BP, and a history of falls rep-
Medications
Poor vision
Poor balance Table 4. DELIRIUMS mnemonic for multiple causes of
Muscle weakness delirium
Gait problems Drugs/dialysis disequilibrium syndrome
Dementia (poor ability to “dual-task”) Emotional (depression and psychosis)
Depression Low PO2 states (pulmonary embolus, myocardial infarction, anemia,
Loss of consciousness and stroke)
Syncope Infection
Carotid sinus massage Retention of urine and feces
Event monitor Ictal or rejection
Seizures Uremic encephalopathy
Grand mal Metabolic (vitamin B12 deficiency, hypothyroidism, thiamine
Petit mal deficiency)
Partial complex Subdural hematoma

2 Geriatric Nephrology Curriculum American Society of Nephrology


Table 5. Treatment of orthostasis associated with markedly increased morbidity and mortality in
Eliminate medications patients on dialysis. The causes of fragility fractures are falls or
Adequate fluid and salt intake other trauma and bone disease. In the general population that
Treat anemia (if present, with erythopoetin) usually means osteoporosis (Table 7). In renal failure, the pic-
Elevate head of bed ture is complicated because of renal osteodystrophy. Osteodys-
Bedside commode trophies include osteomalacia caused by vitamin D deficiency,
Avoid hot showers osteitis fibrosa cystica caused by excess parathyroid hormone
Get up slowly
secretion, and adynamic bone disease caused by aluminum
Orthostatic exercises
toxicity and/or parathyroid hormone oversuppression. Low
Jobst stockings
Midodrine (alpha 1 adrenergic antagonist)
bone mineral density may be seen in either osteoporosis or in
Fludrocortisone osteodystrophy. Bone mineral loss in older men correlates with
elevated cystatin C and lower estimated GFR.13 Bone mineral
resented the major associated factors. Desmet et al.1 reported density may be abnormally elevated in the presence of aortic
that age, diabetes, increased number of drugs, antidepressant calcification. In dialysis patients, the major risk factors for fra-
drug use, and failing a walking test predicted falling in hemo- gility fracture are older age, female sex, diabetes mellitus, heart
dialysis patients. Angalakudi et al.11 found the following co- disease, longer periods on dialysis, race other than African
morbidities to be associated with an in-hospital fall in persons American, lower body mass index, and low parathyroid lev-
with chronic kidney disease (⬍60 ml/min GFR): dementia, els.14 In the Dialysis Outcomes and Practice Patterns Study,
pneumonia, gastrointestinal disease, diabetes, antidepressants, risk factors were older age, female gender, prior kidney trans-
and anticonvulsants. plant, low serum albumin, selective serotonin reuptake inhib-
itors, narcotics, benzodiazepines, adrenal steroids, and very
high parathyroid hormone levels.15 Levels of PTH between 600
TREATMENT
and 900 pg/ml seemed to have the lowest risk.
The use of bisphosphonates in this population is unclear. In
The Cochrane Collaboration examined 62 trials on interven- persons who do not have adynamic bone disease or hyperpara-
tions to reduce fall risk.12 Multidisciplinary, multifactorial thyroidism and normal 25(OH) vitamin D levels, bisphospho-
programs reduced falls in community-dwelling older persons nates at lower doses can be considered. In most cases, a bone
(RR ⫽ 0.73; range ⫽ 0.63 to 0.85) and residential care facilities biopsy with tetracycline labeling would be considered neces-
(RR ⫽ 0.60; range ⫽ 0.50 to 0.73). Programs that had high sary before using bisphosphonates to not aggravate adynamic
efficacy at reducing falls were muscle strengthening and bal- bone disease, although it is feasible that bone markers could be
ance retraining; home hazard assessment and modifications; used in persons where no exposure to aluminum exists.16 Low
withdrawal of psychotropic medications; Tai Chi exercise in- levels of osteocalcin and bone alkaline phosphatase suggest
tervention; and cardiac pacing for falls with cardio-inhibitory adynamic bone.
carotid sinus hypersensitivity (Tables 5 and 6). Hypogonadism is common in older males and in renal fail-
All persons who fall should be enrolled in a home fall pre- ure patients. In males with a low free testosterone (either cal-
vention program. This is reimbursed by Medicare Part A if culated or measured by dialysis) (see www.issam.ch) or bio-
homebound and Medicare Part B if not.
available testosterone (albumin bound and free), the use of
testosterone, which should both increase osteoblastic activity
FRAGILITY FRACTURES and muscle strength, can be considered. Total testosterone
should not be used in view of elevated levels of sex hormone
Fragility fractures (fractures associated with minor trauma) are binding globulin in older persons.
Table 6. Medications associated with falls
Cardiovascular drugs Table 7. Risk factors for osteoporosis
Nitrates Low calcium intake
Antihypertensives Seizure medications (anticonvulsants)
Diuretics Thin build
Antiarrythmics Ethanol (excess alcohol)
Anticonvulsants Hypogonadism
Antidepressants Prior fracture
Sedative/hypnotics Thyroid excess
NSAIDs Race (Caucasian/Asian)
Propoxyphene Other relatives with osteoporosis/fractures
Antipsychotics Steroids
Dopamine agonists Inactivity
(used for restless legs syndrome in dialysis) Smoking

American Society of Nephrology Geriatric Nephrology Curriculum 3


Raloxifene increases bone markers of osteoblastic activity in DISCLOSURES
patients on hemodialysis and thus could be used to treat osteo- None.
porosis in women on dialysis.
After renal transplantation, there is a rapid decrease in bone REFERENCES
mineral density over the first year. This decrease in bone is
associated with increased risk of fractures. The causes of this *Key References
increased loss of bone include renal osteodystrophy, glucocor- 1. Desmet C, Beguin C, Swine C, Jadoul M: Falls in hemodialysis pa-
ticoids, immunotherapy, vitamin D deficiency, hypophos- tients: prospective study of incidence, risk factors, and complications.
phatemia, hypogonadism, and osteoporosis. Bisphosphonates Am J Kidney Dis 45: 148 –153, 2005
2. Li M, Tomlinson G, Naglie G, Cook WL, Jassal SV: Geriatric comor-
(oral and intravenous), vitamin D, and calcitonin have all been bidities, such as falls, confer an independent mortality risk to elderly
shown to slow the rate of bone loss.17 There are inadequate dialysis patients. Nephrol Dial Transplant 23: 1396 –1400, 2008
data to conclude what are their effects on fracture prevention. 3. Ball AM, Gillen DL, Sherrard D, Weiss NS, Emerson SS, Seliger SL,
After a hip fracture, a number of factors have been shown to Kestenbaum BR, Stehman-Breen C: Risk of hip fracture among dialysis
improve outcome in older persons: surgery within 48 h de- and renal transplant recipients. JAMA 288: 3014 –3018, 2002*
4. Morley JE: Falls and fractures. J Am Med Dir Assoc 8: 276 –278, 2007*
creases length of hospitalization and possibly mortality; sur- 5. Roberts RG, Kenny RA, Brierley EJ: Are elderly hemodialysis patients
gery delay for cardiology clearance may increase mortality; bet- at risk of falls and postural hypotension? Int Urol Nephrol 35: 415– 421,
ter outcomes are obtained with spinal or epidural anesthesia; 2003
early mobilization enhances rehabilitation outcomes; delirium 6. Sherman RA, Torres F, Cody RP: Postprandial blood pressure changes
reduces functional outcomes and it is reduced by a geriatric during hemodialysis. Am J Kidney Dis 12: 37–39, 1988*
7. Bischoff-Ferrari HA, Dawson-Hughes B, Willett WC, Staehelin HB,
consult; vitamin D (calcidiol or calcitriol) will improve out- Bazemore MG, Zee RY, Wong JB: Effect of Vitamin D on falls: a
comes; and the use of bisphosphonates depends on the type of meta-analysis. JAMA 291: 1999 –2006, 2004
bone disease. 8. Blair D, Byham-Gray L, Lewis E, McCaffrey S: Prevalence of vitamin D
[25(OH)D] deficiency and effects of supplementation with ergocalcif-
erol (vitamin D2) in stage 5 chronic kidney disease patients. J Ren Nutr
18: 375–382, 2008
9. O’Donnell S, Moher D, Thomas K, Hanley DA, Cranney A: Systematic
CONCLUSIONS review of the benefits and harms of calcitriol and alfacalcidol for
fractures and falls. J Bone Miner Metab 26: 531–542, 2008
Falls are extremely common in patients with end-stage renal 10. Cook WL, Tomlinson G, Donaldson M, Markowitz SN, Naglie G,
failure on dialysis. Fall prevention programs that include a Sobolev B, Jassal SV: Falls and fall-related injuries in older dialysis
patients. Clin J Am Soc Nephrol 1: 1197–1204, 2006*
medication review, prevention of orthostasis, cardiac pacing 11. Angalakuditi MV, Gomes J, Coley KC: Impact of drug use and comor-
where appropriate, home hazard assessment and modifica- bidities on in-hospital falls in patients with chronic kidney disease. Ann
tions, and muscle strengthening and retraining are effective. In Pharmacother 41: 1638 –1643, 2007
addition, making sure the 25(OH) vitamin D level is ⬎30 12. McClure R, Turner C, Peel N, Spinks A, Eakin E, Hughes K: Population-
ng/ml has been found to be effective. based interventions for the pr3evention of fall-related injuries in older
people. Cochrane Data Sys Rev 1: CD004441, 2005
Treatment of bone disease is complicated because of the 13. Ishani A, Paudel M, Taylor BC, Barrett-Connor E, Jamal S, Canales M,
coexistence of osteoporosis and renal osteodystrophy. In per- Steffes M, Fink HA, Orwoll E, Cummings SR, Ensrud KE; Osteoporotic
sons without adynamic bone disease, low-dose bisphospho- Fractures in Men (MrOS) Study Group: Renal function and rate of hip
nates are possibly indicated. There may be a role for testoster- bone loss in older men: the Osteoporotic Fractures in Men Study.
one in males and raloxifene in females. Osteoporos Int 19: 1549 –1556, 2008*
14. Kaneko TM, Foley RN, Gilbertson DT, Collins AJ: Clinical epidemiol-
ogy of long-bone fractures in patients receiving hemodialysis. Clin
Orthopeadics Rel Res 457: 188 –193, 2007
15. Jadoul M, Albert JM, Akiba T, Akizawa T, Arab L, Bragg-Gresham JL,
TAKE HOME POINTS Mason N, Prutz KG, Young EW, Pisoni RL: Incidence and risk factors
for hip or other bone fractures among hemodialysis patients in the
• Fall prevention programs should be instituted for all older dialysis Dialysis Outcomes and Practice Patterns Study. Kidney Int 70: 1358 –
patients 1366, 2006*
• All older kidney failure patients should have 25(OH) vitamin D levels 16. Miller PD: Is there a role for bisphosphonates in chronic kidney dis-
replaced to at least 30 ng/ml ease? Semin Dial 20: 186 –190, 2007*
• Bisphosphonates, testosterone in males, and raloxifene in females can 17. Palmer SC, McGregor DO, Strippoli GF: Intervention for preventing
be used to treat bone disease as long as the type of disease is under- bone disease in kidney transplant recipients. Cochrane Database Syst
stood Rev 18: CD005015, 2007

4 Geriatric Nephrology Curriculum American Society of Nephrology


REVIEW QUESTIONS: FALLS IN ELDERLY PATIENTS a. Midodrine
WITH KIDNEY DISEASE b. Bromergocriptine
c. Propoxyphene
1. To prevent falls, 25(OH) vitamin D levels should be greater
d. Miglitol
than
e. Fludrocortisone
a. 10 ng/ml
b. 15 ng/ml
3. To determine if there is adynamic bone measure
c. 20 ng/ml
a. PTH
d. 25 ng/ml
b. Osteocalcin
e. 30 ng/ml
c. Testosterone
2. Which of the following drugs can be used to treat postprandial d. 25(OH) vitamin D
hypotension? e. Vitamin K

American Society of Nephrology Geriatric Nephrology Curriculum 5


Chapter 34: Association Between CKD and Frailty and
Prevention of Functional Losses
Emaad Abdel-Rahman
Department of Internal Medicine, Division of Nephrology, University of Virginia, Charlottesville, Virginia

The incidence and prevalence of chronic kidney portant concept because it is believed that interven-
disease (CKD) is increasing worldwide and more so tion at that stage of prefrailty may reverse the frailty
in the elderly persons. Among individuals 70 yr of syndrome.9
age or older, the prevalence of CKD increased from Regardless of the definition, it is important to
38% in 1988 to 1994 to 47% in 1999 to 2004.1 This realize two main things. The first is that frailty de-
noted increase in the number of elderly patients velops slowly and that it may be initiated by differ-
with CKD has been associated with new challenges ent factors such as lack of activity, inadequate nu-
where CKD has become increasingly recognized as tritional intake, stress, or triggered by a disease
an important comorbid condition in elderly indi- process such as CKD. The second is that the pres-
viduals leading to death, cardiovascular events, and ence of frailty is associated with several adverse
hospitalizations.2 health outcomes such as functional decline [wors-
Recently, less recognized challenges, such as ening mobility, activities of daily living (ADL) dis-
functional impairment and frailty, became more ability, recurrent falls, hip and nonspine fractures],
apparent as factors that affect the quality of life and hospitalization, and death.8,10 –12 The recognition of
outcomes in the elderly patients with CKD.3– 6 Fried these two issues highlight the importance of early
et al.5 described an association between CKD on identification of the problem, hopefully at the pre-
one hand, and the development of functional im- frailty period, and intervening in such a way to min-
pairment on the other. Roderick et al.7 further imize the poor outcomes that are associated with
showed that the strength of the association with the frailty syndrome.
measures of morbidity and functional impairment
increase as renal function worsened. Identifying the CKD and Frailty
predisposing factors of frailty in these elderly pa- Background.
tients with CKD, to help prevent and manage it, is CKD has long been recognized to be associated with
being recognized more as crucial to improve their various comorbidities. Rocco et al.13showed more
quality of life and their longevity. than a decade ago that patients with moderate to
advanced CKD have a reduced quality of life and an
increased frequency and severity of poor symptoms
FRAILTY such as tiring easily, weakness, lack of energy, diffi-
culty sleeping, muscle cramps, and easy bruising, as
well as psychologic distress. Subsequently, Shidler
Definition
et al.14 showed higher negative perception of illness
Frailty is a term that has been used for some time
to be associated with higher depression scores and
with variable definitions. Earlier definitions were
lower quality of life, even at early stages of CKD.
vague and described an elderly patient who is lack-
Shlipak et al.3 expanded further on that concept
ing general strength and is susceptible to diseases.
to show that CKD is also associated with poor func-
Fried et al.8 suggested a more precise and standard
definition in which three or more of five compo-
nents (Table 1) would define frailty, whereas the
Correspondence: Emaad Abdel-Rahman, Department of Internal
presence of one or two of these frailty components Medicine, Division of Nephrology, University of Virginia, Box
would be termed “prefrailty.” Prefrail elderly per- 800133, Charlottesville, VA 22908. Phone: 434-243-2671; Fax:
sons are more likely than nonfrail elderly persons to 434-924-5848; E-mail: [email protected]

develop the whole frailty syndrome.8 This is an im- Copyright 䊚 2009 by the American Society of Nephrology

American Society of Nephrology Geriatric Nephrology Curriculum 1


Table 1. Definition of frailty patients exhibited a high prevalence of frailty in association
Unintentional weight loss (10 lbs or more in a year) with elevated inflammatory markers.
Self-reported exhaustion
Weakness (measured by grip strength) Hormonal Changes.
Slow walking speed As we age, several changes occur in the hormonal levels and
Low physical activity activities, and these hormonal changes are prevalent in patients
with CKD and may act as contributing factors in the patho-
tional status. In their study, Shlipak et al. showed that CKD is physiology of frailty noted in the elderly patients with CKD.18
associated with a greater prevalence of both frailty and disabil- The levels of both the testosterone hormone levels and vi-
ity in the elderly population, with the prevalence of frailty and tamin D decline19 with aging and with CKD, and this decline
disability being 15 and 12%, respectively, in elderly with CKD, has been noted to be associated with decrease in muscle mass
versus only 6 and 7% in the elderly with normal kidney func- and strength, as well as poor function in older patients.20 Other
tion, respectively. Similarly, frailty was highly prevalent in pa- endocrinological abnormalities occur with aging and with
tients with end-stage kidney disease (ESKD) on hemodialysis CKD such as hypo- and hyperthyroidism and diabetes melli-
(HD).6 It increased more as a function of aging. The propor- tus20 and are also associated with muscle strength and func-
tion of frailty gradually increased in patients with ESKD as they tional decline.
aged from 44.4% in patients younger than 40 yr to 66.4% in Although growth hormone (GH) levels decrease with aging,
patients 50 to 60 yr of age to 78.8% in ESKD patients older than its basal levels and the spontaneous secretion of endogenous
80 yr. Frailty was also noted to be independently associated GH in patients with CKD are either normal or even increased;
with high risk of death and high risk of the combined outcome however, GH action is decreased in these patients with CKD
of death and hospitalization in these patients with ESKD on because of insensitivity to GH.21,22 Giving GH to elderly pa-
HD.6 tients on chronic HD improved muscle performance, suggest-
ing a role of GH in preserving muscle function.23 Similarly,
Mechanism of Frailty in Elderly Patients With CKD. thyroid hormonal abnormalities are prevalent in the elderly24
Frailty is not synonymous with aging; in fact, only 3 to 7% of and in patients with renal impairment25,26 and are associated
elderly persons between the ages of 65 to 75 yr are frail.9 Shuur- with frequent comorbidities27 that can play a role in the noted
mans et al.15 hypothesized that the concept of frailty is more poor outcomes.
predictive of an outcome than the chronological age per se,
suggesting that frailty may be a better criterion to identify older Malnutrition.
persons in need for intervention. Inadequate food intake is a common finding in elderly persons
Several factors have been suggested to predispose to frailty and more so in uremic patients. Alteration in taste, smell, and
in elderly patients in general and more so in elderly patients mental status often found in patients with CKD can lead to
with CKD, including the following (Table 2). malnutrition. This age-related reduction in food intake, accel-
erated further in uremic individuals, can lead to further weight
Increased Inflammatory and Procoagulant Factors. loss together with an increased loss of muscle mass and
Walston et al.16 showed that frailty in elderly was associated strength causing sarcopenia.28 Advanced sarcopenia, associ-
with increased inflammatory and procoagulant markers and ated with malnutrition, is synonymous with physical frailty
that this increase was independent of the disease status. Similar and is estimated to affect 30% of people older than 60 yr and
results were found in elderly patients with CKD, where renal may affect ⬎50% of those older than 80 yr.29
insufficiency was independently associated with elevations in
inflammatory and procoagulant biomarkers.17Taking these re- Anemia.
sults together, it is suggested that CKD in elderly patients, Anemia is highly prevalent in elderly persons30 and in patients
through its effects on inflammatory and procoagulant mark- with CKD31 and has been shown to be associated with mobility
ers, may contribute to the frailty noted in these patients. This dysfunction, physical limitation, and reduced exercise capac-
was further suggested by a pilot study conducted by Swidler et ity.32,33 The Women’s Health and Aging Studies I and II
al.4 on 26 elderly patients with CKD, who showed that these (WHAS) identified anemia as a risk factor for frailty.34 Taking

Table 2. Mechanisms and management of frailty


Mechanisms of Frailty Management of Frailty
Increase inflammatory markers Anti-inflammatory/ACEI
Anemia Erythropoeisis stimulating agents
Anorexia, nausea3poor nutrition, weight loss, sarcopenia Nutritional support
Hormonal changes Hormonal replacement
Decrease muscle strength Exercise/Tai Chi
Depression Psychotherapy/antidepressants

2 Geriatric Nephrology Curriculum American Society of Nephrology


these studies together, anemia is suggested to be a factor pre- Hormonal Replacement.
disposing to frailty in elderly patients with CKD. To combat the progression of sarcopenia, several intervention
strategies including replacement or even supplementation doses
Depression. of hormones such as testosterone,55 GH,56, insulin-like growth
Depression is highly prevalent in patients with CKD,35 occur- factor (IGF),57 and dehydroepiandrosterone (DHEA)58 were
ring in 20 to 30% of these patients.36 It correlates with both used.
hospitalization and mortality rates37 and has been associated Testosterone replacement studies have shown an increase
with frailty.3 in muscle mass in males and improved function in elderly after
hospitalization.19 Similarly, high doses of DHEA improved
Management. muscle strength in males.19 GH treatment to elderly chronic
Several studies have investigated the effects of intervention on HD patients produced anabolic effects, with improved muscle
functional decline in elderly population with various re- performance and better nutritional status,23 increased lean
sults.39 – 42 The first approach in management of elderly pa- body mass at all dosage levels of GH, and improved the quality
tients with CKD is to rule out any underlying disease that could of life.59
be contributing to the symptoms of frailty such as an occult
malignancy, neglect, and alcohol abuse. Identifying patients at Anemia.
the prefrailty stage is very crucial to delay or even prevent fur- Patients treated with recombinant human erythropoietin had
ther deteriorations. The following measures were suggested for a better quality of life,60 significant increases in energy and
management of both prefrailty and frailty stages (Table 2). physical function,61 significant improvement in work capacity,
and an increase in measured exercise capacity.62Thus, identi-
Exercise fying and treating anemia may contribute in improving symp-
The beneficial effects of exercise in the older population toms of frailty in elderly patients with CKD.
have been well documented. Exercise may prevent or slow
down the decline in physical functioning.43,44 Interestingly, Depression.
it was noted that elderly who are physical active45 and those Recent reports have suggested that depressive symptoms can
who undergo resistance exercise46 have lower levels of inflam- be ameliorated by psychiatric counseling, cognitive-behavioral
matory markers, suggesting an anti-inflammatory component therapy, and/or medications.63,64 Good psychologic health, in-
to exercise. cluding positive affect, was found to significantly lower the risk
Studies in the field of renal rehabilitation have shown that of frailty.41 Thus, identifying symptoms of depression by
exercise training in HD patients improves the aerobic capacity screening tests as the Geriatric Depression Scale (GDS) and
and muscle strength.47 Specific types of physical activity have adequately managing depression by therapy and medication
been shown to help reduce frailty.48 Tai Chi exercises has been can be beneficial in the prevention of frailty. However, the
used in China for centuries and consist of a number of stylized treatment of clinical depression in dialysis patients presents
movements in which practitioners continuously change pos- various obstacles. First, it is difficult to encourage patients to
ture. These movements address weakness, slow walking speed, pursue and complete an appropriate evaluation36; in addition,
and low physical activity, three of the five frailty components. many patients are reluctant to take additional medications, the
medications themselves may have side effects, and the dialysis
Nutrition. regimen itself imposes time constraints on patients’ willing-
Malnutrition is common in patients with CKD49 and is associ- ness to obtain appropriate counseling.
ated with low muscle mass and sarcopenia.50 Managing mal-
nutrition and supplementation of vitamin D are promising Future Directives.
means to alleviate some of the components of frailty. This is Several other drugs might have a role in the treatment of frailty
more important in patients with CKD who have decreased and are currently being studied. These include anti-inflamma-
levels of active vitamin D. Vitamin D was shown to improve tory and anabolic agents, psycho-stimulants, and selective an-
balance, reduce falls, improve quadriceps strength, and to pre- drogen receptor modulators
dict better neuromuscular performances.51,52
Of interest, chronic inflammation, which is common in pa-
tients with CKD,4 may contribute to the malnutrition noted in
these patients.50 Hence, new treatment strategies targeting in- CONCLUSIONS
flammatory markers may be beneficial to improve the malnu-
trition and frailty. In this respect, angiotensin converting en- Frailty, an important problem in geriatric medicine, is becom-
zyme inhibitors (ACEIs) have been shown to be associated ing more recognized in patients with CKD, as well as patients
with a better nutritional status and lower levels of inflamma- with ESKD, as a factor that can even worsen the prognosis and
tory markers in patients with CKD,53 as well as improving the quality of life of these patients. It is imperative for the
muscle strength.54 nephrology community, in lieu of the rising numbers of elderly

American Society of Nephrology Geriatric Nephrology Curriculum 3


patients being seen by nephrologists, to recognize this syn- Women’s Health Initiative observational study. J Am Geriatr Soc 53:
1321–1330, 2005
drome of frailty and work diligently to manage it. 11. Walston J, Hadley EC, Ferrucci L, Guralnik JM, Newman AB, Studenski
SA, Ershler WB, Harris T, Fried LP: Research agenda for frailty in older
adults: toward a better understanding of physiology and etiology:
summary from the American Geriatrics Society/National Institute on
TAKE HOME POINTS Aging Research Conference on Frailty in Older Adults. J Am Geriatr
Soc 54: 991–1001, 2006
• Frailty is a prevalent and a newly recognized challenge to patients with
12. Ensrud KE, Ewing SK, Taylor BC: Frailty and risk of falls, fracture, and
CKD that affects the quality of life and outcomes in elderly patients with
mortality in older women: the Study of Osteoporotic Fractures. J
CKD
Gerontol Med Sci 62: 744 –751, 2007
• Frailty develops slowly and its presence is associated with several
13. Rocco MV, Gassman JJ, Wang SR, Kaplan RM: Cross-sectional study
adverse health outcomes as functional decline, hospitalization, and
of quality of life and symptoms in chronic renal disease patients: the
death
Modification of Diet in Renal Disease Study. Am J Kidney Dis 129:
• Strength of association with measures of morbidity and functional
888 – 896, 1997
impairment increase as renal function worsens
14. Shidler NR, Peterson RA, Kimmel PL: Quality of life and psychological
• Several mechanisms were suggested to explain the association be-
relationships in patients with chronic renal insufficiency. Am J Kidney
tween frailty and CKD such as the presence of inflammatory markers,
Dis 32:557–566, 1998
anemia, malnutrition, hormonal changes, depression, and decreased
15. Schuurmans H, Steverink N, Lindenberg S, Frieswijk N, Slaets JP: Old
muscle strength
or frail: what tells us more? J Gerontol Biol Sci Med Sci 59A: 962–965,
• Identifying and treating the underlying mechanisms at the prefrailty
2004
stage can help delay or even prevent further deteriorations
16. Walston J, McBurnie MA, Newman A: Frailty and activation of the
inflammation and coagulation systems with and without clinical co-
morbidities: results from the Cardiovascular Health Study. Arch Intern
Med 162: 2333–2341, 2002*
DISCLOSURES
17. Shlipak MG, Fried LF, Crump C, Bleyer AJ, Manolio TA, Tracy RP,
None.
Furberg CD, Psaty BM: Elevations of inflammatory and procoagulant
biomarkers in elderly persons with renal insufficiency. Circulation 107:
87–92, 2003*
18. Gupta D, Bundschu HD: Testosterone and its binding in the plasma of
REFERENCES male subjects with chronic renal failure. Clin Chim Acta 36: 479 – 484,
1972
*Key References 19. Morley JE, Kim MJ, Haren MT: Frailty and hormones. Rev Endo Met
1. Coresh J, Selvin E, Stevens LA, Manzi J, Kusek JW, Eggers P, Van Dis 6: 101–108, 2005
Lente F, Levey AS: Prevalence of chronic kidney disease in the United 20. Valenti G: Frailty as a dysruption of steroid “syncrinology” in elderly
States, JAMA 298: 2038 –2047, 2007 man. Acta Biomed 78: 222–224, 2007
2. Go AS, Chertow GM, Fan D, McCulloch CE, Hsu CY: Chronic kidney 21. Schaefer F, Chen Y, Tsao T, Nouri P, Rabkin R: Impaired JAK-STAT
disease and the risks of death, cardiovascular events, and hospitaliza- signal transduction contributes to growth hormone resistance in
tion. N Engl J Med 351: 1296 –1305, 2004 chronic uremia. J Clin Invest 108: 467– 475, 2001
3. Shlipak MG, Stehman-Breen C, Fried LF, Song X, Siscovick D, Fried LP, 22. Mak RHK, Pak YK: End-organ resistance to growth hormone and IGF-I
Psaty BM, Newman AB: The presence of frailty in elderly persons with in epiphyseal chondrocytes of rats with chronic renal failure. Kidney Int
chronic renal insufficiency. Am J Kidney Dis 43: 861– 867, 2004* 50: 400 – 406, 1996
4. Swidler MA, Uribarri J, Siu A, Viassara H: Characterization of frailty in 23. Johannsson G, Bengtsson BA, Ahlmen J: Double-blind, placebo con-
older adults with chronic kidney disease (CKD): correlations with trolled study of growth hormone treatment in elderly patients under-
biomarkers of inflammation. J Am Soc Nephrol 16: 813, 2005* going chronic hemodialysis: anabolic effect and functional improve-
5. Fried LF, Lee JS, Shlipak M, Chertow GM, Green C, Ding J, Harris T, ment. Am J Kidney Dis 33: 709 –717, 1999
Newman AB: Chronic kidney disease and functional limitation in older 24. Chiovato L, Mariotti S, Pinchera A: Thyroid diseases in the elderly.
people: Health, Aging and Body Composition Study. J Am Geriatr Soc Baillieres Clin Endocrinol Metab 11: 251–270, 1997
54: 750 –756, 2006 25. Lo JC, Chertow GM, Go AS, Hsu CY: Increased prevalence of subclin-
6. Johansen KL, Chertow GM: Significance of frailty among dialysis pa- ical and clinical hypothyroidism in persons with chronic kidney dis-
tients. J Am Soc Nephrol 2007;18:2960 –2967* ease. Kidney Int 67: 1047–1052, 2005
7. Roderick PJ, Atkins RJ, Smeeth L, Nitsch DM, Hubbard RB, Flectcher 26. Chonchol M, Lippi G, Salvagno G, Zoppini G, Muggeo M, Targher G:
AE, Bulpitt CJ: Detecting chronic kidney disease in older people; what Prevalence of subclinical hypothyroidism in patients with chronic kid-
are the implications? Age Ageing 37: 179 –186, 2008 ney disease Clin J Am Soc Nephrol 3: 1296 –1300, 2008
8. Fried LP, Tangen CM, Walston J, Newman AB, Hirsch C, Gottdiener J, 27. Morley JE: Hormones and the aging process. J Am Geriatr Soc 51(7
Seeman T, Tracy R, Kop WJ, Burke G, McBurnie MA; Cardiovascular Suppl): S333–S337, 2003
Health Study Collaborative Research Group: Frailty in older adults: 28. Bales CW, Ritchie CS: Sarcopenia, weight loss, and nutritional frailty in
evidence for a phenotype. J Gerontol Med Sci 56A: M146 –M156, the elderly. Annu Rev Nutr 22: 309 –323, 2002
2001 29. Baumgartner RN, Koehler KM, Gallagher D, Romero L, Heymsfield SB,
9. Fried LP, Darer J, Walston J: Frailty. In: Geriatric Medicine: An Evi- Ross RR, Garry PJ, Lindeman RD: Epidemiology of sarcopenia among
dence-Based Approach, 4th Ed., edited by Cassel C, Leipzig R, Cohen the elderly in New Mexico. Am J Epidemiol 147: 755–763, 1998
H, Larson EB, Meier D, New York: Springer-Verlag, 2003, pp 1067– 30. Balducci L: Epidemiology of anemia in the elderly: information on
1074 diagnostic evaluation. J Am Geriatr Soc 51(3 Suppl): S2–S9, 2003
10. Woods NF, LaCroix AZ, Gray SL, Aragaki A, Cochrane BB, Brunner RL, 31. McClellan W, Aronoff SL, Bolton WK, Hood S, Lorber DL, Tang KL, Tse
Masaki K, Murray A, Newman AB; Women’s Health Initiative: Frailty: TF, Wasserman B, Leiserowitz M: The prevalence of anemia in patients
emergence and consequences in women aged 65 and older in the with chronic kidney disease. Curr Med Res Opin 20: 1501–1510, 2004

4 Geriatric Nephrology Curriculum American Society of Nephrology


32. Chaves PHM, Ashar B, Fried LP: Looking at the relationship between 48. Wolf SL, Sattin RW, O’Grady M, Freret N, Ricci L, Greenspan AI, Xu T,
hemoglobin concentration and prevalent mobility difficulty in older Kutner M: Study design to investigate the effect of intense Tai Chi in
women: should the criteria currently used to define anemia in older reducing falls among older adults transitioning to frailty. Cont Clin
women in the elderly be evaluated? J Am Geriatr Soc 50: 1257–1264, Trials 22: 689 –704, 2001
2002 49. Ikizler T, Hakim RM: Nutrition in end-stage renal disease. Perspectives
33. Odden MC, Whooley MA, Shlipak MG: Association of chronic kidney in clinical nephrology. Kidney Int 50: 343–357, 1996
disease and anemia with physical capacity: the Heart and Soul Study. 50. Stenvinkel P, Heimbürger O, Lindholm B: Are there two types of
J Am Soc Nephrol 15: 2908 –2915, 2004 malnutrition in chronic renal failure? Evidence for relationships be-
34. Chaves PH, Semba RD, Leng SX, Woodman RC, Ferrucci L, Guralnik tween malnutrition, inflammation and atherosclerosis (MIA syndrome).
JM, Fried LP: Impact of anemia and cardiovascular disease on frailty Nephrol Dial Transplant 15: 953–960, 2000
status of community dwelling older women: the Women’s Health and 51. Larsen ER, Mosekilde L, Foldspang A: Vitamin D and calcium supple-
Aging Studies I and II. J Gerontol A Biol Sci Med Aci 60: 729 –735, mentation prevents severe falls in elderly community dwelling women:
2005 a pragmatic population based 3-year intervention study. Aging Clin
35. Hedayati SS, Jiang W, O’Connor CM, Kuchibhatla M, Krishnan KR, Exp Res 17: 125–132, 2005
Cuffe MS, Blazing MA, Szczech LA: The association between depres- 52. Wicherts IS, van Schoor NM, Boeke AJ, Visser M, Deeg DJ, Smit J,
sion and chronic kidney disease and mortality among patients hospi- Knol DL, Lips P: Vitamin D status predicts physical performance and its
talized with congestive heart failure. Am J Kidney Dis 44: 207–215, decline in older persons. J Clin Endocr Metab 92: 2058 –2065, 2007
2004 53. Stenvinkel P, Andersson P, Wang T, Lindholm B, Bergström J, Palm-
36. Cohen SD, Norris L, Acquaviva K, Peterson RA, Kimmel PL: Screening, blad J, Heimbürger O, Cederholm T: Do AC: E-inhibitors suppress
diagnosis, and treatment of depression in patients with end-stage tumor necrosis factor-␣ production in advanced chronic renal failure.
renal disease. Clin J Am Soc Nephrol 2: 1332–1342, 2007 J Intern Med 246: 503–507, 1999
37. Lopes AA, Albert JM, Young EW, Satayathum S, Pisoni RL, Andreucci 54. Onder G, Penninx B, Balkrishman R: Relation between use of angio-
VE, Mapes DL, Mason NA, Fukuhara S, Wikström B, Saito A, Port FK: tensin-converting enzyme inhibitors and muscle strength and physical
Depression as a predictor of mortality and hospitalization among function in older women: an observational study. Lancet 359: 926 –
hemodialysis patients in the United States and Europe in Dialysis 930, 2002
Outcomes and Practice Patterns Study (DOPPS). Kidney Int 66: 2047– 55. Sheffield-Moore M, Paddon-Jones D, Casperson SL, Gilkison C, Volpi
2053, 2004 E, Wolf SE, Jiang J, Rosenblatt JI, Urban RJ: Androgen therapy in-
38. Andrew MK, Rockwood K: Psychiatric illness in relation to frailty in duces muscle protein anabolism in older women. J Clin Endocrinol
community-dwelling elderly people without dementia: a report from Metab 91: 3844 –3849, 2006
the Canadian Study of Health and Aging. Can J Aging 26: 33–38, 2007 56. Zachwieja JJ, Yarasheski KE: Does growth hormone therapy in con-
39. Fiatarone MA, O’Neill EF, Ryan ND: Exercise training and nutritional junction with resistance exercise increase muscle force production and
supplementation for physical frailty in very elderly people. N Engl muscle mass in men and women aged 60 years or older? Phys Ther 79:
J Med 330: 1769 –1775, 1994 76 – 82, 1999
40. Gill TM, Baker DI, Gottschalk MA: Program to prevent functional 57. Butterfield GE, Thompson J, Rennie MJ, Marcus R, Hintz RL, Hoffman
decline in physically frail elderly persons who live at home. N Engl AR: Effect of rhGH and rhIGF-I treatment on protein utilization in
J Med 347: 1068 –1074, 2002 elderly women. Am J Physiol 272: E94 –E99, 1997
41. Ostir GV, Ottenbacher KJ, Markides KS: Onset of frailty in older adults 58. Nair KS, Rizza RA, O’Brien P, Dhatariya K, Short KR, Nehra A, Vittone
and the protective role of positive affect. Psychol Aging 19: 402– 408, JL, Klee GG, Basu A, Basu R, Cobelli C, Toffolo G, Dalla Man C, Tindall
2004 DJ, Melton LJ 3rd, Smith GE, Khosla S, Jensen MD: DHEA in elderly
42. Singh AS, Chin A Paw MJ, Bosscher RJ, van Mechelen W: Cross- women and DHEA or testosterone in elderly men. N Engl J Med 355:
sectional relationship between physical fitness components and func- 1647–1659, 2006
tional performance in older persons living in long-term care facilities. 59. Feldt-Rasmussen B, Lange M, Sulowicz W, Gafter U, Lai KN, Wiede-
BMC Geriatr 6: 4, 2006 mann J, Christiansen JS, El Nahas M; APCD Study Group: Growth
43. Painter P, Carlson L, Carey S, Paul SM, Myll J: Physical functioning and hormone treatment during hemodialysis in a randomized trial im-
health-related quality-of-life changes with exercise training in hemo- proves nutrition, quality of life, and cardiovascular risk. J Am Soc
dialysis patients. Am J Kidney Dis 35:482– 492, 2000* Nephrol 18: 2161–2171, 2007
44. Cheema B, Abas H, Smith B, O’Sullivan A, Chan M, Patwardhan A, 60. Kleinman KS, Schweitzer SU, Perdue ST, Bleifer KH, Abels RI: The
Kelly J, Gillin A, Pang G, Lloyd B, Singh MF: Progressive exercise for use of recombinant human erythropoietin in the correction of ane-
anabolism in kidney disease (PEAK): a randomized, controlled trial of mia in predialysis patients and its effect on renal function: a double-
resistance training during hemodialysis. J Am Soc Nephrol 18: 1594 – blind, placebo-controlled trial. Am J Kidney Dis 114: 486 – 4950,
1601, 2007* 1989
45. Taaffe DR, Harris TB, Ferrucci L, Rowe J, Seeman TE: Cross-sectional 61. Roth D, Smith RD, Schulman G, Steinman TI, Hatch FE, Rudnick MR,
and prospective relationships of interleukin-6 and C-reactive protein Sloand JA, Freedman BI, Williams WW Jr, Shadur CA: Effects of
with physical performance in elderly persons: MacArthur studies of recombinant human erythropoietin on renal function in chronic renal
successful aging. J Gerontol Series A-Biol Sci Med Sci 55: M709 –715, failure predialysis patients. Am J Kidney Dis 24: 777–784, 1994
2000 62. Clyne N, Jogestrand T: Effect of erythropoietin treatment on physical
46. Greiwe JS, Cheng B, Rubin DC, Yarasheski KE, Semenkovich CF: exercise capacity and on renal function in predialytic uremic patients.
Resistance exercise decreases skeletal muscle tumor necrosis factor in Nephron 60: 390 –396, 1992
frail elderly humans. FASEB J 15: 475– 482, 2001 63. Wuerth D, Finkelstein SH, Finkelstein FO: The identification and treat-
47. Kouidi E, Albani M, Natsis K, Megalopoulos A, Gigis P, Guiba-Tziam- ment of depression in patients maintained on dialysis. Semin Dial 18:
piri O, Tourkantonis A, Deligiannis A: The effects of exercise training 142–146, 2005
on muscle atrophy in hemodialysis patients. Nephrol Dial Transplant 64. Cukor D: Use of CBT to treat depression among patients on hemodi-
13: 685– 699, 1998 alysis. Psychiatr Serv 58: 711–712, 2007

American Society of Nephrology Geriatric Nephrology Curriculum 5


REVIEW QUESTIONS: ASSOCIATION BETWEEN CKD c. Intentional weight loss
AND FRAILTY AND PREVENTION OF FUNCTIONAL d. Low physical activity
LOSSES e. Self-reported exhaustion
1. The following factors have been suggested to predispose to
3. True or False:
frailty in elderly patients with CKD except:
a. Identifying patients at the prefrailty stage is crucial to delay
a. Anemia
or even prevent further deteriorations
b. Hypertension
b. Frailty is less prevalent in elderly with CKD versus elderly
c. Depression
with normal kidney function
d. Malnutrition
c. New treatment startegies as angiotensin converting en-
e. Decrease in growth hormone activity
zyme inhibitors is associated with a better nutritional sta-
2. he following were suggested to define frailty except: tus and lower inflammatory markers in patients with
a. Weakness CKD, and may be beneficial in improving malnutrition
b. Slow walking speed and frailty

6 Geriatric Nephrology Curriculum American Society of Nephrology


Chapter 35: Methods to Assess Quality of Life and
Functional Status and Their Applications in Clinical
Care in Elderly Patients with CKD
Tara I. Chang and Manjula Kurella Tamura
Division of Nephrology, Stanford University School of Medicine, Palo Alto, California

Elderly patients and their caregivers are faced with a ments allow for broad evaluation of overall health
wide variety of complex treatment decisions rang- across many different domains. In addition, be-
ing from the risks and benefits of antihypertensive cause generic assessments are geared toward the
therapy to the decision to pursue renal replacement general population, they allow for comparisons
therapy versus palliative care. There is a great deal of with different groups of patients and interventions.
heterogeneity in health status among elderly pa- However, generic assessments may inadvertently
tients with chronic kidney disease (CKD) and end- bias results toward or against subsets of the general
stage kidney disease (ESKD), and age alone cannot population. For example, a generic assessment with
reliably predict outcomes. In addition to measures an emphasis on physical functioning may suggest
of disease severity and psychosocial function (dis- poorer results for elderly persons or patients with
cussed elsewhere), measures of quality of life (QOL) mobility-limiting conditions such as rheumatoid
and functional status may provide useful informa- arthritis.
tion to aid in prognostic stratification and help In contrast to generic assessments, disease-spe-
guide treatment decisions. QOL and functional sta- cific assessments are designed for patients with a
tus themselves are important outcomes that need to specific disease or undergoing a particular interven-
be carefully considered along with survival when tion. However, disease-specific assessments may
evaluating treatment options. In this context, we not accurately reflect QOL if they lack items per-
review the methods to assess QOL and functional taining to dimensions of the disease that affects
status, and their applications in clinical care, focus- QOL (e.g., an ESKD scale that does not assess pru-
ing on ESKD-related interventions and outcomes. ritus). Another limitation of disease-specific assess-
ments is that because they are developed specifically
for use in a particular disease state or for assessment
HEALTH-RELATED QUALITY OF LIFE of a particular intervention, it is difficult to com-
pare the results to a different population.
Quality of life (QOL) is a concept that most people
intuitively understand, yet is difficult to define pre-
cisely. Most definitions of QOL are centered on the METHODS TO ASSESS HRQOL
notion of health put forth by the World Health Or-
ganization as “a state of complete, physical, mental There are many different types of QOL assessments
and social well-being, and not merely the absence of available. It is important when choosing a HRQOL
disease or infirmity.”1 Health-related QOL assessment tool to understand the context and pa-
(HRQOL) focuses specifically on the influence of tient population in which the tool was developed,
health, illness, and medical treatment on QOL. because this affects the reliability and validity of the

GENERIC VERSUS DISEASE-SPECIFIC


ASSESSMENTS Correspondence: Manjula Kurella Tamura, Division of Nephrol-
ogy, Stanford University School of Medicine, 780 Welch Road,
HRQOL assessments are divided into two general Suite 106, Palo Alto, CA 94304. E-mail: [email protected]

types: generic and disease specific. Generic assess- Copyright 䊚 2009 by the American Society of Nephrology

American Society of Nephrology Geriatric Nephrology Curriculum 1


Table 1. Common HRQOL instruments
Time to Validated in Validated in
Test Domains Tested Resources
Complete (min) ESRD Elderly
SF-36 Physical functioning 5–10 Yes, HD and PD Yes RAND corporation
http://www.rand.org/health/
surveys_tools.html
Limitations in usual role
activities due to physical
health problems
Bodily pain
General health perceptions
Vitality
Social functioning
Limitations in usual activities
because of emotional
problems
General mental health
SIP Physical—ambulation, body 20–30 Yes No Medical Outcomes Trust
care
Psychosocial—alertness, HD and PD http://www.outcomes-trust.org
emotional behavior, social
interactions
Independence– sleep,
eating, work, recreation
NHP Physical mobility 10–15 No Yes
Pain
Social isolation
Emotional reactions
Energy
Sleep
KDQOL- SF Eight generic domains used 20–30 Yes No KDQOL working group
in SF-36
Overall health rating HD only http://www.gim.med.ucla.edu/kdqol/
Eleven ESRD-related
domains: symptoms,
effects of kidney disease,
burden of kidney disease,
work status, cognitive
function, quality of social
interaction, sexual
function, sleep, social
support, dialysis staff
encouragement, patient
satisfaction
CHEP Eight generic domains used 25 Yes No
in SF-36
Eight other generic HD and PD
domains: cognitive
function, sexual function,
sleep, work, recreation,
travel, finances, general
quality of life
Five ESRD-related domains:
diet, freedom, body
image, dialysis access,
symptoms
HD, hemodialysis; PD, peritoneal dialysis; SF-36, Short Form 36; SIP, Sickness Impact Profile; NHP, Nottingham Health Profile; KDQOL-SF, Kidney Disease
Quality of Life Short-Form; CHEP, CHOICE Health Experience Questionnaire.

2 Geriatric Nephrology Curriculum American Society of Nephrology


assessment. Some common assessments are summarized in FUNCTIONAL STATUS
Table 1.2,3
By far the most frequently used generic QOL assessment is Functional status refers to the ability to perform daily activities
the Medical Outcomes Study (MOS) 36-item Short Form required to meet basic self-care needs and to maintain health
Health Survey (SF-36).4 The assessment takes approximately 5 and well being. Functional status reflects both functional ca-
to 10 min to complete and captures information about the pacity, what an individual is capable of doing, and functional
individual’s overall health in eight domains.5 All domains in performance, what an individual actually does in daily life.
the SF-36 are scored from 0 to 100, with higher scores indicat- Functional status may be affected by impairments in physical,
ing higher QOL. The scores are compared with a standardized cognitive, sensory, or social function. For example, impair-
norm in a defined population, such as the US general popula- ments in physical function, (e.g., muscle strength) or in sensory
tion. There are shorter versions of the SF-36 known as the function (e.g., balance) may both result in difficulty in walking.
SF-12 and SF-20, which take 1 to 2 min to administer but are
used less often. The SF-36 has been validated in the ESKD
population and in general elderly populations. The Sickness METHODS TO ASSESS FUNCTIONAL STATUS
Impact Profile (SIP) and Nottingham Health Profile (NHP)
are two other commonly used generic assessment tools. The Karnofsky Performance Status (KPS) was developed to
There are also several kidney disease–specific HRQOL in- measure functional status in terminally ill patients, and in par-
struments available. The Kidney Disease Quality of Life ticular, those with cancer, but has been used in many other
(KDQOL) instruments were initially developed to evaluate the chronic illnesses including ESKD (Table 2).5–9 Patients are as-
impact of erythropoietin in hemodialysis patients. The first signed a score ranging from 0 (dead) to 100 (perfect health).7
assessment, called KDQOL–long form (LF) included 134 ques- The scores are usually assigned by the clinician or interviewer.
tions that spanned 11 kidney disease targeted scales. In re- The scale is easy to use but does not detect early changes in
sponse to criticisms of the KDQOL-LF that it was overly long functional status. The Katz Activities of Daily Living (ADL)
and redundant, the KDQOL-SF (short form) was released. scale was developed to measure functional status in the elderly
This test includes questions from the SF-36 plus an additional and in those with chronic disease (Table 3).8 The observer de-
43 kidney disease–specific items. A shorter version of this in- termines the level of independence on a three-point scale rang-
strument, known as the KDQOL-36 is also available, which ing from independent to dependent in each of the following six
contains the same items as in the generic SF-12 along with an activities: bathing, dressing, toileting, transferring, continence,
additional 24 questions that are kidney disease specific. Some and feeding. It was initially designed for use by direct observa-
recommend the KDQOL-36 as the preferred measurement tion over a period of weeks but has been adapted for use in an
tool for large-scale assessments in dialysis facilities because of interview setting. Because it was used to identify impairments
its ease of administration with relatively minimal patient and in basic skills, it may be most useful in populations with pre-
staff burden.6 The Choices for Healthy Outcomes in Caring for existing impairments (such as a nursing home setting) or to
ESKD (CHOICE) Health Experience Questionnaire (CHEP) is identify care needs after acute events such as hospitalization.
another HRQOL assessment that, unlike the KDQOL instru- The Lawton Brody Instrumental Activities of Daily Living
ments, was designed for both hemodialysis and peritoneal di- (IADL) scale was developed to assess performance in everyday
alysis populations. tasks among community-dwelling elderly and is commonly

Table 2. Karnofsky Performance Status scale


Description Score
Normal no complaints; no evidence of disease. 100 Able to carry on normal activity and to work;
Able to carry on normal activity; minor signs or symptoms of disease. 90 no special care needed
Normal activity with effort; some signs or symptoms of disease. 80
Cares for self; unable to carry on normal activity or to do active 70 Unable to work; able to live at home and
work. care for most personal needs; varying
Requires occasional assistance, but is able to care for most of his 60 amount of assistance needed
personal needs.
Requires considerable assistance and frequent medical care. 50
Disabled; requires special care and assistance 40 Unable to care for self; requires equivalent
Severely disabled; hospital admission is indicated although death not 30 of institutional or hospital care; disease
imminent. may be progressing rapidly
Very sick; hospital admission necessary; active supportive treatment 20
necessary.
Moribund; fatal processes progressing rapidly. 10
Dead 0

American Society of Nephrology Geriatric Nephrology Curriculum 3


Table 3. Methods to assess functional status
Previously used
Test Domains evaluated Resources
in ESRD
Karnofsky Performance Status scale Global assessment of performance Yes See Table 2
status
Katz’s Activities of Daily Living (ADL) Toileting Yes Society of Hospital Medicine
scale
Feeding http://www.hospitalmedicine.org
Dressing
Grooming
Physical ambulation
Bathing
Lawton Brody Instrumental Activities of Ability to use telephone Yes Society of Hospital Medicine
Daily Living (IADL) scale
Shopping http://www.hospitalmedicine.org
Food Preparation
Housekeeping
Laundry
Mode of Transportation
Responsibility for Own
Medications
Ability to Handle Finances
Rosow-Breslau Health scale Walking half a mile No
Walking up and down stairs
Doing heavy work around house
Nagi scale Pulling or pushing large objects No
Stooping, crouching or kneeling
Reaching or extending arms
Writing or handling small objects

used in conjunction with the ADL scale.9 The IADL scale eval- ESKD. For example, in one study of ⬎17,000 hemodialysis
uates skills necessary to live independently, including using the patients, a 10-point lower physical component summary
telephone, food preparation, handling finances, and taking (PCS) score on the KDQOL-SF was associated with a 25%
medications. Compared with Katz’s ADL scale, which assesses increased risk of death, similar to the predictive ability of low
basic functions, it is probably more sensitive to early changes in serum albumin.12 In another study of 1000 hemodialysis pa-
functional status. The Rosow-Breslau Health scale was devel- tients, patients with a PCS score below the median (⬍34) were
oped to assess functional status in the elderly.10 The scale as- twice as likely to die as those patients with PCS scores at or
sesses ability to perform physical tasks requiring mobility and above the median.13
strength, such as walking half a mile, climbing up stairs, and Information gathered by HRQOL and functional status in-
doing heavy housework. Like the Lawton-Brody scale, it is struments, combined with other clinical data, can be used to
most appropriate for community-dwelling elderly patients. estimate prognosis. For example, in one study of 292 patients
The Nagi scale evaluates four types of physical activity: pushing starting dialysis, a risk stratification score based on age, func-
or pulling large objects, stooping, crouching, or kneeling, tional status, comorbidity, and planned versus unplanned di-
reaching or extending arms above shoulder level, and writing alysis initiation was used to identify groups with low, medium,
or handling small objects.11 As opposed to the ADL and IADL and high mortality risk.7 Another study of 146 octogenarians
scales, the latter two methods assess specific physical activities starting dialysis categorized patients into risk groups based on
and therefore may be more useful for identifying areas for in- body mass index, functional status, and early versus late refer-
tervention. ral. Patients with a body mass index ⱕ18 kg/m2, a KPS ⱕ40,
and referred ⬍4 mo before starting dialysis had an estimated
83% risk of mortality in the first year, whereas patients meeting
APPLICATIONS IN CLINICAL CARE none of these criteria had an estimated 1-yr mortality risk as
low as 15%.14
Risk Stratification Estimates of prognosis thus facilitate informed decision
Both HRQOL and functional status are strong predictors of making and advanced care planning. Ideally, when discussing
adverse outcomes among patients with incident and prevalent dialysis decision making, it is useful to provide information

4 Geriatric Nephrology Curriculum American Society of Nephrology


about expected QOL and functional status in addition to sur- REFERENCES
vival; however, data are often limited and sometimes conflict- *Key References
ing in this area.15–17 1. World Health Organization: The Constitution of the World Health
Organization. Geneva, World Health Organization, 1947
Evaluating Treatment Options 2. Haywood K, Garratt A, Fitzpatrick R: Quality of life in older people: a
Information about expected HRQOL and functional status can structured review of generic self-assessed health instruments. Quality
Life Res 14: 1651–1668, 2005
be especially important when interventions may have similar 3. Cagney KA, Wu AW, Fink NE, Jenckes MW, Meyer KB, Bass EB, Powe
efficacy but different effects on HRQOL and/or functional sta- NR: Formal literature review of quality-of-life instruments used in
tus, such as the choice of dialysis modality. For example, in the end-stage renal disease. Am J Kidney Dis 36:327–336, 2000*
CHOICE study, patients on hemodialysis and peritoneal dial- 4. Hickey A, Barker M, McGee H, O’Boyle C: Measuring health-related
ysis had similar overall HRQOL after 1 yr; however, hemodi- quality of life in older patient populations. A review of current ap-
proaches. Pharmacoeconomics 23:971–993, 2005*
alysis patients experienced greater improvements in sleep and 5. Ware JE Jr, Sherbourne C: The MOS 36-item short-form health survey
physical function domains, whereas peritoneal dialysis pa- (SF-36). I Conceptual framework and item selection. Med Care 30:
tients experienced greater improvements in the finance do- 473– 483, 1992
main of QOL.18 6. Kalantar-Zadeh K, Unruh ML: Health related quality of life in patients
with chronic kidney disease. Int Urol Nephrol 37: 367–378, 2005
7. Chandna SM, Schulz J, Lawrence C, Greenwood RN, Farrington K: Is
Monitoring Disease Course and Response to Therapy
there a rationale for rationing chronic dialysis? A hospital based cohort
Serial assessment of HRQOL and functional status may be a study of factors affecting survival and morbidity. BMJ 318: 217–223, 1999
useful way to monitor disease course and response to therapy, 8. Katz S, Ford AB, Moskowitz RW, Jackson BA, Jaffe MW: Studies of
although it should be noted that for many of the measures illness in the aged. The index of ADL: A standardized measure of
discussed here, controversy exists over what constitutes a sig- biological and psychosocial function. JAMA 185: 914 –919, 1963
9. Lawton MP, Brody EM: Assessment of older people: self-maintaining and
nificant change because several measures have not been vali- instrumental activities of daily living. Gerontologist 9: 179 –186, 1969
dated for longitudinal use. Nonetheless, in an elderly patient 10. Rosow I, Breslau N: A Guttman health scale for the aged. J Gerontol
with advanced CKD, declining HRQOL and/or functional sta- 21: 556 –559, 1966
tus in the absence of new medical problems may signal the 11. Nagi S: An epidemiology of disability among adults in the United
need for initiation of dialysis. Similarly, in situations where States. Milbank Mem Fund Q Health Soc 54: 439 – 467, 1976
12. Mapes DL, Lopes AA, Satayathum S, McCullough KP, Goodkin DA,
prognosis is unclear and a time-limited trial of dialysis is Locatelli F, Fukuhara S, Young EW, Kurokawa K, Saito A, Bommer J,
started, declining HRQOL and functional status may prompt Wolfe RA, Held PJ, Port FK: Health-related quality of life as a predictor
consideration of dialysis withdrawal. of mortality and hospitalization: the dialysis outcomes and practice
patterns study (DOPPS). Kidney Int 64: 339 –349, 2003
Other Applications 13. DeOreo P: Hemodialysis patient-assessed functional health status pre-
dicts continued survival, hospitalization, and dialysis-attendance com-
HRQOL and functional status assessments have been shown to pliance. Am J Kidney Dis 30: 204 –212, 1997
improve patient–physician communication19 and can high- 14. Joly D, Anglicheau D, Alberti C, Nguyen AT, Touam M, Grünfeld JP,
light areas of concern that might otherwise be overlooked. For Jungers P: Octogenarians reaching end-stage renal disease: cohort
example, understanding a patient’s ability to perform self-care study of decision-making and clinical outcomes. J Am Soc Nephrol 14:
functions might lead to implementation of assistive devices, 1012–1021, 2003
15. Lamping DL, Constantinovici N, Roderick P, Normand C, Henderson L,
use of structured rehabilitation programs, or referral to an as- Harris S, Brown E, Gruen R, Victor C: Clinical outcomes, quality of life,
sisted living setting. There are some data to suggest that use of and costs in the north thames dialysis study of elderly people on
these assessment methods in clinical practice leads to improve- dialysis: a prospective cohort study. Lancet 356: 1543–1550, 2000
ment in patient satisfaction and overall HRQOL without a sig- 16. Unruh ML, Newman AB, Larive B, Amanda Dew M, Miskulin DC,
nificant increase in consultation time.20 Greene T, Beddhu S, Rocco MV, Kusek JW, Meyer KB; Hemodialysis
Study Group: The influence of age on changes in health-related
quality of life over three years in a cohort undergoing hemodialysis.
J Am Geriatr Soc 56: 1608 –1617, 2008*
TAKE HOME POINTS 17. Ifudu O, Mayers J, Matthew J, Tan CC, Cambridge A, Friedman EA:
• Assessment of HRQOL and functional status can have many applica- Dismal rehabilitation in geriatric inner-city hemodialysis patients.
tions in clinical care of elderly patients: estimating prognosis, evaluat- JAMA 5: 29 –33, 1994
ing treatment options, monitoring disease and/or therapy, and identi- 18. Wu AW, Fink NE, Marsh-Manzi JV, Meyer KB, Finkelstein FO, Chap-
fying occult problems man MM, Powe NR: Changes in quality of life during hemodialysis and
• Tables 1 and 3 list online resources with information on how to access peritoneal dialysis treatment: generic and disease specific measures.
the HRQOL and functional assessment instruments J Am Soc Nephrol 15: 743–753, 2003
• Most measures are simple to use and can be performed during a 19. Detmar SB, Muller MJ, Schornagel JH, Wever LD, Aaronson NK:
routine clinic visit Health-related quality-of life assessments and patient -physician com-
munication. JAMA 288: 3027–3034, 2002*
20. Frost MH, Bonomi AE, Cappelleri JC, Schünemann HJ, Moynihan TJ,
Aaronson NK; Clinical Significance Consensus Meeting Group: Apply-
DISCLOSURES ing quality-of-life data formally and systematically into clinical practice.
None. Mayo Clin Proc 82: 1214 –1228, 2007*

American Society of Nephrology Geriatric Nephrology Curriculum 5


REVIEW QUESTIONS: METHODS TO ASSESS a. Bathing ability
QUALITY OF LIFE AND FUNCTIONAL STATUS AND b. Performance of housework
THEIR APPLICATIONS IN CLINICAL CARE IN c. Handling finances
ELDERLY PATIENTS WITH CKD d. Peak oxygen uptake during a treadmill test
1. Which of the following is true about the Kidney Disease Qual-
3. Which of the following are potential applications of quality-
ity of Life (KDQOL) instrument?
of-life and/or functional status assessment in elderly ESKD
a. It includes generic and disease-specific items
patients?
b. It has been validated in ESKD patients
a. Evaluating peritoneal dialysis versus hemodialysis as a
c. It includes questions from the Medical Outcomes Study
treatment options
Short Form Health Survey (SF-36)
b. Identifying areas of vulnerability prior to entry into a re-
d. All of the above
habilitation program
2. Which of the following would not be considered a measure of c. Determining the appropriateness of dialysis withdrawal
functional status? d. All of the above

6 Geriatric Nephrology Curriculum American Society of Nephrology


Chapter 36: Recognizing Delirium, Dementia, and
Depression
Manjula Kurella Tamura
Division of Nephrology, Stanford University School of Medicine, Palo Alto, California

Neuropsychiatric disorders such as delirium, demen- high risk. Several ESKD-specific syndromes of delir-
tia, and depression are common yet poorly recognized ium deserve special mention:
causes of morbidity and mortality among elderly per-
sons with chronic kidney disease (CKD) including Uremic Encephalopathy.
end-stage kidney disease (ESKD). Patients with neu- Uremic encephalopathy is a syndrome of delirium
ropsychiatric disorders are at higher risk for death, seen in untreated ESKD. It is characterized by lethargy
hospitalization, and dialysis withdrawal. These disor- and confusion in early stages and may progress to sei-
ders are also likely to reduce quality of life and hinder zures and/or coma. It may be accompanied by other
adherence with the complex dietary and medication neurologic signs, such as tremor, myoclonus, or as-
regimens prescribed to patients with CKD. This chap- terixis. Although rarely used for diagnostic purposes,
ter will review the evaluation and management of de- the EEG shows a characteristic pattern in patients with
lirium, dementia, and depression among persons with uremic encephalopathy.2 The syndrome is rapidly re-
CKD and ESKD. versed with dialysis or kidney transplantation.

Dialysis Dysequilibrium.
DELIRIUM This syndrome of delirium is seen during or after
the first several dialysis treatments. It is most likely
Delirium is an acute confusional state characterized to occur in elderly patients with severe azotemia
by a recent onset of fluctuating awareness, impair- undergoing high efficiency hemodialysis; however,
ment of memory and attention, and disorganized it has also been reported in patients undergoing
thinking that can be attributable to a medical con- peritoneal dialysis and long-term hemodialysis.3
dition, intoxication, or medication side effects. A The syndrome is characterized by symptoms of
diagnostic algorithm based on Diagnostic and Sta- headache, visual disturbance, nausea, or agitation,
tistical Manual of Mental Disorders, fourth edition and in severe cases, delirium, lethargy, seizures, and
(DSM-IV) criteria, the Confusion Assessment even coma. The incidence and severity of this syn-
Method, has a sensitivity and specificity for delir- drome are felt to be declining because of earlier ini-
ium detection ⬎90% (Table 1).1 tiation of dialysis and institution of preventative
measures in high-risk patients (Table 2).
Risk Factors
Delirium is typically precipitated by an acute or Prevention and Management of Delirium
subacute event such as a neurologic disorder In hospitalized patients, preventative measures can
(stroke, subdural hematoma, hypertensive encepha- reduce incidence and costs associated with delirium
lopathy), infection, electrolyte disorder (hypoglyce- (Table 3).4 Once identified, management of delirium
mia, hyponatremia, hypernatremia, hypercalcemia), is aimed at identification and treatment of precipitat-
intoxication (alcohol, drugs, star fruit), or sleep disor- ing factors and management of behavioral symptoms.
der. Elderly patients with cognitive or sensory impair- Pharmacologic therapy is indicated only when delir-
ment or those taking multiple medications are
thought to be most vulnerable for delirium. Typically, Correspondence: Manjula Kurella Tamura, Division of Nephrol-
one or more precipitants exist in a vulnerable patient. ogy, Stanford University School of Medicine, 780 Welch Road,
Elderly patients with multiple chronic diseases, such Suite 106, Palo Alto, CA 94304. E-mail: [email protected]

as elderly patients with ESKD, are thus at especially Copyright 䊚 2009 by the American Society of Nephrology

American Society of Nephrology Geriatric Nephrology Curriculum 1


Table 1. Confusion assessment method 60% to 80% of cases of dementia, vascu-
Criteria Assessment lar dementia for 10% to 20%, and other
1. Acute onset and fluctuating course Is there an acute change in mental status from baseline? syndromes such as Parkinson’s disease–
Does mental status fluctuate throughout the day? associated dementia and frontotempo-
2. Inattention Is there difficulty focusing attention? ral dementia for the remainder. In the
3. Disorganized thinking Is thinking disorganized or incoherent? ESKD population, some studies suggest
4. Altered level of consciousness Is there hyperalertness, lethargy, stupor, or coma? that vascular dementia may be much
Diagnosis requires criteria 1, 2, and either 3 or 4. more prominent than in the general
Adapted from Inouye et al.1
population.6,7 However, it should be
ium threatens patient safety or interrupts essential therapy. In noted that the distinction between vas-
these cases, haloperidol is the agent of choice, with a usual starting cular dementia and Alzheimer’s dementia is not always
dose of 0.25 to 0.5 mg twice daily. No dosage adjustment is needed straightforward, because there is considerable overlap in risk
in ESKD. Atypical antipsychotic medications, such as risperidone, factors and clinical features. Dialysis dementia is a syndrome
olanzapine, and quietapine, have also been used for this purpose. related to aluminum intoxication first described several de-
Because of a possible risk of increased mortality when used in cades ago when aluminum contamination of dialysate fluid
patients with dementia, short-term use is recommended. Benzo- 5 and the use of aluminum containing binders were more prev-
diazepines, atypical antipsychotics, and some antidepressants alent; however, this disorder is now felt to be rare.
have also been used for treatment of delirium, but these agents
have side effects that make them less desirable as first-line agents Risk Factors
except in the case of benzodiazepines for alcohol withdrawal. Sup- Older age is a strong risk factor for dementia and mild cognitive
portive care to prevent aspiration, deep venous thrombosis, and impairment. The prevalence of dementia quadruples with each
pressure sores should also be provided in all patients. decade of age above the age of 60, reaching 30% over the age of
85.8 Stroke and vascular risk factors associated with increased
stroke risk, such as hypertension, hyperlipidemia, and diabetes,
DEMENTIA are linked with an increased risk for dementia in most studies.
Anemia, a nontraditional vascular risk factor, has also been linked
Dementia is a chronic confusional state characterized by im- with dementia and cognitive impairment in both the general el-
pairment in memory and at least one other cognitive domain, derly population and among hemodialysis patients.9,10
such as language, orientation, reasoning, or executive func- There is growing recognition that CKD is also a strong risk
tioning. The impairment in cognitive function must represent factor for dementia and mild cognitive impairment, indepen-
a decline from the patient’s baseline level of cognitive function dent of age and traditional vascular risk factors. The prevalence
and must be severe enough to interfere with daily activities and of cognitive impairment seems to rise early in the course of
independence. The term mild cognitive impairment denotes CKD, and increases as GFR declines, reaching 20% for persons
cognitive impairment (usually memory impairment) beyond with a GFR ⬍20 ml/min per 1.73 m2 (Figure 1).11 Among
that associated with normal aging but without associated func- hemodialysis patients, the prevalence of cognitive impairment
tional impairments. Individuals with mild cognitive impair- is even higher, estimated at 27% to 37%.12,13 An issue that re-
ment have a high risk of progression to dementia. mains unresolved is the extent to which neurodegenerative dis-
ease, subclinical cerebrovascular disease, unresolved uremia,
Epidemiology or the dialysis process itself contributes to cognitive impair-
In the general population, Alzheimer’s dementia accounts for ment and dementia.

Evaluation
History taking (ideally from the patient and caregiver) should
focus on the duration and severity of cognitive and behavioral
deficits, as well as use of medications that might interfere with
cognitive function such as antihistamines, antipsychotics, and

Table 2. Prevention of dialysis dysequilibrium


1. Identify high-risk patient: elderly, severe azotemia, initial dialysis
treatment
2. Perform reduced efficiency dialysis: reduce blood and dialysate
flow rates or perform continuous renal replacement therapy
Figure 1. Unadjusted prevalence of cognitive impairment and 3. Increase dialysate sodium concentration
cerebrovascular disease by estimated GFR among 23,405 US 4. Administer mannitol
adults. Adapted from reference 11. Adapted from Owen et al.24

2 Geriatric Nephrology Curriculum American Society of Nephrology


Table 3. Measures for prevention of delirium in also be ruled out. The role of testing for genetic markers of
hospitalized patients dementia risk (e.g., apoE variants) is controversial.
1. Provide orienting communication
2. Use adaptive equipment such as visual and/or hearing aids Management
3. Encourage early mobilization Two classes of medications are now available for treatment of
4. Prevent dehydration both Alzheimer’s and vascular dementia. Cholinesterase in-
5. Provide uninterrupted sleep time hibitors are approved for treatment of mild to moderate de-
6. Avoid psychoactive drugs
mentia, whereas memantine, an N-methyl-D-aspartate recep-
7. Treat pain
tor antagonist, is approved for treatment of moderate to severe
Alzheimer’s dementia and may also have some efficacy in treat-
anticholinergics. Routine screening for dementia in the general ment of vascular dementia. The clinical benefit of both classes
population is controversial. However, given the high preva- of agents seems to be modest, and the effect of treatment on
lence of cognitive impairment in the CKD population and its long-term outcomes such as nursing home placement remains
implications for disease management, screening for cognitive unclear. There is also no data on safety or efficacy of these
impairment in elderly CKD patients is probably warranted. A agents in ESKD patients; thus, therapy decisions should be
large number of screening tests are available with a range of individualized.
administration times and diagnostic accuracy; thus, there is no Behavioral symptoms such as agitation or hallucinations
single best screening test available. The Mini-Mental State should be treated with a stepped approach, beginning with
Exam is perhaps the best known cognitive test for dementia nonpharmacologic approaches such as removal of precipitat-
screening and requires 7 to 10 min to administer.14 A score ing factors (e.g., pain, excessive noise), followed by psychoso-
⬍24 (of a maximum score of 30) has a sensitivity and specific- cial interventions (e.g., music therapy, caregiver education),
ity of ⬎80% for dementia detection in the general popula- and pharmacologic therapy as a last step. A key aspect of de-
tion.15 Other cognitive tests that can be administered in 5 min mentia management is the assessment of patient safety and
or less, such as the clock drawing task, the Mini-cog (consisting ability to perform self-care functions, comply with medical regi-
of the clock drawing task plus uncued recall of three words), or mens, and participate in medical decision making. Goals of care
the Short Portable Mental Status Questionnaire, have similar should be discussed early in the course of disease when possible.
performance characteristics in the general population.16 Im-
pairments in executive function, which can be assessed by the
clock drawing task, may be particularly important to identify, DEPRESSION
because impairments in this domain are strongly linked with
adherence to therapy, functional status, and ability to live in- Depression constitutes the most common neuropsychiatric
dependently. Screening tests of cognitive function may also be disorder affecting ESKD patients, with up to 45% of incident
useful for identifying patients who may lack capacity to pro- dialysis patients and up to 20% of prevalent dialysis patients
vide informed consent for medical procedures. affected.18 Despite its high prevalence, depression frequently
Delirium and depression frequently coexist with dementia; goes undiagnosed in ESKD patients, and as a result, untreated.
however, it is important to exclude these conditions as the sole
cause of cognitive impairment before establishing a diagnosis Evaluation
of dementia. In practice, this can be difficult in hemodialysis Depressive syndromes are categorized as major depression,
patients, because unresolved uremia and subtle dialysis dys- minor depression, and dysthymia. Using DSM-IV criteria, a
equilibrium can lead to temporal fluctuations in cognitive diagnosis of depression requires at least five of nine symptoms
function.17 As such, the optimal timing of cognitive function to be present daily for at least 2 wk (Table 4). Depressed mood
testing for patients on hemodialysis is unknown. Neuropsy- or anhedonia must be included, and the symptoms must be
chologic testing on a nondialysis day can be useful if the diag- severe enough to interfere with functioning.
nosis is uncertain. Screening questionnaires for depression can be difficult to
In addition to cognitive function testing, laboratory testing interpret in patients with ESKD or other coexisting conditions
for B12 deficiency and hypothyroidism is recommended for all because of the overlap between somatic symptoms of depres-
patients with suspected dementia. In ESKD patients, inade- sion and ESKD. Of the available screening tools, the Beck De-
quate dialysis, severe anemia, and aluminum toxicity should pression Inventory is perhaps the best validated screening tool

Additional Resources
http://www.hospitalmedicine.org/geriresource/toolbox Assessment tools for delirium, dementia, and depression
http://www.americangeriatrics.org/education/depression.shtml Assessment and management tools for depression
http://elderlife.med.yale.edu/public/public-main.php Management tools for hospitalized patients with delirium
http://www.psychiatryonline.com/pracGuide/pracGuideTopic_2.aspx Delirium practice guidelines

American Society of Nephrology Geriatric Nephrology Curriculum 3


Table 4. DSM-IV diagnostic criteria for depression should be strongly considered for patients with mild or mod-
1. Depressed mood erate depression.
2. Anhedonia Antidepressants are the most commonly used treatment for
3. Change in sleep depression and a mainstay of most guidelines, although re-
4. Change in appetite or weight cently their efficacy has come into question.19 Small clinical
5. Change in psychomotor activity trials have also reported short-term benefits in patients with
6. Loss of energy ESKD. In clinical trial settings, only 40% to 65% of patients
7. Reduced concentration
respond to treatment, and thus combination therapy may be
8. Thoughts of worthlessness or guilt
needed.18 Selective serotonin-reuptake inhibitors are perhaps
9. Thoughts of death or suicide
the best-studied antidepressants in patients with ESKD and
Major depression requires five or more symptoms including depressed mood
or anhedonia causing impaired functioning to be present daily for at least 2 seem to have similar safety and efficacy profiles as in the gen-
wk. Minor depression requires two to four symptoms including depressed eral population.20,21 Tricyclic antidepressants are generally not
mood or anhedonia causing impaired functioning daily for at least 2 wk.
considered first-line therapy in patients with ESKD because of
Dysthymia requires three to four symptoms including depressed mood
causing impaired functioning for at least 2 yr. their side effect profile and potential for causing cardiac con-
duction problems and orthostatic hypotension. Several excel-
to detect depression in ESKD, with sensitivity and specificity lent reviews on the treatment of depression have been pub-
rates of ⬎90%. However, because of its length, it is used more lished.22,23 In general, initial treatment should begin with low
commonly in research rather than clinical settings. Other ab- doses, and clinical response and side effects should be assessed
breviated tools, such as the Patient Health Questionnaire, frequently in the first several months. If response has been
which has been validated in ESKD and in elderly populations, suboptimal, the dose may be increased after 3 to 4 wk. If symp-
and the five-item Geriatric Depression Scale, which has been toms persist despite a full therapeutic trial of antidepressants
validated in elderly populations, may be useful when clinical (e.g., 8 to 10 wk), psychiatric referral is indicated.
assessment time is limited. Depression may have atypical pre-
sentations in elderly patients, including functional decline as
the most prominent presenting feature.
TAKE HOME POINTS
In addition to assessment of symptom duration and associ-
ated impairment, evaluation should include assessment of co- • Neuropsychiatric disorders such as delirium, dementia, and depres-
existing psychiatric disorders and suicide risk. Coexisting cog- sion are common in elderly persons with ESKD and are likely to
nitive impairment or substance abuse is especially important adversely affect adherence with therapy, quality of life, and other
important outcomes
to identify. If suicidal ideation is endorsed, intent and suicide
risk factors (older, white, male, recently hospitalized) should • Brief screening tools are available for each of these conditions to
be ascertained, and a plan for immediate psychiatric evaluation facilitate identification and management
arranged.
• Elderly patients with ESKD may have two or more coexisting neuro-
psychiatric disorders; thus, neuropsychiatric evaluation may be useful
Management in cases where the diagnosis is uncertain
In the primary care setting, untreated depression is associated
with reduced quality of life, poor adherence with treatment,
and increased morbidity and mortality. Similar findings have
been shown in some, but not all, studies of patients with ESKD. DISCLOSURES
None.
Several evidence-based treatments for depression are avail-
able, including psychotherapy, exercise therapy, pharmaco-
logic therapy, and electroconvulsive therapy.
The initial choice of therapy should be based on treatment REFERENCES
history, coexisting medical conditions, and patient prefer-
ences. Although not extensively evaluated in patients with 1. Inouye SK, van Dyck CH, Alessi CA, Balkin S, Siegal AP, Horwitz RI:
ESKD, a recent meta-analysis reported similar efficacy of psy- Clarifying confusion: the confusion assessment method. A new
method for detection of delirium. Ann Intern Med 113: 941–948, 1990
chotherapy and pharmacologic therapy in older adults. Thus,
2. Teschan PE, Ginn HE, Bourne JR, Ward JW, Hamel B, Nunnally JC,
psychotherapy should be considered when pharmacologic Musso M, Vaughn WK: Quantitative indices of clinical uremia. Kidney
therapy is not desired or not effective. Many types of therapy Int 15: 676 – 697, 1979
are available, including cognitive therapy, behavioral therapy, 3. Miller RB, Tassistro CR: Peritoneal dialysis. N Engl J Med 281: 945–
and family therapy; they differ in the underlying model of ill- 949, 1969
4. Inouye SK: Prevention of delirium in hospitalized older patients: risk
ness and focus on the individual versus family. Short-term ex-
factors and targeted intervention strategies. Ann Med 32: 257–
ercise programs (12 wk) also have proven efficacy in reducing 263, 2000 (a review of delirium risk factors and management strat-
depressive symptoms among older adults. Given the other egies)
benefits of exercise in patients with ESKD, this form of therapy 5. Schneider LS, Dagerman KS, Insel P: Risk of death with atypical

4 Geriatric Nephrology Curriculum American Society of Nephrology


antipsychotic drug treatment for dementia: meta-analysis of random- medical practice: clinical utility and acceptance. Mayo Clin Proc 71:
ized placebo-controlled trials. JAMA 294: 1934 –1943, 2005 829 – 837, 1996
6. Seliger SL, Siscovick DS, Stehman-Breen CO, Gillen DL, Fitzpatrick A, 16. Holsinger T, Deveau J, Boustani M, Williams JW Jr: Does this patient
Bleyer A, Kuller LH: Moderate renal impairment and risk of dementia have dementia? JAMA 297: 2391–2404, 2007 (a review of cognitive
among older adults: the Cardiovascular Health Cognition Study. J Am function screening tests)
Soc Nephrol 15: 1904 –1911, 2004 17. Williams MA, Sklar AH, Burright RG, Donovick PJ: Temporal effects of
7. Fukunishi I, Kitaoka T, Shirai T, Kino K, Kanematsu E, Sato Y: Psychi- dialysis on cognitive functioning in patients with ESRD. Am J Kidney
atric disorders among patients undergoing hemodialysis therapy. Dis 43: 705–711, 2004
Nephron 91: 344 –347, 2002 18. Wang PL, Watnick SG: Depression: a common but underrecognized
8. Jorm AF, Jolley D: The incidence of dementia: a meta-analysis. Neu- condition associated with end-stage renal disease. Semin Dial 17:
rology 51: 728 –733, 1998 237–241, 2004
9. Milward EA, Grayson DA, Creasey H, Janu MR, Brooks WS, Broe GA: 19. Kirsch I, Deacon BJ, Huedo-Medina TB, Scoboria A, Moore TJ, John-
Evidence for association of anaemia with vascular dementia. Neuro- son BT: Initial severity and antidepressant benefits: a meta-analysis of
report 10: 2377–2381, 1999 data submitted to the Food and Drug Administration. PLoS Med 5:
10. Grimm G, Stockenhuber F, Schneeweiss B, Madl C, Zeitlhofer J, e45, 2008
Schneider B: Improvement of brain function in hemodialysis patients 20. Wuerth D, Finkelstein SH, Ciarcia J, Peterson R, Kliger AS, Finkelstein
treated with erythropoietin. Kidney Int 38: 480 – 486, 1990 FO: Identification and treatment of depression in a cohort of patients
11. Kurella Tamura M, Wadley V, Yaffe K, McClure LA, Howard G, Go maintained on chronic peritoneal dialysis. Am J Kidney Dis 37: 1011–
R, Allman RM, Warnock DG, McClellan W: Kidney function and 1017, 2001
cognitive impairment in US adults: the Reasons for Geographic and 21. Koo JR, Yoon JY, Joo MH, Lee HS, Oh JE, Kim SG, Seo JW, Lee YK,
Racial Differences in Stroke (REGARDS) Study. Am J Kidney Dis 52: Kim HJ, Noh JW, Lee SK, Son BK: Treatment of depression and effect
227–234, 2008 of antidepression treatment on nutritional status in chronic hemodial-
12. Murray AM, Tupper DE, Knopman DS, Gilbertson DT, Pederson SL, Li ysis patients. Am J Med Sci 329: 1–5, 2005
S, Smith GE, Hochhalter AK, Collins AJ, Kane RL: Cognitive impair- 22. Wuerth D, Finkelstein SH, Finkelstein FO: The identification and treat-
ment in hemodialysis patients is common. Neurology 67: 216 –223, ment of depression in patients maintained on dialysis. Semin Dial 18:
2006 142–146, 2005 (a review of depression diagnosis and treatment in
13. Kurella M, Chertow GM, Luan J, Yaffe K: Cognitive impairment in patients with ESKD)
chronic kidney disease. J Am Geriatr Soc 52: 1863–1869, 2004 23. Mann JJ: The medical management of depression. N Engl J Med 353:
14. Folstein MF, Folstein SE, McHugh PR: “Mini-mental state.” A practical 1819 –1834, 2005 (a review of depression management in the general
method for grading the cognitive state of patients for the clinician. medical population)
J Psychiatr Res 12: 189 –198, 1975 24. Owen WF, Pereira BJG, Sayegh MH (eds): Dialysis and Transplanta-
15. Tangalos EG, Smith GE, Ivnik RJ, Petersen RC, Kokmen E, Kurland LT, tion: A Companion to Brenner and Rector’s The Kidney, 1st Ed., New
Offord KP, Parisi JE: The Mini-Mental State Examination in general York, Wiley, 2000

American Society of Nephrology Geriatric Nephrology Curriculum 5


REVIEW QUESTIONS: RECOGNIZING DELIRIUM, a. Acute onset
DEMENTIA, AND DEPRESSION b. Interference with activities of daily living
c. Memory impairment
1. Which of the following is NOT a required feature of delirium?
a. Acute onset
3. Which of the following types of therapy may be indicated for
b. Fluctuating course
patients with ESKD with mild to moderate depression?
c. Inattention
a. Exercise therapy
d. Language impairment
b. Additional erythropoietin to raise Hb levels to 12 to 13 g/dl
2. Which of the following is NOT considered a feature of demen- c. Selective serotonin reuptake inhibitors
tia? d. Psychotherapy

6 Geriatric Nephrology Curriculum American Society of Nephrology


Chapter 37: Dialysis Decisions in the Elderly Patient
With Advanced CKD and the Role of Nondialytic
Therapy
Mark Swidler
Renal Division, Mount Sinai School of Medicine, New York, New York

THE GERIATRIC PERSPECTIVE ON Barthel Index), physical performance (Timed Get up


MEDICAL DECISION MAKING and Go test; timed walking speed), frailty testing
(Frailty Phenotype4), cognition (MMSE, mini-cog),
What is most important to older adults is maintain- psychologic status (Geriatric Depression Scale), nutri-
ing independence and quality of life (QOL) through tion, medication review, urinary incontinence, visual/
optimal mental capacity and physical functioning. hearing impairment, and social support. These can
Dialysis decisions in elderly persons must move be- used to generate prognostic models for mortality, hos-
yond the traditional GFR-related indications for di- pitalization, and loss of functional independence.5–9
alysis initiation and incorporate geriatric principles The presence of combinations of functional im-
that focus on assessment of function, disabilities, pairments and geriatric syndromes will influence
comorbidities, and geriatric syndromes (e.g., frailty, prognosis, shared decision-making, and the ability
dementia, delirium, depression, falls, malnutrition, to tolerate renal replacement therapy. As a person
polypharmacy). These geriatric syndromes are ages, functional status becomes as useful as comor-
powerful predictors of adverse outcomes including bidity in risk assessment. It can improve mortality
mortality, hospitalization, nursing home place- prediction in patients ⬎80 yr and lead to more ac-
ment, and hip fractures. Chronic kidney disease curate risk adjustment.6,7 Functional data and the
(CKD), uremia, and dialysis accelerate these out- documentation of limitations are essential to dialy-
comes, especially the expression and progression of sis decision analysis.
frailty in predisposed patients. Chronological age Categorization of elderly patients based on esti-
alone should not be a factor in the dialysis decision mated life expectancy and functional level is helpful
tree, although it is associated with an increased risk to use as a starting framework for informed discus-
of death. Each elderly patient must be approached sions about medical decisions. One model (Clinical
individually. Glidepaths10,11) uses the following four categories:
Geriatricians use validated standardized tools robust older people (life expectancy ⱖ 5 yr, func-
for staging the functional age of their patient and tionally independent, and not needing help from
look for signs that increase risk for disability and caregivers); frail older people (life expectancy ⬎ 5
affect morbidity and mortality. A comprehensive yr, significant functional impairment requiring
geriatric assessment (CGA) obtained at baseline to help from caregivers); moderately demented older
define overall health status forms the basis for indi- people (life expectancy 2 to 10 yr, may or may not
vidualized diagnostic and therapeutic interventions be functionally impaired); and end-of-life older
and allows for both a general and prognostic cate- people (life expectancy of ⬍2 yr).
gorization.1 A CGA can be followed serially and Other models developed in geriatric oncolo-
used in medical decision-making as elderly patients gy1,12–15 use modified geriatric assessment para-
and their families are faced with challenges such as digms to evaluate the risks and benefits of therapy
treatment for cancer, surgery, percutaneous gas-
trostomy tube insertion, nursing home placement,
withdrawal of intensive care unit (ICU) care, and Correpondence: Mark Swidler, Renal Division, Mount Sinai
dialysis decisions. Geriatric assessment tools2,3 include School of Medicine, 1 Gustave L. Levy Place, New York, NY
evaluations of comorbidity (Charleson Comorbidity 10029. E-mail: [email protected]

Index), functional status (Karnofsky scale, Katz and Copyright 䊚 2009 by the American Society of Nephrology

American Society of Nephrology Geriatric Nephrology Curriculum 1


in elderly cancer patients. This has not been formally studied in ESRD exemplifies the disease trajectory characterized by
CKD/ESRD but the concept is similar. Dialysis has been com- long-term complications punctuated by acute medical epi-
pared with cancer in terms of mortality, adverse effects, and sodes (Figure 1).18 Renal replacement may correct uremia but
symptom burden. Classification of a prospective elderly dialy- the disease trajectory continues.
sis patient into one of the following general “functional age” CKD and dialysis are risk factors for adverse geriatric out-
categories begins the dialysis decision process (Table 1). comes. Decreasing GFR and dialysis are associated with step-
• Healthy/usual: this is the most optimal dialysis patient who might also wise increases in rates of the following: functional limita-
be a transplant candidate. tions,21 which, independent of the cause, are associated with
• Vulnerable: this is a more typical dialysis candidate. Geriatric assess- increased mortality, hospitalization, and long-term care22;
ment and intervention plans (e.g., rehabilitation, pain control, treat- frailty syndrome,23 which predicts falls, mobility/activities of
ment of cognitive deficits and depression, limiting polypharmacy, pre-
venting falls, instituting home services) may slow the progression of
daily living (ADL) disability, hospitalization, and death among
geriatric susceptibility factors that will adversely affect prognosis, QOL, community elders4 and ESRD patients24; cognitive dysfunc-
and the dialysis experience. tion25 (hemodialysis patients with dementia have a survival
• Frail: This is a suboptimal dialysis candidate and should be considered probability of 30% at 2 yr versus 60% in their nondemented
for a nondialytic treatment plan or a time-limited dialysis trial. Final counterparts, a 50% reduction in life expectancy)26; falls and
decisions will hinge on patient preferences, QOL, and contextual issues
(see Method for Evaluating Dialysis Decisions).
falls-related injury (including hip fracture), which are signifi-
cant predictors of mortality27; hospitalization,28 which in older
adults is associated with a high incidence of multiple adverse
Epidemiology of CKD/Dialysis in the Elderly
outcomes, including functional decline, delirium, and falls29;
Although dialysis is life-sustaining therapy and extends life, it
nursing home complications, where the yearly death rate is two
may also create, increase, or prolong suffering while not restor-
times that of the general ⬎65-yr-old ESRD population [cogni-
ing or maintaining function in selected subgroups of geriatric
tive/physical impairment is significant, with approximately
patients. Recent studies suggest that dialysis may not offer a
60% showing moderate to severely impaired decision-making
survival advantage in patients over the age of 75 with multiple
skills, with no advanced directives in 60 to 65%, and do not
comorbidities and cardiac ischemia.17
resuscitate (DNR) in 25 to 30%]30; and increasing symptom
Dialysis has the attributes of a serious and progressive
burden.31,32
chronic illness including (1) reduced lifespan versus age-
CKD, uremia, and dialysis will accelerate these geriatric
matched nondialysis patients; (2) progressive disability; (3) re-
outcomes in predisposed patients. The elderly CKD stage 4 to 5
peated hospitalizations; (4) significant comorbidity; (5) high
patient who presents with “geriatric susceptibility factors”
symptom burden; and (6) caregiver stress.

Table 1. Functional age (stages of aging1) categories with clinical measures


ACOVE Stage*
Healthy/Usual Vulnerable Frail
VES-13 score† 0–2 3–6 7⫹
Walking speed (m/s)‡ ⬎0.77 ⬍0.42
Chair stand time¶ (sec)§ ⬍11.2 Unable, ⬎60
Frailty score¶ 0 1–2 3–5
Syndromes** 0 1 2⫹
Remaining life expectancy†† High Middle Low
Common geriatric assessment measures
ADLs 0 1 2⫹
Instrumental ADLs 0 1 2⫹
Mini Mental State Examination ⬎26 23–26 ⬍23
Geriatric Depression Scale 0 5 6⫹
Polypharmacy (no.of medications per day) ⬍5 5–8 9⫹
Comorbidity None limiting Slight Severe
Adapted from reference 1. Reprinted with permission. 䊚 2008 American Society of Clinical Oncology. All rights reserved.
*ACOVE (Assessing Care of Vulnerable Elders) is a project that uses quality indicators to identify and treat noninstitutionalized vulnerable elders (http://
www.rand.org/health/projects/acove/acove3/).

The Vulnerable Elders Survey (VES-13) (http://www.rand.org/health/projects/acove/survey.html) is a self-administered brief function-based tool for screening
community-dwelling populations to identify older persons at risk for adverse outcomes and has been studied in cancer patients. A score of ⱖ3 identifies
vulnerable adults at risk for geriatric deficits, functional decline, and death16

Walking speed-distance 4 m/15 ft at patient’s usual speed.
§
Time it takes for patient to rise from a straight chair five times as quickly as possible.

Validated frailty screening tool with five frailty components: (1) ⬎10-lb weight loss in last year; (2) poor endurance (self-report tool); (3) low physical activity
(self-report tool); (4) slow walking time (s/15 ft); (5) weak grip strength (hand dynamometer); ⱖ3 criteria present ⫽ positive for frailty phenotype.4
**Dementia, delirium, depression, falls.
††
See reference 1, Table 2, p. 1938 and reference 15.

2 Geriatric Nephrology Curriculum American Society of Nephrology


Characterized by:

- functional decline months to years

- episodes of acute (serious) complications

- 1 year probability death .24 /hospitalization .66


general dialysis population19

- 1 year mortality rate 46% octogenarians and


nonagenarians starting dialysis 20

- un-certain course for individual dialysis patient

Figure 1. Dialysis patient disease trajectory. Adapted from ref. 18 (reprinted with permission).

such as frailty, functional disability, and serious comorbidity pairments and geriatric syndromes, decisional incapacity, and
(at least three to four conditions) is a suboptimal dialysis can- no stated prior wishes, health care proxy, or surrogates.
didate because these factors are already significant determi- The reader is referred to the Renal Physicians Association—
nants of mortality in patients on dialysis. Conversely, those American Society of Nephrology (RPA-ASN) shared decision-
patients who are more independent, less frail, and more cog- making guidelines for the accepted nephrology community
nitively intact will have a better prognosis, although still com- standard of care format in dialysis decision making.34 Because
promised in the long term (Tables 2– 4 ). geriatric patients with CKD are a heterogeneous group, the
patient should be stratified into healthy, vulnerable, or frail
based on a geriatric assessment to define and document the
METHOD FOR EVALUATING DIALYSIS DECISIONS IN functional status, presence of geriatric syndromes, comorbid-
THE ELDERLY ity, psycho-social status, and home support system, because
these will impact on prognosis and the ability to process infor-
The elderly patient with CKD being evaluated for dialysis may mation. The “Stages of Aging”1 paradigm described above is
range from a “healthy” cognitively intact and interactive per- comprehensive. For the nongeriatrician nephrologist meeting
son who can verbalize his or her preferences to a “frail” nursing the patient for the first time, the Get Up And Go Test (from a
home patient with multiple chronic conditions, functional im- sitting position, stand without using arms for support and walk
10 ft/3 m and back as quickly as possible) and the Rapid Chair
Rise (stand up from a seated position in a hardback chair with
Table 2. Geriatric susceptibility factors associated with
death in the general dialysis population arms folded) can be considered. In this case, physical frailty is
defined as scoring ⬎10 s for the go test and/or an inability to
Diabetic Nondiabetic
rise from the chair without using the arms. A moderately frail
Age (yr) patient would be unable to complete either test, whereas se-
45–64 1.43 (1.28,1.59)* 1.55 (1.40,1.72)*
verely frail is defined as the inability to complete both.35The
65–74 2.23 (2.00,2.48)* 2.65 (2.39,2.93)*
optimal situation would be a CGA done by the geriatric team
75⫹ 3.10 (2.77,3.47)* 3.91 (3.54,4.32)*
Frailty
and reviewed with the nephrologist.
BMI (kg/m2) The four topics method summarized below is a useful tem-
⬍20 1.38 (1.27,1.49)* 1.30 (1.23,1.38)* plate to address the main components of a dialysis discussion
25 to ⬍30 0.85 (0.80,0.90)* 0.84 (0.79,0.89)* in the elderly (Tables 5 and 6). Each topic is framed by under-
30⫹ 0.80 (0.75,0.85)* 0.81 (0.75,0.87)* lying ethical principles and their associated clinical counter-
Albumin 0.81 (0.78,0.84)* 0.77(0.75,0.80)* parts. Although topic 2 takes precedence, the more topics that
Functional disability can be fully explored and discussed, the better informed and
Inability to ambulate 1.36 (1.22,1.52)* 1.53 (1.36,1.72)* shared decisions will be. Two starter questions to help direct
Inability to transfer 1.46 (1.23,1.74)* 1.15 (0.95, 1.39) the initial flow of information are as follows: (1) what does the
Relative risk with 95% confidence interval).
patient/family need to know from the nephrologist to facilitate
Adapted from USRDS 2002.33
*P ⬍ 0.001. decision making (topics 1 and 3) and (2) what does the neph-

American Society of Nephrology Geriatric Nephrology Curriculum 3


Table 3. Geriatric susceptibility factors associated with death in octogenarians and nonagenarians on
dialysis (relative risk)20
• Older age c/w 80–84 yr • CHF
85–89 yr: RR 1.22 关CI, 1.20 to 1.24兴 RR 1.21 关CI, 1.19 to 1.23兴
⬎90 yr: RR 1.56 关CI, 1.51 to 1.61兴
• Underweight
• Nonambulatory status RR 1.20 关CI, 1.18 to 1.23兴
RR 1.54 关CI, 1.49 to 1.58兴
• # comorbid conditions* c/w 0–1
2 ⫺3: RR, 1.31 关CI, 1.28 to 1.33兴
• Serum albumin concentration
ⱖ 4: RR, 1.68 关CI, 1.64 to 1.72兴
⬍35 g/L RR 1.28 关CI, 1.25 to 1.30兴

*Comorbid conditions: albumin concentration ⬍35 g/L, anemia, underweight, CHF, diabetes, ischemic heart disease, COPD, cancer,
cerebrovascular disease, and PVD.

rologist need to know from the patient/family to frame possi- method, it is decided to forego renal replacement therapy, a
ble scenarios (topics 2– 4)? detailed and consistent plan of action must be imple-
mented. This approach will minimize feelings of abandon-
ment and hopelessness and foster a safe environment. Fam-
NONDIALYTIC TREATMENT/RENAL PALLIATIVE CARE ilies must feel supported, especially if their loved ones are
cognitively impaired. The decision to forego life-sustaining
A conceptual understanding of extending life versus prolong- therapy has powerful symbolic meaning. Although it might
ing the dying process and the ability to explore this with the make sense medically, families may still struggle as the ure-
elderly patient or decision maker(s) is important for shared mic process unfolds. Consistent demonstration of caring,
decision making. This is the palliative care concept of the “big respect, and concern by the team will soften this challenging
picture.” There is a growing literature on nondialytic treat- process and allow an acceptable end of life experience. The
ment (NDT),17,37–39 suggesting that survival may not be signif- medically treated patient with ESRD will require increasing
icantly different in selected subgroups between those on home services and transition into a hospice system (home,
chronic dialysis compared with patients with stage 5 CKD nursing home with hospice, or inpatient hospice depending
treated without renal replacement therapy. Less hospitaliza- on goals of care and adequacy of symptom control). This is
tions and more patient deaths at home may be possible in those best accomplished through a palliative care plan whose phi-
treated medically and using a multidisciplinary team ap- losophy incorporates a patient- and family-oriented ap-
proach.37 This may provide a more humane and dignified end proach that helps achieve their goals.
of life experience for the frail geriatric patient and their family. Renal palliative care40 – 43 is an agreed on management plan
In an important retrospective study, two groups of elderly pa- to optimize QOL and relieve suffering (pain and symptom
tients with CKD stage 5 predicted to need dialysis within 18 mo management); offered simultaneously with all other appropri-
were followed in a multidisciplinary predialysis care clinic with ate medical therapy; not synonymous with end-of-life or hos-
one group initiating renal replacement (RRT) and the other pice care; not just the absence of dialysis provision; suitable in
treated medically. Although the RRT group had a longer overall dialysis patients also; appropriate in all patients with serious
survival, when the groups were stratified into those with high co- illness (high symptom burden, shortened survival, significant
morbidity (more than three comorbid conditions) or ischemic comorbidity) and includes the following: advance care plan-
heart disease versus not, the survival curves were comparable.17 ning (ACP), a process of ongoing communication to update
If through shared decision making using the principles of prognosis/goals of care/preferences as trajectory of decline
the RPA-ASN guidelines framed within the four topics progresses and end-of-life issues become more prominent; pa-

Table 4. Survival of octogenarians and nonagenarians starting dialysis20


Average life expectancy
One-year mortality rate 46%
Median survival after dialysis initiation General population versus age-matched patient initiating dialysis
65–79 yr 24.9 mo
(interquartile range: 8.3–51.8 mo)
80–84 yr 15.6 mo 80–84 yr: 105 or 89 mo/6.7 times longer
(interquartile range: 4.8–35.5 mo)
85–89 yr 11.6 mo 85–89 yr: 78 or 66 mo/6.7 times longer
(interquartile range, 3.7–28.5 mo)
90 yr 8.4 mo 90–94 yr: 57 or 48 mo/6.8 times longer
(interquartile range: 2.8–21.3 mo)

4 Geriatric Nephrology Curriculum American Society of Nephrology


Table 5. Four topics method36 for analysis of a ethical problem in clinical medicine adapted to the geriatric patient with
CKD/ESRD
1. Medical Indications for Intervention 2. Patient Preferences
Beneficence and nonmalfeasance Respect for autonomy
Prognosis/benefits versus burdens Establish general “big picture” goals and outcomes (What is important
What is the functional age of this patient? to you when you imagine the future? e.g., stay at home, no
Is this patient frail? discomfort, live as long as possible).
What are the geriatric susceptibility factors and survival Explore patient’s personal narrative
data? Because higher prevalence of cognitive dysfunction and inability to make
What are the potential adverse geriatric outcomes ? decisions, substituted judgment will be more common. Engage the
Based on the above family.
is the patient a candidate for dialysis? Be prepared that
is the patient a candidate for nondialytic treatment? Preferences may change over time and with new events
3. Quality of Life Some patients will not able to decide or express their preferences
Beneficence and nonmalfeasance; respect for autonomy Some may want to receive limited or no information and delegate to
There is no universal metric for QOL others
QOL is a value judgment and personal 4. Contextual Features
There are some objective criteria (end-stage dementia, Loyalty and fairness
cachexia, advanced cancer) but families may not see Health resources; family dynamics; health care team
it that way Is the family supportive of the patient’s decision?
There is a significant symptom burden31,32 Are there conflicts between family members?
A defined time-limited trial to assess if QOL acceptable Are the descriptions of patient wishes consistent?
on dialysis is an important option to explore What is the cultural, ethnic, or religious belief system and background?
Is there conflict among the healthcare providers or between them and
the family?

tient and family support to create a sense of control over pa- regardless of which decision is taken. If the family decides on
tient’s healthcare, relieve potential burdens on loved ones, and dialysis, time-limited or otherwise, ongoing geriatric assess-
strengthen interfamily relationships; and hospice referral ment and palliative care protocols must be put in place. If an
when appropriate (⬍6 mo estimated survival). NDT approach is taken, follow-up for meticulous renal medi-
Renal palliative care incorporates geriatric principles of the cal management and geriatric-palliative care is imperative. In
interdisciplinary team and holistic approach. Active medical both cases, given the disease trajectory, cooperation among the
treatment of renal complications (e.g., fluid/electrolyte disor- renal and geriatric-palliative teams is essential for the best pa-
ders, renal anemia, fluid overload, CKD mineral bone disease) tient and family outcomes. A geriatric CKD action plan pro-
is continued simultaneously with evaluation and treatment of vides a framework for evaluation and management (Table 6).
geriatric syndromes and symptoms (e.g., pain, depression, fa-
tigue, insomnia, pruritus, constipation) to maximize function
and QOL, avoid unnecessary hospitalizations, and hopefully
allow a dignified death at home. TAKE HOME POINTS
• ESRD/dialysis can be viewed as a form of accelerated aging and has the
attributes of a serious progressive illness
CONCLUSION • The elderly patient with CKD facing dialysis decisions should have a
geriatric assessment to stage the functional age and geriatric syndrome
burden
An effective dialysis decision-making approach involves not • In patients with serious comorbidity, functional impairment, and frailty,
only the process leading up to and including the decision (yes, dialysis may not prolong life but might increase symptom burden and
no, maybe, time-limited) but also a clear postdecision plan ultimately suffering

Table 6. Geriatric CKD dialysis decision action plan


CKD Stage 4 CKD Stage 5
Baseline comprehensive or modified geriatric assessment to stage the Review and update geriatric assessment/functional age stage
functional age and assess for frailty and 4 topics content especially if new acute events/
hospitalizations
Initiate dialysis decision discussions in context of ”big picture” goals Renal replacement therapy (including “Time-limited trial”) or
using the RPA/ASN guidelines and four topics method
Evaluate and treat CKD geriatric susceptibility factors Nondialytic treatment
Renal palliative care assessment and treatment plan Continue renal palliative care
Hospice when estimated prognosis ⬍6 mo

American Society of Nephrology Geriatric Nephrology Curriculum 5


• Renal palliative care that incorporates geriatric principles is part of both 18. Lorenz KA, Lynn J, Dy SM, Shugarman LR, Wilkinson A, Mularski RA,
dialytic and nondialytic treatment plans; it requires not only a targeted Morton SC, Hughes RG, Hilton LK, Maglione M, Rhodes SL, Rolon C,
program of renal specific therapy, but ongoing symptom assessment Sun VC, Shekelle PG: Evidence for improving palliative care at the end
and intervention, and psychosocial support of patient and family, which of life: a systematic review. Ann Intern Med 148: 147–159, 2008*
is best accomplished thru an interdisciplinary team approach 19. US Renal Data System: USRDS 2007 Annual Data Report: Atlas of
End-Stage Renal Disease in the United States. Bethesda, MD, National
Institutes of Health, National Institute of Diabetes and Digestive and
Kidney Diseases, 2007
DISCLOSURES 20. Kurella M, Covinsky KE, Collins AJ, Chertow GM: Octogenarians and
None. nonagenarians starting dialysis in the United States. Ann Intern Med
146: 177–183, 2007*
21. Fried LF, Lee JS, Shlipak M, Chertow GM, Green C, Ding J, Harris T,
Newman AB: Chronic kidney disease and functional limitation in older
REFERENCES people: health, aging and body composition study. J Am Geriatr Soc
54: 750 –756, 2006
*Key References 22. Fried LP, Ferrucci L, Darer J, Williamson JD, Anderson G: Untangling
1. Rodin MB, Mohile SG: A practical approach to geriatric assessment in the concepts of disability, frailty, and comorbidity: implications for
oncology. J Clin Oncol 25: 1936 –1944, 2007* improved targeting and care. J Gerontol A Biol Sci Med Sci 59:
2. Gallo JJ: Handbook of Geriatric Assessment, 4th Ed., Sudbury, MA, 255–263, 2004*
Jones and Bartlett, 2006 23. Shlipak MG, Stehman-Breen C, Fried LF, Song X, Siscovick D, Fried LP,
3. Gupta A: Measurement Scales Used in Elderly Care. Oxford, UK, Psaty BM, Newman AB: The presence of frailty in elderly persons with
Radcliffe, 2008 chronic renal insufficiency. Am J Kidney Dis 43: 861– 867, 2004
4. Fried LP, Tangen CM, Walston J, Newman AB, Hirsch C, Gottdiener J, 24. Johansen KL, Chertow GM, Jin C, Kutner NG: Significance of frailty
Seeman T, Tracy R, Kop WJ, Burke G, McBurnie MA; Cardiovascular among dialysis patients. J Am Soc Nephrol 18: 2960 –2967, 2007*
Health Study Collaborative Research Group: Frailty in older adults: 25. Kurella M, Chertow GM, Fried LF, Cummings SR, Harris T, Simonsick
evidence for a phenotype. J Gerontol A Biol Sci Med Sci 56: M146 – E, Satterfield S, Ayonayon H, Yaffe K: Chronic kidney disease and
M156, 2001* cognitive impairment in the elderly: the health, aging, and body
5. Inouye SK, Peduzzi PN, Robison JT, Hughes JS, Horwitz RI, Concato J: composition study. J Am Soc Nephrol 16: 2127–2133, 2005
Importance of functional measures in predicting mortality among 26. US Renal Data System: 2008 Annual Data Report: Atlas of End-Stage
older hospitalized patients. JAMA 279: 1187–1193, 1998* Renal Disease in the United States. Bethesda, MD, National Institutes
6. Lee SJ, Go AS, Lindquist K, Bertenthal D, Covinsky KE: Chronic of Health, National Institute of Diabetes and Digestive and Kidney
conditions and mortality among the oldest old. Am J Public Health 98: Diseases, Figure 6.91, 2005
1209 –1214, 2008 27. Li M, Tomlinson G, Naglie G, Cook WL, Jassal SV: Geriatric comor-
7. Lee SJ, Lindquist K, Segal MR, Covinsky KE: Development and vali- bidities, such as falls, confer an independent mortality risk to elderly
dation of a prognostic index for 4-year mortality in older adults. JAMA dialysis patients. Nephrol Dial Transplant 23: 1396 –1400, 2008
295: 801– 808, 2006* 28. Go AS, Chertow GM, Fan D, McCulloch CE, Hsu CY: Chronic kidney
8. Reuben DB, Rubenstein LV, Hirsch SH, Hays RD: Value of functional disease and the risks of death, cardiovascular events, and hospitaliza-
status as a predictor of mortality: results of a prospective study. Am J tion. N Engl J Med 351: 1296 –1305, 2004
Med 93: 663– 669, 1992 29. Covinsky KE, Palmer RM, Fortinsky RH, Counsell SR, Stewart AL,
9. Walter LC, Brand RJ, Counsell SR, Palmer RM, Landefeld CS, Fortinsky Kresevic D, Burant CJ, Landefeld CS: Loss of independence in activ-
RH, Covinsky KE: Development and validation of a prognostic index ities of daily living in older adults hospitalized with medical illnesses:
for 1-year mortality in older adults after hospitalization. JAMA 285: increased vulnerability with age. J Am Geriatr Soc 51: 451– 458, 2003
2987–2994, 2001* 30. US Renal Data System: 2004 Annual Data Report: Atlas of End-Stage
10. Flaherty JH, Morley JE, Murphy DJ, Wasserman MR: The development Renal Disease in the United States. Bethesda, MD, National Institutes
of outpatient Clinical Glidepaths. J Am Geriatr Soc 50: 1886 –1901, of Health, National Institute of Diabetes and Digestive and Kidney
2002* Diseases, Figure 3.40 –3.42, 2004
11. Reuben DB: Guidelines, evidence-based medicine, and Glidepaths: 31. Murtagh FE, Addington-Hall J, Higginson IJ: The prevalence of symp-
talking the talk. J Am Geriatr Soc 50: 1905–1906, 2002 toms in end-stage renal disease: a systematic review. Adv Chronic
12. Basso U, Monfardini S: Multidimensional geriatric evaluation in elderly Kidney Dis 14: 82–99, 2007*
cancer patients: a practical approach. Eur J Cancer Care (Engl) 13: 32. Weisbord SD, Carmody SS, Bruns FJ, Rotondi AJ, Cohen LM, Zeidel
424 – 433, 2004 ML, Arnold RM: Symptom burden, quality of life, advance care plan-
13. Balducci L, Yates J: General guidelines for the management of older ning and the potential value of palliative care in severely ill haemodi-
patients with cancer. Oncology (Williston Park) 14: 221–227, 2000 alysis patients. Nephrol Dial Transplant 18: 1345–1352, 2003
14. Balducci L, Extermann M, Carreca I: Management of breast cancer in 33. US Renal Data System: 2002 Annual Data Report: Atlas of End-Stage
the older woman. Cancer Control 8: 431– 441, 2001 Renal Disease in the United States. Bethesda, MD, National Institutes
15. Walter LC, Covinsky KE: Cancer screening in elderly patients: a frame- of Health, National Institute of Diabetes and Digestive and Kidney
work for individualized decision making. JAMA 285: 2750 –2756, 2001 Diseases, Figure 9A, 2002
16. Mohile SG, Bylow K, Dale W, Dignam J, Martin K, Petrylak DP, Stadler 34. Moss AH: Shared decision-making in dialysis: the new RPA/ASN
WM, Rodin M: A pilot study of the vulnerable elders survey-13 com- guideline on appropriate initiation and withdrawal of treatment. Am J
pared with the comprehensive geriatric assessment for identifying Kidney Dis 37: 1081–1091, 2001*
disability in older patients with prostate cancer who receive androgen 35. Gill TM, Baker DI, Gottschalk M, Peduzzi PN, Allore H, Byers A: A
ablation. Cancer 109: 802– 810, 2007 program to prevent functional decline in physically frail, elderly per-
17. Murtagh FE, Marsh JE, Donohoe P, Ekbal NJ, Sheerin NS, Harris FE: sons who live at home. N Engl J Med 347: 1068 –1074, 2002
Dialysis or not? A comparative survival study of patients over 75 years 36. Jonsen AR, Seigler M, Winslade WJ: Clinical Ethics, 6th Ed., New York,
with chronic kidney disease stage 5. Nephrol Dial Transplant 22: McGraw Hill, 2006*
1955–1962, 2007* 37. De Biase V, Tobaldini O, Boaretti C, Abaterusso C, Pertica N, Los-

6 Geriatric Nephrology Curriculum American Society of Nephrology


chiavo C, Trabucco G, Lupo A, Gambaro G: Prolonged conservative 40. Holley JL: Palliative care in end-stage renal disease: illness trajectories,
treatment for frail elderly patients with end-stage renal disease: the communication, and hospice use. Adv Chronic Kidney Dis 14: 402–
Verona experience. Nephrol Dial Transplant 23: 1313–1317, 2008* 408, 2007*
38. Wong CF, McCarthy M, Howse ML, Williams PS: Factors affecting 41. Brown EA, Chambers EJ, Eggeling C: Palliative care in nephrology.
survival in advanced chronic kidney disease patients who choose not Nephrol Dial Transplant 23: 789 –791, 2008
to receive dialysis. Ren Fail 29: 653– 659, 2007 42. Holley JL: Palliative care in end-stage renal disease: focus on advance
39. Smith C, Da Silva-Gane M, Chandna S, Warwicker P, Greenwood R, care planning, hospice referral, and bereavement. Semin Dial 18:
Farrington K: Choosing not to dialyse: evaluation of planned non- 154 –156, 2005
dialytic management in a cohort of patients with end-stage renal 43. Moss AH, Holley JL, Davison SN, Dart RA, Germain MJ, Cohen L,
failure. Nephron Clin Pract 95: c40 – c46, 2003* Swartz RD: Palliative care. Am J Kidney Dis 43: 172–173, 2004

American Society of Nephrology Geriatric Nephrology Curriculum 7


REVIEW QUESTIONS: DIALYSIS DECISIONS IN THE a. 30%
ELDERLY PATIENT WITH ADVANCED CKD AND THE b. 50%
ROLE OF NONDIALYTIC THERAPY c. 70%
d. 80%
1. The order of most important versus less important prognostic
factors for ⱖ80-yr-old dialysis patients associated with death 4. Potential geriatric adverse outcomes associated with CKD/
at 1 yr would be: ESRD include
a. ⱖ4 comorbids, poor functional status, low BMI, CHF a. Frailty
b. Poor functional status, ⱖ4 comorbids CHF, low BMI b. Cognitive dysfunction
c. Low BMI, poor functional status ⱖ4 comorbids, CHF c. Falls
d. CHF, low BMI, poor functional status ⱖ4 co-morbids d. Pain
2. In dialysis discussions, a patient’s age is used primarily to: e. All of the above
a. Make the final decision
5. A renal palliative care assessment is appropriate if:
b. Frame prognosis
a. Patient decides to try a time-limited trial
c. Decide if palliative care assessment is appropriate
b. Patient opts for nondialytic treatment
d. Exclude renal replacement therapy as an option
c. Patient has been on dialysis for 1 yr
3. The life expectancy of an elderly dialysis versus an age matched d. b and c
general population is decreased by approximately: e. a, b, and c

8 Geriatric Nephrology Curriculum American Society of Nephrology


Chapter 38: End of Life and Decision Making in Elderly
Persons With Kidney Failure
Richard Swartz and Erica Perry
Division of Nephrology, Department of Internal Medicine, University of Michigan Health System, Ann Arbor,
Michigan

DEATH AND DYING IS INEVITABLE IN Case 1


NEPHROLOGY PRACTICE Several months after the death of a very loving man
who did home hemodialysis (HD) for many years
Mortality rates for end-stage kidney disease (ESKD) with his wife’s help, the dialysis social worker received
in the United States are well established through an angry and critical letter from his teenage daughter,
decades of national surveillance1 and consistently blaming herself but also asking why no one had pre-
average 15 to 20% per year for all patients who do pared her family for her father’s loss. She described
not receive a kidney transplant. Thus, if one works how she and her sisters regretted not having the chance
in a chronic dialysis unit of 70 patients, one can to say a loving goodbye to their father and how bereft
expect to encounter at least one death per month. In their mother felt at his loss.
the acute hospital setting, multiple studies2,3 have
verified that patients who develop acute renal fail- This particular case occurred early in our career
ure severe enough to require renal replacement with ESKD, and although we had an open relation-
therapy have a 50% chance of dying in the hospital ship with the patient and his family, we failed to talk
irrespective of the type of renal replacement used about death and dying because we felt that it was a
and despite all of the technology presently available taboo subject and because we thought that we
to support attendant multiorgan comorbidity. In might spare this patient and his family anguish by
both circumstances, age increases the odds of dying not talking about the prognosis for chronic dialysis
and frames how we must approach decision making or about end of life. By avoiding such discussion, we
with elderly patients. may have missed an important opportunity to help
Working with patients who have acute or this family prepare and inadvertently contributed
chronic renal failure inevitably forces us to face pa- to the void left behind after his death.
tients who are dying. At the same time, a multitude As a nephrologist, I have learned that death is my
of studies and quality-of-care initiatives put us on companion, and the relationships that I develop
the spot and judge our practice decisions on the with patients necessarily will include talking openly
basis of outcomes that usually include mortality about end of life somewhere along the way. We be-
rates. It is no wonder that, even if we come to un- gan our own inquiry into the importance of these
derstand at some level that death is inevitable in our relationships shortly after Case 1 and during a par-
practice, there is great reluctance to talk about death ticularly discouraging spate of deaths among our
and a persisting view that death is a “failure” of our dialysis outpatients. Around the same time, the Pa-
medical skills or that talking about death will make tient Self-determination Act of 19904 was enacted,
it happen. This view is all too often reinforced by mandating that hospitals offer patients the oppor-
prevailing public sentiment and the media, and tunity to execute advance directives at the time of
practitioners tend to focus on continuing interven- admission and bringing this issue to the fore. In-
tion and cure rather than on the reality of progres-
sive illness that often is at hand. In essence, we have
not “naturalized” death as an inherent and inevita- Correspondence: Richard D. Swartz, MD, University of Michigan,
ble event that we must learn to address if we are to Department Internal Medicine, Division of Nephrology, 3914
Taubman Ctr, Box 0364, University of Michigan Health System,
deal with chronic illness effectively. Ann Arbor, MI 48109-0364. E-mail: [email protected]

Copyright 䊚 2009 by the American Society of Nephrology

American Society of Nephrology Geriatric Nephrology Curriculum 1


stead of just second-guessing ourselves because of the recent Case 2
deaths in our program, we decided to look at our experience A 17-yr-old high school athlete and scholar developed rapidly
with the death and dying process. We reviewed cases to see progressive glomerulonephritis with a multisystem illness that he
whether we had documented recognizing and discussing end survived but that required dialysis and then a kidney transplant.
of life before patients’ deaths.5 What we found was that ad- The transplant failed acutely with another multisystem illness,
dressing death and dying, whether a formal advance directives after which he went on peritoneal dialysis. Chronic pain and
document was executed or nor, significantly affected the ulti- failure to thrive prompted University Hospital consultation. Now
mate outcome for patients, families, and staff. Having such age 19, he switched to HD but continued to suffer pain and
discussions favored a “good death” that was to some degree chronic depression, both of which defied adequate treatment. His
reconciled with a measure of closure and even relief. Other family and medical staff avoided talking about prognosis and
surveys suggest that patients on chronic dialysis expect medical alternatives because he was “too young to give up,” and pushed for
staff to initiate these discussions,6,7 and the more experience we parenteral nutrition and another transplant (from his stepfa-
have dealing with death and dying, either our own personal ther).
experience or that with patients, the more prepared we are to The social worker arranged a family and staff meeting to discuss
undertake such discussions.8 the options, during which his grandmother, who had recently lost her
One approach in the ESKD setting is to undertake more husband to a painful cancer death, “broke the ice” by reminding the
open discussion of these realities earlier in the course of en- family how her husband had withdrawn from futile cancer treat-
counters with patients. An opportune time occurs when the ment and made a choice that our patient understood at the time it
options for chronic kidney disease (CKD) are first presented, happened. His grandmother asked our patient if he had thought
and now we often let folks know that, besides HD, peritoneal about that himself. Suddenly animated and with a clear sense of
dialysis (PD), and transplantation, there is a “fourth option” relief, he related how he thought about this often but worried about
that involves no dialysis but concentrates on comfort care if the impact on his family. His mother and stepfather were tearful but
circumstances so warrant. Of course, most patients usually are did not interrupt him and then shared that they did not know how he
not prepared to talk about the “fourth option” when first fac- had tolerated all that had happened to him. Medical staff ventured
ing CKD, but it is an opportunity to establish that it is indeed that we could not predict the course of another transplant and all that
an available option at the appropriate future time. In the acute might be entailed, and the patient, with his family’s assent, declined
setting, especially in the intensive care unit, it is important to and chose to withdraw from dialysis and spend his last days at home.
keep the “no dialysis” option in mind, especially when other A home visit several days later revealed an entire football team visit-
comorbidity accumulates and further intervention begins to ing and the patient looking more settled than we had ever seen him.
look futile. In both settings, this type of discussion is more His football jersey had been retired and framed, and that same
pertinent for the elderly in view of the prognosis that they face. evening, he was driven around the football field in his wheelchair
during a high school football game dedicated to him. He died peace-
fully the next day.
VOLUNTARY WITHDRAWAL FROM DIALYSIS:
WHOSE CHOICE IS IT?: Although this patient is young, his situation only empha-
sizes that it is not the age but the individual experience and
Neu and Kjellstand9 reported in a compelling article published personal circumstances that determine the appropriateness of
⬎20 yr ago that 20% of deaths among chronic dialysis patients withdrawal from life-sustaining treatments that we use for
in Minnesota resulted from voluntary withdrawal from treat- ESKD. Because this patient was young, we had our own agenda
ment. Soon thereafter, another report confirmed this experi- regarding the next set of decisions, an agenda focused on con-
ence among patients in Michigan,10 and in both studies, ad- tinuing toward the next life-prolonging modality. In the end,
vancing age and the degree of comorbidity were important the patient along with the family member having the most life
cofactors. For patients who withdraw from dialysis, death experience (his grandmother) were able to express how much
comes relatively quickly within 1 to 2 wk.1,9 More recent stud- he was suffering and ventured to open the door to discussing
ies confirmed that a “uremic” death can often be managed in a alternatives and to refusing more and more of what by now
home setting without undue suffering.11,12 However, it is com- looked to be futile medical intervention. With the elderly, we
mon for patients and families to voice concerns that with- are more apt to be open to the options, but age is not the sole
drawal from dialysis constitutes “giving up” or even “suicide,” criterion for this discussion.
thus insisting on aggressive care that does not contribute to
either quality of life or quality of death. Many patients, fami-
lies, or even medical care staff members have not had personal DECISION MAKING: EXPLORING OPENLY AND
experience with chronic illness or end of life in their own lives, WITHOUT AN AGENDA
and they are influenced by media sensationalism surrounding
stories like those of Nancy Cruzan, Terri Schiavo, or Jack Case 3
Kevorkian. A 78-yr-old man develops renal cancer in his single remaining

2 Geriatric Nephrology Curriculum American Society of Nephrology


kidney, 3 yr after the other kidney had been removed and 1 yr after can “cure” or restore health after some “acute” illness such as
angioplasty for acute myocardial infarction (MI) with congestive pneumonia, fracture, or even acute myocardial infarction.
heart failure (CHF). Present quality of life is good, with a creati- However, we cannot cure most “chronic” illnesses and can
nine level of 1.8 mg/dl and some limited exertion tolerance. The only contain them. ESKD is an excellent example of such a
tumor is located so that partial nephrectomy is not feasible. His condition. In any illness, acute or chronic, palliative care is that
urologist suggests nephrectomy and simply going on dialysis. The care focused on promoting quality of life and not simply on
patient’s sister had been a dialysis patient recently and had died prolonging longevity.13,14 If treatment can prolong life while
miserably in the midst of severe complications. The patient came also promoting quality and comfort, then so much the better.
to our clinic seeking a second opinion. Figure 1 shows the interrelation between acute and pallia-
tive care during the course of chronic illness, emphasizing the
Too often in our practice we focus only on curing the dis- increasing importance of controlling symptoms instead of cur-
ease at hand, in this case removing the remaining kidney with ing disease as chronic illness progresses. This scheme does not
cancer in it, without considering that we consign this man to imply that we ignore medical complications that can be cor-
ESKD when we undertake nephrectomy. We already have es- rected but rather that, for each and every therapeutic decision,
tablished that the prognosis for ESKD translates into a life ex- we weigh the impact of our intervention on quality of life and
pectancy of 5 to 6 yr on average, but for this man who is over symptom control versus the quantity of life and its prolonga-
75, the mortality rate could be two to three times higher and tion at all costs. In a sense, it is valuable to view dialysis itself as
translate into a life expectancy of only 2 to 3 yr.1,11 If we con- a “palliative” intervention and to consider its contribution to
sider additional comorbidity factors that increase mortality the quality rather than to the quantity of life.
risk further, including symptomatic heart disease, diabetes These distinctions between quality and quantity of life are
mellitus as the cause of CKD, significant peripheral vascular not always so clear cut, and the process of making choices often
disease, and poor functional status as indicated by the Kar- takes time, requires several discussions, and creates tension as
novsky scale,1,11 the outlook for doing nephrectomy gives one we walk a tightrope between continuing full-court pressure
pause. These statistics do not even address potential impaired and letting go.
quality of life, the intrusiveness, and the daily “hassle” that
often are part of chronic dialysis and that loom ominously for Case 4
all patients but especially for the elderly. Ironically, the overall The Palliative Care team at our hospital was called to see a 43-
prognosis11 for renal cancer in situ is at least as good as that for yr-old man with congenital heart disease and previous stroke,
ESKD11; thus, even if we cannot predict exactly how the clinical who presented with biventricular failure, hypotension, and renal
course or potential complications will unfold, we cannot sim- failure, all progressing despite infusions of cardiac supportive
ply assume a priori that removing the cancer (and the remain- agents. His sister was his major care provider and was the decision
ing kidney) is the optimal treatment. In the final analysis, we maker both for him and for his elderly mother. He had been
must seriously look at aspects of our decision that we might not functional until recent months despite his chronic illness, but be-
have initially considered. came severely confined and continuously breathless except when
Looking at outcome data in this way, there is always the kneeling at his bedside. We discussed comfort care at home with
danger that we begin to take on an impersonal approach, im- Hospice support. We also considered the role of dialysis to improve
part the data, and leave the decision to the patient. At the same his heart failure, noting that HD would pose hemodynamic diffi-
time, it is difficult not to have our own bias and agenda in going culties and that PD was feasible but would require substantial
over situations like this one. Listening to the patient’s view- home-based care. The patient and his sister still clung to prolong-
point and finding the right balance between what we know and ing life unless and until they could see no way out, and they chose
what we don’t know, or what we view as the best medical de- to defer both Hospice and dialysis in favor of one more trial of
cision and what actually fits the patient’s goals, is an art that is cardiac supportive agents infused at home.
only learned through experience and that is poignantly illus- He returned to the hospital within a few days with more dis-
trated by this case. Often there is no one right answer, and
sharing the exploration establishes a trusting relationship in
which we can empathize with the patient’s dilemma and share CURATIVE INTERVENTION
in what may well be an ongoing process aimed at choosing the HEALTH
optimal solution for the individual. DYING
PALLIATIVE CARE

THE PALLIATIVE CARE MODEL: ACUTE VERSUS


CHRONIC ILLNESS
ACUTE ILLNESS CHRONIC ILLNESS
The principles of “palliative care” articulate our understanding
of how chronic illness is distinct from acute illness. Clearly we Figure 1. The trajectory of care during chronic illness.

American Society of Nephrology Geriatric Nephrology Curriculum 3


tress and worse renal failure. Together we agreed on a time-lim- ahead. We can, at a minimum, learn what participants already
ited trial of PD. PD was extremely helpful and improved his CHF know, discuss the goals of treatment, allow for input by family
to the point he could walk, sleep in bed at night, and function at members who need to express, make pertinent suggestions,
home. His sister managed to balance her work and the caring for and assure that all participants hear the same discussion.
him and for mom during the ensuing 14 mo. Eventually, he de-
veloped refractory arrhythmias that required intensive care unit Case 5
treatment and culminated in withdrawal of support by his sister A 67-yr-old man with previous peritoneal dialysis and then sev-
with their mother’s understanding and support. eral years with a chronically failing transplant was now back on
HD. Progressive cardiac and peripheral vascular disease limited
Situations like his one demand shared decision making with his function and well being, making him increasingly dependent
active listening and flexibility rather than an agenda and as- on his wife, who often asked the questions and made the decisions.
sumptions about the best medical course to take. Here a “time- As his condition deteriorated, he asked to withdraw from dialysis.
limited trial” of dialysis provided substantial symptom relief At a family meeting, it was clear that his wife and daughter were
and improved quality of life for an extended period. This re- angry, blamed him for not trying harder, were unwilling to sup-
prieve also resulted in solid grounding from which the eventual port withdrawal of ongoing medical care, and refused to have him
decision to voluntarily withdraw dialysis care became a recon- at home if chose to withdraw from dialysis. After the meeting
ciled and acceptable choice. when his family was gone, we suggested to him that his family
seemed uncomfortable making this decision for him and that
when he died he would leave much anguish and anger. We sug-
SHARED DECISION MAKING IN ESKD: A PROCESS gested that he discuss his feelings more openly with his family,
AND NOT AN EVENT specifically that he was suffering and in pain, that he still loved
them, and that they were in no way to blame for his difficulties.
There is no substitute for undertaking direct discussion with Two days later his wife brought him for scheduled HD, asking
patients and families about quality of life, prognosis, and the if we would admit him for terminal care. His scheduled dialysis
possibility of death and dying. The recent document “Shared was deferred, and a family meeting was quickly arranged. His
Decision Making in the Appropriate Initiation of and With- wife related that now she understood his situation more clearly
drawal from Dialysis: Clinical Practice Guideline”11 is an ex- but that she could not physically care for him at home. She asked
cellent resource that includes a series of recommendations for if we could admit him so that she could stay with him at the
such discussions along with appended medical, psychologic, hospital. Other family members agreed. The patient was admit-
and epidemiologic data relevant to both the acute and chronic ted and died peacefully within several hours with his wife at his
renal failure settings. Such discussions need not wait for a crisis bedside.
situation, and there are many opportunities to explore pa-
tients’ fears and questions or to articulate their goals of care. Learning to navigate the journey through ESKD and end of
Some starting points for discussion might include talking life does not come naturally. The skills are acquired by actually
about CKD planning or advance directives, any juncture at making the journey with patients, by developing authentic re-
which there might be potentially serious treatment complica- lationships that do not hide our humanness, and by allowing
tions, or even simply obtaining informed consent for a proce- dying patients to contribute to us. In addressing goals of care
dure. and discussing medical choices, we learn to accept our own
When critical illness does develop, the discussion takes a limitations as medical practitioners and give some autonomy
different turn and often focuses pointedly on bad news, requir- and responsibility back to the patients. The challenge may
ing a thoughtful and deliberate approach. There are a number seem daunting with end points that are different than those we
of very helpful resources that offer suggestions for organizing originally conceived, but the process can be the very gratifying
family meetings15 and for actually delivering “bad news.”16 Ef- for us and contribute immeasurably to reconciliation for pa-
fective discussions require some planning that includes invit- tients and families.
ing the necessary participants, choosing an appropriate quiet
setting, establishing what participants have already heard, ex-
ploring what new information participants are ready to hear, TAKE HOME POINTS
giving an optimal amount of information that can be ab- • Life span is shortened for patients with ESKD and with AKI as well, and
sorbed, checking out what everyone has heard, and planning death is an inevitable clinical event in nephrology practice
for any subsequent meetings or actions that seem necessary. • Learning to talk about death and dying is an outgrowth of long-term
Listening well and being prepared to follow-up on unexpected relationships with patients and families that serve all parties well in this
process; these issues are obviously nearer and more pressing in elderly
developments may be pivotal to the success of such discus-
persons but are equally germane to dialysis patients of all ages
sions. A family meeting has value even when the situation • Talking about advance directives helps to focus our discussions on
seems unresolved and chaotic because any subsequent discus- chronic illness and on patients’ fears that ultimately favor patient,
sions will have been affected, and will start one or two steps survivor, and staff reconciliation when death does occur

4 Geriatric Nephrology Curriculum American Society of Nephrology


• Patients are often prepared to talk about death and dying, and as many 5. Swartz R, Perry E: Advance directives are associated with “good
as 20% of patients will eventually withdraw voluntarily from dialysis deaths” in chronic dialysis patients. J Am Soc Nephrol 3: 1623–1630,
when quality of life deteriorates; addressing these issues directly and 1993*
viewing treatment decisions from a “palliative care” perspective that 6. Holley J, Nespor S, Rault R: Chronic in-center hemodialysis patients’
focuses on quality rather than quantity of life, sometimes to withdraw attitudes, knowledge and behavior toward advance directives. J Am
from treatment, but sometimes to continue treatment Soc Nephrol 3: 1405–1408, 1993
• Shared decision making is pivotal and includes sharing available prog- 7. Perry E, Buck C, Newsome J, Berger C, Messana J, Swartz R: Dialysis
nostic information, answering questions sensitively and directly, making staff influence patients in formulating their advance directives. Am J
sure all involved parties participate and are heard, and continuing the Kid Dis 25: 262–268, 1995
relationship and discussion throughout the course of illness 8. Perry E, Swartz R, Smith-Wheelock L, Westbrook J, Buck C: Why is it
• Family meetings that include pertinent medical staff and facility at difficult for staff to discuss advance directives with chronic dialysis
delivering “bad news” sensitively are essential tools of the trade that patients? J Am Soc Nephrol 7: 2160 –2168, 1996
are learned only with experience 9. Neu S, Kjellstrand C: Stopping long-term dialysis: an empirical study
of the withdrawal of life-supporting treatment. N Engl J Med 314:
14 –20, 1986*
10. Port FK, Wolfe RA, Hawthorne VM, Ferguson CW: Discontinuation of
DISCLOSURES dialysis as a cause of death. Am J Nephrol 9: 145–159, 1989
None. 11. Renal Physicians Association and American Society of Nephrology:
Clinical Practice Guideline: Shared Decision-Making in the Appro-
priate Initiation of and Withdrawal From Dialysis. Washington, DC,
REFERENCES Renal Physicians Association and American Society of Nephrology,
2001*
*Key References 12. Cohen LM, Germain MJ, Poppel DM, Woods AL, Pekow PS, Kjell-
1. US Renal Data Service: Annual Data Report (2008). Available online at: strand CM: Dying well after discontinuing the life-support treatment of
http://www.usrds.org/adr.htm. Accessed October 24, 2008 dialysis. Arch Int Med 160: 2513–2518, 2000*
2. The VA/NIH Acute Renal Failure Trial Network: Intensity of renal 13. Cohen LM, Moss AH, Weisbord SD, Germain MJ: Renal palliative care.
support in critically ill patients with acute kidney injury. N Engl J Med J Palliat Med 9: 977–992, 2006
359: 7–20, 2008 14. Storey P: Primer of Palliative Care, 3rd Ed., Glenview, IL, American
3. Pannu N, Klarenbach S, Wiebe N, Manns B, Tonelli M; Alberta Kidney Academy of Hospice and Palliative Medicine
Disease Network: Renal replacement therapy in patients with acute 15. Ambuel B, Weissman D: Conducting a family conference EPERC Fast
renal failure. JAMA 299: 793– 805, 2008 Facts, #16. Available online at: http://www.eperc.mcw.edu/fastFact/
4. La Puma J, Orentlicher D, Moss R: Advance directives on admission: ff_016.htm. Accessed April 14, 2009
clinical implications and analysis of the Patient Self-Determination Act 16. Buckman R: How to Break Bad News: A Guide for Health Care Pro-
of 1990. JAMA 266: 402– 405, 1991 fessionals. Baltimore, The Johns Hopkins University Press, 1992*

American Society of Nephrology Geriatric Nephrology Curriculum 5


REVIEW QUESTIONS: END OF LIFE AND DECISION b. Prompt discussion between patients and care givers about
MAKING IN ELDERLY PERSONS WITH KIDNEY medical choices
FAILURE c. Have no particular impact because they are often ignored
d. Have specific legal standing in most jurisdictions
1. Mortality in end-stage kidney disease:
4. Voluntary withdrawal from dialysis:
a. Is affected by age and the presence of heart disease and
a. Occurs rarely in the usual clinical setting
diabetes
b. Results in unacceptable acute symptoms and uncomfort-
b. Averages as much as 20% per year or more
able death
c. Is ⬎50% per year in persons ⬎80 yr old
c. Is not legally acceptable in most states
d. all of the above
d. Is more common with increasing age and comorbidity
2. The odds of in-hospital death for persons suffering acute kid- 5. Family meetings in the clinical setting necessarily involve all of
ney failure: the following except:
a. Are 50% overall a. Inviting all vested parties in family and medical care
b. Average ⬎50% if renal replacement therapy is required team(s)
c. Depend on the type of renal replacement therapy chosen b. Identifying a quiet setting that minimizes interruptions
d. None of the above c. Finishing with a plan that obviates the need for further
meetings
3. Advance directives: d. Verifying what participants already know and still need to
a. Have a major impact on outcome know

6 Geriatric Nephrology Curriculum American Society of Nephrology

You might also like